Fare

Ace your homework & exams now with Quizwiz!

Lance, Inc., a calendar year corporation, reported the following operating income (loss) before income tax and the enacted tax rates for the last three years of operations: Income Tax rate Year 2 $ 100,000 40% Year 3 $(300,000) 30% Year 4 $ 400,000 30% There are no permanent or temporary differences between operating income (loss) for financial and income tax reporting purposes. When filing its year 3 tax return, Lance elected to use only the carryforward provision. What amount should Lance report as income tax refund receivable in year 3? a) $0 b) $40,000 c) $90,000 d) $60,00

answer: a because Lance opted to only use the carryforward provision, not the carryback provision.

Decker Company assigns some of its patents to other enterprises under a variety of licensing agreements. In some instances advance royalties are received when the agreements are signed, and in others, royalties are remitted within 60 days after each license year‐end. The following data are included in Decker's December 31 balance sheet: Year 1 royalty receivable $90,000 Unearned royalties 60,000 Year 2 royalty receivable $85,000 Unearned royalties 40,000 During Year 2, Decker received royalty remittances of $200,000. In its income statement for the year ended December 31, Year 2, Decker should report royalty revenue of a) $195,000 b) $215,000 c) $220,000 d) $225,000

answer: b 200 + 20 - 5 = 215 Royalty collections + reduction in unearned royalties - reduction in royalties receivable = Royalty income

Using direct method for cash flow: The cash flow statement presented using the direct method is easy to read because it lists all of the major operating cash receipts and payments during the period by source. In other words, it lists where the cash inflows came from, usually customers, and where the cash outflows went, typically employees, vendors, etc. After all of the sources are listed, the total cash payments are then subtracted from the cash receipts to compute the net cash flow from operating activities. Then the investing and financing activities added to arrive at the net cash increase or decrease Here's a list of the most common types of receipts and payments used in the direct method format: Receipts received from Customers Payments paid to Suppliers Payments paid to Employees Interest Payments Income Tax Payments As you can see, listing these payments gives the financial statement user a great deal of information where receipts are coming from and where payments are going to. This is one of the main advantages of the direct method compared with the indirect method. Investors, creditors, and management can actually see where the company is collecting funds from and whom it is paying funds to. The indirect method doesn't list these types of details The problem with this method is it's difficult and time consuming to create. Most companies don't record and store accounting and transactional information by customer, supplier, or vendor. Business events are recorded with income statement and balance sheet accounts like sales, materials, and inventory. It's laborious for most companies to compile the information with this method. For example, in order to figure out the receipts and payments from each source, you have to use a unique formula. The receipts from customers equals net sales for the period plus the beginning accounts receivable less the ending accounts receivable. Similarly the payments made to suppliers is calculated by adding the purchases, ending inventory, and beginning accounts payable then subtracting the beginning inventory and ending accounts payable.

Lance Corp.'s statement of cash flows for the year ended September 30, year 1, was prepared using the indirect method and included the following: Net income 60,000 Noncash adjustments: Depreciation expense 9,000 Increase in accounts receivable (5,000) Decrease in inventory 40,000 Decrease in accounts payable (12,000) Net cash flows from operating activities $92,000 Lance reported revenues from customers of $75,000 in its year 1 income statement. What amount of cash did Lance receive from its customers during the year ended September 30, year 1? (this is asking to use direct method approach ) a) $80,000 b) $70,000 c) $65,000 d) $55,000 answer: b notice that net income of $60,000 is from 9/30/year 1, and $75,000 revenue is from 12/30/year 1. $75,000 revenue가 일어났는데, 실제 cash는 뭐냐..뭐 이런것 $75,000 - 5,000 A/R= $70,000. Sales -(End AR − Beg.AR)=Collections Sales-Increase in AR= Collections $75,000 − $5,000 = $70,000

Franchise fee revenue from the initial sale of a franchise is recognized by the franchisor only when all material services or conditions applicable to the sale have been substantially performed. Franchise fee revenue shall be recognized when all material services have been substantially performed by the franchisor (i.e., the franchisor has no remaining obligation to refund any cash received and substantially all of the initial services of the franchisor have been performed <---한글로 번역하자면, franchisor (The company that allows an individual to run a location of their business) 는받은 현금을 환불해야 할 의무가 없으며 franchisor의 최초 서비스의 대부분이 수행되었다면, 우리는 이런 상황을 "all material service have substantially performed by franchisor"라 한다. And this is whey franchise fee revenue must be recognized.) Thus, let's use the above concept in these 2 questions: REMEMBER, if the franchisee pays the initial franchise fees over an extended period of time, the business would use the present value of initial franchise fees.

On December 31, year 1, Reed, Inc. authorized Foy to operate as a franchisee for an initial franchise fee of $75,000. Of this amount, $30,000 was received upon signing the agreement and the balance, represented by a note, is due in three annual payments of $15,000 each beginning December 31, year 2. The present value on December 31, year 1, of the three annual payments appropriately discounted is $36,000. According to the agreement, the nonrefundable down payment represents a fair measure of the services already performed by Reed; however, substantial future services are required of Reed. Collectibility of the note is reasonably certain. On December 31, year 1, Reed should record unearned franchise fees in respect of the Foy franchise of $0 $36,000 $45,000 $75,000 answer: b Franchise fee revenue shall be recognized when all material services have been substantially performed by the franchisor (i.e., the franchisor has no remaining obligation to refund any cash received and substantially all of the initial services of the franchisor have been performed <---한글로 번역하자면, franchisor (The company that allows an individual to run a location of their business) 는받은 현금을 환불해야 할 의무가 없으며 franchisor의 최초 서비스의 대부분이 수행되었다면, 우리는 이런 상황을 "all material service have substantially performed by franchisor"라 한다. And this is whey franchise fee revenue must be recognized.) Of the initial fee of $75,000, the $30,000 down payment applies to the initial services already performed by Reed. Additionally, this amount is not refundable. Therefore, the $30,000 may be recognized as revenue in year 1. The three remaining $15,000 installments relate to substantial future services to be performed by Reed. The present value of these payments, $36,000, is recorded as unearned fees and recognized as revenue once substantial performance has occurred. Cash 30,000 Notes Receivable 45,000 Discount on NR 9,000 Franchise revenue 30,000 Unearned franchise fees 36,000 ***An unearned fee in accounting is money a business collects from a customer up front for services the company has yet to perform, such as a prepaid annual membership. If your small business collects unearned fees, you must record the fees initially as a liability on the balance sheet.****

A company has available-for-sale investments that cost $50,000 and were valued at $45,000 at the beginning of the current period during which the investments were sold for $48,000. Which of the following best reflects the impact of these events on the elements of comprehensive income of the current year? A. Impact on net income: $3,000 gain; Other comprehensive income (reclassification): $5,000 loss; Comprehensive income: $2,000 loss B. Impact on net income: $2,000 loss; Other comprehensive income (reclassification): $5,000 gain; Comprehensive income: $3,000 gain C. Impact on net income: $4,000 gain; Other comprehensive income (reclassification): $6,000 loss; Comprehensive income: $2,000 loss D. Impact on net income: $5,000 loss; Other comprehensive income (reclassification): $5,000 gain; Comprehensive income: $3,000 gain

The correct answer is B. ($50,000 value였던 AFS가 벌써 beginning of the year에 $45,000라고 되었던건, previous year에 $5,000 loss 를 OCI에다 report했다는것. after fair value measurement. 금년에, AFS를 $48,000금액으로 3자한테 팔았다. 그럼 $3,000 gain이 생기고, AFS를 파는 즉시 this gain will be reported in the net income instead of OCI. 하지만, 작년에 $5,000 loss를 OCI에 기록한게 있다. 우선 $3,000 gain recognize를 작년에 기록한 $5,000 OCI Loss한테서 net it off를 한다. it leaves only $2,000 loss that should be reported in the net income directly this year. 그다음에 중요한게 OCI에서 기록했던 $5,000 loss를 gain으로 reclassification을 해줘야한다. I guess in this way, company can reverse the loss reported previous year with new gain recognized this year (making it zero) 마지막 step은 comprehensive income이다. comprehensive income is equal to (net income + OCI) so, if you add $2,000 loss net income + $5,000 gain OCI = 3,000 gain overall. The relationship of net income and comprehensive income can be shown as follows: Other Comprehensive Net income + comprehensive = income income Ignoring any tax effects, these transactions and events affect comprehensive income as follows: Net income: Realized loss ($48,000 - $50,000) $(2,000) Other comprehensive income: Reclassification adjustment gain 5,000 -------- Comprehensive income $ 3,000 ======== A realized loss of $2,000 is recognized because investments costing $50,000 were sold for $48,000. That realized loss is included in net income. The $5,000 reclassification gain is required to offset the previously recognized unrealized loss ($50,000 - $45,000). FASB ASC 220-10-20 defines "other comprehensive income" as "all revenues, expenses, gains, and losses that under generally accepted accounting principles (GAAP) are included in comprehensive income but excluded from net income." Currently, existing GAAP specifies that unrealized holding gains and losses on available-for-sale securities should be reported as direct charges or credits to equity. Thus, these gains and losses constitute other comprehensive income. To prevent including certain items in the determination of comprehensive income twice, reclassification adjustments are required for a transaction or event that has been included as a component of other comprehensive income and later becomes a component of net income. Impact Impact Years Before Year of Sale Sale ------------ -------- Net income -0- $(2,000) Other comprehensive income: Unrealized gain/(loss) $(5,000) Reclassification -0- 5,000 -------- -------- Comprehensive income $(5,000) $ 3,000 ======== ========

Which of the following terms refers to the current harmonization of accounting practices of the United States and the countries represented by the IASB? A. Convergence B. Formal union C. Bilateral harmonization D. Synchronization

the answer is A. The FASB, SEC, and International Accounting Standards Board (IASB) are working toward convergence, as evidenced by the following discussion on IASB's website (www.fasb.org): The International Accounting Standards Board (IASB) is an international organization organized to develop international financial reporting standards (IFRSs). Additional goals of the organization are to promote the use of these standards and to work towards the convergence of the IFRSs and national accounting standards. Convergence is when two or more things come together to form a new whole

Which of the following accounts of a governmental unit is credited to close it out at the end of the fiscal year? a) Appropriations. b) Revenues. c) Fund Balance‐Assigned. d) Encumbrances.

answer: d

On December 1 of the current year, Bann Co. entered into an option contract to purchase 2,000 shares of Norta Co. stock for $40 per share (the same as the current market price) by the end of the next two months. The time value of the option contract is $600. At the end of December, Norta's stock was selling for $43, and the time value of the option is now $400. If Bann does not exercise its option until January of the subsequent year, which of the following changes would reflect the proper accounting treatment for this transaction on Bann's December 31 year‐end financial statements? a) The option value will be disclosed in the footnotes only. b)Other comprehensive income will increase by $6,000. c) Net income will increase by $5,800. d) Current assets will decrease by $200.

answer: c A stock option is a financial instrument that is recorded at its fair value and is remeasured to fair value at the end of each reporting period with the gains and losses reported in income of the period. On December 1, when Bann acquired the option, its fair value was $600. At the end of the reporting period on December 31, the fair value of the stock had increased by $3 per share ($43 − 40) and the fair value of the time value component of the option decreased to $400. Therefore, the fair value of the option at December 31 was ($3 × 2,000 shares + $400) = $6,400. The change in the fair value of the option was $6,400 − $600, resulting in a $5,800 increase in fair value. Net income is increased by the change in the fair value of the option in the amount of $5,800. **Basically, Bann purchased the option to buy shares of Norta Company's stock in certain period.. Meaning, it's not a promise that he will buy it for sure but purcahsed the rights and option to buy when he feels like to buying it.. 이런 자유스러운 option을 위해서 Bann compay paid $600 for option contract. The FMV of option value needs to be remeasure in end of the year, which will affect the net income. option contract definition: Contracts to buy and sell come in all kinds of arrangements. One of the lesser-known varieties of contracts is known as an "option contract." In a typical option contract, the seller agrees to keep an offer open for a certain amount of time. A potential buyer has to give the seller some payment in exchange. In other words, in an option contract, the seller is agreeing to keep the "option" open for the buyer. It is an option, which is a derivative, which is required to be reported in the financial statements at fair value. December 1 - Debit Derivative (current asset) $600, Credit Cash $600 (this is implied) December 31 Debit Derivative (current asset) $5,800, Credit Gain of $5,800 All Dann did was purchase the option to buy stock. But Dann Co. did not exercise its rights at 12/31 so the adjustment of the option-a trading security-to fair value at year end would result in an unrealized gain going into the income statement.

According to ASC 740, Income Taxes, justification for the method of determining periodic tax expense is based on the concept of a) Matching of periodic expense to periodic revenue. b) Objectivity in the calculation of periodic expense. c) Recognition of assets and liabilities. d) Consistency of tax expense measurements with actual tax planning strategies.

answer: c ASC Topic 740 states that the objective of accounting for income taxes is to recognize current taxes payable or refundable and deferred tax assets and liabilities for the future tax consequences of events that have been recognized in an enterprise's financial statements or tax returns. Matching is also a rationale, but a less important one.

Blue Township has a number of outstanding bond issues that include a $4,000,000 general obligation bond that financed City Hall, a $2,000,000 revenue bond that financed upgrades to the water treatment plant, a $1,000,000 special assessment bond for sidewalks, and a $3,000,000 general obligation bond used for streets and roads. Revenues of the water fund, a proprietary fund, are expected to pay off the revenue bond. What should Blue Township report as long-term liabilities in the governmental activities column of the government-wide statement of net position? A. $4,000,000 B. $7,000,000 C. $8,000,000 D. $9,000,000

The correct answer is C. The obligations of the proprietary fund, the revenue bonds of $2,000,000, should be shown as a fund liability. The other bonds should be shown in the government-wide statements as a liability relating to governmental activities, but not shown in the fund statements. The general obligation debt consists of the $4,000,000 and $3,000,000 general obligation bonds as well as the special assessment bonds of $1,000,000. Special assessment debt is usually an obligation of the general government, although some costs may be defrayed with special assessments. General obligation bonds are long-term debt that is backed by the "full faith and credit" of the governmental entity or by the general tax revenues of the government. This type of debt is not reported as a liability in a governmental fund but should be reported only in the governmental activities column in the government-wide statement of net position.

Smith owns several works of art. At what amount should these art works be reported in Smith's personal financial statements? A. Original cost B. Insured amount C. Smith's estimate D. Appraised value

The correct answer is D. Personal financial statements shall present assets at their estimated current values and liabilities at their estimated current amounts at the date of the financial statements. A set of guidelines for determining estimated current values included in this pronouncement includes "use of appraisals." Smith should report the art works at their current values based on appraised value.

Dollar-value LIFO method is also used to alleviate the problems of LIFO liquidation. Under this method, goods are combined into pools and all increases and decreases in a pool are measured in terms of total dollar value. The pools created under this method are, therefore, known as dollar-value LIFO pools. If inflation and other economic factors (such as supply and demand) were not an issue, dollar-value and non-dollar-value accounting methods would have the same results. However, since costs do change over time, the dollar-value LIFO presents the data in a manner that shows an increased cost of goods sold (COGS) when prices are rising, and a resulting lower net income. When prices are decreasing, dollar-value LIFO will show a decreased COGS and a higher net income. Dollar value LIFO can help reduce a company's taxes (assuming prices are rising), but can also show a lower net income on shareholder reports.

The dollar‐value LIFO inventory cost flow method involves computations based on Inventory pools of similar items: YES A specific price index for each year: YES Dollar‐value LIFO uses dollar‐value pools which are made up of "similar" items (in terms of interchangeability, type of material, or similarity in use). Dollar‐value LIFO determines increases or decreases in ending inventory in terms of dollars of the same purchasing power. Ending inventory is deflated to base‐year cost by dividing ending inventory by the current year's specific conversion price index. The resulting amount is then compared with the beginning inventory which has also been stated in base‐year dollars. Example 1: The Fast company adopted dollar-value LIFO method on December 31, 2011. The inventory on current prices at the end of 2011 and 2012 was as follows: December 31, 2011 (End of year prices) $40,000 December 31, 2012 (End of year prices) $52,800 The inventory prices were increased by 25% during the year 2012. (Notice, this is talking about the PRICE of inventory.. Inventory사는데 들은 비용이 시세가 작년보다 25% 늘었다는것. 시세가 늘었으니깐, inventory purchase한 quantity에도 영향이 미치는지 계산을 하는게 Dollar-value LIFO method) Required: Compute the amount of inventory at the end of 2012 using dollar-value LIFO method. Solution: First of all, we need to compute the value of ending inventory at base-year-prices. It is computed using the following formula: Ending inventory at base-year-prices = $52,800/1.25 <- 25%가 증가한것을 1.25로 표시해준다. = $42,240 Now we can compute the real-dollar quantity increase in inventory: = ($42,240 - $40,000) = $2,240 <- this is how much increased in price of buying inventory in year 2 The next step is to value this real dollar quantity increase in inventory at year-end-prices: = $2,240 × 1.25 = $2,800 The real dollar quantity increase in inventory valued at year-end-prices is usually known as dollar-value LIFO layer (or layer). If this layer is added to the beginning inventory of the year 2012, we would get the total inventory at the end of the year 2012. It is shown below: Beginning inventory (first layer) in terms of 100 $40,000 Increase during 2012 (second layer) in terms of 125 $2,800 ——— Dollar value LIFO inventory on December 31, 2012 42,800

Lane Co., which began operations on January 1, year 1, appropriately uses the installment method of accounting. The following information pertains to Lane's operations for year 1: Installment sales $1,000,000 Regular sales 600,000 Cost of installment sales 500,000 Cost of regular sales 300,000 General and administrative expenses 100,000 Collections on installment sales 200,000 The deferred gross profit account in Lane's December 31, year 1 balance sheet should be: a) $150,000 b) $320,000 c) $400,000 d) $500,000

answer: This answer is correct. Under the installment method, gross profit is deferred at the time of sale and is recognized by applying the gross profit rate to subsequent cash collections. At the time of sale, gross profit of $500,000 is deferred ($1,000,000 - $500,000). The gross profit rate is 50% ($500,000 ÷ $1,000,000). Since year 1 collections on installment sales were $200,000, gross profit of $100,000 (50% × $200,000) is recognized in year 1. This recognition of gross profit would decrease the deferred gross profit account to a 12/31/Y1 balance of $400,000 ($500,000 - $100,000). Note that regular sales, cost of regular sales, and general and administrative expenses do not affect the deferred gross profit account. ***문제를 자세히 잘 읽어야한다. if question asked what is the deferred gross profit account as of 1/1/1 instead of 12/31/1 balance? 1/1/1 기준으로는 deferred gross profit is 1,000,000 sales - 500,000 cogs= $500,000 하지만, since question is asking 12/31/year 1 balance, then, reflect the amount that was collected during the year

Tallent Corporation had the following account balances at December 31, year 1: Cash on hand and in banks $975,000 Cash legally restricted for additions to plant (expected to be disbursed in year 3) 600,000 Bank certificates of deposit (due February 1, year 2, purchased September 1, year 1) 250,000 In the current assets section of Tallent's December 31, year 1 balance sheet, what total amount should be reported under the caption "cash and cash equivalents?" a) $1,225,000 b)$ 975,000 c) $1,575,000 d) $1,825,000

answer: b Cash equivalents are investments that can be readily converted to cash. Common examples of cash equivalents include commercial paper, treasury bills, short term government bonds, marketable securities, and money market holdings. ... The investment should be short term. They should mature in less than three months It also includes currency, coins, checks received but not yet deposited, checking accounts, petty cash, savings accounts, money market accounts **Note! cash legally "RESTRICTED" is excluded from cash and cash equivalent. (restriction이 없어야함) Generally C.D. can be considered cash and equivalent if the maturity is less than 3 month but in this case, it is more than 3 month.

On the statement of cash flows in which the operating activities section is prepared under the indirect method, depreciation is treated as an adjustment to reported net earnings because depreciation a) Is a direct inflow of cash from investing activities. b) Reduces reported net earnings but does not involve an outflow of cash. c) Reduces reported net earnings and involves an inflow of cash. d) Is an inflow of cash to a reserve account for replacement of assets.

answer: b Depreciation is deducted in the calculation of net earnings; however, it is a noncash expense item and as such does not affect cash. Therefore, it would be added back in to net earnings on the statement of cash flows when using the indirect method.

Different types of Depreciation 총정리 1) Straight line method 2) Sum-of-the-years'-digits method 3) Declining balance method- DB Method a) Includes 200% bonus depreciation b) Includes 150% Depreciation ----------------------------------------------- 1) Straight Line Depreciation- 제일 쉽게 생각하면, 정확하게 each year에 똑같은 금액이 depreciate된다. 또 중요한게, straight line deducts salvage value. Straight Line Depreciation = (Cost - Residual value) / Useful life On April 1, 2011, Company A purchased an equipment at the cost of $140,000. This equipment is estimated to have 5 year useful life. At the end of the 5th year, the salvage value (residual value) will be $20,000. Company A recognizes depreciation to the nearest whole month. Calculate the depreciation expenses for 2011, 2012 and 2013 using straight line depreciation method. Depreciation for 2011 = ($140,000 - $20,000) x 1/5 x 9/12 = $18,000 Depreciation for 2012 = ($140,000 - $20,000) x 1/5 x 12/12 = $24,000 Depreciation for 2013 = ($140,000 - $20,000) x 1/5 x 12/12 = $24,000 **notice, for straight line depreciation, it is always the same amount that was deducted, 단지 틀린것이라면, depreciation이 정확히 1년, 즉 12개월을 채우지 못햇더라면, monthly 정확히 allocation 구분하기** 2) Sum-of-the-years'-digits method : Depreciation expense = (Cost - Salvage value) x Fraction Company A purchased the following asset on January 1, 2011. What is the amount of depreciation expense for the year ended December 31, 2011? Acquisition cost of the asset --> $100,000 Useful life of the asset --> 5 years Residual value (or salvage value) at the end of useful life --> $10,000 Depreciation method --> sum-of-the-years'-digits method Calculation of depreciation expense Sum of the years' digits = 1+2+3+4+5 = 15 Depreciation for 2011 = ($100,000 - $10,000) x 5/15 = $30,000 Depreciation for 2012 = ($100,000 - $10,000) x 4/15 = $24,000 Depreciation for 2013 = ($100,000 - $10,000) x 3/15 = $18,000 Depreciation for 2014 = ($100,000 - $10,000) x 2/15 = $12,000 Depreciation for 2015 = ($100,000 - $10,000) x 1/15 = $6,000

(3) Declining Balance Depreciation Method. (Both 200% and 150%) Depreciation = Book value x Depreciation rate Book value = Cost - Accumulated depreciation **(핵심) Notice, there are 2 things different. Declining balance는 residual value 또는 salvage value를 빼주지 않는다. 또한, straight line depreciation랑 틀리게, 매년 정확한 금액이 depreciate되는게 아니고, depreciation gets accumulated!! [Example, Double declining balance depreciation- 200%] On April 1, 2011, Company A purchased an equipment at the cost of $140,000. This equipment is estimated to have 5 year useful life. At the end of the 5th year, the salvage value (residual value) will be $20,000. Company A recognizes depreciation to the nearest whole month. Calculate the depreciation expenses for 2011, 2012 and 2013 using double declining balance depreciation method. Useful life = 5 years --> Straight line depreciation rate = 1/5 = 20% per year Depreciation rate for double declining balance method 20% x 2 = 40% Depreciation for 2011 = $140,000 x 40% x 9/12 = $42,000 Depreciation for 2012 = ($140,000 - $42,000) x 40% x 12/12 = $39,200 Depreciation for 2013 = ($140,000 - $42,000 - $39,200) x 40% x 12/12 = $23,520 [Example, Double declining balance depreciation- 150%] On April 1, 2011, Company A purchased an equipment at the cost of $140,000. This equipment is estimated to have 5 year useful life. At the end of the 5th year, the salvage value (residual value) will be $20,000. Company A recognizes depreciation to the nearest whole month. Calculate the depreciation expenses for 2011, 2012 and 2013 using double declining balance depreciation method. Useful life = 5 years --> Straight line depreciation rate = 1/5 = 20% per year Depreciation rate for double declining balance method = 20% x 150% = 20% x 1.5 = 30% per year Depreciation for 2011 = $140,000 x 30% x 9/12 = $31,500 Depreciation for 2012 = ($140,000 - $31,500) x 30% x 12/12 = $32,550 Depreciation for 2013 = ($140,000 - $31,500 - $32,550) x 30% x 12/12 = $22,785

A company reported the following information for Year 1: Net income $34,000 Owner contribution 9,000 Deferred gain on an effective cash- flow hedge 8,000 Foreign currency translation gain 2,000 Prior service cost not recognized in net periodic pension cost 5,000 What is the amount of other comprehensive income for Year 1? A. $5,000 B. $14,000 C. $15,000 D. $43,000

. The correct answer is A. Other comprehensive income includes items such as gains and losses on foreign currency transactions designated as hedges, gains and losses on derivative instruments, and gains or losses associated with pension or other postretirement benefits. Therefore, for this question the correct answer is $5,000: Deferred gain on an effective cash-flow hedge ($8,000) + Foreign currency translation gain ($2,000) − Prior service cost not recognized in net periodic pension cost ($5,000) = $5,000 여기서 Deferred gain on effective cash flow hedge는 gain이라서 positive, Foreign currency translation gain 또한 positive, 하지만, prior service cost not recognized in net periodic pension cost는 goodwill처럼 amortize해야한다. 노조와 미팅후, 직원들에게 지불해야하는 pension cost가 늘어났을때 우린 prior service cost not recognized in net period pension cost라 불른다. 따라서, 이건 liability 이고, minus로 we need to net it out all the gain and losses stated in OCI.

Comprehensive income is the sum of net income and other items that must bypass the income statement because they have not been realized, including items like an unrealized holding gain or loss from available for sale securities and foreign currency translation gains or losses. Comprenhensive income = net income + other comprehensive income. (여기서 "other comprehensive income" includes unrealized holding gain on available‐for‐sale securities) 조심하기

A company reports the following information as of December 31: Sales revenue $800,000 Cost of goods sold 600,000 Operating expenses 90,000 Unrealized holding gain on available‐for‐sale securities, net of tax 30,000 What amount should the company report as comprehensive income as of December 31? a) $30,000 b) $110,000 c) $140,000 d) $200,000 answer: c Comprehensive income is net income plus or minus unrealized gains and losses that are recognized in comprehensive income for the period. Net income is equal to $110,000 ($800,000 − $600,000 − $90,000). An unrealized holding gain on available‐for‐sale securities is classified as other comprehensive income. Therefore, comprehensive income is calculated as net income of $110,000 plus the $30,000 unrealized holding gain on available‐for‐sale securities, which equals $140,000.

A statement of financial position, which reports unrestricted, temporarily restricted, and permanently restricted net assets, is required for which one of the following organizations? I. A public university. II. A private, not‐for‐profit hospital. A. I and II. B. I only. C. Neither I nor II. D. II only. answer: A FASB ASC 958 requires a statement of financial position which reports unrestricted, temporarily restricted, and permanently restricted net assets for nongovernmental, not-forprofit organizations. Therefore, the statement of financial position is required for a private, not-for-profit hospital, but not for a public university, which is supported by government. (애초부터 A statement of financial position was required by non-governmental not for profit entity. 이뜻은, government가 후원은 해주지않고, 아무런 정부랑 상관없는 공기업)

A statement of financial position, which reports unrestricted, temporarily restricted, and permanently restricted net assets, is required for which one of the following organizations? I. A public university. II. A private, for‐profit hospital. A) I and II. B) I only. C) Neither I nor II. D) II only. Answer: c because the hospital is a for‐profit entity and public university is for-profit entity

According to the FASB Conceptual Framework: Relevance: Predictive, Confirming, Materiality, consistency (relevance 뜻: "important to the matter at hand" (F/S작성시에 매우 관련깊은 information) Faithful Representation: Completeness, Neutrality, Freedom from Error (verifiability) Enhance: Understand, Verify, Timeliness, Understandability

According to the FASB Conceptual Framework, predictive value is an ingredient of Relevance: yes Faithful representation: no

The formula to compute accounts receivable turnover is Net credit sales / Average accounts receivable (net credit sales divided by average A/R.. note that "net credit sales" purely excludes cash sale.. CASH Sales를 제외한것..)

Accounts receivable turnover랑은 관련이 없지만, current ratio 구할때 current asset에서 A/R include할때, it must be net A/R, meaning (A/R- Doubtful allowance)

All of the following is underlying: An interest rate index. A security price. An average daily temperature. Each of the other choices meets the basic definition of an underlying, which is any financial or physical variable that has either observable changes or objectively verifiable changes. Underlying- 관찰 가능한 변경 또는 객관적으로 검증 가능한 변경이있는 재무

An underlying is commonly a specified price or rate such as a stock price, interest rate, currency rate, commodity price, or a related index. However, any physical or financial variable with observable changes or objectively verifiable changes qualifies as an underlying.

A county's balances in the general fund included the following: Appropriations $435,000 Encumbrances 18,000 Expenditures 164,000 Vouchers payable 23,000 What is the remaining amount available for use by the county? a) $230,000 b) $248,000 c) $253,000 d)$271,000

Answer: C This is governmental accounting. Appropriation $435,000 - Encumbrance $18,000 - Expenditure $164,000 = $253,000 remained amount available for use by government. You book the appropriation with the budget, as mentioned, as to book the amount of monies the entity is allowed to expend for the fiscal year. An encumbrance is a control account used to book a purchase, usually in the form of a purchase order. I.e. the governmental entity has issued a purchase order to purchase the good/service but hasn't actually expended the money for the purchase yet. When the entity pays for the good/service the encumbrance is negated and the expenditure is booked.

Nickels Hospital, a nonprofit hospital affiliated with a religious group, reported the following information for the year ended December 31, year 1: • Gross patient service revenue at the hospital's full established rates $860,000 • Bad debts expense 10,000 • Contractual adjustments with third‐party payors 100,000 • Allowance for discounts to hospital employees 35,000 On the hospital's statement of operations for the year ended December 31, year 1, what amount should be reported as net patient service revenue? A) $725,000 B) $760,000 C) $715,000 D) $815,000

Answer:a remember, Health Care Organization Accounting (HCOA)- Uses full accrual method. $860,000 Revenue -10,000 bade debt - contractual adjustment to payor 100,000- allowance for discounts to hospital employee 35,000 = $725,000 **if there was endowment, assume it is restricted and don't include in the operating statement which is "net patient service revenue" . also don't include charity care. i believe the endowment goes to financing activity statement rather than operating statement and charity care must be disclosed in F/S note

Miro Co. began business on January, 2, year 2. Miro used the double‐declining balance method of depreciation for financial statement purposes for its building, and the straight‐line method for income taxes. On January 16, year 4, Miro elected to switch to the straight‐line method for both financial statement and tax purposes. The building cost $240,000 in year 2, which has an estimated useful life of 16 years and no salvage value. Data related to the building is as follows: Year Double‐declining balance depre Straight‐line depre Year 2 $30,000 $15,000 Year 3 26,250 15,000 Miro's tax rate is 40%. Which of the following statements is correct? a) There should be no reduction in Miro's deferred tax liabilities or deferred tax assets in year 4. b) Miro's deferred tax liability should be reduced by $1,875 in year 4. c) Miro's deferred tax asset should be reduced by $10,500 in year 4. d) Miro's deferred tax asset should be decreased by $750 in year 4. answer: d

First calculate depreciation expense in year 4. Historical cost of $240,000 less accumulated depreciation of $56,250 = $183,750 book value. $183,750 book value divided by 14 years remaining life = $13,125. Next, calculate the amount in the deferred tax asset account. Year 2 DDB depre Tax S/L Book/tax differ $30,000 $15,000 $15,000 Tax rate tax account Classification 40% $6,000 Deferred tax asset Year 3 DDB depre Tax S/L Book/tax differ $26,250 $15,000 $11,250 Tax rate tax account Classification 40% $4,500 Deferred tax asset Year 4 DDB depre Tax S/L Book/tax differ $13,125 $15,000 $1,875 Tax rate tax account Classification 40% $750 Reduction of DTA The total in the deferred tax asset account of 12/31/Y3 is $10,500. In year 4, when tax depreciation is greater than depreciation expense on the financial statements, the timing difference will "turn around." Therefore, in year 4, the deferred tax asset account will be reduced by $750.

Key to remember that I use: If accounting results are higher than taxable results in the current year - liability If taxable results are higher than accounting results in the current year - asset A deferred tax liability records the fact the company will, in the future, pay more income tax because of a transaction that took place during the current period, such as an installment sale receivable. If financial reporting basis (Book basis) is greater than taxable income, it is deferred tax liability. (Save tax now, pay tax later) If financial reporting basis (book basis) is lesser than taxable income, deferred tax asset is created. (Pay tax now and save tax later)

For calendar year 3, Clark Corp. had depreciation of $300,000 on its income statement. On its Year 3 tax return, Clark had depreciation of $500,000. Clark's income statement also included $50,000 accrued warranty expense that will be deducted for tax purposes when paid. Clark's enacted tax rates are 30% for Year 3 and 25% for future years. These were Clark's only temporary differences. In Clark's Year 3 income statement, the deferred portion of its provision for income taxes should be a) $60,000. b) $45,000. c) $37,500. d) $50,000. answer: c The income tax provision (expense) must be reported in two components: the amount currently payable (current) and the tax effects of temporary differences (deferred). The deferred portion should be based on future enacted tax rates. The depreciation difference results in future taxable amounts totaling $200,000 in Years 3 to 12, while the warranty difference results in future deductible amounts totaling $50,000 in Years 3 to 12. Deferred tax expense is computed as follows: Increase in deferred tax liability $200,000 future taxable amount × 25% = $50,000 (depreciation is treated as deferred tax liability because, according to question, book basis of depreciation is 300,000 but tax basis was 500,000. Which means, current tax income was able to deduct more depreciation which yields to lesser net income which also means, lesser tax due.. So, 회사는 빛쟁이 입장이다. 이번년에 tax를 더 적게 냈으니깐, 다음년에 더 많이 내야되서, deferred tax liability로 표시된다) If financial reporting basis (Book basis) is greater than taxable income, it is deferred tax liability. (Save tax now, pay tax later) Increase in deferred tax asset ($50,000) future deductible amount × 25% = (12,500) Deferred portion of income tax expense $37,500 If financial reporting basis (book basis) is lesser than taxable income, deferred tax asset is created. (Pay tax now and save tax later) Warranty expense is an example of a situation when taxes payable are greater than income tax expense because on the income statement statment estimated warranty expense is deductible and on the tax return only actual warranty expenses are deductible. ***Accrued liabilities. Liabilities are claims against a business, such as contingent liabilities, which is money the company may have to pay out in the future based on events that haven't yet come to fruition. Under financial accounting, a business has to record liabilities when they're most probably incurred and the dollar amount can be reasonably estimated.*** this is why it is causing the temporary difference. Book purpose에서는 accrued liability, estimates, allowance for bad debt같은걸 다 뺴주지만, tax basis에서는 실제로 돈이 나가야지만, expense로처리한다.

A company used the percentage‐of‐completion method to account for a 4‐year construction contract. Which of the following would be used in the calculation of the income recognized in the second year? Income previously recognized: yes Progress billings to date: no

In the calculation of pension expense recognized for a period by an employer sponsoring a defined benefit pension plan, which components will not be included? Interest cost on the projected benefit obligation. Actuarial present value of benefits attributed by the pension benefit formula to employee service during that period. Amortization of the unrecognized net obligation (and loss or cost) or unrecognized net asset (and gain) existing at the date of transition. Excess of accumulated benefit obligation over the fair value of the plan assets.

Health Care Organization Accounting (HCOA)- Uses full accrual method. (ex: nonprofit hospital reports)

Russell Hospital, a nonprofit hospital affiliated with a private university, provided $250,000 of charity care for patients during the year ended December 31, year 1. The hospital should report this charity care? answer: Only in the notes to the financial statements for year 1. According to the AICPA Audit and Accounting Guide, Health Care Organizations, charity care does not qualify for recognition as receivables or revenue in the financial statements. Management's policy for providing charity care, as well as the level of charity care provided, should be disclosed in the financial statements. Such disclosure generally is made in the notes to the financial statements and is measured based on the providers' rates, costs, units of service, or other statistical measure.

On December 31 of the previous and current year, Taft Corporation had 100,000 shares of common stock and 50,000 shares of noncumulative and nonconvertible preferred stock issued and outstanding. Additional information for the current year follows: Stockholders' equity at 12/31 $4,500,000 Net income year ended 12/31 1,200,000 Dividend on preferred stock year ended 12/31 300,000 Market price per share of common stock on 12/31 72 The price-earnings ratio on common stock at December 31 was: A. 5 to 1. B. 6 to 1. C. 8 to 1. D. 9 to 1.

The correct answer is C. The price-earnings ratio is P/E = Stock price ÷ EPS (earnings per share). The net earnings per common share is $9: ($1,200,000 - $300,000) ÷ 100,000 = $9 Price-earnings ratio: $72 ÷ $9 = $8

On December 1, year 1, Branch Corporation leased office space for 10 years at a monthly rental of $15,000. On that date Branch paid the landlord the following amounts: Rent deposit $ 15,000 First month's rent 15,000 Last month's rent 15,000 Installation of new walls and offices 96,000 $ 141,000 The entire amount of $141,000 was charged to rent expense in year 1. What amount should Branch have charged to expense for the year ended December 31, year 1? a) $15,000 b) $15,800 c) $30,800 d) $96,000

The leasehold improvement amortization is $9,600 a year ($96,000 ÷ 10 yrs.) or $800 per month ($9,600 ÷ 12 mos.). Accordingly, December's expenses should have been $15,800, and $95,200 of leasehold improvements should be deferred and amortized over the remainder of the lease. The rent deposit of $15,000 and the last month's rent of $15,000 should be set up as prepaid assets as shown in the following journal entry. Leasehold improvements 96,000 Rent expense 15,000 Rent deposit 15,000 Prepaid rent 15,000 Cash 141,000 Leasehold imp. amort. 800 Leasehold improv. 800

Definition of Functional Currency Under international financial reporting standards, a functional currency is the currency used in the primary economic environment where an entity operates. This is the environment in which an entity primarily generates and expends cash. You should consider the following primary factors in determining an entity's functional currency: The same Standard defines presentation currency as "the currency in which the financial statements are presented". The functional currency is determined by looking at a number of relevant factors. This currency should be the currency in which an entity usually generates and spends cash. Functional currency should be the one in which the business transactions of an entity are normally denominated. All of the transactions which are not in the functional currency are treated as foreign transactions. Following five factors need to be considered when determining a functional currency. Functional currency is the currency: 1) That mainly affects the prices at which the goods or services are sold 2) Of the country whose regulations, market conditions and competitive forces mainly affect the pricing policy of the entity 3) That influences the costs and expenses of the entity 4) In which the funds are usually generated 5) In which receipts from operating activities are retained Market - The functional currency is the foreign currency when the foreign activity has a strong local sales market for products or services even though a significant amount of exports may exist. The functional currency is the parent's currency when the foreign operation's sales market is mostly in the parent's country. An entity can present its financial statements in any currency. Usually they are presented in the functional currency; therefore usually the functional currency and the presentation currency is the same. If the presentation currency differs from the functionally currency, the financial statements should be translated into the presentation currency. Situation 1) 예를 들어서, if U.S. parent have foreign subsidiary in Japan who use its functional currency. <-이뜻은, 일본에 있는 subsidiary가 본인나라의 functional currency를 쓰고있다는것. 즉, 일본에서 일어나는 세일이 미국보다 더 영향력있고, 더 많고, expense또한 일본에서 더 많이 쓰이고, 마켓이 일본이 더 클때, 일본이 yen becomes functional currency. 보통은, functional currency로 Financial statement를 만들지만, parent가 미국회사다 보니 나중에 어쩔수없이, yen에서 dollar로 바꿔야한다. 이런경우엔, we use Current Rate Method If the foreign affiliate's financial statements are in the local currency and this is also the functional currency then the Current-Rate Method must be used. Situation Situation 2) 다른 예를 들어서, 위에 Example이랑 완전 똑같다 단, 일본 subsidiary is using local currency, rather than having Yen as it's functional currency. U.S. parent have foreign subsidiary in Japan who use its local currency. 이뜻은, 그닥, 일본시작이 영향력이 있는곳이 아니고, sales도 일본이 주요 돈을 벌어들이는곳이 아니면, 우린 함부로 yen을 functional currency라고 하지 않느다, 그냥 현지돈 local currency라고 한다. 이런경우 we use Temporal-Rate Method. If the foreign affiliates' financial statements are in the local currency but the functional currency is dollars then the Temporal-Rate Method must be used. Temporal Rate Method는 조심해야하는게 이안에서도 여러가지 조건이 있다. Monetary asset is translated using current rate, income statement is translated using weighted average, and nonmetary asset is translated using historical cost. Situation 3) If the financial statements of the foreign affiliate are using same currency as their parent, then no translation is needed. 그외에 알아둘것: 1) If the functional currency is the same as the presentation currency, gains or losses are reported in profit and loss for the period. (if parent company use USD $ and its subsidiary company in different country also use USD $ to prepare its F/S, and if gain and loss founded, it is reported straightly to Income statement, rather than Other comprehensive Income. 2) If the functional currency is not the same as the presentation currency, gains or losses are reported in OCI. (other comprehensive income)

The two methods for carrying out the translation of foreign accounts are: 1) The Current-Rate Method (also known as the Closing-Rate Method) o All assets and liabilities are converted at the current rate of exchange on the date of the balance sheet. o Items in the income statement, which represent flows over the accounting period, are converted at the exchange rate prevailing at the times of accrual. Since this is often not feasible a weighted average exchange rate over the accounting period is used instead. o Dividends and other distributions are converted at the exchange rate prevailing at the time they were paid. 2) The Temporal Method o Monetary assets and liabilities (cash, liquid securities, accounts payable and receivable, debt) are converted at the current rate of exchange. o Nonmonetary assets and liabilities (fixed assets and inventory) are translated at historical rates. Thus no accounting capital gains or losses arise from these items. o Income state items are converted at the average exchange rate for the accounting period unless, as in the case of depreciation or cost of inventory sold, they are directly associated with nonmonetary items. In this latter case the historical cost is used for the translation. o Dividends and other distributions are converted at the current rate of exchange at the time they were paid. The rules are also affected by the nature of the functional currency of the affiliate. The functional currency of an entity is the dominant currency used in its daily operations. The U.S. accounting practice rules are based upon functional currency rather than the distinction between integrated and self-sustaining foreign affiliates. The U.S. rules are: • If the financial statements of the foreign affiliate are in dollars then no translation is needed. • If the foreign affiliate's financial statements are in the local currency and this is also the functional currency then the Current-Rate Method must be used. • If the foreign affiliates' financial statements are in the local currency but the functional currency is dollars then the Temporal-Rate Method must be used. • If the foreign affiliate's financial statements are in the local currency but the functional currency is not dollars or the local currency but some third currency instead then the financial statements must be translated into the functional currency using the Temporal-Rate Method and then those values translated into dollars using the Current-Rate Method.

When progress billings are sent on a long‐term contract, what type of account should be credited under the completed‐contract method and percentage‐of‐completion method? Completed‐contract: contra asset Percentage‐of‐completion: contra asset .

Under the percentage‐of‐completion method, income is recognized periodically on the basis of the percentage of the job that is complete. The completed‐contract method recognizes income from the job only when the contract is completed. This is the only difference in accounting for the two methods. The difference between methods is simply a question of timing—the percentage method recognizes profit little by little over time, while the completed-contract method defers the entire profit until completion. For both methods, when progress billings are sent, "Billings on construction in progress" is credited for the amount billed. This is shown on the balance sheet as a contra account to Construction in progress. Debit "A/R" Credit" Billings on construction in progress " ***** the amount of income recognized in any year would be added to**** (A/R로 위에 잡아놓았던게 수금이 되면) Construction in progress xxx Income on long‐term contract xxx percentage‐of‐completion method- 이 방법은 프로젝트 완료 단계를 지속적으로 평가하거나 적어도 프로젝트를 완료하기 위해 남은 비용을 추정하는 것이 합리적 일 때 가장 효과적입니다. Conversely, this method should not be used when there are significant uncertainties about the percentage of completion or the remaining costs to be incurred. The estimating abilities of a contractor should be considered sufficient to use the percentage of completion method if it can estimate the minimum total revenue and maximum total cost with sufficient confidence to justify a contract bid. (즉, ESTIMATED COST가 예상이 어느정도 되어야지만, percentage‐of‐completion method 쓸수있다) In essence, the percentage of completion method allows you to recognize as income that percentage of total income that matches the percentage of completion of a project

With respect to the objective of general purpose financial reporting, what are U.S. GAAP and IFRS differences? A. The objective for GAAP is more precise and longer than the objective for IFRS. B. The objective for IFRS is more precise and longer than the objective for U.S. GAAP. C. With the issuance of SFAC 8, chapter 1, U.S. GAAP and IFRS have the same "Objective of General Purpose Financial Reporting." D. FASB and IASB are working toward convergence of the objective.

answer is C. The FASB and IASB have been working toward convergence of this objective. As a result, SFAC 8 replaced SFAC 1. Now U.S. GAAP and IFRS (International Financial Reporting Standards) have the same objective for financial reporting purposes.

Gaffney uses IFRS to prepare its financial statements. During year 4, Gaffney voluntarily changes its accounting method because the new method will provide more reliable and relevant information. Gaffney can estimate the effects of the change. How should Gaffney treat the change in accounting principle? a) On a prospective basis. b) On a retrospective basis. c)By restating the financial statements. d) By a cumulative adjustment on the income statement.

answer:b retrospective basis = change in accounting method but if company can't estimate the effects of the change, it is rather prospective method. <-Going forward change in accounting principle can only be reported on a prospective basis if it is impracticable to determine the effects of the change.

Comprehensive income

is the sum of net income and other items that must bypass the income statement because they have not been realized, including items like an unrealized holding gain or loss from available for sale securities and foreign currency translation gains or losses. Comprehensive income for a corporation is the combination of the following amounts which occurred during a specified period of time such as a year, quarter, month, etc.: Net income or net loss (which is reported on the income statement), plus Other comprehensive income (if this is present, a statement of comprehensive income must be prepared) Examples of other comprehensive income include: Unrealized gains/losses on available-for-sale investments (remember, unrealized gain/losses on trading securities affects income directly, and not go to other comprehensive income) Unrealized gains/losses on hedge/derivative financial instruments Foreign currency translation adjustments Unrealized gains/losses on postretirement benefit plans The amount of other comprehensive income for the period will be added to the accumulated other comprehensive income, which is a separate line within stockholders' equity on the end-of-the-period balance sheet. (The net income or net loss reported on the income statement will be added to retained earnings as usual.)

Conn Company purchased a new machine for $480,000 on January 1, 2007, and leased it to East the same day. The machine has an estimated 12-year life, and will be depreciated $40,000 per year. The lease is for a 3-year period expiring January 1, 2010, at an annual rental of $85,000. Additionally, East paid $30,000 to Conn as a lease bonus to obtain the 3-year lease. For 2007 Conn incurred insurance expense of $8,000 for the leased machine. What is Conn's 2007 operating profit on this leased asset? Answers A : $67,000 B : $55,000 C : $47,000 D : $37,000

Answer C is correct. This lease is an operating lease because it does not meet any of the four criteria to be a capital lease as described in SFAS 13. The lessor (Conn) should recognize as revenue the 2007 rental payment ($85,000) plus a proportionate fraction of the lease bonus ($30,000/ 3-year lease term = $10,000 per year). Therefore, total revenue for 2007 is $95,000 ($85,000 + $10,000). 2007 expenses total $48,000 (depreciation of $40,000 and insurance of $8,000). Thus, operating profit on the leased asset is $47,000 ($95,000 revenues less $48,000 expenses). This answer is incorrect. Refer to the correct answer explanatio

hich of the following statements is correct regarding reporting comprehensive income? A. Accumulated other comprehensive income is reported in the stockholders' equity section of the balance sheet. B. A separate statement of comprehensive income is required. C. Comprehensive income must include all changes in stockholders' equity for the period. D. Comprehensive income is reported in the year-end statements but not in the interim statements.

The correct answer is A. FASB ASC 220-10-45 requires that accumulated other comprehensive income be reported in the stockholders' equity section of the balance sheet:

To determine the accounting treatment for a transaction, a governmental entity must first refer to: A. GASB Technical Bulletins. B. the Codification of Governmental Accounting and Financial Reporting Standards. C. AICPA Practice Bulletins. D. Implementation Guides published by the GASB staff.

the answer is B. GASB has now codified all of its standards in the Codification of Governmental Accounting and Financial Reporting Standards. Governmental entity경우, 제일 먼저 refer할 source는 "ENTITY MUST FIRST REFER TO" is "Codification of Governmental Accounting and Financial Reporting Standards" For-profit또한 똑같다. 제일 먼저 refer할 source는 "Codification of FASB" (앞에 CODIFICATION이 들어가야한다)

On January 2, 2006, Morey Corp. granted Dean, its president, 20,000 stock appreciation rights. On exercise, Dean is entitled to receive cash for the excess of the stock's market price on the exercise date over the market price on the grant date. The rights are exercisable beginning on January 2, 2008 and expiring on December 31, 2008. The market price of Morey's stock was $30 on January 2, 2006 and $45 on December 31, 2006. Morey used the Black-Sholes-Merton pricing model and estimated the values of each right at $16 each. As a result of the stock appreciation rights, the company should recognize compensation expense for 2006 of: A) $300,000 B) $320,000 C) $150,000 D) $160,000

Correct Answer: D Explanation: The service period is the period from the grant date, January 2, 2006, to the exercise date, January 2, 2008, or two years. Therefore, the total compensation of $320,000 (20,000 SARS X the estimated fair value of $16 each) divided by two years equals the compensation expense of $160,000 each year.

Hanson Corporation's income statement for the year ended December 31, year 3, shows pretax book income of $400,000. The following items for year 3 are treated differently on the tax return and on the books: Per tax return Per books Royalty income $ 20,000 $ 40,000 Depreciation expense 125,000 100,000 Officers life insurance premium None 15,000 Assume that Hanson's enacted tax rate for year 3 is 40% and 30% for all future years. Of Hanson's total income tax expense, how much should be reported as deferred income taxes in Hanson's year 3 income statement? a) $ 6,000 b) $ 7,500 c) $ 9,000 d) $13,500 answer: d The deferred tax component of income tax expense reported in the income statement is computed by determining the change in the balance sheet account(s) for deferred income taxes. Since no beginning balance due to temporary differences originating in prior years is given, the deferred portion of income tax expense will be the amount of the ending balance sheet account(s). Depreciation expense and royalty income are temporary differences and cause pretax book income to exceed taxable income, while the officers life insurance premium is a permanent difference and has no effect on deferred taxes. Because book income exceeds taxable income, taxes on this difference will be due in future years when reversal occurs. The two future taxable amounts that represent the temporary differences are tax affected using future rate of 30%. This gives the desired ending balance in the deferred income tax liability account and the deferred portion of income tax expense for year 3. Temporary difference × Tax rate = Deferred tax Royalties: ($ 40,000 − $ 20,000) × 30% = $6,000 Depreciation: ($125,000 − $100,000) × 30% = $7,500 Thus, deferred tax expense for year 3 would be $13,500.

..a timing difference arises because accounting "rules" differ from the "rules" applied by the tax authorities. For example, royalty income under accounting principles should be shown as pre-tax income in the year in which it is earned but the tax authorities will (typically) follow the principle of taxing royalty income in the year in which it is received. It's the difference between accruals based accounting and cash based accounting

There are 3 types of application 1) retrospective 2) propsective 3) restate

1) A retrospective application is the application of a new accounting principle as if that principle had always been applied. The concept is used when the financial statements for multiple periods are being presented. (Ex: LIFO, FIFO, Change in entity, change the completion contract term, **Retrospective는 re-visiting all the previous years that are affected by new principles and changing all the F/S 2) Prospective application is the application of a new accounting policy to transactions after the date of the policy change, with recognition of the effect of changes in accounting estimates in the current and future periods. The change is not applied to prior periods. (prospective ex: change in salvage value, depreciation, decreased in estimated useful life of asset, Estimates are based on the judgment and specialized knowledge derived from past experience. **Changes in accounting estimates are accounted for in the period of change and future periods if the change affects both. ***(change가 해당하는 이번년서부터 앞으로 쭉 going forward) 3) Restate- Error correction. (Dont get confused with Restate and Retrospective) Ex: Effect of a failure to provide for uncollectible accounts in the previous period. If this is the case, there must be adjustment to the opening balance of retained earnings A restatement is the process of revising previously issued financial statements to correct an error. A retrospective application is the application of a different accounting principle to previously issued financial statements, as if that principle had always been used. Retrospective application is required for changes in accounting principle and changes in reporting entity.

A business interest that constitutes a large part of an individual's total assets should be presented in a personal statement of financial condition as: A. a separate listing of the individual assets and liabilities at cost. B. separate line items of both total assets and total liabilities at cost. C. a single amount equal to the proprietorship equity. D. a single amount equal to the estimated current value of the business interest.

ANSWER: D

When a firm elects not to bifurcate a hybrid financial instrument, how should changes in fair value be recognized? (Definition of hybrid financial instrument: A single financial security that combines two or more different financial instruments. Hybrid securities, often referred to as "hybrids," generally combine both debt and equity characteristics) 이런 두가지의 틀린 security 1나의 financial instrument로 묶여있지만, 회사는, bifurcate, <-두개로 나눠서 구분하지 않을거가한다..) a) As other comprehensive income. b) On a prospective basis in the current year earnings and future year's earnings. c)As a prior period adjustment with restatement of previous years' financial statements. d) As a cumulative effect adjustment to the beginning balance of retained earnings for the period. answer: b The difference between the total carrying amount of the components of the bifurcated hybrid financial instruments and the fair value of the combined hybrid instruments should be recognized in earnings for the period. remember, financial instrument are valued at FMV.

According to ASC Topic 815, hybrid instruments must be accounted for: At fair value if an election is made not to bifurcate the hybrid instrument.

In a statement of cash flows, receipts from sales of property, plant, and equipment and other productive assets should generally be classified as cash inflows from A) Operating activities. B) Financing activities. C) Investing activities. D) Selling activities.

Answer: c Receipts from sales of "productive assets" are not classified as cash inflows from operating activities. Receipts from sales of property, plant, and equipment and other productive assets are categorized as cash flows from investing activities. 여기서, Productive asset란, assets whose value appreciates over some period of time, or which produce a cash return. Such assets might be a portfolio of shares, a rental property, or plant and machinery (pp&e) for a business. 회사소유의 물건인데 팔면, 돈이 되는것들.)

Deferred tax liability vs. Deferred tax asset 요약: Deferred tax is difference in tax liability calculated for temporary difference between the profit as per income tax and profit as per accounting. The temporary difference can either be a tax liability to be met in future (save tax now, pay tax later), or a tax asset (pay tax now and save tax later). If financial reporting basis (Book basis) is greater than taxable income, it is deferred tax liability. (Save tax now, pay tax later) If financial reporting basis (book basis) is lesser than taxable income, deferred tax asset is created. (Pay tax now and save tax later)

As a result of differences between depreciation for financial reporting purposes and tax purposes, the financial reporting basis of Noor Co.'s sole depreciable asset, acquired in year 3, exceeded its tax basis by $250,000 at December 31, year 3. This difference will reverse in future years. The enacted tax rate is 30% for year 3, and 40% for future years. Noor has no other temporary differences. In its December 31, year 3 balance sheet, how should Noor report the deferred tax effect of this difference? a)As an asset of $75,000. b) As an asset of $100,000. c) As a liability of $75,000. d) As a liability of $100,000. answer: d This answer is correct. At 12/31/Y3, the financial reporting basis of the depreciable asset exceeds its tax basis by $250,000. This means that in future years tax depreciation will be less than book depreciation, resulting in future taxable amounts. The existence of future taxable amounts requires recognition of a deferred tax liability at 12/31/Y3 based on future enacted tax rates. Therefore, Noor should report a 12/31/Y3 deferred tax liability of $100,000 ($250,000 × 40%). Note that the deferred tax liability should reflect the future tax consequences of events which have been recognized in the financial statements, so its computation is based on current tax rates, unless future tax rates have been enacted and are different from the current rate. 다른 설명: When the pretax GAAP basis is greater than the taxable income, the temporary differences will result in a deferred liability. In this example, 'exceed its tax basis by $250,000', means that pretax GAAP income is $250,000 greater than the taxable income -> liability. $250,000 x .40 = $100,000 deferred liability 내가 생각하는 설명: This questions tells us financial reporting basis (Book basis) has higher depreciable asset than taxable income. Let's say book basis depreciable asset is $500,000, then tax basis asset is $250,000, because the questions tells us book basis's depreciable asset exceed tax basis by $250,000. ($500,000 -$250,000 = $250,000) Having higher depreciable asset yields higher income because it is income producing assets.. so, if tax return reports lesser depreciable asset this year, it will create lesser tax. this is deferred tax liability because the company is saving tax now but have to pay tax later year as our book basis shows higher depreciable asset .

The first way that GAAP and IFRS differ when it comes to inventory is the allowable inventory costing methods. There are three costing methods that are widely used in accounting. Those three methods are FIFO, LIFO, and average cost. lIIFO & FIFO는 내가 알고, Average cost just takes the average cost of the inventory sold. Under GAAP, a company can chose to use any three of these methods. However, IFRS has banned the use of LIFO because they believe that it allows for income manipulation. (So, for IFRS, no LIFO allowed) 이게 진짜 핵심 포인트: Another way that GAAP and IFRS differ is how inventory is to be presented and valued on the balance sheet. There are two ways in which inventory can be valued: lower of cost or market (LCM) or lower of cost or net realizable value (NRV). 1) LCM means that inventory should be recorded at the lower of either the cost to produce it, the cost to repurchase it, or the market value of the inventory. <-LCM only is required by GAAP. 그냥 단순하게, current replacement cost를 market value로 보는게 아니고, current replacement cost랑 net realized value (which is estimated selling price less estimated costs of completion and estimated costs to sell.) 를 비교하기.. Market = Current replacement cost If Current Replacement Cost > Net Realizable Value (NRV), then NRV is considered as Market. If Current Replacement Cost < (NRV - Normal Profit Margin), then (NRV - Normal Profit Margin) is Market. *여기서 뽑힌, market value will be compare to historical cost and whichever has the lower value will be the answer. 2) NRV is the estimated selling price minus the estimated disposal cost. <-IFRS use NRV(Lower of NRV or historical cost)

At the end of the year, Ian Co. determined its inventory to be $258,000 on a LIFO (last in, first out) basis. The current replacement cost of this inventory was $230,000. Ian estimates that it could sell the inventory for $275,000 at a disposal cost of $14,000. If Ian's normal profit margin for its inventory was $10,000, what would be its net carrying value? answer: $251,000 Under, GAPP, ASC Topic 330 requires the use of lower of cost or market (LCM) for reporting inventory. The market value of inventory is defined as the replacement cost (RC), as long as it is less than the ceiling, net realizable value (NRV), and more than the floor NRV less a normal profit (NRV − NP). In this case, the amounts are computed as follows: Ceiling: NRV = ($275,000 est. selling price − $14,000 cost to sell) = $261,000 Floor: NRV − NP = $261,000 − $10,000 = $251,000 The replacement cost is $230,000, which is lower than the floor. Therefore, the net carrying value of the inventory should be reported at the floor value of $251,000, which is lower than the cost of $258,000.

An underlying asset is a term used in derivatives trading, such as with options. A derivative is a financial instrument with a price that is based on (that is, derived from) a different asset. The underlying asset is the financial instrument (such as stock, futures, a commodity, a currency or an index) on which a derivative's price is based.

Below is distinguishing characteristic of a derivative instrument: 1. One or more underlyings and one or more notional amounts 2. No initial net investment or smaller net investment than required for contracts with an expected similar response to market changes, and 3. Terms that require or permit net settlement, net settlement by means outside the contract, and delivery of an asset that is substantially the same as net settlement.

Beal, Inc. intends to lease a machine from Paul Corp. Beal's incremental borrowing rate is 14%. The prime rate of interest is 8%. Paul's implicit rate in the lease is 10%, which is known to Beal. Beal computes the present value of the minimum lease payments using a) 8% b) 10% c) 12% d) 4% answer: b ASC Topic 840 states that the lessee should compute the PV of the minimum lease payments using the lesser of the lessee's incremental borrowing rate (14% in this case) or the implicit rate used by the lessor if known (10% in this case). The PV of the minimum lease payments should be computed using the implicit rate of 10% because it is known by the lessee and is lower than the incremental rate.

Discount/Interest Rate to be used by lessee : LESSER of i.) Rate implicit in the lease (if known) ii.) Rate available in the market to lessee, which is incremental borrowing rate (not prime) The same rule would apply to lessor also. An implicit interest rate is one that is not explicit; in other words, the rate is not stated. For example, if I lend you $5,000 and you agree to repay me $1,000 at the end of each year for six years you are obviously paying interest. ... Hence, this loan has an implicit interest rate of 5.5%. **since implict interest rate is not in "word", i guess there are many times when rate implicit in the lease is not known... however, if rate implict in the lease is known make sure to compare with incremental borrowing rate and select the lower rate to calculate the present value of minimum lease payment.

Earnings per share data must be reported on the face of the income statement for Income from continuing operations: yes Discontinued operations: no *Earnings per share amount presented for income from continuing operating MUST, MANDATORY to be reported on face of income statement but, *Earnings per share amount presented for income from discontinuing operating MAY be reported Either on (1) face of income statement OR (2) Footnote

Earnings per share data shall be shown on the face of the income statement. Earnings per share amounts must be presented for (1) income from continuing operations, and (2) net income. Earnings per share date on discontinued operations must be shown either on the face of the income statement or in the footnotes.

Fair value accounting, also known as mark-to-market accounting, can change values on the balance sheet as market conditions change. In contrast, historical cost accounting, based on past transactions, is simpler, more stable, and easier to perform, but does not represent current market value. It summarizes past transactions instead. Reporting Stock Investments of Less Than 20% of Shares Ownership of less than 20% of a company's stock dictates that the investor is not able to exercise significant influence in the company or participate in shareholder meetings where business decisions affecting the company are made. Ownership of this quantity of stock is recorded using the cost method. The following is an example of how to report investments of less than 20% of shares -- assume ABC Corporation purchases 10% of XYZ's Corporation's common stock, or 50,000 shares. The market price of the stock is $1. When purchasing less than 20% of a company's stock, the cost method is used to account for the investment. Journal entry upon purchasing investment: Investment (Debit) $50,000 cash (credit) $50,000 ABC records a journal entry for the purchase by debiting Investment in XYZ Corp. for $50,000 and crediting Cash for $50,000. The investment in XYZ Corporation is reported at cost in the asset section of the balance sheet. If the investee declares dividends, the investor records a journal entry for their share of the investment. Assume XYZ Corporation declares a dividend of $ 1 per share. Dividend receivable (debit) $50,000 Dividend Income (credit) $50,000 ABC records a journal entry debiting Dividends Receivable for $50,000 and crediting Dividend Income for $50,000. The Dividend Receivable is reported on the balance sheet under current assets and Dividend Income is reported on the income statement under a section for other income 핵심 !!! "the value of an investment accounted for under the cost method should be adjusted to current fair value at the end of each accounting period, in cases where the fair value is readily determinable. " *****Changes in fair value are debited (for gains in fair value) or credited (for losses) to a fair value adjustment account reported on the balance sheet to adjust the investment account balance to its end of period fair value. <- this is like unrealized loss/gain but it actually affects Income. 보통 unrealized loss/gain은 comprehensive income으로 it goes into Other comprenhensive income in the B/S not affecting income... Investment type can be either (1) trading security or (2) available for sale" If the investment is considered a "trading security" or stock purchased for the purpose of selling it in the near term, (you plan to trade within a year), the balancing debit or credit is charged to an unrealized loss or gain reported on the income statement. (AFFECTING INCOME STATEMENT. REPORTING ON INCOME STATEMENT) If the investment is an "available for sale" security, the balancing debit or credit goes to an unrealized loss or gain account reported in the other comprehensive income section of owner's equity on the balance sheet. (REPORTING ON OTHER COMPREHENSIVE INCOME SECTION OF B/S) When the investment is sold, all losses or gains from the transaction become realized and flow through into the income statement to adjust revenues for the period. ***Trading securities are always current assets. Available-for-sale securities may be either current assets or noncurrent assets, depending on how long management intends to hold them.

Equity method: If you own between 20 percent and 50 percent of the investee's voting shares, you automatically qualify for equity method accounting. Under this method, you book your portion of the investee's income or losses on your income statement and update the asset's book value accordingly. Treat any dividends as a return of capital -- do not book them as income but rather subtract them from the carrying value of the investment. However, under the fair value option to the equity method, you recognize as income changes to the stocks' fair value rather than your share of investee income. You treat dividends as income under the fair value option. The original investment is recorded on the balance sheet at cost (fair value). Subsequent earnings by the investee are added to the investing firm's balance sheet ownership stake (proportionate to ownership), with any dividends paid out by the investee reducing that amount. The dividends received from the investee by the investor however are recorded on the income statement.

A "statement of functional expenses" is REQUIRED for which one of the following private nonprofit organizations? a) Colleges. b) Hospitals. c) Voluntary health and welfare organizations. d) Performing arts organizations. answer: c

FASB117 requires all Not-for-Profits to prepare the following: 1) Statement of Financial Position (Balance Sheet) 2) Statement of Activities (Income Statement / Profit & Loss Statement) 3) Statement of Cash Flows (for accrual basis entities) 4) Statement of Functional Expenses (for voluntary heath and welfare organizations only) The statement of functional expenses divides a non profit's expenses into three categories: Program Expenses: goods and services distributed to fulfill the purpose of the organization. Administrative expenses: costs of business management, record keeping, budgeting, and finance and other management and administrative activities. Fund raising expenses: costs of fund-raising campaigns and event

1. What is a reportable segment? There are three rules to follow when determining what constitutes a segment that must be separated from the rest of the business for reporting purposes. First, a segment must generate revenues and incur expenses. Second, the segment's results must be reviewed by the chief operations officer. Third, the segment must have readily identifiable financial information. You should be aware that only publicly traded corporations need to report segment results. 2. How many segments need to be reported separately? The FASB has established several tests for companies to use when determining which reportable segment results actually need to be separately disclosed. The first set is the three "10% threshold" tests. First, a segment should be separated if it comprises at least 10% of the company's total assets. Second, a segment should be separated if it brings in at least 10% of the company's total revenue - this includes both external revenue and revenue from intersegment transactions. Finally, a segment should be separately reported if the numerical value of its profit or loss is at least 10% of the greater of the combined profit for all profitable segments or the combined loss of all unprofitable segments (3조건중에 하나만 맞아 떨어져도 okay) 3) The last rule says that at least 75% of external revenue needs to be covered by separately reported segments. Any remaining segments can be combined into an "Other" category in the notes. Information and procedures about what to report for each segment is beyond the scope of this article.

Finally, a segment should be separately reported if the numerical value of its profit or loss is at least 10% of the greater of the combined profit for all profitable segments or the combined loss of all unprofitable segments. Let's look at a quick example of this last test below: Segment Segment Profit (Loss) A $58,000 B ($25,000) C ($71,000) D $90,000 E $11,000 F $46,000 G ($9,000) According to the table of profits and losses, profitable segments brought in $205,000 while unprofitable segments lost $105,000. If you add up all the segment profit that has positive profit which is (Adding A, D, E, F = $205,000) If you add up all the segment profit that has negative Loss which is (Adding B, C, G = $105,000) That means a segment must be separately reported if profit or loss was greater than $20,500 ($205,000 x 10%). Since $205,000 is greater than -$105,000. *segment should be separately reported if the numerical value of its profit or loss is at least 10% of the greater of the combined profit for all profitable segments or the combined loss of all unprofitable segments* ($105,000 loss가 단지 negative라서 $205,000 profit한테 진게 아니고, negative, positive를 다 떠나서, 그냥 숫자 자체가 $105,000이 $205,000 보다 작다고 무식하게 생각하는게 룰이다) For this example, results for all but Segment E and G need to be disclosed separately because both E and G are less than $20,500. 그리고 75% rule이라는건. if there is external revenue reported in segment, reportable segments must be at least 75% of external revenue라는것 같다. EX: The following information pertains to Been Corp. and its operating segments for the year ended December 31, year 1: Total revenues $80,000,000 Sales to external customers (included in total) $30,000,000 External revenue reported by reportable operating segments must be at least $22,500,000 <-- $30,000,000 X 75% Per ASC Topic 280, there must be enough segments reported so that at least 75% of unaffiliated revenues is shown by reportable segments (75% test). Sales to external customers total $30,000,000, so external revenues reported by reportable operating segments must be at least $22,500,000 ($30,000,000 × 75%) *75% Rule only applies when there is external service revenue*

What is commercial substance? A business transaction is said to have commercial substance when it is expected that the future cash flows of a business will change as a result of the transaction. A change in cash flows is considered to be when there is a significant change in any one of the following (not including tax considerations): Risk. Such as experiencing an increase in the risk that inbound cash flows will not occur as the result of a transaction; for example, a business accepts junior secured status on a debt in exchange for a larger repayment amount. Timing. Such as a change in the timing of cash inflows received as the result of a transaction; for example, a business agrees to a delayed payment in exchange for a larger amount. Amount. Such as a change in the amount paid as the result of a transaction; for example, a business receives cash sooner in exchange for receiving a smaller amount. The concept of commercial substance is also applied to exchanges of assets between businesses. When there is commercial substance (which is when there is a change in cash flow resulting from the transaction), the parties should recognize a gain or loss on the exchange. If there is no commercial substance, record the acquired asset at the book value of the asset given up in the exchange. There are additional issues related to the recognition of a gain or loss when a transaction has no commercial substance

If an exchange has no commercial substance, the accounting treatment will depend on the circumstances. The simplest circumstance is when two assets are traded with no cash payment from either side. In this case, the new asset is recorded at the carrying amount of the old asset (original cost minus accumulated depreciation). If cash is given along with the old asset, the new asset is similarly recorded at the carrying amount of the old asset plus the amount of money paid. If cash is received in an exchange that has no commercial substance, part of the realized gain is recognized if the cash is less than 25% of the total consideration received. (Watch out for this 25% rule) For example, if a machine with a fair value of $20,000 and a carrying value of $15,000 is exchanged for a similar machine and $3,000 in cash, part of the $5,000 ($20,000 - $15,000) realized gain will be recorded. In this case, since the cash is 15% ($3,000 ÷ $20,000) of the fair value of the old asset, 15% ($750 = $5,000 x 15%) of the realized gain will actually be recognized. If cash is more than 25% of the total assets received, the exchange is treated as a cash sale and the entire difference between the fair value and carrying amount of the old asset is booked as an income statement gain. If the carrying amount of the old asset is greater than the fair value of the assets received, the entire loss is booked and the new asset is recorded at the lower fair value. ex: Amble, Inc. exchanged a truck with a carrying amount of $12,000 and a fair value of $20,000 for a truck and $4,000 cash. The fair value of the truck received was $16,000. At what amount should Amble record the truck received in the exchange? Answer: $9,600 DR: Cash 4,000 (Cash received) DR: New truck: 9,600 (plug) CR: Gain: 1,600 (gain recognized: 8,000 realized * (4,000/20,000)) CR: Old truck: 12,000 (Carrying amount) So basically the new truck should be recorded as the fair value less the unrecognized gain ( 16,000 - 6,400 = 9,600). *If boot is received ,a portion of the gain is recognized * and watch out for the 25% rule for boot. in this case, 4000/20000 = 20% < 25%, therefore gain is recognized in proportion to cash recd

제일 쉽게 생각할려면, 1) Change in current asset- operating activities 2) Change in current liability- operating activity 3) Change in long-term asset- investing activities 4) Change in long-term liability- financing activity 5) Change in stockholder's equity- financing activity. Operating Activities- 1) cash receipts from customers 2) cash paid to supplies and employees 3) cash generated from operations 4) Dividends received *** 5) Interest paid 6) Interest received 7) tax paid 8) Collection of a tax refund from the government. 10) depreciation of equipment and amortization of the patent 11) trading security *notice, Available for sale & held to maturity investment are classified as investment activity but trading security is in operating activity. (Accounts Receivable Inventory Supplies Prepaid Insurance Other Current Assets Notes Payble Accounts Payable Wages Payable Payroll Taxes Payable Interest Payable Income Taxes Payable Unearned Revenues Other Current Liabilities) *net cash flow flow from operating activity 란, 위에 나온 item 모두 더하고 뺀금액. ("netting out cash received and cash paid as expense")

Investment Activities- 1) purchasment of equipment -> 돈이 나가는것이니깐 outflow 2) replacement of equipment 3) proceeds from sale of equipment-> 팔아서 돈이 들어오는것 inflow 4) available‐for‐sale securities purchased->돈주고 산것이니깐, outflow. 5) available‐for‐sale securities sold->돈주고 판건-> inflow 6) Collection of a note receivable from a related party.-> inflow 돈이 수금된것이니깐. 7) held‐to‐maturity investments sold -> 돈주고 판건 cash inflow 8) cash proceeds from the sale of investment 9) Common stock purchased from other company Financing Activities-This section of the cash flow statement reports changes in balances of the long-term liability and stockholders' equity accounts, such as: 1) proceeds from capital contributed 2) proceeds from loan 3) payment from loan 4) Dividend Paid** 5) Issuing common stock 6) mortgage repayment 7) Issuing company's bonds 8) Collection of proceeds from a note payable. **(Notes Payable (generally due after one year. *if notes payable is current liability, meaning due before 1 yr, it is operating activity rather than financing activity) 그외에, Bonds Payable Deferred Income Taxes Preferred Stock Paid-in Capital in Excess of Par-Preferred Stock Common Stock Paid-in Capital in Excess of Par-Common Stock Paid-in Capital from Treasury Stock Retained Earnings Treasury Stock)

governmental unit'S basis of accounting method: Enterprise Fund & Internal service use full-accrual basis method. recognizes revenues and expenses on the accrual basis, not on the modified accrual basis. Therefore, these two funds do not use the same basis of accounting. Special Revenue, Permanent, Capital Projects are governmental fund and recognizes revenues and expenditures on the modified accrual basis.

Modified accrual accounting combines aspects of accrual basis accounting with cash basis accounting. The purpose of this type of accounting is to measure the flows of current financial resources in governmental fund financial statements. Revenues are recognized when they become available and measurable. Availability arises when the revenue is available to finance current expenditures to be paid within 60 days. Measurability occurs when the cash flow from the revenue can be reasonably estimated. Expenditures are recognized when liabilities are incurred. This is the same approach used under the accrual basis of accounting, though inventory and prepaid items can be recognized as expenditures when purchased, rather than first being capitalized as an asset. In addition, depreciation expense is not recognized. Instead, assets are expensed when purchased.

Sale-Leaseback Transactions Leases SFAS 13, November 1976 "Accounting for Leases" Sale-Leaseback 1. Entity A sell a property to Entity B 2. Entity B leases the property to Entity A 3. Entity A is the seller-lessee 4. Entity B is the purchaser-lessor Classification of sale-leaseback by seller-lessee 1. Sale-capital-leaseback 2. Sale-operating-leaseback Classification of sale-leaseback by purchaser-lessor 1. Purchase-direct-financing-leaseback 2. Purchase-operating-leaseback Sale-capital-leaseback 1. The lease meets one of (A), (B), (C), (D) 2. A gain or loss on the sale of property is deferred 3. Deferred gain or loss on the sale of property is amortized --> in proportion to the amortization of the leased asset Sale-operating-leaseback 1. The lease does not meets any of (A), (B), (C), (D) 2. A gain or loss on the sale of property is deferred 3. Deferred gain or loss on the sale of property is amortized --> in proportion to the rent expense charged over the lease term Purchase-direct-financing-leaseback 1. The lease meets one of (A), (B), (C), (D) and both of (E), (F) 2. Record the purchase of property 3. Apply the accounting for a direct financing lease Purchase-operating-leaseback 1. The lease does not meet the requirement for a direct financing lease 2. Record the purchase of property 3. Apply the accounting for an operating lease Exceptions Case 1. The seller-lessee leases only a minor portion --> of the remaining use of the property Case 2. The seller-lessee retains --> more than a minor portion --> but less than substantially all --> of the remaining use of the property and --> gain on the sale > (2a) or (2b) (2a) recorded amount of leased asset (2b) present value of minimum lease payments Case 3. Fair value of the property < undepreciated cost Case 1 The sale and the leaseback are recorded --> as "separate" transactions Case 2 1. Sale-capital-leaseback --> Gain on the sale - (2a) is --> recognized at the time of sale 2. Sale-operating-leaseback --> Gain on the sale - (2b) is --> recognized at the time of sale Case 3 A loss is recognized at the time of sale --> up to the amount of (3a) (3a) = undepreciated cost - fair value Capital lease criteria (A) Ownership transfer --> Ownership is transferred by the end of the lease term (B) Bargain purchase option -> Lessee has an option purchase at the price lower than the fair value (C) Lease term: 75% rule --> Lease term ≥ 75% of economic life of the lease property (D) Minimum lease payment: 90% rule --> Present value of minimum lease payments > 90% of fair value of the lease property Additional criteria for lessor (E) Collectibility of minimum lease payment --> reasonably predictable (F) No important uncertainties --> about the additional costs to be incurred by lessor --> when such costs are not reimbursable

On December 31, year 1, Bain Corp. sold a machine to Ryan and simultaneously leased it back for 1 year. Pertinent information at this date follows: Sales price $360,000 Carrying amount 330,000 Present value of reasonable rentals ($3,000 for 12 months @ 12%) 34,100 Estimated remaining useful life 12 years In Bain's December 31, year 1 balance sheet, the deferred revenue from the sale of this machine should be a) $34,100 b) $30,000 c) $ 4,100 d) $0 answer: d ASC Topic 840 generally treats a sale‐leaseback as a single financing transaction in which any profit on the sale is deferred and amortized by the seller. However, ASC Topic 840 amends this general rule when either only a minor part of the remaining use of the leased asset is retained (case 1), or when more than a minor part but less than substantially all of the remaining use of the leased asset is retained (case 2). Case 1 occurs when the PV of the lease payments is 10% or less of the FV of the sale‐leaseback property. Case 2 occurs when the leaseback is more than minor but does not meet the criteria of a capital lease. This is an example of case 1, because the PV of the lease payments ($34,100) is equal to or less than 10% of the FV of the asset ($360,000). ASC Topic 840 specifies that under these circumstances, the full gain ($360,000 − $330,000 = $30,000) is recognized, and none is deferred.

Materiality and relevance are both defined by (여기서 애기 하는 relevance란, relevant information) :What influences or makes a difference to a decision marker.

Relevant information is capable of making a difference in a user' s decision. Materiality is entity specific and related to relevance—if omitting it or misstating it could influence a user' s decision. Therefore, materiality and relevance are defined by what influences or makes a difference to a decision maker. Relevance 뜻 -"important to the matter at hand" (관련깊은 information)

During January year 1, Vail Co. made long‐term improvements to a recently leased building. The lease agreement provides for neither a transfer of title to Vail nor a bargain purchase option. The present value of the minimum lease payments equals 85% of the building's market value, and the lease term equals 70% of the building's economic life. Should assets be recognized for the building and the leasehold improvements? Building: No Leasehold improvements: Yes

Per ASC Topic 840, a lease is classified as a capital lease by the lessee if the lease terms meet any one of the following criteria: (1) transfer of ownership to the lessee by the end of the lease term; (2) bargain purchase option; (3) lease term greater than or equal to 75% of the economic life of the property; or (4) the present value of the minimum lease payments is greater than or equal to 90% of the property's fair market value. Since the terms of Vail's lease do not meet any of the criteria, the lease should be accounted for as an operating lease. Thus, Vail should not recognize the building as an asset. However, Vail should recognize the cost of leasehold improvements as assets because in an operating lease, these costs are capitalized and amortized over the shorter of their useful lives or the term of the lease.

In a sale‐leaseback transaction, the seller‐lessee retains the right to substantially all of the remaining use of the equipment sold. The profit on the sale should be deferred and subsequently amortized by the lessee when the lease is classified as a(n) Capital lease: yes Operating lease: yes

Per ASC Topic 840, any profit related to a sale‐leaseback transaction in which the seller‐lessee retains the right to substantially all of the remaining use of the equipment sold shall be deferred and amortized in proportion to the amortization of the leased asset if the transaction is classified as a capital lease. If the transaction is classified as an operating lease (e.g., if the lease begins in the last 25% of the asset's economic life), the profit shall be deferred and amortized in proportion to the related gross rental charged to expense over the lease term. It is important to note that losses are recognized immediately for either a capital or operating lease. definition of sales lease type: sale-and-leaseback is a financial transaction, where one sells an asset and leases it back for the long-term; therefore, one continues to be able to use the asset but no longer owns it. The transaction is generally done for fixed assets, notably real estate, as well as for durable and capital goods such as airplanes and trains (sale-and-leaseback은 금융 거래로 자산을 팔아 장기적으로리스합니다. 따라서 자산을 계속 사용할 수는 있지만 더 이상 자산을 소유 할 수는 없습니다)The sale of a property in which the seller immediately begins to rent the property from the buyer. That is, the seller no longer has ownership of the property, but maintains residence and/or use for the duration of the rental agreement. A sale-leaseback gives the seller profit from the sale while the buyer is guaranteed income from the rental agreement in the medium or long-term. Sometimes, a sale-leaseback occurs in order to grant the seller access to capital to make improvement on the property; for example, the seller may use the proceeds from the sale to build a factory.

Solen Co. and Nolse Co. exchanged similar trucks with fair values in excess of carrying amounts. In addition, Solen paid Nolse to compensate for the difference in truck values and the transaction lacks commercial substance. As a consequence of the exchange, Solen recognizes A. A gain equal to the difference between the fair value and carrying amount of the truck given up. B. A gain determined by the proportion of cash paid to the total consideration. C. A loss determined by the proportion of cash paid to the total consideration. D. Neither a gain nor a loss. answer: d Per APB 29 and SFAS 153, exchanges that lack commercial substance are recorded at book value. Therefore, gains on these exchanges are recognized only to the extent that boot is received. Since no boot is received by Solen, neither a gain nor loss would be recognized

Slate Co. and Talse Co. exchanged similar plots of land with fair values in excess of carrying amounts. In addition, Slate received cash from Talse to compensate for the difference in land values. The cash flows of the plots of land are not expected to be significantly different. As a result of the exchange, Slate should recognize A. A gain equal to the difference between the fair value and the carrying amount of the land given up. B. A gain in an amount determined by the ratio of cash received to total consideration. C. A loss in an amount determined by the ratio of cash received to total consideration. D. Neither a gain nor a loss. Answer: B APB 29 and SFAS 153 state that when the exchange of nonmonetary assets lacks commercial substance and includes a cash payment (boot), the entity which received cash has realized a partial gain on the exchange. To compute the partial gain to be recognized, first the total gain is computed by subtracting the book value of the nonmonetary asset given up from its fair value. (your asset that you were exchange to other party) This total gain is then multiplied by the ratio of the cash received to the total consideration received (Cash + FMV of nonmonetary asset received). In this case, since Slate received cash, it would recognize a partial gain as described above. ***두 example를 나란히 compare한 이유는, the difference between two questions is who received cash/compensation. 1st question: Solen paid/ not received, so for him no Gain or Loss. the person who receives boot, recognizes the partial gain, so in 2nd question, Slate company must recognize gain on cash (boot)

Watson Company acquired available‐for‐sale securities at a cost of $150,000 in year 1. At December 31, year 1, the securities had a market value of $172,000. In year 2, Watson sold all of its available‐for‐sale securities for $185,000. Watson does not elect the fair value option for reporting its available‐for‐sale securities. As a result of the information presented, what amount of gain should be reported in Watson's net income for year 1 and year 2? Ignore income taxes. Income statement for year 1: $0 Income statement for year 2: $35,000

The amount of gain reported in Watson's net income would be $0 for year 1 and $35,000 for year 2. The gain reported in net income for year 2 is the excess of the selling price of $185,000 over the cost of $150,000. The unrealized gain of $22,000 ($172,000 less $150,000) for year 1 is reported as a component of other comprehensive income in year 1. Since the securities were available‐for‐sale, unrealized gains and losses prior to their sale are not reported in net income. This is why there was $0 reported for the gain in year 1.

Munn Corp.'s records included the following stockholders' equity accounts: Preferred stock, par value $15, authorized 20,000 shares -$255,000 Additional paid‐in capital, preferred stock- $15,000 Common stock, no par, $5 stated value, 100,000 shares authorized- $300,000 In Munn's statement of stockholders' equity, the number of issued and outstanding shares for both common stock and preferred stock?

The balances in the common stock and preferred stock accounts equal the total par or stated value of the shares issued. Therefore, when the account balance is divided by the par or stated value per share, the result is the number of shares issued. *여기서 tricky한것은, question only tells us about the number of shares authorized. 사실상 authorized shares는 큰 의미가 없다. authorized란, 회사가 issue할수있는 maximum의 share를 애기한다.. 쉽게 문제를 풀수있는것은, issued price divided by stated value 또는 par value가 되겠다. Preferred stock ($225,000 ÷ $15), 17,000 shares issued Common stock ($300,000 ÷ $ 5), 60,000 shares issued

Which of the following is not disclosed on the statement of cash flows when prepared under the direct method, either on the face of the statement or in a separate schedule? A. The major classes of gross cash receipts and gross cash payments B. The amount of income taxes paid C. A reconciliation of net income to net cash flow from operations D. A reconciliation of ending retained earnings to net cash flow from operations

The correct answer is D. A reconciliation of ending retained earnings to net cash flow from operations would serve no useful purpose and is not required to be disclosed on the statement of cash flows. Each of the other items mentioned would be disclosed under the direct method.

A. statement of changes in stockholders' equity includes the following: (i guess this is separate statement that is disclosed in financial statement which provides information on changes that are made in retained earning, stockholder equity section) a. Column headings identify individual stockholders' equity accounts. b. Events changing stockholders' equity accounts are listed chronologically to the left. c. The body of the statement is presented in terms of the dollar impact of various transactions and events. d. The impact of the transactions on the number of shares of stock, if any, is presented in the descriptions to the left. e. The ending balances tie to the items presented in the stockholders' equity section of the balance sheet on the same dates.

The full set of financial statements for a period should show the following: Financial position at the end of the period (balance sheet) Earnings (loss) for the period (income statement) Comprehensive income or loss for the period (income statement) Cash flows during the period (statement of cash flows) Investments by and distributions to owners during the period (statement of changes in owners' equity and the statement of retained earnings)

Which of the following rates may be used to translate the cash flow statement? I. Historical exchange rates. II. Current exchange rates. III. Weighted‐average rates. answer: I and III because the cash flow statement may be translated at the rates in effect at the time the transaction occurred (historical exchange rates) or at weighted‐average exchange rates if not substantially different. Current exchange rate method measure the current market value so this not acceptable.

Translated at Current rates Historical AR long‐term $120,000 $100,000 Prepaid insurance 55,000 50,000 Copyright 75,000 85,000 $250,000 $235,000 The subsidiary's functional currency is the currency of the country in which it is located. What total amount should be included in Post's December 31, year 1 consolidated balance sheet for the above accounts? Answer: $250,000 When the functional currency of a foreign subsidiary is the local foreign currency, balance sheet accounts are translated using the current exchange rate (the rate in effect at the balance sheet date). Therefore, these accounts should be included in the balance sheet at $250,000. Note that if the functional currency was the US dollar, balance sheet accounts would be remeasured using a combination of historical and current rates. If the foreign affiliate's financial statements are in the local currency and this is also the functional currency then the Current-Rate Method must be used. (이뜻은, 일본에 있는 subsidiary가 본인나라의 functional currency를 쓰고있다는것. 즉, 일본에서 일어나는 세일이 미국보다 더 영향력있고, 더 많고, expense또한 일본에서 더 많이 쓰이고, 마켓이 일본이 더 클때, 일본이 yen becomes functional currency. 보통은, functional currency로 Financial statement를 만들지만, parent가 미국회사다 보니 나중에 어쩔수없이, yen에서 dollar로 바꿔야한다. 이런경우엔, we use Current Rate Method) If the foreign affiliates' financial statements are in the local currency but the functional currency is dollars then the Temporal-Rate Method must be used. 이뜻은, local currency가 functional currency로 감히 불리지 않을만큼, 마켓의 영향력이 없다는뜻. (Temporal Rate Method는 조심해야하는게 이안에서도 여러가지 조건이 있다. Monetary asset is translated using current rate, income statement is translated using weighted average, and nonmetary asset is translated using historical cost. )

Adjustments Within The Operating Activities Section When we use the indirect method to prepare a statement of cash flows we begin with the net income figure from the company's income statement as our starting point. We then make adjustments to that figure to arrive at the cash amount. (most of company use indirect method) If all of a company's revenues were cash sales (no credit sales), and if the company paid out cash for all of its expenses, then net income would equal the cash from operating activities. However, since some of the revenues and expenses on the income statement were not cash transactions, we must include depreciation, gain or losses on sales of assets, and the changes in current assets and current liabilities.

Under the indirect method of presenting the statement of cash flows, the presentation of this statement begins with net income or loss, with subsequent additions to or deductions from that amount for non-cash revenue and expense items, resulting in net income provided by operating activities.

Fair Value Option(FVO) --> Option to measure eligible items at fair value Eligible items: (a) Financial assets and financial liabilities --> except the items not eligible listed in asc 825-10-15-4 (b) Other eligible items listed in asc 825-10-15-4 Items not eligible: (a) Investment in a subsidiary to be consolidated (b) Investment in a Variable Interest Entities (VIE) to be consolidated Election of FVO FVO is applied "instrument by instrument." FVO is elected when the eligible item is "first recognized." Once FVO is elected --> the decision is "irrevocable." Items Fair Value Option was elected for Measured at "fair value" Unrealized gains and losses are recognized in "earnings." FVO may be elected for the following: 1. Investment in HTM securities 2. Investment in AFS securities 3. Investment in equity securities for "equity method" --> 20% or more, but no more than 50% of ownership 4. Financial liabilities HTM: Held-to-Maturity AFS: Available for Sale

When practicable to estimate, an entity must disclose the value of financial instruments at :Fair value. ASC Topic 825 requires the disclosure, either in the body of the financial statements or in the notes, of the fair value of financial instruments for which it is practicable to estimate that value. Methods and significant assumptions used to make the estimate should also be disclosed. Quoted market prices, if available, are the best estimate of fair value.

How would the amortization of premium on bonds payable affect each of the following? Carrying value of bond: Decrease Net income Increase

When the premium on bonds payable is amortized, the following entry is made: (ex: company issued bond of $1,000,000 and premium interest for $60,000 for 20 years) Cash $1,060,000 Bonds Payable $1,000,000 Premium on Bonds Payable $60,000. Premium on bonds payable $3,000 Interest expense 3,000 $60,000 / 20 year= 3,000 amortize each year if using straight method. This entry has several effects. First, it reduces the amount of the premium. Because the carrying value of the bonds is the face value of the bonds plus the unamortized premium, amortization of the premium serves to reduce the carrying value. Second, amortization of the premium decreases interest expense, thus increasing net income. The bond premium occurred because the bonds' stated interest rate was slightly greater than the interest rate required by the investors in the bond market. Since the premium is related to the interest rates when the bonds were issued, the amortization of the premium involves the account Interest Expense.

Albee Township's fiscal year ends on June 30. Albee uses encumbrance accounting. On April 5, year 1, an approved $1,000 purchase order was issued for supplies. Albee received these supplies on May 2, year 1, and the $1,000 invoice was approved for payment. What journal entry or entries should Albee make on May 2, year 1, upon receipt of the supplies and approval of the invoice?

With General Ledger you can record pre-expenditures commonly known as encumbrances. The primary purpose of tracking encumbrances is to avoid overspending a budget. Encumbrances can also be used to predict cash outflow and as a general planning to fiscal year이 시작되면, gov section에서 비용으로 쓸수있는 허락된 금액이 있다. set aside하는.. 이걸 appropriation이라한다. 물건을 오더하고, purchase order를 받고, contract에 sign 이 되는 순간, appropriation에서 encumbrance로 변한다. Step 1: book entry when purchase order is received & contract is signed: Debit Encumbrce. control $1,000 Credit Reserved for encum $1,000 Step 2: Book entry when product arrive/received. We must cancel encumbrance by reversing previous J/E and convert A/P to expenditure. Debit Reserved for encum $1,000 Credit Encumbrac control $1,000 Debit Expenditure $1,000 Credit A/P $1,000 Step 3: Book entry when payment is sent Debit A/P $1,000 Credit Cash: $1,000

Accounting for Non-exchange transaction. (말그래로, non-exchange, 즉 government가 아무런 서비스르 제공하지 않아도 돈을 받는법) 이뜻은, 얼래 ACCRUAL GAAP에선, ENTITY가 어떠한 service를 제공했을때 sales revenue를 이러내고 그것에 맞는 value를 in exchange로 (돈)을 받는데, gov accounting에서는 꼭 어떤한 service를 주지 않아도 저절로 돈을 받을수가 있다는 그런 내용 (1) derived tax revenue-citizen's sales tax, income tax. (뭐뭐를 인해서 발생하는 택스. 시민들이 물건을 retailer에서 구입하면, 거기서 붙는 sales tax. 시민들의 income w-2에서 발생한 income tax) (2) imposed non-exchanged transaction- 가만히 있어도 tax가 들어오는것.. house purchase가 없더라고 기존에 property owner들이 property tax내야된다. *여기서 "imposed"뜻은, forcing someone. 특별나게 activity가 없었더라도, 시민들의 tax를 빼서갈수있는 이유중 하나는, collecting house property tax다. 10년전에 샀던 집이라도 집이 있다는 이유만으로도 평생 시민들을한테 tax를 받아갈것이다. Forcing to collect. (3) grant-Mandated non-exchange transaction. This is also known as "restricted grants". Providing governments (Federal, state) requires receiving Government (city) to expend funds for specific purpose. grant is recognized as "revenue" when the money has been used. (Rather than when money is received) (4) Voluntary- this is unrestricted DONATIONS, grants from the state to city. There's no specific requirpement and no restriction. 말그대로 voluntary! 그냥 내가 마음이가서 다른 government sector한테 fund를 주는것. 그리고 그 fund 쓰임에 대해 자유를 준다.

Which of the following items is an example of imposed nonexchange revenue for a governmental entity? answer: Property taxes. "imposed" = "property taxes" In accordance with GASB 33, Accounting and Financial Reporting for Nonexchange Transactions, personal income taxes are an example of what type of nonexchange transaction? answer: Derived. Derived = personal income tax

A liquidating entity must present which of the following? A. A statement of net assets in liquidation B. A statement of changes in net assets in liquidation C. Both a statement of net assets in liquidation and a statement of changes in net assets in liquidation D. Neither a statement of net assets in liquidation nor a statement of changes in net assets in liquidation

You are correct, the answer is C. FASB ASC 205-30-45 requires that, when liquidation becomes imminent and at subsequent reporting dates, the entity must present both a statement of net assets in liquidation and a statement of changes in net assets in liquidation. 내가 이해 하기론, Statement of net asset란, Total Asset -Total Liability.

During year 1, Mitchell Corp. started a construction job with a total contract price of $600,000. The job was completed on December 15, year 2. Additional data are as follows: Year 1 Year 2 Actual costs incurred $225,000 $255,000 Estimated remaining costs 225,000 Billed to customer 240,000 360,000 Received from customer 200,000 400,000 Under the completed‐contract method, what amount should Mitchell recognize as gross profit for year 2? a) $ 45,000 b) $ 72,000 c) $ 80,000 d) $120,000 answer: d When a company uses the completed‐contract method of accounting for construction projects, all revenue and expense recognition is deferred until the project is complete or substantially complete (ASC Topic 605). Also, note that neither customer billings nor payments on account are used to determine the revenue recognized under the completed‐contract method (or under the percentage‐of‐completion method). Since the project was complete in year 2, Mitchell should recognize $120,000 ($600,000 − $480,000) in gross profit for year 2.

Year 1 Contract Price: 600,000 Less Total Cost: 450,000 Gross Profit: 150,000 where 450,000 is (Actual Cost -> 225,000 + Prior Period Cost -> 0 + Estimated remaining cost -> 225,000) Percent of Completion: Costs to date/Total Costs 225,000 / 450,000 = 50% Gross Profit to be recognized in year 1: 50% x 150,000 = 75,000 but we won't recognize this since we are using completed-contract method Year 2 Contract Price: 600,000 Less Total Cost: 480,000 Gross Profit: 120,000 where 480,000 is (Actual Cost -> 255,000 + Prior Period Cost -> 225,000 + Estimated remaining cost -> 0) Percent of Completion: Costs to date (Yr 1 & 2)/Total Costs 480,000 / 480,000 = 100% Gross Profit to be recognized in year 2: 100% x 120,000 = 120,000 ***핵심: 만약에, Under the completed‐contract method, what amount should Mitchell recognize as gross profit for year 1? 이라고 물어보면, 답은 $0 이다. Completed contract method는 percentage completion method랑은 틀리게, contract이 완성이 되어야지만, profit을 recognize한다. ***

When preparing a draft of its year 2 balance sheet, Mont, Inc. reported net assets totaling $875,000. Included in the asset section of the balance sheet were the following: Treasury stock of Mont, Inc. at cost $24,000 Idle machinery 11,200 Cash surrender value of life insurance on corporate executives 13,700 At what amount should Mont's net assets be reported in the December 31, year 2 balance sheet? a) $851,000 b)$850,100 c)$842,600 d) $834,500

answer: a Idle machinery ($11,200) and cash surrender value of life insurance ($13,700) are both assets. The only item listed which should not be included in the asset section of the balance sheet is the treasury stock ($24,000). Although the treasury stock account has a debit balance, it is not an asset; instead, it is reported as a contra equity account. Therefore, the $24,000 must be excluded from the asset section, reducing the net asset amount to $851,000 ($875,000 − $24,000).

The statement of cash flows is required for which of the following entities? I. Private, not‐for‐profit hospital. II. Private, not‐for‐profit college. III. Governmental police station. a) I only. b) II only. c) I and II. d) I, II, and III.

answer: c because GASB does not require a statement of cash flows for this governmental unit.

On January 1, year 2, an intangible asset with a 35‐year estimated useful life was acquired. On January 1, year 6, a review was made of the estimated useful life, and it was determined that the intangible asset had an estimated useful life of 45 more years. As a result of the review a) The original cost at January 1, year 2, should be amortized over a 50‐year life. b) The original cost at January 1, year 2, should be amortized over the remaining 30‐year life. c) The unamortized cost at January 1, year 6, should be amortized over a 40‐year life. d) The unamortized cost at January 1, year 6, should be amortized over a 45‐year life.

answer: d 쉽게생각해서, 그냥, year 6 부터는 새롭게 determined된 estimated useful life 로 나누기.. here, it is 45 yrs.

On January 2 of the current year, LTTI Co. entered into a three‐year, noncancellable contract to buy up to 1 million units of a product each year at $.10 per unit with a minimum annual guarantee purchase of 200,000 units. At year‐end, LTTI had only purchased 80,000 units and decided to cancel sales of the product. What amount should LTTI report as a loss related to the purchase commitment as of December 31 of the current year? a) $0 b) $8,000 c) $12,000 d) $52,000

answer: d 이 질문의 핵심포인트는 "3 yr non cancellable contract"이다. 따라서, 지금 계약을 파괴하고 싶어도 계약맺었던 금액들을 다 토해내야됨 (200,000 units less 80,000 units = 120,000 units. 120,000 units × $0.10 = $12,000) <--current year penalty 200,000 minimum annual purchase x 0.10 = 20,000 <- 2nd yr 200,000 minimum annual purchase x 0.10 = 20,000 <- 3rd yr total loss = $12,000 + 20,000 + 20,000 = $52,000

A private not‐for‐profit college has both regular and term endowments. On the college's statement of financial position, how should the net assets of each type of endowment be reported? Term Endowments-> Temporarily restricted Regular Endowments-> Permanently

certain assets that must be maintained permanently. only income from the assets may be expended. Regular Endowment: represent resources that must be invested permanently with income to be used for either restricted or unrestricted purpose. (ex: 만약에 1 million contribution을 받았다. 원금을 절때 쑬수없지만, 원금을 발생하는 이자수입은 unrestricted or temporary restriction으로 reclassify된다) To be restricted, resources must restricted by donors or grantors- Internally designated (by governing board) resources are unrestricted Only contributed resources may be restricted Temporarily restricted net position (resources) -Term endowment: resources are to be invested for a period time (a kind of time restriction) contribution으로 받은 돈으로, 예를들어서, 암 치료 리서치를 위해 돈을 받았다. 정해진 시간이 지나면, restriction에서 풀려지고 unrestrict이 된다. we call this "reclassification" ※ Resources are presumed to be unrestricted unless evidence exist that donor - imposed restrictions exist

What is the purpose of reporting comprehensive income? A. To summarize all changes in equity from nonowner sources B. To reconcile the difference between net income and cash flows provided from operating activities C. To provide a consolidation of the income of the firm's segments D. To provide information for each segment of the business

correct answer is A. SFAC 6 defines comprehensive income as "the change in equity [net assets] of a business enterprise during a period from transactions and other events and circumstances from nonowner sources. It includes all changes in equity during a period except those resulting from investments by owners and distributions to owners." Comprehensive income is the change in equity of a business enterprise during a period from transactions and other events from non-owner sources. It includes all non-owner changes in equity (in contrast to net income which does not include some changes in equity). 여기서 nonowner source란, -->The income of a company from any transaction that does not involve an owner's investment or distribution to an owner

A company has the following accrual-basis balances at the end of its first year of operation: Unearned consulting fees $ 2,000 Consulting fees receivable 3,500 Consulting fee revenue 25,000 The company's cash-basis consulting revenue is what amount? A. $19,500 B. $23,500 C. $26,500 D. $30,500

correct answer is B. The company's "cash-basis" consulting revenue is $23,500: Accrual basis consulting fee revenue $25,000 Unearned consulting fee-- cash received with no revenue 2,000 Consulting fees receivable-- revenue with no cash received (3,500) -------- Cash basis revenue $23,500

The SEC's rulemaking procedures identified on their website include which of the following steps? A. Issue identification B. Commissions deliberation C. Rule adoption D. None of the answer choices are correct.

correct answer is C. The SEC website lists the following steps in the rulemaking process: Concept Release: The rulemaking process usually begins with a rule proposal, but sometimes an issue is so unique and/or complicated that the SEC seeks out public input on which, if any, regulatory approach is appropriate. A concept release is issued describing the area of interest and the SEC's concerns, usually identifying different approaches to addressing the problem, followed by a series of questions that seek the views of the public on the issue. The public's feedback is taken into consideration as the SEC decides which approach, if any, is appropriate. Rule Proposal: The SEC publishes a detailed formal rule proposal for public comment. Unlike a concept release, a rule proposal advances specific objectives and methods for achieving them. Typically, the SEC provides between 30 and 60 days for review and comment. Just as with a concept release, the public comment is considered vital to the formulation of a final rule. Rule Adoption: Finally, the SEC Commissioners consider what they have learned from the public exposure of the proposed rule, and seek to agree on the specifics of a final rule. If a final measure is then adopted by vote of the full Commission, it becomes part of the official rules that govern the securities industry.

Deferred gross profit definition and usage The deferred gross profit concept arises when a business uses the installment sales approach to recognizing its sales transactions. Under the installment method, only the gross profits on those sales for which cash payment has been received are recognized. All gross profits associated with uncollected receivables are parked on the balance sheet as an offset to receivables, where they remain until customer payments are received. The deferred amount of gross profit is stated on the balance sheet as an offset to the accounts receivable account. As such, the deferred profit appears as a contra account immediately below the accounts receivable line item in the assets section of the balance sheet. When this approach is used, the content of the relevant line items in the balance sheet are: Accounts receivable (contains cost of sales + profit) Less: Deferred gross profit (contains unrealized profit) = Net accounts receivable (contains cost only) For example, ABC International sells $100,000 of goods under a periodic payment plan. The cost of the goods sold is $70,000, so there is $30,000 of gross profit associated with the sale. The initial presentation in ABC's balance sheet is: Accounts receivable = $100,000 Less: Deferred gross profit = $(30,000) Net accounts receivable = $70,000 After one month, the customer makes an initial payment of $10,000. Based on the 30% gross profit margin, this payment is comprised of $7,000 cost reimbursement and $3,000 of profit. ABC can now recognize $3,000 of gross profit, which reduces the balance in the deferred gross profit contra account to $27,000.

ex: Tillary Company, which began business on January 1, year 1, appropriately uses the installment sales method of accounting. The following data are available for year 1: Installment accounts receivable, Dec 31, year 1: $200,000 Deferred gross profit, Dec.31, year 1 (before recognition of realized gross profit): $140,000 <-- this should be treated as total gross profit Gross profit on sales 40% The cash collections and the realized gross profit on installment sales for the year ended December 31, year 1, should be: Answer: Cash collections: $150,000 Realized gross profit: $60,000 As this is the first year of operations, all $140,000 is from year 1 sales. In the absence of any defaults and repossessions during the year, this represents the total gross profit (GP) for year 1. Therefore, the total debits to installment AR for year 1 sales (1) can be computed by dividing the deferred GP by the GP ratio, or $350,000 ($140,000/40%). Next, cash collections (2) can be calculated as: $350,000 total debits − $200,000 ending balance = $150,000 cash collections. Finally, GP realized in year 1 (3) would be 40% times cash collections of $150,000 for $60,000 GP realized. **total gross profit과 realized gross profit의 차이점을 알아두기. total gross profit은 말그대로, total sales- total cost = total profit이다.. total profit을 %로 인식하는게 installment method에서는 편할것이다. %로 convert하는건 쉽다. it's (total profit/total sale) 명심해야할게, installment method 에서는, we need to report total gross profit as "deferred gross profit" until partial cash payment from A/R is collected and reduce the balance by the percentage collected. This is called realized gross profit

Different Types of Revenue recognition method: 1) Point of Sale method <- most common method used by company 2) Installment method 3) Cost Recovery Method 1. Point of sale method Debit Credit Accounts Receivable xxx Sales Revenue xxx Cost of Goods Sold xxx Inventory xxx 2. Installment method ex: For example, let's say a company sells a machine costing $180,000 for $300,000. The sales contract calls for 4 annual payments of $75,000. Let's assume the same facts as above for a machinery sale. If collection of the full account is somewhat uncertain <- 핵심: When we are not sure if we can all collect the full amount, we use installment method, we need to use the installment method for the contract. We will only make one journal entry at first: debit Accounts Receivable for $300,000, credit Inventory for $180,000, and credit Deferred Gross Profit for $120,000. Debit Accounts Receivable $300,000 Credit Inventory $180,000 Credit Deferred Gross profit $120,000 *Installment method에서는, A/R 상대계정이 revenue가 아니고, Deferred Gross profit이다. 그리고, Cost (debit) Inventory (Credit)이 아니다. 무족건, Inventory (credit)만 표시하기.* Revenue and cost of goods sold are recognized as each installment payment is received. In addition, deferred gross profit is reduced. (Revenue랑 COGS계정은 돈이 수금될때 잡아주라는것. Deferred gross profit은 돈이 수금되면, 줄여주라는것) Under the installment method, we must split each cash payment between those three accounts using a gross profit percentage. In the example given, $120,000 out of the $300,000 sale is profit, so the percentage is (120,000 ÷ 300,000) x 100, or 40%. For each cash payment, we would debit Cash and credit Accounts Receivable for $75,000. In addition, we would debit Cost of Goods Sold for $45,000, debit Deferred Gross Profit for $30,000, and credit Sales Revenue for $75,000. In this way, profit is recognized throughout the entire payment schedule.

example of recovery method: The following information pertains to a sale of real estate by Ryan Co. to Sud Co. on December 31, year 1: Carrying amount $2,000,000 Sales price: Cash $ 300,000 Purchase money mortgage 2,700,000 3,000,000 The mortgage is payable in nine annual installments of $300,000 beginning December 31, year 2, plus interest of 10%. The December 31, year 2 installment was paid as scheduled, together with interest of $270,000. Ryan uses the cost recovery method to account for the sale. What amount of income should Ryan recognize in year 2 from the real estate sale and its financing? a) $570,000 b) $370,000 c) $270,000 d) $0 answer:d Under the cost recovery method no profit of any type is recognized until the cumulative receipts (principal and interest) exceed the cost of the asset sold. This means that the entire gross profit ($3,000,000 - $2,000,000 = $1,000,000) and the year 2 interest received ($270,000) will be deferred until cash collections exceed $2,000,000. Therefore, no income is recognized in year 2.

On both December 31, 20X1, and December 31, 20X2, Kopp Co.'s only marketable equity security had the same fair value, which was below cost. Kopp considered the decline in value to be temporary in 20X1 but other than temporary in 20X2. At the end of both years, the security was classified as a noncurrent available-for-sale security. Kopp could not exercise significant influence over the investee. What should be the effects of the determination that the decline was other than temporary on Kopp's 20X2 net noncurrent assets and net income? A. No effect on both net noncurrent assets and net income B. No effect on net noncurrent assets and decrease in net income C. Decrease in net noncurrent income assets and no effect on net income D. Decrease in both net noncurrent assets and net income

the answer is B. In accordance with requirements of FASB ASC 320-10-45-5, Kopp Co. would have created a valuation allowance in 20X1 to lower the carrying value of the security to fair value. Since there was no change in the fair value between December 31, 20X1, and December 31, 20X2, there would be no effect on net noncurrent assets in 20X2. The recorded value in 20X2 is the 20X1 fair value. FASB ASC 320-10-35-34 also provides that:

SEC Regulation S-X provides guidance for the issuer regarding: A. nonfinancial forms and disclosures required by the SEC. B. instructions on electronically filing the forms required by the SEC. C. the use of EDGAR by SEC registrants. D. format and content of financial information submitted to the SEC.

the answer is D. Regulation S-X contains information regarding the financial statements that must be submitted to the SEC. Regulation S-X is a prescribed regulation that lays out the specific format and content of financial reports. Regulation S-K contains the instructions for filing the nonfinancial statement forms required by the SEC. Regulation S-T contains instructions for the electronic filing of required SEC forms. Both Regulation S-K and S-T should be read together, as some parts of Regulation S-X may supersede the instructions in Regulation S-K.

Orleans Co., a cash-basis taxpayer, prepares accrual basis financial statements. In its 2007 balance sheet, Orleans' deferred income tax liabilities increased compared to 2006. Which of the following changes would cause this increase in deferred income tax liabilities? I. An increase in prepaid insurance. II. An increase in rent receivable. III. An increase in warranty obligations. Answer is I and II only.

*Book purpose, accounting follows GAAP so it treats A/R when sales occur even if cash has not been collected and treat accrual liability such as warranty obligation when liability accrues, even if no cash payment was made. However, tax basis follows cash basis, it only recognize revenue when actual cash is received and recognize expense when actual payment was sent. This is why it's creating temporary differences. In the first instance, if prepaid insurance increases, cash was out flowed (deductible for tax purposes NOW), but the expense has yet to be recognized for financial purposes. Therefore, you received benefit in the form of lower taxes now that will reverse as the asset is used up/expensed. In the second case, you received income on the accrual basis NOW, but have not received the cash that will be taxed on the cash basis. Accordingly, when you receive the cash, you'll be taxed on it = future taxable amounts. In the third case, you recognize the expense NOW for financial purposes. But for tax purposes, only when the cash is spent on the warranty. Therefore, when spent, it is a "future deductible amount", which is a DTA by definition. 제일 이해하기쉽게: when revenue is recognized for book purpose when it is not recognized for tax basis, it is DTL when expense is recoginised for book purpose when it is not expense for tax basis, it is DTA

Miro Co. began business on January 2, 20X0. Miro used the double-declining balance method of depreciation for financial statement purposes for its building, and the straight-line method for income taxes. This was the only difference between tax and financial statement accounting. On January 16, 20X2, Miro elected to switch to the straight-line method for both financial statement and tax purposes. The building, which cost $240,000 in 20X0, had a useful life of 15 years and no salvage value. Data related to the building is as follows: Double-declining balance Straight-line Year Depreciation Depreciation 20X0 $30,000 $16,000 20X1 $20,000 $16,000 Miro's tax rate is 40%. Which of the following statements is correct? a. There should be no reduction in Miro's deferred tax liability. b. There should be no increase in Miro's deferred tax asset. c. Miro's deferred tax asset should be reduced by $7,200 in 20X2. d. Miro's deferred tax asset should be increased by $7,200 in 20X2. Correct Answer: B

1) Year 0 and Year 1, Total temporary difference due to the different depreciation is $18,000 ($50,000-$32,000) Which created Deferred Tax Assets: $18,000 x 40% = $7,200 2) in Year 2, the company switch to using straight line for both income tax and financial reporting purpose. Remember, Double Decline and Straight line are allowable depreciation methods under GAAP, therefore, on financial reporting side, they will treat it like change in estimates. No retro adjustments needed. Then, depreciation on Financial report is ($240,000 - $50,000)/13 = $14,615 Tax depreciation is $16,000 Temporary difference = $1,385 Therefore, in Year 02, there is no additional DTA created , and the DTA will be reversed by $1,385 x 40% = $554 3) Then, the balance sheet still carries DTA with debit balance at the amount of $7,200 - $554 =$6,646 Even this DTA will eventually be fully reversed in the future years, but not happened in year 2 only. Therefore, B is the best answer. A change in depreciation method is treated prospectively, meaning that you do not go back and restate it. The original cost is $240,000 and for two years they used DDB - year 1 was $30,000 and year 2 was $20,000. In year 3 when the change is made, the building had a carrying cost of $190,000 with 13 years of life remaining. $190,000/13 = $14,615.38. Different explanation: Miro's deferred tax asset will not increases. Miro will continue to have a deferred tax asset that will decrease on a straight-line basis over the life of the asset. The change in depreciation method represents a change in accounting estimate inseparable from a change in accounting principle. The change from the double-declining balance method of depreciation for financial statement purposes to the straight-line method consistent with it tax treatment will be treated prospectively. At the beginning of 20x2, Miro will have a financial statement net book value of $190,000 and a tax basis of $208,000. Book depreciation of $14,615 will be consistently less than tax depreciation of $16,000, which will result in an annual decrease of the deferred tax asset.

Segment reporting is the reporting of the operating segments of a company in the disclosures accompanying its financial statements. Segment reporting is required for publicly-held entities, and is not required for privately held ones. (also not required to not-for-profits) Under Generally Accepted Accounting Principles (GAAP), an operating segment engages in business activities from which it may earn revenue and incur expenses, has discrete financial information available, and "whose results are regularly reviewed by the entity's chief operating decision maker for performance assessment and resource allocation decisions". (Regularly reviewed by Chief operating is very important when determining what needs to be in segment reporting) Follow these rules to determine which segments need to be reported: Aggregate the results of two or more segements if they have similar products, services, processes, customers, distribution methods, and regulatory environments. Report a segment if it has at least 10% of the revenues, 10% of the profit or loss, or 10% of the combined assets of the entity. If the total revenue of the segments you have selected under the preceding criteria comprise less than 75% of the entity's total revenue, then add more segments until you reach that threshold. You can add more segments beyond the minimum just noted, but consider a reduction if the total exceeds ten segments. The information you should include in segment reporting includes: -The factors used to identify reportable segments. -The types of products and services sold by each segment -The basis of organization (such as being organized around a geographic region, product line, and so forth) -Revenues -Interest expense -Depreciation and amortization -Material expense items -Equity method interests in other entities -Income tax expense or income -Other material non-cash items -Profit or loss

A convention some companies use in their financial statements in which they report information by sector. For example, a single publicly-traded company that engages in agriculture, mining, and manufacturing may report revenue, sales, and profits/losses for each field individually. This allows a company to show its investors what parts of the company are performing better relative to the others. It also prevents one sector that is overperforming or underperforming from unduly influencing a financial statement. What is 'Business Segment Reporting' Giving separate accounts of a company's individual divisions, subsidiaries or other segments. In an annual report, the purpose of business segment reporting is to provide an accurate picture of a public company's performance to its shareholders. For upper management, business segment reporting is used to evaluate each segment's income, expenses, assets, liabilities and so on in order to assess profitability and riskiness. ex: The following information pertains to Been Corp. and its operating segments for the year ended December 31, year 1: Total revenues $80,000,000 Sales to external customers (included in total) $30,000,000 In its financial statements, Been should disclose major customer data if sales to any single customer amount to at least : $ 8,000,000 Because if 10% or more of the revenue of an enterprise is derived from sales to any single customer, that fact and the amount of revenue from each significant customer shall be disclosed. In this problem, Been reported revenues of $80,000,000 and thus should disclose major customer data if sales to any single customer amount to $8,000,000 ($80,000,000 × 10%). The segment generating the sales must be disclosed but the name of the customer does not need to be disclosed.

Pride, Inc. owns 80% of Simba, Inc.'s outstanding common stock. Simba, in turn, owns 10% of Pride's outstanding common stock. What percentage of the common stock cash dividends declared by the individual companies should be reported as dividends declared in the consolidated financial statements? Dividends declared by Pride : 90% Dividends declared by Simba: 0%

A reciprocal ownership relationship exists between the two companies such that Pride (the acquirer) owns 80% of Simba (the acquiree), and Simba owns 10% of Pride. When Pride declares a cash dividend, 90% of it is distributed to outside parties and 10% goes to Simba. Because Simba is part of the consolidated entity, its 10% share is eliminated; thus, only 90% of dividends declared by Pride are reported in the consolidated statements. When Simba declares a dividend, 80% is distributed to Pride and 20% to outside parties. Pride's 80% share is eliminated as an intercompany transaction and the remaining 20% is also excluded because, from the acquirer's point of view, acquiree dividends do not represent dividends of the consolidated entity and must be eliminated. (여기서 중요한점은, parent who acquires more than 50% of stock of subsidiary is consolidated and 이러한 parent는 양아치다. 본인이 sub한테 받아먹는 dividend는 intercompany transaction으로 주장해서 eliminate한다. 그리고, 본인이 오직 outside shareholder한테 delcare한 dividend 부분만, consolidated return에 dividend pay도 표시한다. subsidiary가 자기들의 outside shareholder한테 declare한 dividend는 무시한다. )

On March 1, year 1, Agront Corporation issued 10,000 shares of its $1 par value common stock for all of the outstanding stock of Barcelo Corporation, when the fair market value of Agront's stock was $50 per share. In addition, Agront made the following payments in connection with this business combination: Finder's and consultant's fees $20,000 SEC registration costs 7,000 Agront's acquisition cost would be capitalized at A) $0 B) $500,000 C) $520,000 D) $527,000

ANSWER: B Per ASC Topic 805 the finder's and consultant's fees should be expensed. The SEC registration costs should be netted against the additional paid‐in capital account JOURNAL ENTRY: Dr. Investment in B $500K ($50 x #10K shrs) Cr. CS $10K ($1 x #10K shrs) Cr. APIC ; $490K ($500K-$10K) Dr. APIC ; $7K Dr. Expense $20K (hit I/S) Cr. Cash ; $27K Must treat the finder's and consultant's fees as non-direct cost, which to be expensed as incurred. 1. Direct out-of-pocket costs such as finder's fee or a legal fee are expensed. 2. Stock registration and issuance costs such as SEC filling fees are a direct reduction of the value of the stock issued (Debit to APIC) of the parent/purchasing company. **Direct stock related fee must be deducted from APIC** All other costs associated with the acquisition must be expensed, including reimbursements to the acquiree for bearing some of the acquisition costs. Examples of costs to be expensed include finder's fees; advisory, legal, accounting, valuation and other professional or consulting fees; and general administrative costs, including the costs of maintaining an internal acquisitions department. **내가 햇갈렸던 부분은, Common stock을 issue할때 weighed average로 계산을 안했다는것.. 3/1에 issue했는데 왜 10/12 month를 par value에 반영을 안했는지 궁굼하다...common stock weighted average계산하는건 아마, b/s common stock outstanding 을 계산할때만인가? ** Weighted-average outstanding을 쓰는 situation은 언제냐면, To determine the number of shares used for the calculation of basic earnings per share (BEPS) and diluted earnings per share (DEPS), first determine the weighted‐average shares outstanding. <- BEPS, DEPTS를 계산할때

In the calculation of pension expense recognized for a period by an employer sponsoring a defined benefit pension plan, which components will not be included? A) Interest cost on the projected benefit obligation. B) Actuarial present value of benefits attributed by the pension benefit formula to employee service during that period. B) Amortization of the unrecognized net obligation (and loss or cost) or unrecognized net asset (and gain) existing at the date of transition. D) Excess of accumulated benefit obligation over the fair value of the plan assets. Answer: d notice ,the question is asking about pension expense that needs to be subtracted from plan asset. Pension expense included in the PBO which needs to be subtracted from plan asset. If plan asset is greater than PBO company must record current liability, which is way to record for unfunded benefit.

Answer A is incorrect because per SFAS 87, interest cost should be included as a component of pension expense. B. Answer B is incorrect because per SFAS 87 service costs determined by actuarial present value should be included as a component of pension expense. Answer C is incorrect because per SFAS 87, amortization of the unrecognized net obligation or unrecognized net asset should be included as a component of pension expense. Answer D is correct. Excess of accumulated benefit obligation over the fair value of the plan assets should not be included as a component of pension expense because it represents the unfunded accumulated benefit obligation which could require recognition of an additional minimum liability.

If losses in the amount of $2,750 (net of tax) on available‐for‐sale securities have been previously included in other comprehensive income, what amount would be the reclassification adjustment when the securities are sold? Assume a 30% tax rate. A) $2,750 B) $(2,750) C)$3,575 D) $(3,575)

Answer: A $2,750 had been a deduction of other comprehensive income in prior years. When the securities are sold, the loss will be included in net income. Adding -2,750 to net income. 여기서 끝나는게 아니고, $2,750 must be added back to other comprehensive income to avoid taking the loss twice. During prior year, -2,750 was included as "other comprenhensive income" <--remember this comes after net income so doesn't affect net income. Now, Current year, -2,750 has been sold so we need to "recalssify" this loss to be in net income. Simply, other comprehensive income에서 net income으로 전환하는것. 근데 이게 끝이 아니다. 다시 Current year's other comprehensive income에 $2,750 (positive)를 add해준다. 따랏, it net it out with previous other comprehensive income entry??

The following expenses were among those incurred by Sayre Company during year 1. Accounting and legal fees $160,000 Interest 60,000 Loss on sale of office equipment 25,000 Rent for office space 200,000 One‐quarter of the rented premise is occupied by the sales department. How much of the expenses listed above should be included in Sayre's general and administrative expenses for year 1? A) $310,000 B) $335,000 C) $360,000 D) $370,000

Answer: a The accounting and legal fees ($160,000) and the portion of office rent not allocable to sales (3/4 × $200,000 = $150,000) are all considered general and administrative expenses. Therefore, general and administrative expense should be $310,000 ($160,000 + $150,000). In addition, the interest expense ($60,000) would be included with financial expense or other expenses. The $25,000 loss on the sale of office equipment should be included in other expenses and losses. The office rent for the sales department (1/4 × $200,000 = $50,000) is an operating expense and included in selling expenses rather than general and administrative expenses.

Korn Company incurred the following costs during year 1: -Modification to the formulation of a chemical product $135,000 -Troubleshooting in connection with breakdowns during commercial production 150,000 -Design of tools, jigs, molds and dies involving new technology 170,000 -Seasonal or other periodic design changes to existing products 185,000 -Laboratory research aimed at discovery of new technology 215,000 In its income statement for the year ended December 31, year 1, Korn should report research and development expense of a) $520,000 b) $470,000 c) $385,000 d) $335,000

Answer: a 쉽게 생각하면, R&D includes new idea, new testing, salary of researcher who is working on new item, new technology, formulation of product (여기서 formulation은 창조하다, 새로 만든다.) Any work that relates to existing product such as maintenance, fixing troubleshooting is not R&D. 여기서 TRICK은, Modification to the formulation of a chemical product이다. Modification이라서 햇갈리만, 핵심포인트는, "formulation"이다. 어떤걸 창조할려고 하는데, 그때, 잠시 필요했던 수정작업도 R&D. 조심해야하는게, if it says "reformulations of a chemical compound" , this is not considered research and development because it is re-doing, re-formulating. Re-formulating= rechanging something that is already formulated.

A company issued rights to its existing shareholders to purchase for $15 per share, 5,000 unissued shares of common stock with a par value of $10 per share. Common stock will be credited at a) $15 per share when the rights are exercised. b) $15 per share when the rights are issued. c) $10 per share when the rights are exercised. d) $10 per share when the rights are issued.

Answer: c At issuance, the issuer makes only a memorandum entry indicating the number and price at which the shares may be purchased. Upon exercise, the company receives the cash and makes the following entry: Cash (proceeds) Common stock (par value) APIC (plug) **common stock은 par value로 갑고, common stock이 issue되서, shareholder/creditor한테팔려서 돈이 받으면, 만약에 이익이 더 생기면, 차액을 APIC (Additional paid In Capital)로 잡아죽. Par value for a share refers to the stock value stated in the corporate charter. Additional paid-in capital is any payment received from investors for stock that exceeds the par value of the stock.

On October 1 of the current year, a US company sold merchandise on account to a British company for 2,000 pounds (exchange rate, 1 pound = $1.43). At the company's December 31 fiscal year‐end, the exchange rate was 1 pound = $1.45. The exchange rate was 1 pound = $1.50 on collection in January of the subsequent year. What amount would the company recognize as a gain (loss) from foreign currency translation when the receivable is collected? a) $0 b) $100 c) $140 d) $(140)

Answer: b **currency covert로 인해 생긴 gain/loss는 it goes into net income. **다른 나라의 F/S를 translate하다가 생긴 gain/loss는 it goes into "other comprehensive income" and doesn't affect the net income. 핵심포인트: 이번문제는 요약하자면, 맨처음에 미국이 영국에 있는 회사에 팔고, A/R에 달아놓을때, 그때 currency exchange 반영해서 잡아두기. 하지만, 연말에 currency가 떨어지거나 바뀌면, adjust해줘야 한다. F/S는 최대한 정확히 보고 해야하니깐. 그래서 이때 book entry에 12/31 기준 currency exchange rate에 맟추어 다시 adjust하기. 그다음에 다음년에 A/R이 수금되면, 수금 날짜에 맟춘 currecny rate랑 비교하고, 거기서 발생하는 gain or loss는, it will affect the net income! On October 1 of the current year, the company would record the accounts receivable and sale at the spot rate of $2,860 (2,000 pounds × $1.43). At the end of the year, the account receivable would be adjusted to the end of year spot rate. In this case, the account receivable would be revalued to $2,900 (2,00 pounds × $1.45 ), and an adjustment would be made to accounts receivable by debiting accounts receivable by $40 and crediting a gain on foreign currency transactions for $40. This gain on foreign currency transactions would be recognized in the income statement of the current year. In the subsequent year, when the receivable was collected, the exchange rate was $1.50. Accordingly, the account receivable would be revalued to $3,000 (2,000 pounds × $1.50), and a gain of $100 ($3,000 less $2,900 fair value at end of previous year) would be recognized in income in the year that the receivable is collected.

Mr. and Mrs. Gasson own 100% of the common stock of Able Corp. and 90% of the common stock of Baker Corp. Able previously paid $4,000 for the remaining 10% interest in Baker. The condensed December 31, year 2 balance sheets of Able and Baker are as follows: Able Baker Assets $600,000 $60,000 Liabilities $200,000 $30,000 Common stock 100,000 20,000 Retained earnings 300,000 10,000 $600,000 $60,000 In a combined balance sheet of the two corporations at December 31, year 2, what amount should be reported as total stockholders' equity? A) $430,000 B) $426,000 C) $403,000 D) $400,000

Answer: b Combined financial statements is the term used to describe financial statements prepared for companies which are owned by the same parent company or individual. They are prepared by combining all of the subsidiaries' financial statement classifications. Intercompany transactions should be eliminated in the same way as consolidated statements. Combining the stockholders' equity accounts of Able and Baker results in a total of $430,000 ($100,000 + $20,000 + $300,000 + $10,000). The intercompany balances (investment in Baker, $4,000; and Common Stock, $4,000) must be eliminated, which reduces combined stockholders' equity to $426,000 ($430,000 - $4,000) Able bought the remaining 10% of Baker's share for 4,000 thus they now own 100% DR Equity purchase for 10% Baker share 4,000 CR Cash paid 4,000 Thus your total assets is 660,000-4,000=656,000 Your Liabilities= 230,000 For a shareholders equity is $430,000 - $4,000= 426,000 **Stock purchase한 party의 retained earning을 줄여주고, 그돈을 받은 (stock seller)는 돈받은만큼, cash (즉 asset)에서 다시 토해 내야한다.

On January 1, year 1, Frost Co. entered into a two‐year lease agreement with Ananz Co. to lease 10 new computers. The lease term begins on January 1, year 1 and ends on December 31, year 2. The lease agreement requires Frost to pay Ananz two annual lease payments of $8,000. The present value of the minimum lease payments is $13,000. Which of the following circumstances would require Frost to classify and account for the arrangement as a capital lease? a) The economic life of the computers is three years. b) The fair value of the computers on January 1, year 1, is $14,000. c) Frost Co. does not have the option of purchasing the computers at the end of the lease term. d) Ownership of the computers remains with Ananz Co. throughout the lease term and after the lease ends.

Answer: b 이 질문의 purpose는, which factor makes this lease agreement to be capital lease agreement.. For a lessee to account for a lease as a capital lease under U.S. GAAP, the terms of the lease MUST MEET AT LEAST ONE of the capital lease criteria: (1) Ownership transfers at the end of the lease (2) Written option for bargain purchase (3) PV of minimum lease payments ≥ 90% of FV of leased property (4) Lease term ≥ 75% of asset useful life If the fair value of the computers at lease inception is $14,000 and the present value of the minimum lease payments is $13,000, then the lease will be accounted for as a capital lease: 13,000/14,000 = 93%

A change in the periods benefited by a deferred cost because additional information has been obtained is A) A correction of an error. B) An accounting change that should be reported by restating the financial statements of all prior periods presented. C) An accounting change that should be reported in the period of change and future periods if the change affects both. D) Not an accounting change.

Answer: c ASC Topic 250 states that a change in the periods benefited by a deferred cost should be treated as a change in accounting estimate. Changes in accounting estimates are accounted for in the period of change and future periods if the change affects both (if you can't tell if the question is talking about retrospective or prospective (Estiatmes) ,If the problem has a deferred cost that occurred in an earlier period treat it as an estimate. I also noticed that in the estimate problems wiley uses the phrase "additional information has been obtained". They use "Additional info" because they were was uncertainity and more research had to be performed) In most cases estimates will allow you to play with income and expenses by using information that wouldn't of been available in that fiscal period to your advantage. The main difference is estimates are made when certain information is missing and once the fiscal period closes and new information is attained you can no longer change an estimate in prior periods. You change the estimate and future periods are affected and in most cases when the change is made you still have a level of uncertainty with the new method of making an estimate. Uncertainty is the nature of estimates. Once time goes by and you become certain of events you can't go back and change them. The main reason we change most accounting principles in old periods is so the comparative financials use the same major accounting rules. The reason we don't change estimates in prior periods is because that level of uncertainty in estimates should be present in each periods financials.

On November 15, year 2, Celt, Inc., a US company, ordered merchandise FOB shipping point from a foreign company for 200,000 LCUs. The merchandise was shipped and invoiced to Celt on December 10, year 2. Celt paid the invoice on January 10, year 3. The spot rates for LCUs on the respective dates are as follows: November 15, year 2 $ .4955 December 10, year 2 .4875 December 31, year 2 .4675 January 10, year 3 .4475 In Celt's December 31, year 2 income statement, the foreign exchange transaction gain is A) $9,600 B) $8,000 C)$4,000 D) $1,600

Answer: c No journal entry is prepared on 11/15/Y2 when the goods are ordered, so the spot rate for LCUs on that date ($.4955) is not relevant to the solution of this problem. On 12/10/Y2, Celt would record the purchase and related accounts payable at $97,500 (200,000 LCUs × $.4875). A foreign exchange transaction gain (loss) is to be recognized if the spot rate on the settlement date (or any intervening balance sheet date) is different from the rate on the transaction date. At 12/31/Y2, the spot rate is $.4675, which means the account payable must be adjusted down to $93,500 (200,000 LCUs × $.4675), resulting in a gain of $4,000. A shortcut approach is to multiply the change in the exchange rate ($.4875 − $.4675 = $.02) by the payable balance in LCUs ($.02 × 200,000 LCUs = $4,000). 여기서 핵심 포인티: shipping FOB term이면, 계약 맺자마자, A/P를 record하는게 아니고, 배가 물건을 실고 떠나날 기준으로 잡아야한다. 그리고, 아무리 진짜돈은 다음년 1월초에 나가도 adjustment는 언제나 current year's at year end에 adjust가 되어햔다.

The original cost of an inventory item is above the replacement cost. The replacement cost is above the net realizable value. Under the lower of cost or market method, the inventory item should be priced at its A) Replacement cost. B) Original cost. C) Net realizable value. D) Net realizable value less the normal profit margin.

Answer: c This answer is correct because under LCM, market is the replacement cost provided that replacement cost is lower than net realizable value (ceiling) and higher than the net realizable value less a normal profit margin (floor). Since the replacement cost is above the ceiling, the ceiling represents the market value to be compared with cost. The ceiling (market price) is less than cost so the inventory would be priced at net realizable value. For LCM example, we have to choose Lower of cost or market. Basically replacement cost cannot be higher than celling and lower than floor. (Celling > replacement cost > floor) Here, celling price means (NRV=selling price-cost to complete the sell which include cost to dispose) Here, Floor means (NRV - MARGIN) So, replacement cost must be in between Ceilling and Floor. Replacement가 왜 중요하나면, it determines the "Market" value later on. Market value가 왜 중요하냐고? LCM method requires us to choose Lower of Cost or Market. (여기서 cost는 historical cost를 애기한다) Going back to replacement cost.. replacement cost는 given에서 줄것이다. 그치만 if replacement cost is lower than floor value, we have to use whichever has the greater amount because this number will be our "market value"

Which of the following funds of a governmental unit records depreciation? A) Capital projects fund. B) Debt service fund. C) Internal service fund. D) Special revenue fund.

Answer: c because internal service funds are the only ones listed that record depreciation. 내 생각에는, internal service fund에서는 차를 구입하거나, 복사기를 구입하고, 다른 부서들한테 사용하는 권한을 reimbursement 식으로 보상받은 곳이라서, 구입하는 tangible pp&e들이 분명히 존재할거라 믿는다. Debt service는 그냥 long term lialibity를 갚는곳이고, special revenue fund는 motor tax, hotel tax등등 specific tax를 관리하는곳이고, captial project는 그냥 말그대로, 오랫동안 공사에 들어가는 fund..

A city council designates funds in the enterprise fund for future equipment replacement. The enterprise fund should report this as: A) A restricted component of net position. B) A net investment in capital assets. C) A designated component of net position. D) An unrestricted component of net position.

Answer: d because the fund should be presented as an unrestricted component of net position. To be presented as restricted the fund must be externally restricted by creditors, or restricted by law. In order to be Restricted Fund, must be restricted by creditors, donor, or by law. (city 정부가 스스로 restriction을 걸어놓는게 아닌가보다) Enterprise Fund란, accounts for activities by which government provides goods and services to general public (a feel is charged to external user) Enterprise ex: water, public transportation, sewer, airport, waste, toll road.. 정부가 시민들을 위해 운영하는 사업체들. Of course, people have to pay for these services.

On January 1 of the current year, Lean Co. made an investment of $10,000. The following is the present value of $1.00 discounted at a 10% interest rate: ($10,000 <- is future amount인가보다.. 향휴 $10,000 value의 investment의해서 계약을 맺은거니깐 ,treat $10,000 as future amount upon the end of the investment) Periods PV of $1.00 discounted at 10% 1 .909 2 .826 3 .751 What amount of cash will Lean accumulate in two years? A) $12,000 B) $12,107 C) $16,250 D) $27,002

Answer:b *Note, PV (present value) is used when we know the future amount but when we wants to converted/calculated to present value. 이 문제는 거꿀고, present value는 알고있는데, future amount를 몰라서 거꿀로, 찾는것. FV factors would be used to work this problem. Because PV factors are the only information given, the formula, PV amount equals the PV factor times the future amount, must be used. Let x = the future value. Substituting into this formula, 10,000 = .826(x). Solving for x, the future value is $12,107.

Antidilutive securities would generally be used in the calculation of Basic earnings per share: no Diluted earnings per share:no

Basic earnings per share should be based on the weighted‐average outstanding shares. Diluted EPS should not include any security whose conversion, exercise, or contingent issuance would increase diluted EPS. By definition, antidilutive common stock equivalents would increase diluted EPS if they were included in its calculation.

In a statement of cash flows, which of the following items is reported as a cash outflow from financing activities? I. Payments to retire mortgage notes. II. Interest payments on mortgage notes. III. Dividend payments.

Both I and III are financing cash outflows. Principal payments on loans from financial institutions are financing because they are a return of a source of long-term financing. The dividends are a return to shareholders who have provided a considerable portion of total firm financing. Interest payment on mortgage notes is operating activities.

All entities are exposed to some form of market risk. For example, gold mines are exposed to the price of gold, airlines to the price of jet fuel, borrowers to interest rates, and importers and exporters to exchange rate risks. Many financial institutions and corporate businesses (entities) use derivative financial instruments to hedge their exposure to different risks (for example interest rate risk, foreign exchange risk, commodity risk, etc.).

Cash flow hedges A cash flow hedge is designed to minimize the risk that a company will have to pay more than it expects. let's say that your company will need to buy 1000 tons of copper next year, which sells for $4,585 per ton as of this writing. So you expect an expense of $4,585,000 for your copper needs, and you use this figure when planning your budget. If the price of copper spikes to $6,000 per ton between now and next year, this would mean more than $1.4 million in unanticipated expenses. To mitigate this risk, you could buy some copper futures contracts, which would increase in value if the price of copper were to rise. Fair value hedges On the other hand, a fair value hedge is a type of hedging instrument designed to limit exposure to changes in the value of an asset or liability. For example, if your company owns a large stock portfolio, you could buy put options on the stocks in the portfolio. And, the value of these options would increase if the stock's price were to fall, reducing or even eliminating your potential losses.

A firm has basic earnings per share of $1.29. If the tax rate is 30%, which of the following securities would be dilutive? a) Cumulative 8%, $50 par preferred stock. b) 10% convertible bonds, issued at par, with each $1,000 bond convertible into 20 shares of common stock. c) 7% convertible bonds, issued at par, with each $1,000 bond convertible into 40 shares of common stock. d) 6%, $100 par cumulative convertible preferred stock, issued at par, with each preferred share convertible into four shares of common stock.

Choice "c" is correct. A dilutive security will produce an earnings per share number below basic earnings per share. (핵심: Dilutive EPS will be always be lower than Basic EPS) The formula for basic earnings per share is income available to common shareholders divided by the weighted average number of common shares outstanding. Basic earnings per share is $1.29, and a dilutive security will result in a lower earnings per share number. If the seven percent convertible bonds are converted, the company will save $49 on each bond ($1,000 x .07 x (1 - .30)), but 40 new shares of stock will be issued. This equates to $1.225 per 1 new share, which is a lower ratio than $1.29 per share. So these securities will be dilutive. Choice "a" is incorrect. There is no indication given that the shares are convertible, so they will not be dilutive. Choice "b" is incorrect. If the ten percent convertible bonds are converted, the company will save $70 on each bond ($1,000 x .10 x (1 - .30)) and 20 new shares of stock will be issued. This equates to $3.50 per 1 new share, which is a higher ratio than $1.29 per share. So these securities will be anti-dilutive. Choice "d" is incorrect. If the convertible preferred stock is converted, the company's earnings per share will increase in the numerator by the $6 dividend that will no longer be paid, while the denominator will increase by 4 for the new shares of common stock issued. That equates to $1.50 per share, which is higher than $1.29.

Combined statements may be used to present the financial position and the results of operations of a group of unconsolidated subsidiaries or companies under common management. A combined financial statement shows financial results of different subsidiary companies from that of the parent company. The complete financial statement of one subsidiary is shown separately from another as a stand-alone company. The benefit of combined financial statements is that it allows an investor to analyze the results and gauge the performance of the individual subsidiary companies separately. A financial statement that merges the assets, liabilities, net worth, and operating figures of two or more affiliated companies. A combined statement is distinguished from a consolidated financial statement of a company and subsidiaries, which must reconcile investment and capital accounts.

Consolidated Financial Statements Consolidated financial statements aggregate the financial position of a parent company and its subsidiaries. This allows an investor to check the overall health of the company in a holistic manner rather than viewing the individual company's financial statements separately. In other words, the consolidated financial statements agglomerates the results of the subsidiary businesses into the parent company's income statement, balance sheet and cash flow statement

An example of a notional amount is a) Number of barrels of oil. b) Interest rates. c) Currency swaps. d) Stock prices. answer: a Notional amounts are the referenced associated asset or liability that is commonly a number of units such as barrels of oil.

For instance, in an options contract of 100 shares of an underlying asset, the notional amount is the contract size (100) times the price of the underlying asset. So, if the option is purchased at $3 per share and the price of the underlying security is $24, the notional amount is $3 x 100 x $24 = $7,200.

A subsidiary's functional currency is the local currency, which has not experienced significant inflation. The appropriate exchange rate for translating the depreciation on plant assets in the income statement of the foreign subsidiary is the a) Exit exchange rate. b) Historical exchange rate. c) Weighted‐average exchange rate over the economic life of each plant asset. d) Weighted‐average exchange rate for the current year. answer: d because ASC Topic 830 specifies that when the functional currency is the foreign currency, exchange rates for expenses should be those in effect at the time transactions are recorded during the current period. However, the statement also says that weighted‐average rates can be used for items occurring numerous times during the accounting period. Such weighted‐average rates may also be used for accounting allocations such as depreciation. If the foreign affiliate's financial statements are in the local currency and this is also the functional currency then the Current-Rate Method must be used. (이뜻은, 일본에 있는 subsidiary가 본인나라의 functional currency를 쓰고있다는것. 즉, 일본에서 일어나는 세일이 미국보다 더 영향력있고, 더 많고, expense또한 일본에서 더 많이 쓰이고, 마켓이 일본이 더 클때, 일본이 yen becomes functional currency. 보통은, functional currency로 Financial statement를 만들지만, parent가 미국회사다 보니 나중에 어쩔수없이, yen에서 dollar로 바꿔야한다. 이런경우엔, we use Current Rate Method) If the foreign affiliates' financial statements are in the local currency but the functional currency is dollars then the Temporal-Rate Method must be used. 이뜻은, local currency가 functional currency로 감히 불리지 않을만큼, 마켓의 영향력이 없다는뜻. (Temporal Rate Method는 조심해야하는게 이안에서도 여러가지 조건이 있다. Monetary asset is translated using current rate, income statement is translated using weighted average, and nonmetary asset is translated using historical cost. )

Gordon Ltd., a 100% owned British subsidiary of a US parent company, reports its financial statements in local currency, the British pound. A local newspaper published the following US exchange rates to the British pound at year‐end: (U.S. parent company owes subsidiary in British and subsidiary need to follow U.S Exchange rate) Current Rate $1.50 Historical rate (acquisition) 1.70 Average rate 1.55 Inventory (FIFO) 1.60 Which currency ratio should Gordon use to convert its income statement to US dollars at yearend? a) 1.50 b) 1.55 c)1.60 d) 1.70 answer: b the current rate method requires income statement items (revenues and expenses) to be translated using the weighted‐average rate, $1.55. Per ASC Topic 830, if the functional currency equals the local currency, (meaning, functional currency refers to the main currency used by a business or unit of a business, generally the currency that parent company uses), company'the current rate method is used. the current rate method requires income statement items. If British subsidiary also followed US dollar (in this case, functional currency that its parents use), then British subsidiary can use current rate. However, since, British sub used Pound instead of dollar, we need to use "average rate" to convert

With respect to the footnote disclosure, what are U.S. GAAP and IFRS differences? A. U.S. GAAP requires more footnote disclosure than IFRS. B. IFRS requires more footnote disclosure than U.S. GAAP. C. U.S. GAAP and IFRS require about the same amount of footnote disclosure. D. IFRS requires very little footnote disclosure.

The correct answer is B. IFRS (International Financial Reporting Standards) is principle-based, with fewer rules and standards than GAAP. Consequently, disclosure of the reasoning behind the information on the financial statements requires a great deal of footnote disclosure, including a footnote for accounting policies. International f/s 이 더 복잡하네..아무래도 인터네셜널 수준과 기대치가 더 높아서 그런가

enterprise‐wide disclosures about products and services, geographic areas, and major customers are required for all enterprises. It is required for public company and may required for private company but not required for non-for-profit. It is required to company even if the company has 1 reporting segment. (Segment reporting is required for publicly-held entities, and is not required for privately held ones.) (segment reporting을 require하는것..세분화하게 F/S를 리포트 하는형식이다.) A division is a reportable segment if it is significant. A division is significant if it satisfies at least one of the three 10% tests. (3개 항목에서 하나라도 해당되면 된다는것) 1. Revenue is 10% or more of the combined total segment revenue (including intersegment revenue). 2. Operating profit (loss) is 10% or more of the greater of the total combined segment profit or loss. 3. Identifiable assets are 10% or more of the combined total segment assets.

Enterprise report management is a methodology that involves providing substantial information to the managers in an organization to help them make business decisions. The main goal of enterprise reporting or management reporting is to supply important timely information to managers in an effective way. An enterprise must disclose all of the following about each reportable segment if the amounts are used by the chief operating decision maker: 1) Intersegment revenues. 2) Unusual items. 3) Income tax expense. but it doesn't report Cost of goods sold. <-문제에서 cogs가 나오면 오답

A company used the percentage‐of‐completion method to account for a 4‐year construction contract. Which of the following would be used in the calculation of the income recognized in the second year? Income previously recognized: yes Progress billings to date: no This answer is correct because ASC Topic 605 suggests a cost‐to‐cost measure, which is a method of recognizing income based on costs incurred. The formula used for the calculation is Current year's profit = [(Costs to date/ Total expected cost) x Expected profit] -Profit recognized in previous periods Based on this formula, only income previously recognized is required in the calculation. Progress billings to date is an accumulation of amounts billed, and the balance in the account does not normally coincide with the costs incurred to date.

Ex: Hadley Construction Company has consistently used the percentage‐of‐completion method of recognizing income. During year 1, Hadley started work on a $3,000,000 construction contract which was completed in year 2. The accounting records provided the following data: Year 1 Year 2 Progress billings $1,100,000 $1,900,000 Costs incurred 900,000 1,800,000 Collections 700,000 2,300,000 Estimated cost to complete 1,800,000 0 How much income should Hadley have recognized in year 1? Answer: $100,000 Formula to use: [(Cumulative actual costs up to date/ Estimated remaining cost for this this) x Expected profit] -Profit recognized in previous periods Under the percentage‐of‐completion method, income is recognized based on the progress the company has made toward completion of the project. Progress is measured by comparing costs incurred with the total estimated costs of the project. In year 1 costs incurred were $900,000, and the total costs were estimated at $2,700,000 ($900,000 + $1,800,000). Therefore, the project was 1/3 ($900,000/$2,700,000) completed at 12/31/Y1. Since the estimated total income from the project is $300,000 (contract price of $3,000,000 less total estimated cost of $2,700,000), income of $100,000 (1/3 × $300,000) should have been recognized in year 1.

The first section of a cash flow statement, known as cash flow from operating activities, can be prepared using two different methods known as the direct method and the indirect method. Here we will study the indirect method to calculate cash flows from operating activities. In indirect method, the net income figure from the income statement is used to calculate the amount of net cash flow from operating activities. Since the income statement is prepared on accrual basis in which revenue is recognized when earned and not when received therefore net income does not represent the net cash flow from operating activities and it is necessary to adjust earnings before interest and tax (EBIT) for those items which effect net income although no actual cash is paid or received against them Formula The following is the indirect method formula to calculate net cash flow from operating activities: Cash Flows from Operating Activities: Net Income + Non-Cash Expenses: (Depreciation, Depletion & Amortization Expense) + Non-Operating Losses: (Loss on Sale of Non-Current Assets) − Non-Operating Gains: (Gain on Sale of Non-Current Assets) + Decrease in Current Assets: (Accounts Receivable, Prepaid Expenses, Inventory etc.) − Increase in Current Assets + Increase in Current Liabilities: (Accounts Payable, Accrued Liabilities, Income Tax Payable etc.) − Decrease in Current Liabilities = Net Cash Flow from Operating Activities The following example shows the format of the cash flows from operating activities section of cash flows statement prepared using indirect method:

Example Use the following information to calculate net cash flow from operating activities using indirect method: Net Income $7,000 Depreciation Expense 1,000 Increase in AR 4,400 Increase in Prepaid Rent 7,000 Decrease in Prepaid 1,300 Increase in Accounts Payable 14,000 Increase in Wages Payable 1,000 Decrease Income Tax Payable700 Gain on Sale of Equipment 1,800 Solution: Cash Flows from Operating Activities: Net Income $7,000 Depreciation Expense 1,000 Gain on Sale of Equipment −1,800 Increase in AR −4,400 Increase in Prepaid Rent −7,000 Decrease in Prepaid 1,300 Increase in Accounts Payable 14,000 Increase in Wages Payable 1,000 DecreaseIncome Tax Payable −700 Net Cash Flow from Operating Activities $10,400

Example#1: Macklin Co. entered into a franchise agreement with Heath Co. for an initial fee of $50,000. Macklin received $10,000 when the agreement was signed. Heath signed an 8% interest bearing-note for $40,000. The note was to be paid at a rate of $10,000 per year, starting the next year. All services were performed by Macklin and the refund period had expired. Operations started in the current year. What amount should Macklin recognize as revenue in the current year? The answer is $50000 $50,000 in revenue should be recognized. Revenue on a franchise agreement should be recognized when the franchisor has substantially performed all material services and conditions, and collectibility is reasonably assured. All services are performed and the refund period has expired. 여기서 핵심 포인트는 "All services were performed by franchisor and the refund period had expired" <- this is when you recognize total initial fee as gross revenue. Example #2 Baker Co. has a franchise restaurant business. On January 15 of the current year, Baker charged an investor a franchise fee of $65,000 for the right to operate as a franchisee of one of Baker's restaurants. A cash payment of $25,000 towards the fee was required to be paid to Baker during the current year. Four subsequent annual payments of $10,000 with a present value of $34,000 at the current market interest rate represent the balance of the fee which is expected to be collected in full. The initial cash payment is nonrefundable and no future services are required by Baker. What amount should Baker report as franchise revenue during the current year? the answer is $59,000 (25000+34000) 핵심:************************* Franchise fee revenue from the initial sale of a franchise is recognized by the franchisor only when all material services or conditions applicable to the sale have been substantially performed. Substantial performance is indicated by: (1) absence of intent to refund cash received or forgive any unpaid balance; (2) performance of substantially all initial Again, here we would recognize $25,000 + PV of remaining fees $34,000 = $59,000 as "the initial cash payment is nonrefundable and no future services are required by Baker" REMEMBER, if the franchisee pays the initial franchise fees over an extended period of time, the business would use the present value of initial franchise fees.

Example#3: On December 31, year 1, Reed, Inc. authorized Foy to operate as a franchisee for an initial franchise fee of $75,000. Of this amount, $30,000 was received upon signing the agreement and the balance, represented by a note, is due in three annual payments of $15,000 each beginning December 31, year 2. The present value on December 31, year 1, of the three annual payments appropriately discounted is $36,000. According to the agreement, the nonrefundable down payment represents a fair measure of the services already performed by Reed; however, substantial future services are required of Reed. Collectibility of the note is reasonably certain. On December 31, year 1, Reed should record "unearned franchise fees" in respect of the Foy franchise of : (*이문제는 햇갈리수 있는게, "According to the agreement, the nonrefundable down payment represents a fair measure of the services already performed by Reed; however, substantial future services are required of Reed." meaning, subsequent payment can't be recognized as revenue until substantial future services are provided.. 따라서, revenue로는 바로 record할수없지만, J/E에는 "Unearned revenue"로 표시하기.) Even the question is asking how company should record "unearned franchise fee" in 12/31/Year 1. $0 $36,000 $45,000 $75,000 answer: b Franchise fee revenue shall be recognized when all material services have been substantially performed by the franchisor (i.e., the franchisor has no remaining obligation to refund any cash received and substantially all of the initial services of the franchisor have been performed <---한글로 번역하자면, franchisor (The company that allows an individual to run a location of their business) 는받은 현금을 환불해야 할 의무가 없으며 franchisor의 최초 서비스의 대부분이 수행되었다면, 우리는 이런 상황을 "all material service have substantially performed by franchisor"라 한다. And this is whey franchise fee revenue must be recognized.) Of the initial fee of $75,000, the $30,000 down payment applies to the initial services already performed by Reed. Additionally, this amount is not refundable. Therefore, the $30,000 may be recognized as revenue in year 1. The three remaining $15,000 installments relate to substantial future services to be performed by Reed. The present value of these payments, $36,000, is recorded as unearned fees and recognized as revenue once substantial performance has occurred. Cash 30,000 Notes Receivable 45,000 Discount on NR 9,000 Franchise revenue 30,000 Unearned franchise fees 36,000 ***An unearned fee in accounting is money a business collects from a customer up front for services the company has yet to perform, such as a prepaid annual membership. If your small business collects unearned fees, you must record the fees initially as a liability on the balance sheet.****

When an intercompany sale/purchase of a fixed asset occurs, such assets remain within the consolidated group. Intercompany profits on the sale and/or acquisition of fixed assets between affiliates are eliminated in consolidation so as to reflect the carrying value of the fixed assets at cost to the consolidated group. A similar adjustment for intercompany profit is made for depreciable and nondepreciable long-lived assets. An adjustment must also be made for any depreciation recorded on the intercompany profit so that depreciation is adjusted based on cost of the asset to the consolidated entity. Example Intercompany Sale of Equipment: A parent had equipment (original purchase price $60,000, accumulated depreciation of $30,000) with a five-year remaining life. The parent sold the equipment to its subsidiary for $40,000. The parent's journal entry for the sale was: Dr. Cash 40,000 Dr. Accumulated depreciation 30,000 Cr. Equipment 60,000 Cr. Gain on sale of equipment 10,000 The subsidiary recorded its purchase as: Dr. Equipment 40,000 Cr. Cash 40,000 The subsidiary will record depreciation expense each year at $8,000 ($40,000/5 years). The gain on sale of $10,000 must be eliminated along with the difference in depreciation of $2,000 ($8,000 - $6,000). The $6,000 in depreciation is based on a $30,000 carrying value. The elimination entry is: Dr. Gain on sale of equipment 10,000 Dr. Accumulated depreciation 2,000 Cr. Depreciation expense 2,000 Cr. Equipment 10,000 The depreciation adjustment will be required at each year-end over the life of the asset. In subsequent years, the debit to gain on sale instead will be to retained earnings for an amount that is reduced by the accumulated excess amortization. An affiliate experiencing an intercompany profit on the sale of a long-lived asset to another affiliate may have to pay income taxes on that gain. In this instance, the intercompany profit on the sale should be reduced by the tax effect in making the consolidated adjusting entry.

Example: Dunn Corp. owns 100% of Grey Corp.'s common stock. On January 2, year 2, Dunn sold to Grey for $40,000 machinery with a carrying amount of $30,000. Grey is depreciating the acquired machinery over a 5‐year life by the straight‐line method. The net adjustments to compute year 2 and year 3 consolidated income before income tax would be an increase (decrease) of? Answer: Year 2 Year 3 $ (8,000) $ 2,000 When computing consolidated income, the objective is to restate the accounts as if the intercompany transactions had not occurred. In year 2, the gain on the sale of the equipment ($40,000 - $30,000 = $10,000) must be eliminated, since the consolidated entity has not realized any gain. Also in year 2, some depreciation expense must be eliminated because the subsidiary computed depreciation based on a cost of $40,000 rather than on the original carrying amount of $30,000. Additional depreciation of $2,000 [($40,000 - $30,000) / 5] must be eliminated. Therefore, year 2 consolidated income must be adjusted downward by $8,000 ($10,000 - $2,000). Year 3 consolidated income must also be adjusted for the additional depreciation, so it must be adjusted upward by $2,000.

when government makes F/S, they usually report 2 level. 1) Fund ->for each major funds, each separate F/S must be made. for minor funds, you just need sum of all minor funds and add that sum to F/S...dont need to create each F/S for minors. (for funds, depend on the type of fund, you decide if you need to use either economic resources & full accrual method or current financial resource & modified accrual method) 2) Government wide- this is consoldiated all the funds and making one F/S. (For this type, we use "full accrual method" and use "economic resource" method. This means, "governmental fund"같은경우, current financial resource로 b/s를 작성하고, "modified accrual"로 I/S를 작성해서, 나중에 government wide로 F/S를 consolidate할때, 다시 "full accrual method" and use "economic resource" method로 바꿔져야한다. 이러한 process를 '"reconciliation"이라고 불른다) 여기서 명심할께, governemend wide consolidation안에 fidicuary fund는 포함 안됨. because it's not gov's own fund.

Government wide 는 두개로 나눠져있다. 1) statement of net position 2) statement of cash flow

There are 3 types of governmental accounting funds. 1) Government fund 2) propreitary fund 3) fiduciary fund. *Government Fund includes: * (Gov fund는 말그대로, Fund를 받는것이다. gov가 별다른 service를 제공하지 않아도, funding을 받는것이라고 생각하자) A) General fund-accounts for those resources not required to be in another fund. (밑에 fund에 해당하지 않는건 대부분 general fund라고 생각하기) <- example: property tax B) Special Revenue Fund- specific revenue sources that are legally restrctied to expenditures for specific purpose (ex: motor fuel tax, hotel tax) C) Capital project Fund- accounts for financial resources to be used for acquistion or construction of major capital facilities. D) Debt Service Fund- accounts for accumulation of resources for, and payment of general long-term debt principal and interest. E) Permanent Fund- resources that are legally restricted to extent that only earnings and not principal may be used to support gov program **Propreitary fund includes: (gov 부서가 service를 제공하면, 상대방으로부터 user charge fee를 받고, 그게 revenue가 되는 fund를 뜻한다. It is total opposite to government fund)** A) Internal Service Fund- Internal Service Funds are used to account for activities in which one department in a governmental unit provides services exclusively to other departments within the governmental unit, generally on a not‐for‐profit (cost‐reimbursement) basis. ex: motor pools. gov 차량을 하나 사면, 이차가 놀고있을때 다른 entity가 차량을 빌려가서 쓰고 돈을 준다.. 그럼 여러대를 안사도 되고 같이 돌려쓰면 좋고.. B) Enterprise Fund- Government가 시민들한테 서비스를 제공하고 받는돈. 주로, utility, water, highway, governmental hospital, waste 쓰래기 처리 등등으로 시민들한테 서비스를 제공하고, 거기서 버는돈은 revenue로 인식하기 . ** Fiduciary fund includes: A) Pension and (other employee benefit trust funds)- 정부 직원들의 pension을 관리하는것. city 직원들의 pension을 state에서 관리하는것. B) Investment trust fund- city에서 돈이 모였다. 그럼 state가 참견을 한다. 그 짜자란거 다 여기로 모와서 같이 크게 invest하자. 그럼 그게 investment trust fund가 된다. c) Private-purpose trust fund- 는 자기 정부에 쓰여지는돈이 아니고 다른 정부나 private org에 쓰이는 돈 (ex: local homeless shelter에다 쓰일 돈) D) Agency fund- 정부가 돈을 받긴 받았는데, 돈을 어따가 쓸지에 대한 power가 없으면, 정부의 agency fund로 들어온다. (ex: sales tax. board of equalization collects sales tax on behalf of city and county...acting as agent)

Governmental Accounting Fund는 결국 3 가지로 나눠진다 (1) gov fund, (2) proprietary fund (3) fiduciary fund. at the end of year, government fund and proprietary fund gets combined and reported on F/S but fiduciary fund doesn't get combined because this is not the fund that gov owns. A fiduciary fund is an account with funds from assets that the government holds as a trustee and that it cannot use to fund its own programs. Fiduciary funds include pension and employee benefit trust funds, agency funds, external investment trust funds and private-purpose trust funds Out of these 3 types of fund, (1) government fund, (2) propretarship (3) fidiciuary. Only government fund follows "current financial resource and adopts modified accrual." This is refers to Governmental accounting's BS which only includes current liab and current asset. This focus on fiscal accountability. (Meaning it is like cash basis. Report earning when it is cash is actually received or when earning is collected within 60 days of end of fiscal year end and report expense when cash is actually paid. expenditures generally should be recognized in the accounting period in which the fund liability is incurred, if measurable)***government fund uses modified accrual basis which uses concept of "expenditure" rather than expense" propretarship & fidicuary fund같은 경우는, full accrual method를 쓰는데, 이경우에는 expense라고 인식한다. expenditure가 아니고 Propretarship and Fidicuary funds use full accrual basis accounting. (Meaning report earnings when it is earned and report expense is incurred) Under the modified accrual basis, revenues are recognized in the period in which they are both measurable and available, while under the full accrual basis they are recognized when measurable and earned.

Sterling Corporation prepares its financial statements in accordance with IFRS. Sterling paid $10,000 of interest during the year. Sterling must report these finance costs on the statement of cash flows: a) In operating activities. b) Either in operating activities or financing activities. c) Only in financing activities. d) In investing activities or financing activities.

IFRS permits finance costs (interest expense) to be reported in either in the operating or financing section of the statement of cash flows. However, once it is disclosed in a particular section, it must be reported on a consistent basis. In GAAP, Interest payment always need to be reported to "Operating activities". but IFRS allows interest payment to be "Either Operating activities of Financing activities"

Cost Method & Equity Method - when you make investment in other company

If your company invests in another firm, whether it's to form a business alliance or just to make a profit, that investment must be accounted for on your balance sheet. Accounting rules dictate the method to use to report the investment. The cost method and the equity method apply when your ownership interest in the other company is less than a controlling stake. Level of Influence The method a company must use to account for a less-than-controlling stake in another business depends on how much of that other business it own. If it owns less than 20% it must use cost method. If it owns more than 20%, it must use equity method Cost method: If you receive any dividends from the investment, those dividends get treated as revenue. Equity Method: With the equity method, the balance-sheet value of the investment changes according to the net income (the profit) of the "owned" company. Say your company owns 30 percent of a firm, and that firm reports net income of $100,000. You would increase the balance-sheet value of your investment by $30,000 -- 30 percent of $100,000 -- and report the gain as revenue on your income statement. If the firm had a net loss, you'd decrease the value of the investment by your share of the loss and report the decline as an expense. Finally, dividends from the stock are considered a return of invested capital, not revenue. You would decrease the value of the investment by the amount of any dividends received.

Impairment of loss- According to U.S. accounting rules (US GAAP), the value of an asset is impaired when the sum of estimated future cash flow from that asset is less than the book value of the asset. At this point an impairment loss should be recognized, which is done by taking the difference between the fair market value and the book value and recording this amount as the loss. This basically records the asset as if it were being acquired brand new at its fair market value, recording this as its new book value. This is a common occurrence for goodwill where a company will purchase another company for more than the value of the net assets of the target company. Under US GAAP, goodwill is tested annually for impairment

Must measure the fair value of balance sheet items such as asset impairments, business combinations, and goodwill. Ex: On April 1, year 2, Hart, Inc. paid $1,700,000 for all the issued and outstanding common stock of Ray Corp. On that date the costs and fair values of Ray's recorded assets and liabilities were as follows: Cost Fair value Cash $ 160,000 $ 160,000 Inventory 480,000 460,000 PP&E (net) 980,000 1,505,000 Liabilities (360,000) (360,000) Net assets $1,260,000 $1,300,000 In Hart's March 31, year 3 balance sheet, what is the amount of goodwill that should be reported as a result of this business combination, assuming that goodwill is not impaired? a) $390,000 b) $400,000 c) $429,000 d) $440,000 answer: b *여기서 중요한것은, if my acquistion cost is greater than FMV of acquired cost then it is goodwill.. 내가 손해 본거라고 생각이 들어진다.. (in this case, $1,700,000 was paid to acquire the asset but FMV of these acquired asset during the acquisition time is $1,300,000, meaning $400,000 was 손실.. we classify this as goodwill.. but let's say, FMV of acquired asset was higher than what i actually paid for acuqisition. let's say FMV of asset was $1,500,000 and i only paid $1,200,000. then $300,000 is considered as bargain purchase.. 시세보다 더 싸게 산것으로 인식되며, 이건, revenue로 인식된다. goodwill이랑 bargain purchase랑 햇갈리지 말기) The excess of the cost of the investment over the FV of the net identifiable assets is allocated to goodwill, $400,000 ($1,700,000 ‐1,300,000). If it is not impaired it remains at $400,000 at year‐end. **여기서 TRICK은 question에서 보여주는 balance sheet은 Hart corp께 아니고, Hart copr가 사들인 Ray corp's 장부 Balance sheet이다. **여기서 또 TRICK은 Asset은 언제나 positive, liabilty는 언제나 minus로 표시해서 계산하기. Fair value Cash $ 160,000 Inventory 460,000 PP&E (net) 1,505,000 Liabilities (360,000) **NET : $1,300,000 NOTICE, net amount is sum of Asset minus liability. 바보처럼, FMV ASSET + LIABLITY 더한것같고 Acquired amount를 빼지말기. ******핵심! If acquisition cost is greater than FV of acquired asset during the acquisition period, it is considered as 손실, and we call it as "goodwill" however, if acquisition cost is lower than FV of acquired asset cost during the acquisition period, we call this as revenue 이익, "bargain purchase". Be careful with goodwill vs bargain purchase

1) If the functional currency is the same as the presentation currency, gains or losses are reported in profit and loss for the period. (if parent company use USD $ and its subsidiary company in different country also use USD $ to prepare its F/S, and if gain and loss founded, it is reported straightly to Income statement, rather than Other comprehensive Income. 2) If the functional currency is not the same as the presentation currency, gains or losses are reported in OCI. (other comprehensive income)

Nonmonetary items are those assets and liabilities appearing on the balance sheet that are not cash, or cannot be readily converted into cash. Generally, nonmonetary assets include fixed assets such as property, plant and equipment as well as intangible items such as goodwil Monetary items refers to those assets and liabilities whose value is measured and stated in cash. Examples of monetary assets include cash, accounts receivable, notes receivable, and investments.

When a capital project is financed entirely from a single bond issue, and the proceeds of the bond issue equal the par value of the bonds, the Capital Projects Fund would record this transaction by debiting cash and crediting a) Other Financing Sources—Bond Issue Proceeds. b) Fund Balance. c) Appropriations Control. d) Bonds Payable. Answer: A: because per GASB Codification Section 1500, the proceeds of long‐term debt issues not recorded as fund liabilities (for example, proceeds of bonds or notes expended through capital projects funds) should be reported as bond issue proceeds.

Other Financing source- revenue가 아닌건 다 여기로 들어온다. OPERATING 목적인 transfer in and bond proceed. 이뜻은 돈이 STRICT purpose로 들어왔다고해도, 더이상 갑자기 purpose가 없어지게 되면, general fund가 될수가 있다..그러다가, 또 그돈이 갑자기 capital fund에 다리 지을때 쓰일수도 있다.. 그래서 왔다가 갔다 하는돈을 애기한다. 시험문제에서 조심해야할게, (ex: if the same fund was transferred within LA CITY, meaning same entity, this operating transfers in and treat as other financing source) but (ex: if California trasnfer fund to LA City (different entity to different entity) this is called "GRANTS" and it becomes revenue) revenue에 나온 grants는 fed나 state가 local애들한테 돈을 주는것. (ex: california 주랑 FED가 OC에 다리공사를 위해 돈은 좀 주는것..)

Pension expense

Pension expense is the amount that a business charges to expense in relation to its liabilities for pensions payable to employees. The amount of this expense varies, depending upon whether the underlying pension is a defined benefit plan or a defined contribution plan. The characteristics of these plan types are as follows Defined benefit plan- an employee knows the terms of the benefit that he or she will receive upon retirement. (ex: 401k) Under this plan, the employer provides a pre-determined periodic payment to employees after they retire. The amount of this future payment depends upon a number of future events, such as estimates of employee lifespan, how long current employees will continue to work for the company, and the pay level of employees just prior to their retirement. In essence, the accounting for defined benefit plans revolves around the estimation of the future payments to be made, and recognizing the related expense in the periods in which employees are rendering the services that qualify them to receive payments in the future under the terms of the plan. Here is a summary of the relevant costs associated with a defined benefit pension plan, which sum to the net periodic pension cost that is recognized in each accounting period: Service cost + interest cost +actual return on plan asset + Amortization of prior service costs + Gain or loss= Net periodic pension cost Service Cost Explanation: This is the actuarial present value of benefits related to services rendered during the current reporting period. The cost includes an estimate of the future compensation levels of employees from which benefit payments will be derived. Interest cost Explanation: This is the interest on the projected benefit obligation. It is a financial item, rather than a cost related to employee compensation. Actual return on plan assets Explain: This is the difference between the fair values of beginning and ending plan assets, adjusted for contributions and benefit payments. It is a financial item, rather than a cost related to employee compensation. (The term return on plan assets refers to the dividends, interest, and capital gains generated by assets held in a company's pension fund. Accounting rules require companies to differentiate between the expected and actual return on their plan's assets.) Amortization of prior service costs: When an employer issues a plan amendment, it may contain increases in benefits that are based on services rendered by employees in prior periods. If so, the cost of these additional benefits are amortized over the future periods in which those employees active on the amendment date are expected to receive benefits. Gain or loss Explanation: This is the gain or loss resulting from a change in the value of a projected benefit obligation from changes in assumptions, or changes in the value of plan assets.

The retirement of long‐term debt by the issuance of common stock should be presented in a statement of cash flows as a Financing activity Investing activity No No

Per ASC Topic 230, financing and investing activities which have no effect on cash flows shall be shown either in a separate schedule of noncash financing and investing activities or in narrative form in the footnotes, not in the body of the statement. Noncash investing and financing activities are significant investing and financing activities that do not "directly affect cash". These activities involve only long-term assets, long-term liabilities, and stockholders' equity, and they appear at the bottom of the statement of cash flows. Examples of noncash investing and financing activities include issuance of common stock to retire long-term debt, purchase of equipment with a note payable, and issuance of stock to acquire land. "핵심 포인트" -> Statement of cash flows reports only those operating, investing and financing activities that affect cash or cash equivalents. However, some non-cash investing and financing activities may be much important for the users of financial statements because they may have a significant impact on the current and future performance in terms of revenues, profits and the ability of the entity to generate positive cash flows. Therefore, both IFRS and US GAAP require companies to disclose all significant non-cash investing and financing activities either at the bottom of the statement of cash flows as a footnote or in the notes to the financial statements. Examples: Some examples of non-cash investing and financing activities that may become significant for the users of financial statements are given below: 1) Issuance of stock to retire a debt 2) Purchase of an asset by issuing stock, bonds or a note payable. 3) Exchange of non-cash assets. 4) Conversion of debt to common stock. 5) Conversion of preferred to common stock

The present value of the minimum lease payments should be used by the lessee in the determination of a(n) Capital lease liability: Yes Operating lease liability: No

Per ASC Topic 840, the present value of the minimum lease payments should be used to determine the liability under a capital lease. Under an operating lease, a liability arises when rent expense is recorded but has not been paid. Furthermore, it is recorded at the actual amount of cash to be paid, not its present value.

What is the primary purpose of the statement of activities of a nongovernmental not‐for‐profit organization? To report the change in net assets for the period. The change in net assets is the rough equivalent of the net profit figure on an income statement; it is used by nonprofit entities. The measure reveals the change in assets derived from revenues, expenses, and any releases on the restrictions of assets during the period. not-for-profit에서 중요시 여기는 , "change in net asset"은 income statement랑 매우 흡수하다.

Statement of Net Position- like B/S Statement of Activities- like I/S

The following trial balance of Trey Co. on December 31, 20X1, has been adjusted except for income tax expense. Debit Credit ----------- ---------- Cash $ 550,000 Accounts receivable (net) 1,650,000 Prepaid taxes 300,000 Accounts payable $ 120,000 Common stock 500,000 Additional paid-in capital 680,000 Retained earnings 630,000 Foreign currency translation adjustment 430,000 Revenues 3,600,000 Expenses 2,600,000 ---------- ---------- $5,530,000 $5,530,000 ========== ========== Additional Information During 20X1, estimated tax payments of $300,000 were charged to prepaid taxes. Trey has not yet recorded income tax expense. There were no differences between financial statement and income tax income. Trey's tax rate is 30%. Included in accounts receivable is $500,000 due from a customer. Special terms granted to this customer require payment in equal semiannual installments of $125,000 every April 1 and October 1. In Trey's December 31, 20X1, balance sheet (statement of financial position), which amount should be reported as total current assets? A. $1,950,000 B. $2,200,000 C. $2,250,000 D. $2,500,000

The correct answer is A. Cash $ 550,000 Accounts receivable $1,650,000 - (2 x $125,000) 1,400,000 ---------- Total current assets $1,950,000 ========== Note Current assets consist of cash and other assets reasonably expected to be realized in cash or sold or consumed in operations within one year or an operating cycle, whichever is longer. It is important to note that Trey's trial balance has not been adjusted for income taxes. Since there are no temporary differences, Trey's taxable income (for tax purposes) and income before income taxes (for financial reporting purposes) is $1,000,000 (revenues of $3,600,000 less expenses of $2,600,000). Trey's income tax for 20X1 is $300,000 ($1,000,000 × 30%). Therefore, Trey will have no "prepaid taxes" once the trial balance is adjusted for income taxes. The balance in the unadjusted Prepaid Taxes account will be transferred to the Income Tax Expense account. Accounts receivable reported in current assets should include only the installments due within the next year, which includes the $125,000 installments due on April 1, 20X1, and October 1, 20X1. The other two installments should not be included in current assets; rather, they should be included in noncurrent assets as a long-term receivable. Thus, the amount of accounts receivable to be included in current assets is $1,650,000 less $250,000 (i.e., $125,000 × 2 installments due after 20X1), or $1,400,000.

The Town of Starbuck's general fund received a notice of a federal grant award for an expenditure-driven (reimbursement) grant in the amount of $2,000,000. Included with the notice was an advance of $1,000,000. During the year, the Town incurred $1,400,000 of program expenditures of which $800,000 were considered eligible qualifying expenditures. No additional money had been received from the grantor during the year. What would be the amount of intergovernmental receivables reported by the general fund at the close of the fiscal year? A. $0 B. $150,000 C. $600,000 D. $750,000

The correct answer is A. Insofar as the amount of the advance exceeded the qualifying expenditures, the federal government has no liability to the Town at the end of the year (advance grant received $1,000,000 exceed $800,000 eligible qualifying expenditure, therefore, federal government don't have liability to pay any further.

What types of rules are generally issued by the SEC? A. The SEC issues Financial Reporting Releases that usually agree with U.S. GAAP. B. The SEC issues U.S. GAAP. C. The SEC does not issue rules. D. None of the answer choices are correct.

The correct answer is A. The Securities and Exchange Commission (SEC) is a governmental entity created to protect the interest of investors by ensuring full and adequate disclosure by publicly traded companies. Although the SEC has the authority to establish standards, it has generally deferred to the Financial Accounting Standards Board (FASB) or its predecessors to generate U.S. accounting standards. The SEC does issue its own rules in the form of Financial Reporting Releases, which generally agree with U.S. GAAP. (SEC doesn't issue it's own rules.) 증권 거래위원회 (SEC)는 상장 기업이 완전하고 적절하게 공개함으로써 투자자의 이익을 보호하기 위해 설립 된 정부 기관입니다. SEC는 기준을 설정할 권한을 가지고 있지만, 일반적으로 미국 회계 기준을 창출하기 위해 재무 회계 기준위원회 (FASB) 또는 그 전임 위원에게 이관합니다. SEC는 일반적으로 미국 GAAP에 동의하는 재무보고 자료 형태로 자체 규칙을 발행합니다. SEC gives rights to FASB for establishing the accounting rules. (SEC는 그냥 U.S. GAAP의 동의하는 그런 입장. SEC's main duty is to protect interest of investors by making sUure publicly traded company to fully disclose it F/S)

Which of the following assets of a nongovernmental not-for-profit charitable organization must be depreciated? A. A freezer costing $150,000 for storing food for the soup kitchen B. Building costs of $500,000 for construction in progress for senior citizen housing C. Land valued at $1,000,000 being used as the site of the new senior citizen home D. A bulk purchase of $20,000 of linens for its nursing home

The correct answer is A. The freezer is a long-lived, tangible asset currently being used in operations. It also has a limited life. Assets not being used in operations and land are not depreciated. The linens are not material enough in amount and, if used daily, may not have a useful life in excess of a year or slightly more. FASB ASC 958-360-35-1 provides that a not-for-profit entity "shall recognize the cost of using up the future economic benefits or service potentials of its long-lived tangible assets—depreciation *depreciate할수있는 asset은 어느정도의 useful life가 있고, tangible 이여야한다. land는 당연히 depreciate 안되서 제외한다. Assets not being "CURRENTLY" used in operations should be excluded from depreciation. 따라서 현제 빌딩 공사가 한참인 construction은 depreciation 이 해당안됨. 현재 비지니스에서 씌여질게 아니니깐... The linens (천, 침대 천, 식탁보) 같은건, 매일매일 쓰는거지, useful life asset이 있는게 아니라서 제외하기.

At the end of the year, Town City's general fund included, among the assets, an inventory of office supplies worth $1,500 and prepaid fire insurance covering the first two months of the subsequent year of $600. As a result of these assets, the fund balance presented in the balance sheet of the general fund would include which of the following? A. Fund balance—nonspendable: $2,100 B. Fund balance—nonspendable: $1,500; Fund balance—committed: $600 C. Fund balance—assigned: $1,500; Fund balance—restricted: $600 D. Fund balance—restricted: $2,100

The correct answer is A. The portion of fund balance that reflects equity for amounts that cannot be spent because they are not in spendable form would be termed "nonspendable." Both supplies inventory and prepaid expenses are not spendable. Committed fund balance would result from an action of the highest level of the government and assigned fund balance would reflect action by a government designee. Restrictions of fund balance would reflect specific purposes externally imposed by creditors, grantors, or enabling legislation. Nonspendable meaning: Certain resources of governmental funds are not monetary in nature (monetary 란, cash or cash equivalent로 알고있다) and will not lead to a collection of monetary resources in the future. These assets, usually prepaid items and inventories, are considered nonspendable. At year-end, a category of Fund balance is set aside equaling the total of such assets. Fund balance is the fund equity or the difference between the asset and liability accounts of a governmental fund. Fund balance should be reported in specific categories as circumstances require: Nonspendable reflects the value of prepaid or inventory items that are not monetary and will not support spending or the value of monetary resources legally contracted to be kept intact. Restricted reflects the value of assets whose use has previously been designated by an external party such as a grant. Committed reflects the value of assets whose use has previously been designated by the highest level of government, typically the legislature or city council. Assigned reflects the value of assets whose use has been previously designated by management without action by the highest level of authority in the government. The amount of resources to be set aside to cover encumbrances at year-end is usually designated either committed or assigned. Unassigned reflects the value of General fund assets whose use has not been previously designated. Only the General Fund has an Unassigned category of Fund balance.

On December 30, 20X1, Rafferty Corp. leased equipment under a capital lease. Annual lease payments of $20,000 are due December 31 for 10 years. The equipment's useful life is 10 years, and the interest rate implicit in the lease is 10%. The capital lease obligation was recorded on December 30, 20X1, at $135,000, and the first lease payment was made on that date. What amount should Rafferty include in current liabilities for this capital lease in its December 31, 20X1, balance sheet? Incorrect A. $6,500 B. $8,500 C. $11,500 D. $20,000

The correct answer is B. Initial lease obligation on December 31, 20X1 $135,000 Less payment made on December 31, 20X1 - 20,000 -------- Lease obligation during 20X2 $115,000 ======== Portion of December 31, 20X2, payment that is interest = rate x obligation x time = 10% x $115,000 x 1 = $11,500 Portion of December 31, 20X2, payment that is related to lease obligation = payment - interest portion = $20,000 - $11,500 = $8,500 This amount is a current liability since it is payable within the current period. The remaining lease obligation is noncurrent.

Which of the following must be done when an entity is required to use the liquidation basis of accounting? A. Measure assets at fair value B. Recognize previously unrecognized items that the entity expects to sell C. Recognize costs and income expected to be incurred or earned through liquidation only after the liquidation is complete D. Recognize the costs of liquidation only after the liquidation is complete

The correct answer is B. "Recognize previously unrecognized items that the entity expects to sell" is correct. Assets must be measured at the estimated amount of cash expected to be collected. Costs and income should be recognized when it becomes apparent that liquidation is imminent. Liquidation is a form of relief granted under the uniform bankruptcy laws (Chapter 7) that results in the distribution of the debtor's nonexempt, unsecured assets to creditors and a final discharge of the individual debtor from its obligations. Liquidation is the ultimate remedy, the remedy that embodies finality, providing the debtor with a "fresh start." All entities, except governmental units, family farms, railroads, insurance companies, and financial institutions, may use Chapter 7. An entity can liquidate voluntarily or involuntarily, and Chapter 7 can be converted to a Chapter 11, 12, or 13 proceeding.

Dunbarn Co. had the following activities during the year: Purchase of inventory $120,000 Purchase of equipment 80,000 Purchase of available-for-sale securities 60,000 Purchase of treasury stock 70,000 Issuance of common stock 150,000 What amount should Dunbarn report as cash provided (used) by investing activities in its statement of cash flows for the year? A. $(120,000) B. $(140,000) C. $(210,000) D. $150,000

The correct answer is B. A statement of cash flows reflects an entity's cash receipts and cash payments classified by major uses (i.e., operating, investing, and financing activities). The investing activities section shows positive and negative cash flows for transactions involving assets that are not held for resale (i.e., inventory), such as investments in debt and equity securities; plant, property, and equipment; and intangible assets. Dunbarn should report $(140,000) in investing activities related to the purchase (i.e., use of cash) of equipment and AFS (available for sale) securities. Inventory transactions are classified as operating activities; treasury stock and common stock transactions are classified as financing activities.

According to the FASB's conceptual framework, asset valuation accounts are: A. assets. B. neither assets nor liabilities. C. part of stockholders' equity. D. liabilities.

The correct answer is B. A valuation account is neither an asset nor a liability. The elements of the financial statements such as assets and liabilities are described in SFAC 6. Valuation accounts are discussed in paragraph 6.34: A separate item that reduces or increases the carrying amount of an asset is sometimes found in financial statements. For example, an estimate of uncollectible amounts reduces receivables to the amount expected to be collected, or a premium on a bond receivable increases the receivable to its cost or present value. Those "valuation accounts" are part of the related assets and are neither assets in their own right nor liabilities.

Reed Co.'s 20X1 statement of cash flows reported cash provided from operating activities of $400,000. For 20X1, depreciation of equipment was $190,000, impairment of goodwill was $5,000, and dividends paid on common stock were $100,000. In Reed's 20X1 statement of cash flows, what amount was reported as net income? A. $105,000 B. $205,000 C. $305,000 D. $595,000

The correct answer is B. Dividends paid are reported as financing activities. The reconciliation of net income and cash provided by operating activities would reflect both of the other items as they are noncash expenses and losses. Net income + Depreciation expense + Goodwill impairment loss = Cash provided by operating activities X + $190,000 + $5,000 = $400,000 X = $205,000

Which is the most appropriate financial statement to use to determine if a company obtained financing during a year by issuing debt or equity securities? A. Balance sheet B. Statement of cash flows C. Statement of changes in stockholders' equity D. Income statement

The correct answer is B. FASB ASC 230-10-45-1 requires cash flows to be presented in three categories in the statement of cash flows—operating, investing, and financing. Paragraph 6 states: "A statement of cash flows should report the cash effects during a period of an enterprise's operations, its investing transactions, and its financing transactions." (Emphasis added) Hence, one of the key purposes of the statement of cash flows is to disclose how a business financed its operations.

Jen has been employed by Komp, Inc., since February 1, 20X0. Jen is covered by Komp's Section 401(k) deferred compensation plan. Jen's contributions have been 10% of salaries. Komp has made matching contributions of 5%. Jen's salaries were $21,000 in 20X0, $23,000 in 20X1, and $26,000 in 20X2. Employer contributions vest after an employee completes three years of continuous employment. The balance in Jen's 401(k) account was $11,700 at December 31, 20X2, which included earnings of $1,200 on Jen's contributions. What amount should be reported for Jen's vested interest in the 401(k) plan in Jen's December 31, 20X2, personal statement of financial condition? A. $11,700 B. $8,200 C. $7,000 D. $1,200

The correct answer is B. FASB ASC 274-10-35-11 states that nonforfeitable rights to receive future sums that have all the following characteristics shall be presented as assets at their discounted amounts: The rights are for fixed or determinable amounts. The rights are not contingent on the holder's life expectancy or the occurrence of a particular event, such as disability or death. The rights do not require future performance of service by the holder. Since the employer's contributions have not vested and are forfeitable (Jen will not complete three years of employment until February 1, 20X3), the current value of Jen's 401(k) plan is: $7,000 Jen's contributions 10% ($21,000 + $23,000 + $26,000) + 1,200 Earnings on Jen's contributions ------ $8,200 Total current value ======

Blue City's finance staff is analyzing expenses for presentation on the government-wide statement of activities. The City has some notes payable that clearly benefit specific governmental functions, but most of the city's long-term debt consists of general long-term liabilities. How should the interest on the debt be reported? A. Combine all interest payments together and report on a separate line as a direct expense. B. Report the interest from the notes payable benefiting specific functions as a direct expense of those functions and report the remaining interest as a separate line as an indirect expense. C. Combine all interest payments together and report separately at the bottom of the statement. D. Report the interest from the notes payable benefiting specific functions as a direct expense of those functions and include the remaining interest in the cost allocations of indirect expense. .

The correct answer is B. Interest that clearly derives from borrowing that is essential to support a program should be reported as a direct expense of that program, and interest that does not qualify as a direct expense should be reported as a separate line. The two types of interest should not be combined. Interest is reported as a component of other functions or a separate line, but not at the bottom of the statement. Although interest on long-term debt is usually an indirect expense, it should be reported as a separate line and not allocated.

The last member of Cross Corners' founding family, Ezra Cross, left his collection of early American art to the City for permanent display in city office buildings. The fair value of the collection at donation was $2,000,000. The Cross Corners city council formally accepted the collection and set a policy that the art would (a) be held for public exhibition, (b) be protected, cared for, and kept unencumbered, and (c) not be sold except for the purposes of acquiring different items for the collection. The city council agreed that the collection should not be capitalized for financial reporting purposes. The city's maximum depreciation horizon for capital assets is 40 years. How would the collection affect the government-wide financial records in the first year? A. Because the art will be displayed in general government buildings, the governmental activities would show revenue from donations of $2,000,000. B. Because the art will be displayed in general government buildings, the governmental activities would show revenue from donations of $2,000,000 and an expense of $2,000,000. C. Because the art will be displayed in general government buildings, the governmental activities would show revenue from donations of $2,000,000 and expenses of $2,050,000, as well as a capital asset of $2,000,000. D. There would be no impact on the governmental activities section of the government-wide financial statements.

The correct answer is B. The city council's formal policy for the donated collection of American art meets the three criteria that permit the city to avoid capitalization of the art collection. If the collection is not capitalized, it would not be listed among the capital assets used for governmental activities, and depreciation would not be reported. In the year of the donation, a recipient government should recognize a revenue as well as an expense in the same amount.

Martin Co. had net income of $70,000 during the year. Depreciation expense was $10,000. The following information is available: Accounts receivable increase $20,000 Equipment gain on sale increase 10,000 Nontrade notes payable increase 50,000 Prepaid insurance increase 40,000 Accounts payable increase 30,000 What amount should Martin report as net cash provided by operating activities in its statement of cash flows for the year? A. $0 B. $40,000 C. $50,000 D. $100,000

The correct answer is B. The most common method for presenting cash flows from operating activities is the indirect method, in which net income is adjusted for noncash items that have been included in its determination, as well as the change in current assets and current liabilities. Martin should report $40,000 as net cash provided by operating activities, as follows: Net income $70,000 + Depreciation expense 10,000 - Accounts receivable increase (20,000) - Equipment gain on sale increase (10,000) - Prepaid insurance increase (40,000) + Accounts payable increase 30,000 $40,000 여기서 중요한것은, 2가지다. (1) operating activity의 cash flow의 대해서 물어보는것이니깐, we have to exclude nontrade notes payable because this is financing activity. The nontrade N/P (notes payable) is a financing activity. (2) cash flow의 의미는 정말로, cash가 만져지는것만 카운트 된다. here, we add back depreciation expense because depreciation is just reducing value of asset because of time has passed. (우리눈에 실제로 cash payment가 왔다갔다한게 아니다) here, we deduct A/R increase. (accrual method에서는 A/R increase된만큼 sale를 increase해서, 우리의 net income이 크게 보여지게 하지만, 실제로, cash income이 나한테 들어온게 아니니깐 빼주기 here, we deduct equipment gain (sales proceed 로 실제로 돈이 들어오면, cash flow로 인정되지만 단순히 sales proceed - adjusted carrying basis로 생긴 profit gain은 우리가 cash flow에서 제외 해아한다. here, we deduct prepaid expense. (accrual method에서는 이게 prepaid 라서 expense로 인식일 하지 않았지만, 실제로 돈이 나간거니깐, cash flow에서는 expense로 net income에서 빼줘야한다 here, we add the increased in accounts payable because, accrual method에서는 , 실제로 돈이 나가지 않았지만, A/P에 accrual 한만큼 돈이 net income계산시에 빠져나가니깐, cash flow add하기

Each of the following is a component of the changes in the net assets available for benefits of a defined benefit pension plan trust, except: A. the net change in fair value of each significant class of investments. B. the net change in the actuarial present value of accumulated plan benefits. C. contributions from the employer and participants. D. benefits paid to participants.

The correct answer is B. The statement of changes in net assets BPO trust must include the following: The change in fair value of each significant type of investment Investment income Contributions from employers Contributions from participants Contributions from other identified sources Benefits paid to participants Payments to insurance entities to purchase contracts Administrative expenses Only the net change in the actuarial present value of accumulated plan benefits is not included in this list.

New England Co. had cash provided by operating activities of $351,000; cash used by investing activities of $420,000; and cash provided by financing activities of $250,000. New England's cash balance was $27,000 on January 1. During the year, there was a sale of land that resulted in a gain of $25,000 and proceeds of $40,000 were received from the sale. What was New England's cash balance at the end of the year? A. $27,000 B. $40,000 C. $208,000 D. $248,000

The correct answer is C. The main point of this question is that the "cash flows focuses on the cash received" (cash flow only counts actual cash received and doesn't care about gain recognized) in the transaction, not on the gain recognized in the transaction. The amount of the gain on the sale of the land does not affect investing activities, as the $420,000 includes all money for investing. Therefore, given the other information about cash and cash flows in the problem, the ending cash balance may be computed as follows: Cash from operating activities $351,000 Cash from investing activities: Cash used for investing activities (420,000) Cash provided by financing activities 250,000 --------- Net increase in cash 181,000 Beginning cash balance 27,000 --------- Ending cash balance $208,000

Halderman County levies an imposed nonexchange form of tax in the year prior to the year of its intended collection and use. An enforceable legal claim does not arise until the period after the period of its intended collection and use. The following facts apply: On September 1, 20X1, the county levied $2 million of tax for FY 20X2—50% of the tax is due on January 15, 20X2, and the remainder is due July 15, 20X2. It is estimated 5% of the levy will be uncollectible. An enforceable legal claim for the September 1, 20X1, levy does not attach until January 15, 20X3. It is estimated 90% of the September 1, 20X1, levy will be collected during the period January 1, 20X2, through February 28, 20X3. The balance will be collected at a later date, or go uncollected. The County uses an "availability period" equal to two months following the close of the fiscal year, and has a fiscal year-end of December 31. How much revenue would be recognized at the entity-wide level for FY 20X2? A. $1,500,000 B. $1,800,000 C. $1,900,000 D. $2,000,000

The correct answer is C. At the entity-wide level Halderman County accounts for the revenue using accrual-based accounting, net of any allowance for doubtful accounts in the period for which the taxes were levied ($2,000,000 × 0.95 = $1,900,000). The availability requirement does not apply. remember, it is acceptable for governmental fund to allow AR that has been collected within 60days after the year end. 따라서, in this case, it is okay to include A/R collected on Feb. 29th, 2003 when Sales recognized and A/R was first posted it on 9/1/2001. The question clearly told us that 5% of tax levied will not be collectible. 2/29/2003까지 돈이 다 수금이 됬다. 단지 5%에 해당되는 AR은 수금 불가를 판정받아서, GROSS A/R $2,000,000 X 95% = $1,900,000만 수금이 됬다고 보면 된다.

XYZ Corporation pays an insurance premium of $5,000 on a whole life policy on the life of its president. The cash surrender value of the policy increases from $22,000 to $25,000 during the period covered. Which of the following is included in the entry to record the payment of the premium? A. Cash is debited for $50,000. B. Life insurance expense is credited for $3,000. C. Life insurance expense is debited for $2,000. D. Cash surrender value of life insurance is debited for $5,000

The correct answer is C. Enterprises often carry life insurance policies on the lives of key officers and employees. If the enterprise is the beneficiary, the cash surrender value of the policy is an asset of the enterprise. The amount to be charged to expense is the amount of such premiums paid less the increase in cash surrender value during the period. asset이니깐, debit side에 $2,000기록 하나보다

During a period when an enterprise is under the direction of a particular management, its financial statements will directly provide information about: A. both enterprise performance and management performance. B. management performance but not directly provide information about enterprise performance. C. enterprise performance but not directly provide information about management performance. D. neither enterprise performance nor management performance.

The correct answer is C. Financial statements provide direct information about enterprise performance because the primary focus of the statements is to provide information about the financial performance of that enterprise by providing information about earnings. The same cannot be said, however, in regard to management performance. The financial statements depict only indirect information concerning management performance. (Direct information related to management performance would be provided in internal managerial performance reports but not in the external financial statements.)

River City has a defined contribution pension plan. How should River report the pension plan in its financial statements? A. Amortize any transition asset over the estimated number of years of current employees' service. B. Disclose in the notes to the financial statements the amount of the pension benefit obligation and the net position available for benefits. C. Identify in the notes to financial statements the types of employees covered and the employer's and employees' obligations to contribute to the plan. D. Accrue a liability for benefit earned but not paid to plan participants.

The correct answer is C. In this question, only one response relates to a defined contribution plan rather than a defined benefit plan: that among the note disclosures required are the types of employees covered and the employer's and employees' obligations to contribute to the plan. Note Transition assets relate to the measurement of defined benefit plan assets and liabilities in the private sector (FASB ASC 715-60-35-38). Disclosure of pension benefit obligations as well as the requirement to accrue all earned benefits relate to defined contribution plans (GASB Pe5.118).

Ace Co. sold to King Co. a $20,000, 8%, 5‑year note that required five equal annual year-end payments. This note was discounted to yield a 9% rate to King. The present value factors of an ordinary annuity of $1 for five periods are as follows: 8%: 3.993 9%: 3.890 What should be the total interest revenue earned by King on this note? A. $9,000 B. $8,000 C. $5,560 D. $5,050

The correct answer is C. The first thing one needs to answer this question is the annual payment needed to pay the note. Because the note yields a higher rate (9%) than it pays (8%), the note should have a discount. Since the note has a stated rate of 8%, the annual payments will be based on the present value of an ordinary annuity based on the 8%: Thus, the annual payment is $20,000 ÷ 3.993, or $5,009 annually. The present value of the note, however, and thus the initial discount is based on the yield percentage of 9%. Therefore, the note's initial present value is the payment amount multiplied by 3.89 ($5,009 × 3.89), or $19,485. The total amount of interest revenue one earns on a note is related to the total payments and also the present value of the note, with a discount recognized here initially, on this note. The total amount to be received on this note is 5 × $5,009, for a total of $25,045. Interest is generally the amount returned over and above the amount originally recognized, which was the $19,485 originally. Thus, the total interest revenue is $25,045 − $19,485, or $5,560.

On April 1, 20X1, Ivy began operating a service proprietorship with an initial cash investment of $1,000. The proprietorship provided $3,200 of services in April and received full payment in May. The proprietorship incurred expenses of $1,500 in April which were paid in June. During May, Ivy drew $500 against her capital account. What was the proprietorship's income for the two months ending May 31, 20X1, under the following methods of accounting? A. Cash basis: $1,200; Accrual basis: $1,200 B. Cash basis: $1,700; Accrual basis: $1,700 C. Cash basis: $2,700; Accrual basis: $1,200 D. Cash basis: $3,200; Accrual basis: $1,700

The correct answer is D. Under the cash method, income is recognized as received in cash, and expenses are recognized as paid in cash. Under the accrual basis, revenues are recognized as earned (services rendered) and expenses as incurred. Draws are not expenses. Cash Basis Accrual Basis ---------- ------------- Revenue $3,200 $3,200 Less: Expenses - (1,500) ------ ------- Net Income $3,200 $1,700 ====== =======

What are the Statements of Financial Accounting Concepts intended to establish? A. Generally accepted accounting principles in financial reporting by business enterprises B. The meaning of "present fairly in accordance with applicable financial reporting framework" C. The objectives and fundamental concepts that will be the basis for development of financial accounting and reporting guidance. D. The hierarchy of sources of generally accepted accounting principles

The correct answer is C. The opening paragraph of the introduction to SFAC 8, Statements of Financial Accounting Concepts, states: Quote More specifically, Concepts Statements are intended to establish the objectives and fundamental concepts that will be the basis for development of financial accounting and reporting guidance. The fundamentals are the underlying concepts of financial accounting—concepts that guide the selection of transactions and other events and conditions to be accounted for; their recognition and measurement; and the means of summarizing and communicating them to interested parties. The Accounting Standards Codification (ASC) establishes generally accepted accounting principles (GAAP) in financial reporting by business enterprises. The meaning of "present fairly in accordance with the applicable financial reporting framework" is presented in Statement on Auditing Standards (SAS) 122 (AU-C 700.35).

In preparing its cash flow statement for the year ending December 31, 20X1, Reve Co. collected the following data: Gain on sale of equipment $ (6,000) Proceeds from sale of equipment 10,000 Purchase of A.S., Inc., bonds (par value $200,000) (180,000) Amortization of bond discount 2,000 Dividends declared (45,000) Dividends paid (38,000) Proceeds from sale of Treasury stock (carrying amount $65,000) 75,000 In its December 31, 20X1, statement of cash flows, what amount should Reve report as net cash provided by financing activities? A. $20,000 B. $27,000 C. $30,000 D. $37,000

The correct answer is D. Cash inflows from financing activities: Proceeds from sale of treasury stock $75,000 Cash outflows from financing activities: Dividends paid (38,000) -------- Net cash provided by financing activities $37,000 Dividends declared created a liability, but until they are paid, no cash flows out of the corporation. **여기서 핵심 함정은, dividend declared이다. 문제에서 보면, dividend declare랑 dividend paid가 있다. 실제로 돈이 dividend로 지불된것은, dividend paid금액으로 cash flow financing activity계산을 해야한다.

When a full set of general purpose financial statements are presented, comprehensive income and its components should: A. appear as a part of discontinued operations, extraordinary items, and cumulative effect of a change in accounting principle. B. be reported net of related income tax effect, in total and individually. C. appear in a supplemental schedule in the notes to the financial statements. D. be displayed in a financial statement that has the same prominence as other financial statements.

The correct answer is D. FASB ASC 220-10 requires that all items that are recognized as components of comprehensive income be reported in a financial statement that has the same prominence as other financial statements. However, FASB ASC 220-10-45-7 does not prescribe a specific format for the display of such information. Note that the concept of "extraordinary" items has been eliminated from GAAP; the presentation for items that are unusual in nature or occur infrequently will be expanded to include items that are both unusual in nature and infrequently occurring.

A not-for-profit voluntary health and welfare entity received a $500,000 contribution at the start of 20X2 with donor instructions to maintain the principal as a permanent endowment and use all income for a mental health program; the donor stipulated that investment gains remain part of the endowment. The endowment principal was invested in a number of equity securities and mutual funds using an investment management firm. The investment manager remits the proceeds of the investment quarterly. At the end of 20X2, the investment value had increased to $530,000. Dividends for the year totaled $20,000 and custodial and transaction fees were $375. The increase in the investment value would be reported as an increase to permanently restricted net assets and: A. the dividends should be reported as a $20,000 increase in unrestricted net assets with a $375 expense decrease to unrestricted net assets. B. the dividends should be reported as a $20,000 increase in temporarily restricted net assets with a $375 expense decrease to temporarily restricted net assets. C. the dividends may be reported as a $19,625 increase in unrestricted net assets. D. the dividends may be reported as a $19,625 increase in temporarily restricted net assets.

The correct answer is D. FASB ASC 958-320-45-4 indicates that investment revenues may be reported net of related expenses provided that there is disclosure of the expenses elsewhere in the financial statements. In this case, the dividend revenues are subject to specific use by the donor and should be considered an increase to temporarily restricted net assets. No expenses are reported in the temporarily restricted net assets category. If the investment expenses are to be reported separately, the expense would be a decrease in unrestricted net assets accompanied by a reclassification from the temporarily restricted net assets category, leaving a net increase of $19,625 to temporarily restricted net assets ($20,000 - $375).

The presentation of financial statements must be applied within an identifiable framework (AU-C 800.A15-.A18). Normally, the framework is provided by generally accepted accounting principles (GAAP). However, in some circumstances, a different framework may be used. Which of the following would not be indicative of an acceptable framework for the presentation of financial statements? A. Reporting on cash basis but capitalizing fixed assets and recording depreciation B. Ignoring accrued income and expenses C. Measuring inflation and reporting it on the financial statements D. Modifying items on the cash flow statement based on definite criteria

The correct answer is D. Financial statements may be prepared using a comprehensive basis of accounting other than generally accepted accounting principles (often referred to as a special purpose framework). Examples given by AU-C 800.07 are as follows: Cash basis. A basis of accounting that the entity uses to record cash receipts and disbursements and modifications of the cash basis having substantial support (for example, recording depreciation on fixed assets). Tax basis. A basis of accounting that the entity uses to file its income tax return for the period covered by the financial statements. Regulatory basis. A basis of accounting that the entity uses to comply with the requirements or financial reporting provisions of a regulatory agency to whose jurisdiction the entity is subject (for example, a basis of accounting that insurance companies use pursuant to the accounting practices prescribed or permitted by a state insurance commission). Contractual basis. A basis of accounting that the entity uses to comply with an agreement between the entity and one or more third parties other than the auditor. Modifications to the cash flow statement without including all material items on the financial statements (and modifications that do not have substantial support) would not qualify as a special purpose framework.

Escheat property held for another governmental entity should only be reported in an agency fund: A. when the holding period is expected to be short. B. if the assets are not required to be reported elsewhere. C. if the unclaimed property ultimately reverts to another government. D. All of the answer choices are correct.

The correct answer is D. GASB E70.102 indicates that escheat property held for another government should be reported as an asset in a private-purpose trust fund or an agency fund, as appropriate, or in the governmental or proprietary fund in which the escheat property is otherwise reported. Paragraph 26 of GASB Statement 37 (basis for conclusions) clarifies that an agency fund can be used to report amounts held for other governments, provided the holding period is expected to be short.

Cey Company has a defined benefit pension plan. Cey's policy is to fund net periodic pension cost annually, payment to an independent trustee being made 2 months after the end of each year. Data relating to the pension plan for 2008 are as follows: Net pension cost for 2008 - $190,000 Unrecognized prior service cost, 12/31/08 - $150,000 Accumulated benefit obligation, 12/31/08- $480,000 Fair value of plan assets, 12/31/08- $500,000 Projected benefit obligation 12/31/08- $500,000 How much should appear on Cey's balance sheet at December 31, 2008, for pension liability? Current Noncurren $190,000 $0

The current liability for pensions at 12/31/Y5 consists of the year 5 net pension cost ($190,000), which will not be paid until 2 months after year‐end. There is no noncurrent liability. A noncurrent liability would exist if the projected benefit obligation ($480,000) exceeds the fair value of plan assets ($500,000). ASC Topic 715 requires the recording of a liability when the projected benefit obligation is greater than the fair value of plan assets. The unrecognized prior service cost ($150,000) is not directly recorded as a liability. It is amortized to pension cost over future periods. Remember, non current liability will exist if PBO exceed Fair value of plan asset. Plan asest보다 더 PBO가 많아지면, 그땐 차액만큼 ,liability를 잡아줘야한다. Recognition of Net Funded Status of the Plana Asset: FMV of plan assets > PBO. In this case never recorded as a current asset (always recorded as noncurrent asset) This is overfunded plans FMV of plan assets < PBO. Normally recorded as noncurrent liability. This is underfunded plan

The stockholders of Meadow Corp. approved a stock‐option plan that grants the company's top three executives options to purchase a maximum of 1,000 shares each of Meadow's $2 par common stock for $19 per share. The options were granted on January 1 when the fair value of the stock was $20 per share. Meadow determined that the fair value of the compensation is $300,000 and the vesting period is three years. What amount of compensation expense from the options should Meadow record in the year the options were granted? a) $ 20,000 b) $ 60,000 c) $ 100,000 d) $ 300,000 answer: c

The fair value of a fixed option plan at grant date is the fair value of the option. Typically the fair value of one option is given and that is multiple by the number of options, but this problem provides the entire fair value. That total fair value is the total compensation expense to be recognized over the service period- the number of years form grant date to vesting. Once the options vest, no more compensation expense is recognized because the manager has provided the necessary service. Compensation expense per year is the total $300,000 compensation expense divided by 3 years, or $100,000 per year. Stock compensation is a way corporations use stock options to reward employees. Employees with stock options need to know whether their stock is vested and will retain its full value even if they are no longer employed with that company. Vesting When vesting, companies let employees purchase a predetermined number of shares at a set price. Companies may vest on a specific date or on a monthly, quarterly or annual schedule. The timing may be set according to company-wide or individual performance targets being met, or both time and performance criteria. Vesting periods are often three to four years, typically beginning after the first anniversary of the date an employee became eligible for stock compensation. After being vested, the employee may exercise his stock-purchasing option any time before the expiration date. For example, an employee is given the right to purchase 2,000 shares at $20 per share. The options vest 30% per year over three years and have a term of 5 years. The employee pays $20 per share when buying the stock, regardless of the stock price, over the five-year period. For example, an employee is given the right to purchase 2,000 shares at $20 per share. The options vest 30% per year over three years and have a term of 5 years. The employee pays $20 per share when buying the stock, regardless of the stock price, over the five-year period.

Lower of Cost and Net Realizable Value (LCNRV) Rule

The lower of cost or market rule states that a business must record the cost of inventory at whichever cost is lower - the original cost or its current market price. There is an ongoing need to examine the value of inventory to see if its recorded cost should be reduced, due to the negative impacts of such factors as damage, spoilage, obsolescence, and reduced demand from customers Formula for net realizable value is: Inventory market value - Costs to complete and sell goods = Net realizable value ABC International has a green widget in inventory with a cost of $50. The market value of the widget is $130. The cost to prepare the widget for sale is $20, so the net realizable value is $60 ($130 market value - $50 cost - $20 completion cost). Since the cost of $50 is lower than the net realizable value of $60, you continue to record the inventory item at its $50 cost. In the following year, the market value of the green widget declines to $115. The cost is still $50, and the cost to prepare it for sale is $20, so the net realizable value is $45 ($115 market value - $50 cost - $20 completion cost). Since the net realizable value of $45 is lower than the cost of $50, you should record a loss of $5 on the inventory item, thereby reducing its recorded cost to $45. If this calculation does result in a loss, you should charge the loss to the cost of goods sold expense with a debit, and credit the inventory account to reduce the value of the inventory account Replacement Cost: The cost to replace the assets of a company or a property of the same or equal value.

There are 2 primary methods of accounting to determine when revenue is recognized for long-term contracts: 1) completed contract method (CCM) 2) percentage of completion method (PCM) Because the CCM allows the deferral of taxes, a large contractor must usually choose the PCM, but a small contractor can choose CCM if the estimated life of the contract is 2 years or less. If a contractor receives advanced payments during the course of the project, then the contractor can defer the reporting of the advance payments until they are recognized as income under the taxpayer's method of accounting.

The percentage‐of‐completion method for long‐term contracts is preferable over the completed‐contract method when estimates of cost to complete and extent of progress made toward completion are reasonably dependable. Under the percentage‐of‐completion method, income would be periodically recognized as the contract is completed. Ex: The calculation of the income recognized in the third year of a five‐year construction contract accounted for using the percentage‐of‐completion method includes the ratio of: The amount is computed as the ratio of total costs incurred to date to the total estimated costs.

capital lease requirement. 4개중에 하나의 조건이 맞는다. To be considered a capital lease, a lease must satisfy any one of the four criteria. (1) The lease has bargain purchase option (2) have transfer transfer title. (3) The lease term is 75% or more of the useful life (4) the PV of the lease payments is 90% or more of the FV of the asset *In this case, Nobb company has right to bargain purchase option. (remember, 만약에 4가지중에 단 하나라도 해당이 안되면, 그냥 operating lease다) The lease shall be accounted for as an operating lease if none of the four requirements applicable to both lessees and lessors is met. Even if one or more was met, the lease would still be classified as an operating lease if the payments are not reasonably predictable. (EX:The Morn Company leased equipment to the Lizard Company on May 1, year 1. At that time the collectibility of the minimum lease payments was not reasonably predictable.) --> this is when it becomes operating lease

Under operating lease accounting, the lessee does not own the asset, which has the following implications: Lease payments are considered operational expenses for the business. The asset/lease is not reported on the balance sheet. The firm cannot claim depreciation on the asset. In contrast, accounting for a capital lease (or finance lease in IFAC terminology) treats the lessee as the owner of the asset, which means: The lease is considered a loan. Interest payments are considered operational expenses. The asset is included in the balance sheet: the outstanding loan amount (net present value of all future lease payments) is included as a liability, and the present market value of the asset is included as an asset. The lessee can claim depreciation on the asset every year. Pros and Cons Advantages of an operating lease Operating leases provide much-needed flexibility to companies that frequently update or replace their equipment. The lessee is protected from the risk of obsolescence. Accounting is simpler: the asset does not have to be included in the balance sheet. The corresponding debt liability does not have to be calculated or included either. Lease payments are operational expenses, so they are fully tax deductible. It provides improved Return On Asset (ROA) without capital budgeting restraints. Advantages of a capital lease Capital leases recognize expenses sooner than equivalent operating leases. The lessee is allowed to claim depreciation each year on the asset. In addition to depreciation, the interest expense component of the lease payment can also be deducted as an operational expense.

There are four primary methods that accountants use to recognize business sales revenue: percentage of completion, completed contract, cost recovery and installment sales Businesses determine which method to use based on the type of transaction and the type of revenue collection uncertainty they face. Where there is extreme uncertainty, accountants use either the installment sale method or the cost recovery method. If a product is sold through an installment plan, in which the customer is allowed to make payments over a long period of time, then a company would use an installment sales method. The cost recovery method is used in much more uncertain transactions, in which accountants are either unable to assume payment confidently or if the value of the sale is difficult to estimate. Businesses determine which method to use based on the type of transaction and the type of revenue collection uncertainty they face. Where there is extreme uncertainty, accountants use either the installment sale method or the cost recovery method. If a product is sold through an installment plan, in which the customer is allowed to make payments over a long period of time, then a company would use an installment sales method. The cost recovery method is used in much more uncertain transactions, in which accountants are either unable to assume payment confidently or if the value of the sale is difficult to estimate. Installment Method When a sale is made, but payments are delayed over a period of time, the transaction is called an installment sale. Accountants do not want to recognize the full amount of the sale on the outset, because there is a sufficient risk of not collecting that makes the receivables questionable. Therefore, both revenue and costs are only recognized when payments are received by the company from the customer. Each payment is further broken down into two components: an amount used to show a partial recovery of the cost of item sold and an amount dedicated to gross profit. Cost Recovery Method Cost recovery is an even more conservative method of revenue recognition. Here, all gross profit is deferred until the cost of the item sold is recovered. The initial journal entry, however, is identical to installment method. It is really only acceptable to use the cost recovery method if bad debts cannot be reasonably estimated. Otherwise, delaying revenue recognition violates the realization principle.

Under the cost-recovery method, the seller can't record profit on the sales transaction until the customer's payments cover at least the cost of the goods sold. According to the cost recovery method of accounting, gross profit on an installment sale is recognized in income: After cash collections equal to the cost of sales have been received. (EX: gross profit is not recognized until cash collection exceed the amount of COGS. The total contract is $200,00 and COGS is $100,000 the profit is $100,000. In year 1, cash $800 was received. No gain can be recognize in year 1 as, $800 is less than COGS $100,000. In year 2, $500 was received. Thus, profit can be recognized as ($800 received in year 1 + $500 received in year 2= $13,000 exceed cogs $100,000. So, profit of 3,000 is recognized) For financial statement purposes, the installment method of accounting may be used if the: Installment amount collectible is indeterminate.<- answer The profit on a sale in the ordinary course of business is considered to be realized at the time of the sale unless it is uncertain whether the sales price will be collected. The Board concluded that use of the installment method of accounting is not acceptable unless this uncertainty exists. Gemini prepares its financial statements using International Financial Reporting Standards (IFRS). If Gemini cannot reliably estimate the income and expenses of customer contracts, what accounting method should be used to recognize revenues? Answer: Cost recovery method. <- because if the outcome of rendering services cannot be measured reliably, IFRS requires use of the cost recovery method. Note that, installment method is a revenue recognition method used under US GAAP, not IFRS.

On February 1, Year 1, Kew Corp., a newly formed company, had the following stock issued and outstanding: Common stock, no par, $1 stated value, 10,000 shares originally issued for $15 per share. Preferred stock, $10 par value, 3,000 shares originally issued for $25 per share. Kew's February 1, Year 1, statement of stockholders' equity should report What is Common stock: Preferred stock: Additional paid‐in capital:

answer: Common stock: $ 10,000 Preferred stock: $30,000 Additional paid‐in capital: $ 185,000 *핵심* Company must record the stock using PAR or Stated value. Issued value란, investor/shareholder한테 stock을 판 가격 value이고, PAR/Stated는 cost value라고 생각하기. Additional paid-in capital은, difference between excessive value of FMV selling price (issue price) -Par value (Cost) Common stock and preferred stock are reported at par or stated value, and any excess invested above par or stated value is recorded as additional paid‐in capital. In this case, common stock is recorded at stated value (10,000 × $1 = $10,000), and preferred stock is recorded at par value (3,000 × $10 = $30,000). Additional paid‐in capital from common stock is $140,000 [10,000 × ($15 − $1)], and additional paid‐in capital from preferred stock is $45,000 [3,000 × ($25 − $10)], so additional paid‐in capital is $185,000 ($140,000 + $45,000).

On January 1, year 1, JCK Co. signed a contract for an 8‐year lease of its equipment with a 10‐year life. The present value of the 16 equal semiannual payments in advance equaled 85% of the equipment's fair value. The contract had no provision for JCK, the lessor, to give up legal ownership of the equipment. Should JCK recognize rent or interest revenue in year 2, and should the revenue recognized in year 2 be the same or smaller than the revenue recognized in year 1? A. year 2 rent revenue recognized; year 2 amount recognized the same as year 1 B. year 2 rent revenue recognized; year 2 amount recognized smaller than year 1 C. year 2 interest revenue recognized; year 2 amount recognized the same as year 1 D. year 2 interest revenue recognized; year 2 amount recognized smaller than year 1

answer: I think if you look at the # of yrs they will lease makes it a capital lease.8 yrs of a 10 yr lease makes it 80% therefore a capital lease and if it's a capital lease you recognize interest revenue. Since it qualifies as a capital lease, interest revenue is recorded. If this were an operating lease, then rent revenue would be recorded. Also, think of it like this: If you are renting a place, rent is usually fixed for a certain period of time (and even if it isn't fixed, rent revenue is recorded on a SL basis). So it doesn't make sense for the rent recognized to decrease from year to year. So that already gets rid of option B. Then, on the flip side, if you are receiving interest on anything, not just a lease, than the amount from year to year will vary because interest is based off principal with decreases with every payment. So that eliminates option C. Now all you need to know is if this qualifies as a capital lease or an operating lease. Due to the fact that 8 yrs of a 10 yr lease makes it 80%, this is a capital lease, and interest will need to be recorded.

A company records items on the cash basis throughout the year and converts to an accrual basis for year-end reporting. Its cash-basis net income for the year is $70,000. The company has gathered the following comparative balance sheet information: Beginning of Year End of Year ----------------- ----------- Accounts payable $ 3,000 $ 1,000 Unearned revenue 300 500 Wages payable 300 400 Prepaid rent 1,200 1,500 Accounts receivable 1,400 600 What amount should the company report as its accrual-based net income for the current year? A. $68,800 B. $70,200 C. $71,200 D. $73,200

correct answer is C. Accrual-basis net income is computed as follows: Beginning Effect on of Year End of Year Accrual Net Income --------- ----------- ------------------ Cash-basis net income $70,000 Accounts payable $ 3,000 $ 1,000 2,000 Unearned revenue 300 500 -200 Wages payable 300 400 -100 Prepaid rent 1,200 1,500 300 Accounts receivable 1,400 600 -800 ------- Accrual net income $71,200

Kemp Company provides a defined benefit postretirement plan for its employees. Kemp adopted the plan on January 1, year 1, in accordance with the provisions of ASC Topic 715. Data relating to the pension plan for year 1 are as follows: Service cost for year 1 28,000 Interest on the accumulated postretirement benefit obligation 5,000 Amortization of the unrecognized transition obligation 8,000 At the end of year 1, Kemp makes a benefit payment of $10,000 to employees. In its December 31, year 1 balance sheet, Kemp should record accrued postretirement benefit cost of a) $35,000 b) $31,000 c) $51,000 d) $15,000

answer: To determine the accrued postretirement benefit cost, the net periodic postretirement benefit cost must first be calculated as follows: Service cost 28,000 Interest on the accumulated postretirement benefit obligation 5,000 Amortization of the unrecognized transition obligation 8,000 Net periodic postretirement benefit cost $41,000 An adjusting entry is required at year‐end to record net periodic postretirement benefit cost and the cash benefit payments made to employees. An accrued postretirement benefit cost will be recorded if the net postretirement benefit cost exceeds the cash payments to employees. The journal entry would be Postretire benefit cost 41,000 Cash 10,000 Accrued postretire benefit cost 31,000 Therefore, the balance in the accrued postretirement benefit cost account would be $31,000.

The comparative balance sheets for Wellington Inc. reported the following information at December 31, year 1 and year 2: Year 2 Year 1 Retained earnings $210,000 $140,000 AOCI 30,000 35,000 *AOCI (Accumulated Other comprehensive Income) Wellington declared cash dividends of $20,000 in year 2. The decrease in accumulated other comprehensive income for year 2 was due to unrealized losses on available‐for‐sale securities. For the year ended December 31, year 2, what was Wellington's comprehensive income? a) $65,000 b) $95,000 c) $90,000 d) $85,000

answer: Increase in retained earnings from year 1 to year 2 is $70,000. ($210,000 less $140,000) 여기서, we know that company declared dividend in year 2. (meaning during the year 2, not end of the year 2) "declared dividend"란, 정말로, dividend가 pay됬던 안됬던간에, declare이 됬다고 나오면, 무족건, pay한것으로 생각하기. J/E for this dividend is: Debit: Retained earning $20,000 Credit: Dividend payable $20,000 $210,000 retained earning for year 2 means, after dividend declared is affected. so, this actually tells us year 2's retained earning is $230,000. ($210,000 +20,000) $230,000 RE year 2 - $140,000 RE Year 1 = $90,000 net income before dividend declared. ***그리고 핵심 포인트는, the question asks you what is comprehensive income. Remember, comprehensive income is (net income + other comprehensive income) in accumulated other comprehensive income for year 2 ($35,000 less $30,000) (5,000). So, we know other comprehensive income for year 2 was -5,000 loss and when we add our year 2 net income $90,000, it is $85,000. $85,000 is year 2's comprehensive income.

Bond Company leased equipment from Howe, Inc. on December 31, year 1, for a 10‐year period (the useful life of the asset) expiring December 30, year 11. Equal annual payments under the lease are $100,000 and are due on December 31 of each year. The first payment was made on December 31, year 1, and the second payment was made on the due date. The present value at December 31, year 1, of the minimum lease payments over the lease term discounted at 10% (the implicit rate computed by Howe and known by Bond) was $676,000. Bond's incremental borrowing rate was 12% at December 31, year 1. The lease is appropriately accounted for as a capital lease by Bond. What should be the balance in Bond's liability under capital lease account at December 31, year 2? a) $533,600 b) $545,120 c) $607,960 d) $800,000

answer: a The first payment (paid on the date the lease is signed) contains no interest and is, therefore, all reduction of principal. The second payment includes interest of $57,600 (10% × $576,000) and principal of $42,400 ($100,000 − $57,600). Note that because Howe's implicit interest rate of 10% is known by Bond and is lower than Bond's incremental rate, it is used to compute the interest payment. (If there Known implicit interest, meaning if implicit interest is stated in the question, we would compare this implicit rate against the incremental rate and we will use the lesser rate) 여기서 또하나의 조심해야할것은, the question is asking "What should be the balance in Bond's liability under capital lease account at December 31, year 2?" Balance in liability가 조심해야하는것은, 매년 annual payment가 deduct되지만, 여기서, only principle decrease the total bond liability! interest payment는 deduct가 될수가없다. The first payment (paid on the date the lease is signed) contains no interest and is, therefore, all reduction of principal. The second payment includes interest of $57,600 (10% × $576,000) and principal of $42,400 ($100,000 − $57,600). Note that because Howe's implicit interest rate of 10% is known by Bond and is lower than Bond's incremental rate, it is used to compute the interest payment. **notice, we only deduct principle payment from Carrying value of the lease receivable, which is also known as present value minimum liability payment**

On October 1, year 1, Price Corp., a real estate developer, sold land to Greene Co. for $5,000,000. Greene paid $600,000 cash and signed a 10‐year $4,400,000 note bearing interest at 12%. The carrying amount of the land was $4,000,000 on date of sale. The note was payable in forty quarterly principal installments of $110,000 beginning January 2, year 1. Price appropriately accounts for the sale "under the COST RECOVERY METHOD". On January 2, year 2, Greene paid the first principal installment of $110,000 and interest of $132,000. For the year ended December 31, year 1, what total amount of income should Price recognize from the land sale and financing? a) $0 b) $120,000 c) $132,000 d) $252,000

answer: a (여기서 핵심 패스 포인트는, it says it is using cost recovery method, which doesn't recognize profit or revenue until cumulative receipt exceed the company's cost of assets sold, which is carrying book value of asset that sold) Under the cost recovery method no profit of any type is recognized until the cumulative receipts exceed the cost of the asset sold. This means that the entire gross profit ($5,000,000 - $4,000,000 = $1,000,000) and the year 1 interest revenue ($132,000) will be deferred until cash collections exceed $4,000,000. Therefore, no income is recognized in year 1.

Benedict Company leased equipment to Mark, Inc. on January 1, year 2. The lease is for an 8‐year period expiring December 31, year 9. The first of 8 equal annual payments of $600,000 was made on January 1, year 2. Benedict had purchased the equipment on December 29, year 1, for $3,200,000. The lease is appropriately accounted for as a sales‐type lease by Benedict. Assume that the present value at January 1, year 2, of all rent payments over the lease term discounted at a 10% interest rate was $3,520,000. What amount of interest income should Benedict record in year 3 (the second year of the lease period) as a result of the lease? a) $261,200 b) $296,000 c) $320,000 d )$327,200

answer: a This answer is correct. The income recorded would be 10% of the present value of the lease receivable balance outstanding in year 2 (year 3). The interest can be computed using an amortization table. Notice that interest is accrued on December 31 of each year, and is paid the following day on January 1. Therefore, interest income in year 3 should be $261,200. 1/1/year 2 Cash received : $600,000 Interest Income: 0 (1년이 안되어 이자없음) Amount of payment applied to principal: $600,000 Carrying value of the lease receivable : $3,520,000 - $600,000 = $2,920,000 **note, 12/31/year2에 year2의 해당되는 이자를 accrual 하고 1/1/year3에 돈이 지불될것이다. When this interest income is paid, it will be paid with the payment of year 3's $600,000 annual payment. 1/1/Y3 Cash received : $600,000 Interest Income: 292,000 ($2,920,000 x 10%) Amount of payment applied to principal: $ 308,000 ($600,000 -292,000 interest) Carrying value of the lease receivable : $2,920,000 - $308,000 = $2,612,000 **notice, we only deduct principle payment from Carrying value of the lease receivable, which is also known as present value minimum liability payment** The question was asking "What amount of interest income should Benedict record in year 3 as a result of the lease?" Benedict이 돈을 받는 lessor입장이면, year 3에 받을 interest accrual을 A/R이랑 interest income으로 book 에다 달아놓아야 한다. 실제 돈이 1/1/year4에 들어온다고 해도... 12/31/year3 기준으로 accrual 된 interest income은 $2,612,000 balance x 10% interest = $261,200

In year 6, Spirit, Inc. determined that the 12‐year estimated useful life of a machine purchased for $48,000 in January year 1 should be extended by three years. The machine is being depreciated using the straight‐line method and has no salvage value. What amount of depreciation expense should Spirit report in its financial statements for the year ending December 31, year 6? a) $2,800 b) $3,200 c) $4,200 d) $4,800

answer: a A change in estimated life of an asset is a change in accounting estimate and is accounted for on a prospective basis in the current year and future years. To account for this change in estimate, the carrying value of the asset is calculated as of the beginning of the year of the change. Spirit acquired the asset in year 1; therefore, Spirit has depreciated the asset for five years (year 1, year 2, year 3, year 4, and year 5). Depreciation expense using the straight‐line method and no salvage value was $4,000 ($48,000 ÷ 12 years) per year. Therefore, accumulated depreciation at January 1, year 6, was $20,000 ($4,000 × 5 years). The book value at January 1, year 6, is equal to $28,000 ($48,000 cost - $20,000 accumulated depreciation), and the remaining life is 10 years (7 years remaining + 3 years extended life). Therefore, this answer is correct because depreciation expense for year 6 is equal to $2,800 ($28,000 ÷ 10 years).

In December year 1, Mill Co. began including one coupon in each package of candy that it sells and offering a toy in exchange for 50 cents and five coupons. The toys cost Mill 80 cents each. Eventually 60% of the coupons will be redeemed. During December, Mill sold 110,000 packages of candy and no coupons were redeemed. In its December 31, year 1 balance sheet, what amount should Mill report as estimated liability for coupons? a) $ 3,960 b)$10,560 c)$19,800 d) $52,800

answer: a Mill expects 60% of the 110,000 coupons to be redeemed, resulting in redemption of 66,000 coupons. Since five coupons must be presented to receive a toy, it is expected that 13,200 toys (66,000 / 5) will be mailed in the future as a result of December year 1 sales. The net cost per toy for Mill is 30 (80 − 50), so the estimated liability for coupons is $3,960 (13,200 ÷ 30)

Brass Co. reported income before income tax expense of $60,000 for year 2. Brass had no permanent or temporary timing differences for tax purposes. Brass has an effective tax rate of 30% and a $40,000 net operating loss carryforward from year 1. What is the maximum income tax benefit that Brass can realize from the loss carryforward for year 2? a) $12,000 b) $18,000 c) $20,000 d) $40,000

answer: a (여기서 핵심: previous year에 carry over된 operating loss는 current year의 income에서 먼저 deduct가 된다. Deduction이 있고난후, 그다음에 tax rate를 부가 하는것. $60,000 -$40,000 =$20,000 x 30% = $12,000) The $40,000 net operating loss carryforward is the amount of loss in year 1 which may be carried forward to future years to offset the tax due in those years. The value of the carryforward is the net operating loss multiplied by the tax rate for the year in which the carryforward is expected to be realized. Therefore, the maximum income tax benefit that Brass can realize from the carryforward in year 2 is $12,000 ($40,000 × 30%).

Cory, Inc. uses the accrual method of accounting for financial reporting purposes and appropriately uses the installment method of accounting for income tax purposes. Installment income of $250,000 will be collected in the following years when the enacted tax rates are Collection of income Enacted tax rates Year 3 $ 25,000 35% Year 4 50,000 30% Year 5 75,000 30% Year 6 100,000 25% The installment income is Cory's only temporary difference. What amount should be included in the deferred income tax liability in Cory's December 31, year 3 balance sheet? a) $62,500 b) $71,250 c) $78,750 d) $87,500

answer: a *여기서 trick은, What amount should be included in the deferred income tax liability in Cory's December 31, year 3 balance sheet? 인데, 몇년도의 tax income %를 반영할꺼냐가 핵심이다. 흔들리지말고, 각 future의 tax income %를 반영하기 For accounting purposes, installment income of $250,000 is recognized using the accrual method. For tax purposes, the installment income is recognized over 4 years (year 3‐year 6). Using the installment method results in future taxable amounts at 12/31/Y3. Cory's deferred tax liability should be based on future enacted tax rates (30% for year 4 and year 5 and 25% for year 6). Therefore, the 12/31/Y3 deferred tax liability is $62,500, as computed below. Year Taxable amount Rate DT liability Year 4 $ 50,000 × 30% = $15,000 Year 5 75,000 × 30% = 22,500 Year 6 100,000 × 25% = 25,000 $62,500

In a statement of cash flows in which the operating activities section is prepared under the indirect method, a gain on the sale of an investment in available‐for‐sale securities should be presented as a(n) a) Deduction from net income. b) Addition to net income. c)Inflow and outflow of cash. d) Outflow of cash.

answer: a A gain on the sale of an investment is not an addition to net income. Any gain on the sale of an investment other than trading securities should be included as part of the total proceeds reported in investing activities. However, this gain has been included in net income. Under the indirect method, net income is adjusted for items which affect income but not cash. Therefore, the amount of the gain must be deducted from net income to remove the book gain from the cash flows from operating activities, thereby avoiding double counting the gain.

When equipment held under an operating lease is subleased by the original lessee, the original lessee would account for the sublease as a(n) a) Operating lease. b) Sales‐type lease. c) Direct financing lease. d) Capital lease.

answer: a A sublease arises when the lease agreement between the two original parties remains in effect, and the leased property is released to a third party by the original lessee. Per ASC Topic 840, when equipment held under an operating lease is subleased by the original lessee, the sublease is still considered to be an operating lease by the original lessee. 최초리스 이용자에 의해 양도되는 경우, 전 환 사는 여전히 최초리스 이용자가 운용리스로 간주됩니다 (Lessor랑 original lessee가 있었다. 그런데 중간에, original lessee가 다른 lessee한테 sublease를 줘버렸다. 그래도, 모든 책임감은, original lessee한테 남아있다는걸 강조하고있다) sale-and-leaseback is a financial transaction, where one sells an asset and leases it back for the long-term; therefore, one continues to be able to use the asset but no longer owns it. The transaction is generally done for fixed assets, notably real estate, as well as for durable and capital goods such as airplanes and trains Direct-financing lease is basically the coupling of a sale and financing transaction. In this case, the lessor removes the leased asset from its books and replaces it with a receivable from the lessee. The only income recognized by the lessor is the interest received.

Under IFRS, all of the following are types of hedges except for: a) Foreign currency hedge. b) Cash flow hedge. c) Fair value hedge. d) Hedge of net investment.

answer: a Although a foreign currency hedge is a type of hedge under US GAAP, it is not, under IFRS. The three types of hedge classification under IFRS are cash flow hedge, fair value hedge, and hedge of net investment.

Harland County received a $2,000,000 capital grant to be equally distributed among its five municipalities. The grant is to finance the construction of capital assets. Harland had no administrative or direct financial involvement in the construction. In which fund should Harland record the receipt of cash? a) Agency fund. b) General fund. c) Special revenue fund. d) Private purpose trust fund. You Answered Correctly!

answer: a An agency fund is used to account for activities where the government is acting as an agent for other entities.

Adam Co. reported sales revenue of $2,300,000 in its income statement for the year ended December 31, year 2. Additional information was as follows: 12/31/Y1 12/31/Y2 Accounts receivable $500,000 $650,000 Allowance for uncollectible (30,000) (55,000) Uncollectible accounts totaling $10,000 were written off during year 2. Under the cash basis of accounting, Adam would have reported year 2 sales of a) $2,140,000 b)$2,150,000 c)$2,175,000 d) $2,450,000

answer: a An increase in receivables ($150,000) means that the amount of cash collected was less than sales, and this amount should be subtracted from accrual basis sales revenue to arrive at the cash basis sales revenue. Also, the $10,000 written off during year 2 means that the 12/31/Y2 receivable balance is $10,000 less than it would have been had no write‐offs been made. In other words, this $10,000 represents recognized sales that will not result in the collection of cash and should be subtracted from accrual basis sales to determine cash basis sales revenue. Therefore, Adam should report $2,140,000 ($2,300,000 - $150,000 - $10,000) for year 2 cash basis sales 얼래, accrual basis엿는데, 갑자기 cash basis로 전환하게 된다면, YEAR 2 sales revenue $2,300,000에서 A/R를 뺴져야된다.. cash basis only recognize sales when cash is actually received. $2,300,000 - $150,000 (650,000 year 2 A/R -500,000 year 1 AR = 150,000) 150,000을 빼는 이유는 year2에 잡아놓은 A/R을 없에주는것. 하지만,벌써 $10,000 write off는 일어났다고 한다. 그럼. 답은 ($2,300,000- 150,000 - 10,000 = 2,140,000)

Baker Co. began its operations during the current year. The following is Baker's balance sheet at December 31: Baker Co. BALANCE SHEET Assets Cash $192,000 Accounts receivable 82,000 Total assets $274,000 Liabilities and stockholders' equity Accounts payable $24,000 Common stock 200,000 Retained earnings 50,000 Total liabilities and stockholders' equity $274,000 Baker's net income for the current year was $78,000 and dividends of $28,000 were declared and paid. Common stock was issued for $200,000. What amount should Baker report as cash provided by operating activities in its statement of cash flows for the current year? a) $ 20,000 b) $ 50,000 c) $192,000 d) $250,000

answer: a Because Baker is in its first year of operations, the beginning balances of all accounts are zero. Cash flow from operating activities is calculated as net income minus the increase in accounts receivable, plus the increase in accounts payable (78,000 − 82,000 + 24,000) = increase in cash from operating activities of $20,000. Cash flows from common stock and dividends are included in financing activities. indirect method를 쓸때, net income에서 시작할때, operating activity에서는, cash를 제외한, current liability, current asset를 impact하는걸 더하고 빼는것으로 알고있다.

Effective with the year ended December 31, year 1, Grimm Company "adopted a new accounting method" for estimating the allowance for doubtful accounts at the amount indicated by the year‐end aging of accounts receivable. The following data are available: Allowance for doubtful accounts, 1/1/Y1- $24,000 Provision for doubtful accounts during year 1 (2% on credit sales of $1,000,000)- 20,000 Bad debts written off, 11/30/Y1 - 19,500 Estimated uncollectible accounts per aging 21,000 After year‐end adjustment, the bad debt expense for year 1 would be a) $16,500 b)$19,500 c)$20,000 d) $21,000

answer: a Before the year‐end adjustments, the allowance account had a balance of $24,500. Beginning balance of allowance for doubtful account $24,000 +Provision for doubtful account during year 1 $20,000 -Bad debt written (벌써 쓴금액) $19,500 ------------------------------------- $24,500 하지만, question says, company adopted new accounting method that states allowance for doubtful account at end year have equal to "year-end againg of A/R" which is $21,000 in this case. To adjust the account to the appropriate balance indicated by the year‐end aging ($21,000), it must be reduced by $3,500 ($24,500 − $21,000). Unadjusted bad debt expense is $20,000. After the adjustment, bad debt expense is $16,500 ($20,000 − $3,500). Provision for doubtful account during year 1 $20,000에서 $3,500을 빼주는것. 다른데서 뺄수는 없다.. $19,500 bad debt은 이미 claim 한 비용이고. 그렇다고 작년에 carry over 된 beginning balance $24,000에서도 뺄수는 없는것이니깐.

Comprehensive income can be disclosed in various formats. Which of the following is an acceptable format for disclosing comprehensive income? I. At the bottom of the income statement, continue from net income and add other comprehensive income to arrive at comprehensive income for the year. II. In a separate statement, start with net income and add other comprehensive income to arrive at comprehensive income for the year. III. In the statement of stockholders' equity, net income is adjusted for other comprehensive income to arrive at comprehensive income for the year. IV. After retained earnings in the stockholders' equity section of the statement of financial position, start with net income and add other comprehensive income to arrive at comprehensive income for the year. a) I and II b) II and III c) III and IV d) All of the above are acceptable

answer: a Comprehensive income can be disclosed in one of the following two ways: 1. On a combined income statement where other comprehensive income is added to net income to arrive at comprehensive income for the period; or 2. On a two income statement format in which a separate statement follows the typical income statement. The separate statement starts with net income and adds other comprehensive income to arrive at comprehensive income for the period. Reporting comprehensive income on the statement of financial position after retained earnings is not an acceptable format for displaying comprehensive income for the period, nor is displaying it on the statement of stockholders' equity. ** separate statement of comprehensive income may be presented, but it is not required.***

Bay Manufacturing Co. purchased a 3-month U.S. Treasury bill. In preparing Bay's statement of cash flows, this purchase would: A. have no effect. B. be treated as an outflow from financing activities. C. be treated as an outflow from investing activities. D. be treated as an outflow from operating activities.

answer: a Examples of items commonly considered to be cash equivalents are Treasury bills, commercial paper, money market funds, and federal funds sold (for an enterprise with banking operations). Thus, the purchase of U.S. Treasury bills would have no effect on the statement of cash flow The reason why purchasing 3 month U.S. treasury bill is not impacting cash flow is that you are pretty much swapping cash for cash, so there is no inflow or outflow.

During year 2, Teb, Inc. had the following activities related to its financial operations: Payment for the early retirement of long‐term bonds payable (carrying value $740,000) $750,000 Distribution in year 2 of cash dividend declared in year 1 to preferred shareholders 62,000 Carrying value of convertible preferred stock in Teb, converted into common shares 120,000 Proceeds from sale of treasury stock (carrying value at cost, $86,000) 95,000 In Teb's year 2 statement of cash flows, net cash used in financing activities should be a) $717,000 b) $716,000 c) $597,000 d) $535,000

answer: a Financing activities include all cash flows involving liabilities and equity, other than operating items. The only item given which is not classified as a cash flow from financing activities is the conversion of preferred stock into common stock ($120,000). This is a noncash item, reported in a separate schedule at the bottom of the statement of cash flows. The other items are cash financing activities, resulting in net cash used in financing activities of $717,000. *이렇게 non cash item 이있으면, it must be in "separate schedule" at bottom of statement of cash flow.

A change in accounting principle that would require retrospective application to all prior periods would be a change a) From using the percentage‐of‐completion method of accounting for long‐term construction contracts to the completed contract method. b) In the salvage value of a depreciable asset. c) From the straight‐line method of depreciation to the double‐declining balance method. d) From reporting revenues on a cash basis to reporting on an accrual basis.

answer: a From reporting revenues on a cash basis to reporting on an accrual basis. <-- This is incorrect because a change from a non‐GAAP accounting method to GAAP is a correction of an error. non-GAAP에서 GAAP으로 고쳐지는것은 당연히 맞는 것이라서, we don't consider this as change in accounting method.. change from an accounting principle that is not generally accepted to one that is generally accepted should be treated in the same manner as a correction of an error. A correction of an error should be reported as a prior period adjustment. This means that the cumulative effect at the beginning of the period of change is entered directly as an adjustment to the opening balance of retained earnings. When comparative statements are presented, prior years' statements are retroactively restated. 차라리 percentage of completion method에서 long term construction으로 바뀌는게 더 "change in accounting method"이다.

On January 2, City of Walton issued $500,000, 10‐year, 7% general obligation bonds. Interest is payable annually, beginning January 2 of the following year. What amount of bond interest is Walton required to report in the statement of revenue, expenditures, and changes in fund balance of its governmental funds at the close of this fiscal year, September 30? a) $0 b) $17,500 c) $26,250 d) $35,000

answer: a General obligation bonds are accounted for in debt service funds, which use the modified accrual basis of accounting. Only the interest cost actually paid in the period is included in interest expense. Accordingly, this answer is correct. Since no interest was paid during the year, interest expense is $0. General obligation bonds = Debit service fund = Government Fund Type = Modified accrual basis = Recognize expense only when actually cash was paid out.

Alpha Company is in the process of determining whether the carrying amounts of the long‐lived assets and identifiable intangibles acquired in a business combination are recoverable. Alpha accounted for the business combination under the acquisition method and allocated part of the acquisition price to goodwill. In determining the recoverability of the long‐lived assets and the identifiable intangibles acquired in the business combination, the goodwill should be a) Allocated to the long‐lived assets and identifiable intangible assets. b)Ignored. c)Allocated only to the identifiable intangible assets. d) Allocated only to the long‐lived assets.

answer: a Goodwill is created when one company acquires another for a price higher than the fair market value of its assets; for example, if Company A buys Company B for more than the fair value of Company B's assets and debts, the amount left over is listed on Company A's balance sheet as goodwill. The account for goodwill is located in the assets section of a company's balance sheet. It is an intangible asset, as opposed to physical assets like buildings and equipment. **핵심! If acquisition cost is greater than FV of acquired asset during the acquisition period, it is considered as 손실, and we call it as "goodwill" however, if acquisition cost is lower than FV of acquired asset cost during the acquisition period, we call this as revenue 이익, "bargain purchase". Be careful with goodwill vs bargain purchase

Which of the following is the characteristic of a perfect hedge? a) No possibility of future gain or loss. b) No possibility of future gain only. c) No possibility of future loss only. d) The possibility of future gain and no future loss.

answer: a If a hedge is effective, the gain or loss on the hedging instrument would offset any gain or loss for the hedged item. A perfect hedge would result in no possibility of a future gain or future loss. A hedge which completely eliminates the risk of another investment. While perfect hedges eliminate risk, they also greatly reduce or sometimes eliminate the potential for a return. (so resulting, no loss and no gain) What is a 'Perfect Hedge' A perfect hedge is a position undertaken by an investor that would eliminate the risk of an existing position, or a position that eliminates all market risk from a portfolio. In order to be a perfect hedge, a position would need to have a 100% inverse correlation to the initial position. As such, the perfect hedge is rarely found

Arrow Company purchased a machine on January 1, year 1, for $1,440,000 for the purpose of leasing it. The machine is expected to have an 8‐year life from date of purchase, no residual value, and be depreciated on the straight‐line basis. On February 1, year 1, the machine was leased to Baxter Company for a 3‐year period ending January 31, year 4, at a monthly rental of $30,000. Additionally, Baxter paid $72,000 to Arrow on February 1, year 1, as a lease bonus. What is the amount of income before income taxes that Arrow should report on this leased asset for the year ended December 31, year 1? a) $172,000 b) $187,000 c) $222,000 d) $237,000

answer: a Income from the lease is the monthly rental plus a proportionate fraction of the lease bonus less any depreciation expense. Rental income = 11 months × $30,000 = $ 330,000 Lease bonus income = $72,000 × 11/36 = $ 22,000 Depreciation expense = $1,440,000/8 years = $(180,000) Income from leased asset $ 172,000 Note that the lease bonus is recognized as income proportionately over the 36‐month lease period. The leased asset is depreciated for a full year since it has an 8‐year life from the date of purchase (January 1).

Initial direct costs are a) Expensed currently for sales‐type leases. b) Capitalized and amortized to expense over the lease term for all leases. c) Capitalized only if the related lease qualifies as a capital lease. d) Presented on the balance sheet as a contra account to capitalized leased assets.

answer: a Initial direct costs are costs incurred in connection with the negotiation and consummation of leases, such as legal fees, commissions, etc. For sales‐type leases, profit or loss is recognized upon inception of the lease. In keeping with the matching principle, the costs of consummating that lease should be taken into income at the same time as the resulting profit or loss. Therefore, initial direct costs for sales‐type leases are expensed currently.

For installation of street lights, to be assessed against properties benefited $300,000 For construction of public swimming pool; bonds to be paid from pledged fees collected from pool users 400,000 If Ariel Village is not potentially liable for the bonds for the assessments, how much should be accounted for through Debt Service Funds for payments of principal over the life of the bonds? $0 $300,000 $400,000 $700,000

answer: a Installing street lights for particular city's resident is "special assessment" 그지역사람들이 해택받는것이면, tax must be levied on them. Building public swimming pool is "enterprise fund" 정부가 시민들에게 public 수영장을 제공하고, 시민들이 이용비를 지불하게 되는것이니깐, it is enterprise fund. 말그대로, enterprise란 정부가 사업체처럼 관리하는 돈이 들어올수있는 business item. Debit service fund란, obligation to pay long term debt.

On August 1, year 1, Kern Company leased a machine to Day Company for a 6‐year period requiring payments of $10,000 at the beginning of each year. The machine cost $48,000, which is the fair value at the lease date, and has a useful life of 8 years with no residual value. Kern's implicit interest rate is 10% and present value factors are as follows: Present value of an annuity due of $1 at 10% for 6 periods 4.791 Present value of an annuity due of $1 at 10% for 8 periods 5.868 Kern appropriately recorded the lease as a direct financing lease. At the inception of the lease, the gross lease receivables account balance should be a) $60,000 b) $58,680 c) $48,000 d) $47,910

answer: a Lease payments receivable is debited for the gross investment in the lease, which includes the minimum lease payments plus any unguaranteed residual value. Since there is no residual value in this problem, gross investment is simply the minimum lease payments (6 rentals at $10,000 each, or $60,000). Minimum lease payments are rental payments over the lease term including the amount of any bargain purchase option, premium, and any guaranteed residual value and excluding any rental relating to costs to be met by the lessor and any contingent rentals. We are asked to find Gross Receivable Balance. So, we are interested in the accounting for the lessor. (이게 핵심: lessee입장에서가 아니고, lessor입장에서의 gross A/R을 말하는것이다. Lessor 입장에서는 PV value가 붙지 않은 실제로 본인회사가 매년 받을 minimum payment를 통틀어서 AR로 첫번쨰 year에 잡아야된다. this minimum payment is annual payment which doesn't include the interest. The lease receivable is calculated as follows: (Annual Rental Payment x Lease Term) + Guaranteed Residual Value, if any In our problem we have annual rental payment of 10,000 and lease term of 6 years. 10,000 x 6 = 60,000 Lessor's current J/E will be: Dr. Lease receivable 60,000 Cr. Asset Leased 48,000 Cr. Unearned Int revenue 12,000

During the current year, Onal Co. purchased 10,000 shares of its own stock at $7 per share. The stock was originally issued at $6. The firm sold 5,000 of the treasury shares for $10 per share. The firm uses the cost method to account for treasury stock. What amount should Onal report in its income statement for these transactions? a) $0 b)$5,000 gain. c)$10,000 loss. d) $15,000 gain.

answer: a Onal Co. should record the treasury shares at cost, and debit treasury stock for $70,000 when the 10,000 shares of treasury stock were acquired for $7 per share. When the 5,000 shares of treasury stock were subsequently sold at $10 per share, Onal would debit cash for $50,000, credit the treasury stock account for $35,000 ($7 cost × 5,000 shares), and credit additional paid‐in capital for $15,000. No gain or loss is recognized on the income statement for treasury stock transactions. 핵심: When a company buys and sells its own stock, you might think there is a possibility of income statement gains and losses when purchase and sale prices are different. Although gains and losses are realized, they are never recognized on the income statement because companies shouldn't report current income or loss on transactions involving their own ownership shares. They can, however, be recognized as direct additions to and reductions from stockholder's equity on the balance sheet

No net deferred tax asset (i.e., deferred tax asset net of related valuation allowance) was recognized in the year 2 financial statements by the Chaise Company when a loss from discontinued segments was carried forward for tax purposes because it was more likely than not that none of this deferred tax asset would be realized. Chaise had no temporary differences. The tax benefit of the loss carried forward reduced current taxes payable on year 3 continuing operations. The year 3 income statement would include the tax benefit from the loss brought forward in a) Income from continuing operations. b) Gain or loss from discontinued segments. c) Extraordinary gains. d) Cumulative effect of accounting changes.

answer: a Per ASC Topic 740, the tax benefit of an operating loss carryforward or carryback shall be reported in the same manner as the source of income (loss) in the current year. The problem states that the tax benefit of the loss reduced taxes on continuing operations. Thus, in year 3, the tax benefit shall be reported under income from continuing operations

On June 30, year 2, Needle Corporation purchased for cash at $10 per share all 100,000 shares of the outstanding common stock of Thread Company. The total appraised value of identifiable assets less liabilities of Thread was $1,400,000 at June 30, year 2, including the appraised value of Thread's property, plant, and equipment (its only noncurrent asset) of $250,000. The consolidated income statement of Needle Corporation and its wholly owned subsidiary for the year ended June 30, year 2, should reflect a) A gain from bargain purchase of $400,000. b) goodwill of $150,000. c) A deferred credit (negative goodwill) of $400,000. d) Goodwill of $400,000.

answer: a Per ASC Topic 810, the excess of FV over acquisition cost is recognized as a gain from a bargain purchase in the period of acquisition. Cost $ 1,000,000 FMV of identifiable assets and liabilities (1,400,000) Bargain purchase $ (400,000)

The following events relating to the City of Albury's Debt Service Funds occurred during the year ended December 31, year 1: Debt principal matured $2,000,000 Unmatured (accrued) interest on outstanding debt at 1/1/year 1 50,000 Interest on matured debt 900,000 Unmatured (accrued) interest on outstanding debt at 12/31/year 1 100,000 Interest revenue from investments 600,000 Cash transferred from General Fund for retirement of debt principal 1,000,000 Cash transferred from General Fund for payment of matured interest 900,000 All principal and interest due in year 1 were paid on time. How much revenue should Albury's Debt Service Funds record for the year ended December 31, year 1, using modified accrual accounting? a) $ 600,000 b) $1,600,000 c) $1,900,000 d) $2,500,000

answer: a The GASB Codification Section 1800, states that transfers from a fund receiving revenue to a fund through which the resources are to be expended is considered another financing source. Interfund transfers of this nature should be distinguished from revenues, expenses, or expenditures in the financial statements. Financial resources received in other funds for transfer to the Debt Service Funds do not constitute revenue to the Debt Service Fund. Therefore, the only revenue to be recorded is the $600,000 investment income.

Utica Company's net accounts receivable were $250,000 at December 31, year 1, and $300,000 at December 31, year 2. Net cash sales for year 2 were $100,000. The accounts receivable turnover for year 2 was 5.0. What were Utica's total net sales for year 2? a) $1,475,000 b) $1,500,000 c) $1,600,000 d) $2,750,000

answer: a The amount of cash sales ($100,000) was given, so credit sales must be computed to calculate total net sales. The formula for accounts receivable turnover is AR turnover = Credit sales Average AR *여기서 중요한것은, A/R turnover는 "credit sales"/ average AR.따라서, cash sale는 빼야한다. The information given can be inserted into the above equation. Credit sales = 5.0 (250,000 + 300,000/2) Credit sales are $1,375,000. Thus, total sales are $1,475,000 ($1,375,000 credit sales + $100,000 cash sales).

Pine Corp.'s books showed pretax income of $800,000 for the year ended December 31, year 3. In the computation of federal income taxes, the following data were considered: Gain on an involuntary conversion (Pine has elected to replace the property within the statutory period using total proceeds) $350,000 Depreciation deducted for tax purposes in excess of depreciable deducted for book purposes 50,000 Federal estimated tax payments, year 3 70,000 Enacted federal tax rate, year 3 30% What amount should Pine report as its current federal income tax liability on its December 31, year 3 balance sheet? a) $ 50,000 b) $ 65,000 c) $120,000 d) $135,000

answer: a The current federal income tax liability is based on taxable income, which is computed in the "book to tax reconciliation" below. Accounting income $800,000 Nontaxable gain (350,000) Excess tax depreciation (50,000) Taxable income $400,000 The gain on involuntary conversion was included in accounting income but is deferred for tax purposes. Depreciation deducted for tax purposes in excess of book depreciation also causes taxable income to be less than accounting income. Taxes payable before considering estimated tax payments is $120,000 ($400,000 × 30%). Since tax payments of $70,000 have already been made, the 12/31/Y3 current federal income tax liability is $50,000 ($120,000 − $70,000) **핵심**First please know you do NOT create a deferred tax liability on a gain from involuntary conversion if the proceeds are to be used to buy replacement property within the allowed time. Temporary difference. For financial reporting purposes (book basis), any gain experienced in an involuntary conversion of a nonmonetary asset to a monetary asset must be recognized in the period of conversion. For tax purposes, this gain may be deferred if the total proceeds are reinvested in replacement property within a certain period of time. When such a gain is deferred, the tax basis of the replacement property is less than its carrying value and this difference will result in future taxable amounts. Hence, this is a temporary difference. DEFINITION of 'Involuntary Conversion' A process where a taxpayer is involuntarily forced to dispose of property that has been stolen, condemned, destroyed or repossessed, and another piece of property or cash is received in lieu of the property.

Bond Company leased equipment from Howe, Inc. on December 31, year 1, for a 10‐year period (the useful life of the asset) expiring December 30, year 11. Equal annual payments under the lease are $100,000 and are due on December 31 of each year. The first payment was made on December 31, year 1, and the second payment was made on the due date. The present value at December 31, year 1, of the minimum lease payments over the lease term discounted at 10% (the implicit rate computed by Howe and known by Bond) was $676,000. Bond's incremental borrowing rate was 12% at December 31, year 1. The lease is appropriately accounted for as a capital lease by Bond. What should be the balance in Bond's liability under capital lease account at December 31, year 2? a) $533,600 b) $545,120 c) $607,960 d) $800,000

answer: a The first payment (paid on the date the lease is signed) contains no interest and is, therefore, all reduction of principal. The second payment includes interest of $57,600 (10% × $576,000) and principal of $42,400 ($100,000 − $57,600). Note that because Howe's implicit interest rate of 10% is known by Bond and is lower than Bond's incremental rate, it is used to compute the interest payment.

The Morn Company leased equipment to the Lizard Company on May 1, year 1. At that time the collectibility of the minimum lease payments was not reasonably predictable. The lease expires on May 1, year 3. Lizard could have bought the equipment from Morn for $900,000 instead of leasing it. Morn's accounting records showed a book value for the equipment on May 1, year 1, of $800,000. Morn's depreciation on the equipment in year 1 was $200,000. During year 1 Lizard paid $240,000 in rentals to Morn. Morn incurred maintenance and other related costs under the terms of the lease of $18,000 in year 1. After the lease with Lizard expires, Morn will lease the equipment to the Cold Company for another 2 years. The income before income taxes derived by Morn from this lease for the year ended December 31, year 1, should be a) $ 22,000 b) $100,000 c) $122,000 d) $240,000

answer: a The lease shall be accounted for as an operating lease because none of the four requirements applicable to both lessees and lessors is met. Even if one or more was met, the lease would still be classified as an operating lease as the payments are not reasonably predictable (ASC Topic 840). The calculation for lease income for year 1 would be as follows: 조심해야하는점은, lessor의 입장에서, lessor가 얼마나 벌었는지 income을 보는것이다. not lessee's..왜 중요하나면, lessor는 operating lease같은경우, lessor입장에서 depreciation을 deduct한다. 그리고 또 조심해야하는것은, the question states that "collectibility of the minimum lease payments was not reasonably predictable" 라는 단어가 나오면, 무족건 operating lease이다.. even if capital lease qualification 조항이 맞아도, minimum lease payment not reasonably predictable가 나오면 자동적으로 operating lease. Rental income $ 240,000 Less: year 1 depreciation (200,000) Maintenance costs (18,000) Income from lease for year 1 $ 22,000

Diego Corporation values its inventory at the lower of cost or net realizable value as required by IFRS. Diego has the following information regarding its inventory. Historical cost $100,000 Estimated selling price 98,000 Estimated costs to complete and sell 3,000 Replacement cost 90,000 What is the amount for inventory that Diego should report on the balance sheet under the lower of cost or net realizable value method? a) $100,000 b) $95,000 c) $98,000 d) $97,000

answer: b IFRS requires inventory to be reported at the lower of cost or net realizable value (LCNRV). This method requires net realizable value to be calculated as the estimated selling price less estimated costs of completion and estimated costs to sell. Therefore, the NRV is $95,000 ($98,000 — $3,000). The lower of cost or net realizable value is determined by comparing the cost of $100,000 to the NRV of $95,000, and using the lower amount. Inventory should be reported at $95,000 여기서 중요한 점은, lower of cost or net realizable value method only applies to Inventory. 그리고, net realizable value method can be calculated as (Estimated selling Price - Estimated costs to complete & sell) GAAP는 IFRS rule이랑 조금 달르게, requires inventory to be reported at the lower of cost or FMV

Taft Corp. uses the equity method to account for its 25% investment in Flame, Inc. During year 2, Taft received dividends of $30,000 from Flame and recorded $180,000 as its equity in the earnings of Flame. Additional information follows: All the undistributed earnings of Flame will be distributed as dividends in future periods. The dividends received from Flame are eligible for the 80% dividends received deduction. There are no other temporary differences. Enacted income tax rates are 30% for year 2 and thereafter. In its December 31, year 2 balance sheet, what amount should Taft report for deferred income tax liability? a) $ 9,000 b) $10,800 c) $45,000 d) $54,000

answer: a This answer is correct. The deferred income tax liability is the result of the undistributed earnings of an equity investee, which are expected to be distributed as dividends in future periods. For accounting purposes (book purpose), investment revenue is $180,000, while for tax purposes, dividend revenue is $30,000, which will be partially offset by the 80% dividends received deduction. Because of this deduction, the difference ($180,000 − $30,000 = $150,000) is partially a permanent difference (80% × $150,000 = $120,000) and partially a temporary difference (20% × $150,000 = $30,000 which will be taxable in future years). This future taxable amount will be taxed at 30%, resulting in a deferred tax liability of $9,000 (30% × $30,000). Equity method results in Deferred Tax. The amount of dividends received under equity method usually reduces your investment account. Equity in earnings is reported on the I/S as Investment Income. In this case, the company reported equity in sub earnings of 180,000 and received dividends of 30,000. Difference between equity in Sub Earnings and Dividends is 150,000 (hence, this would be distributed as dividends in the future period). However, the company only owns 25% and there is 80% DRD, that means, only 20% is taxable and future tax rate is 30%. If you take 150,000 * 20% * 30% = 9,000 DTL I hope I'm about to say this right and not confuse both of us. The equity earnings are added to Taft Corps books (book purpose) for the year. They received dividends that were eligible for the DRD. The amount of Flame's earnings times the DRD rate is what Taft will eventually receive from Flame. They will get to take the DRD when they receive them. Since Flame reported $180,000 in income that they did not distribute in dividends then Taft will eventually have to pay tax on their portion of Flame's income less the DRD.

The following data relate to a construction job started by Syl Co. during year 1: Total contract price $100,000 Actual costs during year 1 20,000 Estimated remaining costs 40,000 Billed to customer during year 1 30,000 Received from customer during year 1 10,000 Under the completed‐contract method, how much should Syl recognize as gross profit for year 1? a) $0 b) $4,000 c) $10,000 d) $12,000

answer: a When a company uses the completed‐contract method of accounting for construction projects, all revenue and expense recognition is deferred until the project is complete or substantially complete (ASC Topic 605). Because there is an estimated $40,000 of remaining costs, the contract cannot be considered to be substantially complete. Thus, no revenue, expenses, or gross profit would be recognized by Syl Co. in year 1 using this method. Don't get confused with percentage method & completed-contract method The difference between methods is simply a question of timing—the percentage method recognizes profit little by little over time, while the completed-contract method defers the entire profit until completion.

On January 1, year 1, Rodriguez Corp. granted stock options to corporate executives for the purchase of 10,000 shares of the company's $20 par value common stock at 70% of the market price on the exercise date, December 30, year 1. On January 1, year 1, no market price or estimate could be made for the value of the options. All stock options were exercised on December 30, year 1. The quoted market prices of Rodriguez Corp.'s $20 par value common stock were as follows: January 1, year 1 $50 per share December 30, year 1 $60 per share As a result of the exercise of the stock options and the issuance of the common stock, Rodriguez should recognize compensation expense in year 1 of a) $180,000 b) $200,000 c) $500,000 d) $600,000

answer: a When there is no observable market price of the option or estimate of the value of the option, the intrinsic value is measured at the end of each reporting period. The intrinsic value of the option is net of any amounts that the employee must pay. The employees must pay 70% of the value of the stock on the exercise date. Therefore, the employees will pay 70% × $60 = $42 per share. On the exercise date, the compensation expense recognized is $60 − $42 = $18 × 10,000 options = $180,000.

Lind Corp. declared a cash dividend of $50,000 on March 10, year 2, to stockholders of record March 25, year 2, payable on April 5, year 2. As a result of this cash dividend, working capital a) Decreased on March 10 by $50,000. b) Decreased on March 25 by $50,000. c) Decreased on April 5 by $50,000. d) Did not change.

answer: a Working capital equals current assets less current liabilities. Any transactions which affect either current assets or current liabilities, but not both, will affect working capital. If a transaction affects both current assets and current liabilities, the effects will offset and there will be no change in working capital. When a corporation declares a cash dividend on its stock, its retained earnings are decreased and its current liabilities (Dividends Payable) are increased. When the cash dividend is paid, the Dividends Payable account is decreased and the corporation's Cash account is decreased.

Falton Company had the following first-year amounts related to its $9,000,000 construction contract: Actual costs incurred and paid $2,000,000 Estimated costs to complete 6,000,000 Progress billings 1,800,000 Cash collected 1,500,000 What amount should Falton recognize as a current liability at year end, using the percentage-of completion method? a. $0 b. $200,000 c. $250,000 d. $300,000

answer: a Zero. The company has billed the customer $1,800,000, which is less than the total expenses incurred to date, $2,000,000 (similiar to prepaid?) This will be an asset, similar to accounts receivable. If the customer had billed more than the expenses incurred, it would be a liability, similar to unearned revenue. let's say, company billed customer 2,500,000 instead of of billing 1,800,000. If $2,500,000 was billed, then $500,000 will be current liability. (2,500,000 -2,000,000 Actual costs incurred and paid) Journal Entry is: When bills were sent to customer: DR Account Receivable $2500k CR Billings on Construction $2500k Billings on Construction is a contra-asset accounts of Construction In Progress (CIP), an asset account. It reduces CIP: On the balance sheet: Construction In Progress $2000k Less: Billings on Construction $(2500k) Balance on Billings on Construction is $(500) Construction expenses are recorded in CIP. When Billings on Construction is higher than CIP, that means you bills customer more than your expenses, the extra money you bill is not yet earned.

Which of the following inventory methods is not allowed under IFRS? a) LIFO. b) FIFO. c) Weighted‐average. d) Specific identification.

answer: a because the LIFO method for inventory is not allowed under IFRS. However, FIFO, weighted‐average, and specific identification (for certain inventory) are allowed.

Initial direct costs are: a) Expensed currently for sales‐type leases. b) Capitalized and amortized to expense over the lease term for all leases. c) Capitalized only if the related lease qualifies as a capital lease. d) Presented on the balance sheet as a contra account to capitalized leased assets.

answer: a initial direct costs are often incurred and include amounts such as commissions, legal fees, and internal costs that are incremental and directly attributable to negotiating and arranging a lease.

In a statement of cash flows, interest payments to lenders and other creditors should be classified as cash outflows for a) Operating activities. b)Borrowing activities. c)Lending activities. d) Financing activities.

answer: a interest payments to lenders and other creditors are categorized as cash flows from operating activities. (1) Operating activities include cash activities related to net income. For example, cash generated from the sale of goods (revenue) and cash paid for merchandise (expense) are operating activities because revenues and expenses are included in net income. (Also, it includes paying interest payment to creditors or lender. <--착각하기 쉬운게 interest payment to creditor may sounds likes it financing activity but it's actually operating activity) (2) financing activities include cash activities related to noncurrent liabilities and owners' equity. Noncurrent liabilities and owners' equity items include (1) the principal amount of long-term debt, (2) stock sales and repurchases, (ex: Issued 500 shares of common stock) and (3) dividend payments. (Note that interest paid on long-term debt is included in operating activities.)

Hospital, Inc., a not‐for‐profit organization with no governmental affiliation, reported the following in its accounts for the current year ended December 31: Gross patient service revenue from all services provided at the established billing rates of the hospital (note that this figure includes charity care of $25,000) $775,000 Provision for bad debts 15,000 Difference between established billing rates and fees negotiated with third‐party payors (contractual adjustments) 70,000 What amount would the hospital report as net patient service revenue in its statement of operations for the current year ended December 31? a) $665,000 b) $690,000 c) $705,000 d) $735,000

answer: a not-for-profit hospital 경우엔, net patient service revenue in its statement of operations must not include any charity care. charity care란, 어려운 사람들한테 거의 돈을 안받고 의료 서비스를 줄때를 얘기한다. 따라서, 병원의 operating income에서 charity care를 제외한 그외에 환자를 돌보면서 생긴 이익이 main revenue가 될껏이다. 여기서, bad debt제외하고, contractual adjustment를 제외하면 된다. $775,000 - $25,000 charity care -$15,000 bad debt- $70,000 contractual adjustment = $665,000 note that charity care $25,000 should be disclosed in the notes to the financial statements.

Fogg Co., a US company, contracted to purchase foreign goods. Payment in foreign currency was due 1 month after the goods were received at Fogg's warehouse. Between the receipt of goods and the time of payment, the exchange rates changed in Fogg's favor. The resulting gain should be included in Fogg's financial statements as a(n) a) Component of income from continuing operations. b)Extraordinary item. c) Deferred credit. d) Component of "other comprehensive income" and stockholders' equity.

answer: a the gain should be recognized as income. Typically, other comprehensive income (OCI) are: Unrealized holding gains or losses on investments that are classified as available for sale. Foreign currency translation gains or losses. Pension plan gains or losses. 여기서 OCI에 포함되서 나오는 "Foreign Currency Translation"은, when when you are redoing the financial statement from the functional currency (your local operating currency) to a different reporting currency.* <-이렇게 Convert때 생긴 이익/손액은, OCI로 들어간다. 중요한건, "Do not confuse translation for transactions. All foreign currency transactions hit income statement." 질문에서 나온, foreign currency transaction 은 income statement로 고고씽!! F/S를 translate할때, 즉, convert할시에 생긴 차액 이익은, "foreign tranlsation"이라고 불른다. (translate my F/S to other country's F/S...) "foreign tranlsation's" gain or loss report to OCI (remember, OCI comes after net income therefore doesn't not affect I/S but it goes to B/S instead). For the simple "foreign currency transaction", the loss/gain affects income statement.

Harland County received a $2,000,000 capital grant to be equally distributed among its five municipalities. The grant is to finance the construction of capital assets. Harland had no administrative or direct financial involvement in the construction. In which fund should Harland record the receipt of cash? a) Agency fund. b) General fund. c) Special revenue fund. d) Private purpose trust fund. You Answered Incorrectly.

answer: a 여기서, 중요하게 생각해야 하는것은, Harland county의 주요 목적은, it equally "distribute" the funds/grants to other government sector. 그냥 전달해주는 임무를 갖고있고, it doesn't have direct involvement with them. The cash should be recorded in the agency fund. Agency funds are used to account for the assets held for distribution by the state as an agent for another entity for which the government has custodial responsibility and accounts for the flow of assets.

What is a reclassification adjustment as used when reporting comprehensive income? a) Adjustment made to avoid double counting items. b)Adjustment made to reclassify an item of comprehensive income as another item of comprehensive income. c) Adjustment made to make net income equal to comprehensive income. d) Adjustment made to adjust for the income tax effect of reporting comprehensive income.

answer: a 이게 무슨뜻이나면, Other comprehensive income contains all changes that are not permitted to be included in profit or loss. It is particularly valuable for understanding ongoing changes in the fair value of a company's assets. Items that you should insert in other comprehensive income include: (1.) Available-for-sale securities fair value changes that were previously written down as impaired (2) Available-for-sale securities unrealized gains and losses (3) Cash flow hedge derivative instrument gains and losses (4) Debt security unrealized gains and losses arising from a transfer from the available-for-sale category to the held-to-maturity category (5) Foreign currency gains and losses on intra-entity currency transactions where settlement is not planned or anticipated in the foreseeable future (6)Foreign currency transaction gains and losses that are hedges of an investment in a foreign entity (7)Foreign currency translation adjustments (8) Pension or post-retirement benefit plan gains or losses (9) Pension or post-retirement benefit plan prior service costs or credits (10) Pension or post-retirement benefit plan transition assets or obligations that are not recognized as a component of the net periodic benefit or cost ***If an item listed in other comprehensive income becomes a realized gain or loss, you then shift it out of other comprehensive income and into net income or loss. This can happen, for example, when you sell an investment security for which you already recorded an unrealized gain in other comprehensive income. At the point of sale, this is now a realized gain, which shifts into net income. A company can display this reclassification adjustment either on the face of the financial statements, or in the accompanying notes.*** A reclassification adjustment is an adjustment made to avoid double counting in comprehensive income items that are displayed as part of net income for a period that also had been displayed as part of other comprehensive income in that period or earlier periods.

Redwood Co.'s financial statements had the following information at year‐end: Cash $60,000 Accounts receivable 180,000 Allowance for uncollectible accounts 8,000 Inventory 2 40,000 Short‐term marketable securities 90,000 Prepaid rent 18,000 Current liabilities 400,000 Long‐term debt 220,000 What was Redwood's quick ratio? a) 0.81 to 1 b) 0.83 to 1 c) 0.94 to 1 d) 1.46 to 1

answer: a 핵심: Quick ration formula = (Cash + net accounts receivable + short‐term marketable securities) / Current Liability. *여기서, net accounts receivable란, A/R minus allowance for uncollectible accounts. The quick ratio is calculated by dividing quick assets by current liabilities. Quick assets are those current assets that can be quickly converted to cash. Because inventory and prepaid rent are not quickly converted to cash, they are not classified as quick assets. Cash, net accounts receivable, and short‐term marketable securities are the quick assets in this case. Therefore, this answer is correct because the quick ratio is equal to 0.81 to 1 [($60 + $180 − $8 + $90)/$400].

Gains and losses on the hedged asset/liability and the hedged instrument for a fair value hedge will be recognized a) In current earnings. b) In other comprehensive income. c) On a cumulative basis from the change in expected cash flows from the hedged instrument. d) On the balance sheet either as an asset or a liability.

answer: a . Gains and losses of a fair value hedge will be recognized in current earnings. The effective portion of a cash flow hedge is reported in other comprehensive income and the ineffective portion is reported on a cumulative basis to reflect the lesser of the cumulative gain/loss on the derivative or the cumulative gain/loss from the change in expected cash flows from the hedged instrument. Gains and losses from a change in the fair value of derivative instruments are not assets and liabilities and should not be reported on the balance sheet. Fair value hedge is a hedge of the exposure to changes in fair value of a recognized asset or liability or unrecognized firm commitment, or a component of any such item, that is attributable to a particular risk and could affect profit or loss.

The year 1 financial statements of Bice Company reported net income for the year ended December 31, year 1, of $2,000,000. On July 1, year 2, subsequent to the issuance of the year 1 financial statements, Bice changed from an accounting principle that is not generally accepted to one that is generally accepted. If the generally accepted accounting principle had been used in year 1, net income for the year ended December 31, year 1, would have been decreased $1,000,000. On August 1, year 2, Bice discovered a mathematical error relating to its year 1 financial statements. If this error had been discovered in year 1, net income for the year ended December 31, year 1, would have been increased $500,000. What amount, if any, should be included in net income for the year ended December 31, year 2, because of the items noted above? a) $0. b) $ 500,000 decrease. c) $ 500,000 increase. d) $1,000,000 decrease.

answer: a The change from an unacceptable accounting principle to an acceptable accounting principle is considered a correction of an error per APB 20. Thus both of these items are corrections of errors and as such are reported as prior period adjustments. Prior period adjustments are reported in the retained earnings statement and not in the income statement 핵심: change from unacceptable accounting to acceptable accounting principle is correction of error which requires prior period adjustment in retained earning which is in Balance sheet and not in income.

Rollin Corporation purchases 100 shares of stock in Boyle Corp., and classifies the investment as trading securities. Rollin should report these trading securities at a) Lower of cost or market, with holding gains and losses included in earnings. b) Lower of cost or market, with holding gains included in earnings only to the extent of previously recognized holding losses. c) Fair value, with holding gains included in earnings only to the extent of previously recognized holding losses. d) Fair value, with holding gains and losses included in earnings. Trading securities are securities purchased and held principally for the purpose of generating gains on current resale. Trading securities should be reported in the balance sheet at fair value, with holding gains and losses included in earnings.

answer: answer: d Trading securities are securities purchased and held principally for the purpose of generating gains on current resale. Trading securities should be reported in the balance sheet at fair value, with holding gains and losses included in earnings. remember, if investment is less than 20% it is cost method, and if investment is more than 20% but less than 50% it is equity method. Anyway, there are 2 types of investment types. Investment type can be either (1) trading security or (2) available for sale" If the investment is considered a "trading security" or stock purchased for the purpose of selling it in the near term, (you plan to trade within a year), the balancing debit or credit is charged to an unrealized loss or gain reported on the income statement. (AFFECTING INCOME STATEMENT. REPORTING ON INCOME STATEMENT) If the investment is an "available for sale" security, the balancing debit or credit goes to an unrealized loss or gain account reported in the other comprehensive income section of owner's equity on the balance sheet. (REPORTING ON OTHER COMPREHENSIVE INCOME SECTION OF B/S) When the investment is sold, all losses or gains from the transaction become realized and flow through into the income statement to adjust revenues for the period. ***Trading securities are always current assets. Available-for-sale securities may be either current assets or noncurrent assets, depending on how long management intends to hold them.

Hiller Company manufactures equipment which is sold or leased. On December 31, year 1, Hiller leased equipment to Drake Company for a 5‐year period expiring December 31, year 6, at which date ownership of the leased asset is transferred to Drake. Equal payments under the lease are $20,000 and are due on December 31 of each year. The first payment was made on December 31, year 1. Collectibility of the remaining lease payments is reasonably assured, and Hiller has no material cost uncertainties. The normal sales price of the equipment is $77,000 and Hiller's cost is $60,000. For the year ended December 31, year 1, how much income should Hiller recognize from the lease transactions? a) $0 b) $17,000 c) $20,000 d) $23,000

answer: b The lease is a sales‐type lease because title to the leased asset transfers, collectibility is reasonably assured, there are no material cost uncertainties, and a manufacturer's profit exists. Therefore, the lessor would recognize sales of $77,000 and cost of sales of $60,000, resulting in a profit of $17,000. There is no interest income in year 1 since the sale occurs on the last day of the year. Sales-type lease accounting. A lease is classified as a sales-type lease by the lessor when the fair value of the leased property at the start of a lease varies from its carrying amount, it involves real estate, and there is a transfer of ownership to the lessee by the end of the lease term.

On the December 31, year 1 balance sheet of the Stat Company, the current assets were comprised of the following items: Cash $ 70,000 Accounts receivable 120,000 Inventories 60,000 An examination of the accounts revealed that the accounts receivable ($120,000) were composed of the following items: Trade accounts $ 93,000 Allowance for uncollectible accounts (2,000) Claim against shipper for goods lost in transit (11/Y1) 3,000 Selling price of unsold goods sent by Stat Company on consignment at 130% of cost (and not included in Stat's ending inventory) 26,000 What is the correct amount of current assets as of 12/31/Y1? a) $221,000 b) $224,000 c) $244,000 d) $250,000

answer: b the value of Marie's AR includes items out on consignment AS IF THEY HAD BEEN SOLD. This is not GAAP treatment. The value of these items should be included in inventory. Since they are valued at 130% of cost, the real cost can be determined as 26000/1.30 = 20,000. The final calculation of current assets is as follows: 70,000 cash 96,000 ar (trade accounts) 20,000 inventory on consignment (26000/1.30) 60,000 inventory given (2,000) allowance for uncollectible accts. _________ 244,000 "Selling price of Marie's unsold goods" means the consignment inventory is recorded at its expected sales price which is projected to be 130% of cost. it should just be in inventory at cost.

Based upon its past collection experience, Alden Company provides for bad debt expense at the rate of 2% of credit sales. On January 1, year 1, the allowance for doubtful accounts balance was $10,000. During year 1 Alden wrote off $18,000 of uncollectible receivables and recovered $5,000 of bad debts written off in prior years. If credit sales for year 1 totaled $1,000,000, the allowance for doubtful accounts balance at December 31, year 1, should be a) $12,000 b) $17,000 c) $20,000 d) $30,000

answer: b (쉽게 생각해서 이 계산번의 계념은 내가 regulation 시험공부 할떄 자주 나왔던, cogs 계사하는거랑 똑같다. Beginning balance + acquired item - cosg (actual use) = ending balance) The ending balance of the allowance for doubtful accounts includes the Beginning balance + Recoveries of bad debts written off in prior years + Current year's bad debt expense − Write‐offs of uncollectibles. The beginning balance was $10,000 and write‐offs (debits) were $18,000. Recoveries of bad debts written off in prior years ($5,000) would be a credit to the allowance account and a debit to accounts receivable. Finally, the credit to the allowance account for bad debts expense would be $20,000 (2% of $1,000,000), leaving a 12/31/year 1 balance of $17,000.

On January 1, year 1, Hooks Oil Co. sold equipment with a carrying amount of $100,000, and a remaining useful life of 10 years, to Maco Drilling for $150,000. Hooks immediately leased the equipment back under a 10‐year capital lease with a present value of $150,000 and will depreciate the equipment using the straight‐line method. Hooks made the first annual lease payment of $24,412 in December year 1. In Hooks' December 31, year 1 balance sheet, the unearned gain on equipment sale should be a) $50,000 b) $45,000 c) $25,588 d) $0

answer: b According to ASC Topic 840, sale‐leaseback transactions are treated as though two transactions were a single financing transaction, if the lease qualifies as a capital lease. Any gain on the sale is deferred and amortized over the lease term (if possession reverts to the lessor) or the economic life (if ownership transfers to the lessee). Since this is a capital lease, the entire gain ($150,000 − $100,000 = $50,000) is deferred at 1/1/Y1. At 12/31/Y1 an adjusting entry must be prepared to amortize 1/10 of the unearned gain (1/10 × $50,000 = $5,000), because the lease covers 10 years. Therefore, the unearned gain at 12/31/Y1 is $45,000 ($50,000 − $5,000). In this problem, this is clearly, capital lease agreement because PV $150,000 is greater than 90% of 100,000. Also, useful life of equipment is 10 year and agreemen tis 10 year.

During the current year, Comma Co. had outstanding: 25,000 shares of common stock, 8,000 shares of $20 par, 10% cumulative preferred stock, and 3,000 bonds that are $1,000 par and 9% convertible. The bonds were originally issued at par, and each bond was convertible into thirty shares of common stock. During the year, net income was $200,000, no dividends were declared, and the tax rate was 30%. What amount was Comma's basic earnings per share for the current year? a) $3.38 b) $7.36 c) $7.55 d) $8.00

answer: b Basic earnings per share is calculated by dividing earnings available to common shareholders by the weighted‐average number of common shares outstanding. In this case, earnings available to common shareholders is equal to net income ($200,000) minus preferred dividends $16,000 (8,000 × $20 × 10%). Therefore, this answer is correct because basic earnings per share is equal to $7.36 [($200,000 − $16,000) ÷ $25,000]. On Basic EPS you must remove from NI the: ---Declared dividends on non-cumulative prefered stock(whether paid or not) ---Accumulated dividends on cumulative prefered stock(whether declared or not) In determining basic earnings per share, dividends on nonconvertible cumulative preferred stock should be Deducted from net income whether declared or not. if the preferred stock is cumulative, then any dividends not paid in 1 year must be paid in the following years before the common shareholder can receive a dividend. Therefore, in computing EPS, the dividends on nonconvertible cumulative preferred stock must be subtracted from net income before the adjusted amount is divided by the weighted‐average of common shares outstanding.

Pak Co.'s professional fees expense account had a balance of $82,000 at December 31, year 1, before considering year‐end adjustments relating to the following: • Consultants were hired for a special project at a total fee not to exceed $65,000. Pak has "recorded" $55,000 of this fee based on billings for work performed in year 1. • The attorney's letter requested by the auditors dated January 28, year 2, indicated that legal fees of $6,000 were billed on January 15, year 2, for work performed in November year 1, and unbilled fees for December year 1 were $7,000. What amount should Pak report for professional fees expense for the year ended December 31, year 1? a) $105,000 b) $ 95,000 c) $ 88,000 d) $ 82,000

answer: b Before making any adjustment at year-end, professional fee expense account has balance of $82,000. $82,000안에, 이미 company가 "recorded" $55,000 fee를 기록한것으로 보여진다. 다만, legal fee $6,000 +7,000= $13,000은 accrual기록이 안해났다. 아마 legal fee에 대한 letter를 year end가 지난후에 받아서 그런지.... 따라서 $82,000 +13,000 legal fee= $95,000 should be adjusted professional fee at year end.

During year 2, Kam Co. began offering its goods to selected retailers on a consignment basis. The following information was derived from Kam's year 2 accounting records: Beginning inventory $122,000 Purchases 540,000 Freight‐in 10,000 Transportation to consignees 5,000 Freight‐out 35,000 Ending inventory—held by Kam 145,000 —held by consignees 20,000 In its year 2 income statement, what amount should Kam report as cost of goods sold? a) $507,000 b)$512,000 c)$527,000 d)$547,000

answer: b Cost of goods sold is $512,000 as computed below. Beginning inventory $122,000 Purchases $540,000 Freight‐in 10,000 Transportation to consignees 5,000 555,000 Cost of goods available for sale 677,000 Less ending inventory 165,000 ($145,000 + $20,000) $512,000 Cost of goods sold **Consignee hold inventory랑 transportation cost incurred to for consignee project must all need to add back.**아직 consginee가 그 물건을 성공적으로 팔지 않았더라면, consginee가 갖고있는 물건들은 아직 나의 소유라고 생각한다** Additional Explanation: Transportation-in or freight-in costs are part of the cost of goods purchased. The cost of goods (or any asset) includes all costs necessary to get an asset in place and ready for use. Transportation-in costs are allocated to the products purchased and will "cling" to the products. Those products in inventory (items not yet sold) will include their share of the transportation-in costs (as part of the inventory cost). The products that have been sold, will include their share of the transportation-in costs (as part of the cost of goods sold). Transportation-out or freight-out costs are not product costs and are not inventoriable. Transportation-out costs are costs of selling the products and will appear as a selling expense (perhaps as Delivery Expense) in the period in which they occur.

Information concerning the capital structure of the Petrock Corporation is as follows: December 31, Year 1 Year 2 Common stock 90,000 shares 90,000 Convertible preferred stock 10,000 shares 10,000 shares During year 2, Petrock paid dividends of $1.00 per share on its common stock and $2.40 per share on its preferred stock. The preferred stock is convertible into 20,000 shares of common stock. The net income for the year ended December 31, year 2, was $285,000. Assume that the income tax rate was 30%. What should be the diluted earnings per share for the year ended December 31, year 2, rounded to the nearest penny? a) $2.53 b) $2.59 c) $2.90 d) $2.61

answer: b Diluted EPS = (net income - preferred dividend) + convertible preferred dividend + (convertible bond x interest %) -(income tax rate on (convertible bond x interest %))/ weighted average of dilutive common shares + unexercised employee stock options + convertible preferred shares + convertible debt. Diluted EPS includes the effect of any dilutive security. Both the convertible bonds and convertible preferred stock must be tested for dilution. To calculate the basic EPS, in the numerator, reported income of $285,000 would be reduced by the $24,000 of preferred dividends which gives $261,000 available for common stockholders. In the denominator 90,000 shares is the weighted‐average of shares outstanding during the year. The conversion of the preferred stock will have an income effect of $24,000 (there is no tax effect) to the numerator and increase the denominator by 20,000 shares. DEPS is calculated as follows: ($261,000 + $24,000) / (90,000 shs + 20,000 shs) = $2.59

In a sale‐leaseback transaction, a gain resulting from the sale should be deferred at the time of the sale‐leaseback and subsequently amortized when I. The seller‐lessee has transferred substantially all the risks of ownership. II. The seller‐lessee retains the right to substantially all of the remaining use of the property. a) I only. b) Neither I nor II.. c) II only. d) Both I and II.

answer: b Explanation Choice "c" is correct. Recognition of a gain resulting from the sale in a sale-leaseback should be deferred when the seller-lessee retains the right to substantially all of the remaining use of the property (as in a capital lease). Choice "a" is incorrect. When the seller-lessee transfers substantially all the risks of ownership (as in a true sale), any gain resulting from the sale should be recognized immediately.Choice "d" is incorrect. When the seller-lessee transfers substantially all the risks of ownership, any gain resulting from the sale should be recognized rather than being deferred.Choice "b" is incorrect. Recognition of the gain should be deferred when the seller-lessee retains the right to substantially all of the remaining use of the property

Kollar Corp.'s transactions for the year ended December 31, year 2, included the following: Purchased real estate for $550,000 cash which was borrowed from a bank. Sold investment securities for $500,000. Paid dividends of $600,000. Issued 500 shares of common stock for $250,000. Purchased machinery and equipment for $125,000 cash. Paid $450,000 toward a bank loan. Reduced accounts receivable by $100,000. Increased accounts payable by $200,000. Kollar's net cash from financing activities for year 2 was a) $ 50,000 b)$250,000 c)$450,000 d) $500,000

answer: b Financing activities include all cash flows involving liabilities and equity other than operating items. The financing activities are Bank borrowing $ 550,000 Dividend payment (600,000) Issuance of stock 250,000 Bank loan payment (450,000) Net cash used in financing activities $(250,000) (핵심: bank borrow는 inflow of cash. 빌린돈이라도 내 통장으로 들어왔으니 우선 내 cash. Dividend는 내가 갖고있었던, long term liability중에 하나의 임무.. Dividend 지불되면, cash가 준다. 하지만, 내가 stock을 investor/shareholder한테 issue하면 그만큼 팔리는 돈이 수금이 된다. Notice, issuance of stock is positive, in-flow. Bank long payment도 나의 long term liabilty이다. Notice, "net cash used in fianancing activities" means, sum of all the inflow/outflow. ) The purchase of real estate ($550,000), sale of investment securities ($500,000), and purchase of machinery and equipment ($125,000) are investing activities. The reduction of accounts receivable ($100,000) and the increase in accounts payable ($200,000) are operating items.

Assume fixed assets purchased from General Fund revenues are received; what account, if any, should be debited in the General Fund? a) No journal entry should be made in the General Fund. b) Expenditures Control. c) Fixed Assets. d) Asset Expense.

answer: b Fixed Assets purchased with General Fund revenues are not assets of the General Fund but are assets of the governmental unit as a whole which will be reported in the government‐wide financial statements. when fixed asset is received, journal entry will be Expenditure control (debit) Accounts Payable (Credit) *basically, expenditure controls means same as expenditure.*

On January 1, year 1, Grade Company paid $300,000 for 20,000 shares of Medium Company's common stock which represents a 15% investment in Medium. Grade does not have the ability to exercise significant influence over Medium. Medium declared and paid a dividend of $1 a share to its stockholders during year 1. Medium reported net income of $260,000 for the year ended December 31, year 1, and had a "market value of $300,000 at December 31, year 1". The balance in Grade's balance sheet account "Investment in Medium Company" at December 31, year 1, should be a) $280,000 b) $300,000 c) $319,000 d) $339,000 .

answer: b Grade Company will account for this investment using the "fair value method". This method is used because Grade owns less than 20% of Medium and cannot exercise significant influence over the company. **Fair value method option" only applies to c/s holding less than 20% there, fore it only applies to cost method, (does not apply to equity and consolidated)

On January 1, year 1, Grade Company paid $300,000 for 20,000 shares of Medium Company's common stock which represents a 15% investment in Medium. Grade does not have the ability to exercise significant influence over Medium. Medium declared and paid a dividend of $1 a share to its stockholders during year 1. Medium reported net income of $260,000 for the year ended December 31, year 1, and had a market value of $300,000 at December 31, year 1. The balance in Grade's balance sheet account "Investment in Medium Company" at December 31, year 1, should be a) $280,000 b) $300,000 c) $319,000 d) $339,000

answer: b Grade Company will account for this investment using the fair value method. This method is used because Grade owns less than 20% of Medium and cannot exercise significant influence over the company. security (common stock도 포함)한게 less than 20%를 소유하고 있으면, equity method가 아니고, cost method...그리고, less than 20%소유여야지만, Fair value method를 행사할수있다. Under US GAAP, when purchasing less than 20% of a company's stock, the cost method is used to account for the investment. As required by FAS 115, investments accounted for under the cost method should be adjusted to current fair value at the end of each accounting period, in cases where the fair value is readily determinable. "under cost method, which is owning securities less than 20%, is required to ADJUST its investment balance to current Fair Value at end of the year, when Fair value is readily determinable" !!!!!!!!!!

On October 1, year 1, Dean Company leased office space at a monthly rental of $30,000 for 10 years expiring September 30, year 11. As an inducement for Dean to enter into the lease, the lessor permitted Dean to occupy the premises rent‐free from October 1 to December 31, year 1. For the year ended December 31, year 1, Dean should record rent expense of a) $0 b) $29,250 c) $87,750 d) $90,000

answer: c ASC Topic 840 states that rent on operating leases should be expensed on a straight‐line basis unless another method is better suited to the particular benefits and costs associated with the lease. In this lease, the lessee must pay rent of $30,000 monthly for 10 years less the first 3 months, or 117 months (120 − 3). Therefore, total rent expense for the 10 years is $3,510,000 (117 × $30,000). Recognizing rent expense on a straight‐line basis, year 1 rent expense is $87,750 ($3,510,000 × 3/120).

A lessee incurred costs to construct walkways to improve leased property. The estimated useful life of the walkways is 15 years. The remaining term of the nonrenewable lease is 20 years. The walkway costs should be a) Capitalized as leasehold improvements and depreciated over 20 years. b) Capitalized as leasehold improvements and depreciated over 15 years. c) Capitalized as leasehold improvements and expensed in the year in which the lease expires. d) Expensed as incurred.

answer: b Leasehold improvements are properly capitalized and amortized over the remaining life of the lease, or the useful life of the improvements, whichever is shorter. Since the useful life of the walkways is only 15 years and the remaining term of the lease is 20 years, the cost should be depreciated over the 15‐year period. Leasehold improvements are defined as the enhancements paid for by a tenant to leased space. Examples of leasehold improvements are: Interior walls and ceilings Electrical and plumbing additions Built-in cabinetry Carpeting and tiles Leasehold improvements generally revert to the ownership of the landlord upon termination of the lease, unless the tenant can remove them without damaging the leased property. An example of leasehold improvements is offices constructed in unfinished office space. When you pay for leasehold improvements, capitalize them if they exceed the corporate capitalization limit. If not, charge them to expense in the period incurred. If you capitalize these expenditures, then amortize them over the shorter of their useful life or the remaining term of the lease. The remaining term of the lease for amortization purposes can be extended into additional lease renewal periods if the renewal is reasonably assured (such as when there is a bargain renewal option).

In a statement of cash flows, payments to acquire debt instruments of other entities (other than cash equivalents) which will be held until maturity should be classified as cash outflows for a) Operating activities. b) Investing activities. c) Financing activities. d) Lending activities.

answer: b Making payments to acquire debt instruments of other entities is not a financing activity. Per ASC Topic 230, investing activities include making and collecting loans and acquiring and disposing of debt or equity instruments and property, plant, and equipment. Per ASC Topic 230, cash flows from transactions in held‐to‐maturity and available‐for‐sale securities are to be classified as cash flows from investing activities. Conversely, amounts related to securities held for trading are classified as operating activities.

On January 1, year 1, Brecon Co. installed cabinets "to display its merchandise" in customers' stores. Brecon expects to use these cabinets for 5 years. Brecon's year 1, multi‐step income statement should include (여기서 핵심은, Brecon Company installed cabinet "to display it's merchandise" in customer's store이다. 키친 싱크대를 만드는 회사가 IREA 매점에 본인의 디자인의 싱크대 샘플을 갖다 놓고 전시 해놓는이유는, 본인 회사의 물건을 promote할려고 하는 이유이다) a) One‐fifth of the cabinet costs in cost of goods sold. b) One‐fifth of the cabinet costs in selling, general, and administrative expenses. c) All of the cabinet costs in cost of goods sold. d) All of the cabinet costs in selling, general, and administrative expenses.

answer: b One‐fifth of the cabinet costs would be reported as depreciation expense in selling, general, and administrative expenses. Four‐fifths of the cabinet cost would remain capitalized as fixed assets at the end of year 1. (A capitalized cost is an expense associated with a fixed asset that is added to the basis of that asset and expensed over its depreciable life.) SG&A is the acronym for selling, general and administrative. SG&A are the expenses incurred to 1) promote, sell, and deliver a company's products and services, and 2) manage the overall company. Examples of SG&A include sales commissions, advertising, promotional materials, compensation of the company's officers as well as the marketing, sales, finance and office staffs, the rent, utilities, supplies, computers, etc. that are outside of the manufacturing function

The board of trustees of Blue College, a private not‐for‐profit college, established a $500,000 quasi endowment on September 1, year 1. On the college's statement of financial position at December 31, year 1, the assets in this quasi endowment should be included in which of the following classifications? a) Temporarily restricted net assets. b) Unrestricted net assets. c)Permanently restricted net assets. d) Either temporarily or permanently restricted net assets, depending on the expected term of the quasi endowment.

answer: b Quasi endowments do not create real restrictions. Quasi endowment is generally unrestricted invested income. This is resources that needs to be invested permanently. 내부적으로 사용될 목적이있다. resources are presumed to be unrestricted unless evidence exist that donor-imposed restriction exist.

In which of the following funds would it be appropriate to record depreciation of fixed assets? a) Capital Projects. b) General. c) Internal Service. d) Special Revenue.

answer: c Fixed assets arising from expenditures by this governmental fund are displayed only in the government‐wide statements. 참고로, internal service fund란, reimbursement basis로, 아무런 이익안남기고 그냥 cost 비용만 돌려받는건...왜냐면 user들이 internal gov entity니깐.. ex: motor pools. gov 차량을 하나 사면, 이차가 놀고있을때 다른 entity가 차량을 빌려가서 쓰고 돈을 준다.. 그럼 여러대를 안사도 되고 같이 돌려쓰면 좋고..

Carlson Hospital, a nonprofit hospital affiliated with Carlson College, had the following cash receipts for the year ended December 31, 2012: Collections of health care receivables $850,000 Contribution from donor for term endowment $150,000 Tuition from nursing school $50,000 Dividends permanent endowment investment $75,000 The dividends received are restricted by the donor for hospital building improvements. No improvements were made during 2012. On the hospital's statement of cash flows for the year ended December 31, 2012, what amount of these cash receipts would be included in the amount reported for net cash provided (used) by operating activities? a. $1,050,000 b. $900,000 c. $975,000 d. $850,000

answer: b The cash flows from revenues, gains, and other support, which are reported on the hospital's statement of operations, would be included in the net cash provided (used) by operating activities on the statement of cash flows. Both are included in the amount reported for revenue, gains, and other support on the hospital's statement of operations. Accordingly, cash received from patient service revenue and from tuition revenue are both included in the amount reported for cash flows from operating activities. The cash received for the term endowment as well as the cash received from dividends would not be included in the amount reported for net cash provided (used) by operating activities. Both of these cash receipts would be reported as increases in cash flows provided by financing activities. Cash contributions that are donor-restricted for long-term purposes are reported as financing activities on the statement of cash flows. In addition, the AICPA Audit and Accounting Guide, Health Care Organizations, states that cash received for long-term purposes, for example, the cash received for the term endowment and the building improvements, is not reported as a current asset. So that the statement of cash flows will reconcile with the change in cash and cash equivalents reported as current assets on the balance sheet, an amount equal to the cash received for the two financing activities is included in the amount reported for cash flows from investing activities. For Carlson Hospital, this would mean that $225,000, the sum of the $150,000 for the term endowment and the $75,000 of restricted dividends, is reported as a negative amount in the investing activities' section of the statement of cash flows. Note that both of these amounts are reported as investing activities whether the cash was spent this period or in subsequent period(s).

Which of the following accounts should Moon City close at the end of its fiscal year? a) Vouchers Payable. b) Expenditures. c) Fund balance—Unreserved. d) Fund balance‐Assigned.

answer: b The expenditures account is a temporary account that is closed at the end of the fiscal year. Expenditures occur within a given fiscal period and are not carried over into a new fiscal period. fund balance account shows the difference between the assets and liabilities. Its balance is carried forward to the next fiscal period. so, no need to close down fund balance.

What are the three types of period costs that a lessee experiences with capital leases? a) Lease expense, interest expense, amortization expense. b) Interest expense, amortization expense, executory costs. c) Amortization expense, executory costs, lease expense. d) Executory costs, interest expense, lease expense.

answer: b The three costs incurred by a lessee with respect to capital leases are interest expense, amortization expense, and executory costs. Each payment consists of principal reduction and interest expense. The amount capitalized must be amortized over the useful life of the asset. Executory costs, such as insurance, maintenance, etc., are borne by the lessee. The basic premise in capital leases is the risks and responsibilities of ownership are transferred from lessor to lessee.

During the first quarter of the calendar year, Worth Co. had income before taxes of $100,000, and its effective income tax rate was 15%. Worth's effective annual income tax rate for the previous year was 30%. Worth expects that its effective annual income tax rate for the current year will be 25%. The statutory tax rate for the current year is 35%. In its first quarter interim income statement, what amount of income tax expense should Worth report? a) $15,000 b) $25,000 c) $30,000 d) $35,000

answer: b The year‐to‐date income is multiplied by the effective annual income tax rate of the current year (25%) to calculate income tax expense on the interim income statement. Worth should report income tax expense of $25,000 ($100,000 × 25%).

HG, Inc., a calendar year corporation, reported the following operating income (loss) before income tax and the enacted tax rates for the last three years of operations: Income Tax rate Year 2 $ 100,000 40% Year 3 $ (300,000) 30% Year 4 $ 400,000 30% There are no permanent or temporary differences between operating income (loss) for financial and income tax reporting purposes. When filing its Year 3 tax return, HG did not forgo to carryback the Year 3 loss. What amount should HG record in year 3 to account for the income tax refund receivable? a) $0 b) $40,000 c) $30,000 d) $120,000

answer: b This answer is correct because HG elected to carryback the loss. Loss carrybacks can be carried back to the two immediate past periods' income. Because HG only has one year of income before the loss, the refund is calculated based on that income. HG can only recover taxes already paid (taxable income multiplied by the tax rate in effect). Therefore, the refund receivable would be calculated as $100,000 x 40% = $40,000. The remainder of the loss ($300,000 - $100,000 = $200,000) would be carried forward as a deferred tax asset of $60,000 using the future enacted tax rate ($200,000 x 30% = $60,000). > HG did not forego to carryback the year 3 loss. > This answer is correct because HG elected to carryback the loss.

Marr Co. had the following sales and accounts receivable balances, prior to any adjustments at year‐end: Credit sales $10,000,000 Accounts receivable 3,000,000 Allowance for uncollectible accounts 50,000 Marr uses 3% of accounts receivable to determine its allowance for uncollectible accounts at year‐end. By what amount should Marr adjust its allowance for uncollectible accounts at year‐end? a) $0 b) $ 40,000 c) $ 90,000 d) $140,000

answer: b This answer is correct because the allowance for uncollectible amounts should be equal to $90,000, and it is currently $50,000. Therefore the required adjustment is $40,000 ($90,000‐$50,000)

Diego Corporation values its inventory at the lower of cost or net realizable value as required by IFRS. Diego has the following information regarding its inventory. Historical cost $100,000 Estimated selling price 98,000 Estimated costs to complete and sell 3,000 Replacement cost 90,000 What is the amount for inventory that Diego should report on the balance sheet under the lower of cost or net realizable value method? a) $100,000 b) $95,000 c) $98,000 d) $97,000

answer: b This answer is correct. IFRS requires inventory to be reported at the lower of cost or net realizable value (LCNRV). This method requires net realizable value to be calculated as the estimated selling price less estimated costs of completion and estimated costs to sell. Therefore, the NRV is $95,000 ($98,000 — $3,000). The lower of cost or net realizable value is determined by comparing the cost of $100,000 to the NRV of $95,000, and using the lower amount. Inventory should be reported at $95,000.

North, Inc. uses the equity method of accounting for its 50% investment in Mill Corp.'s common stock. During year 3, Mill reported earnings of $600,000 and paid dividends of $200,000. Assume that: (1) all undistributed earnings of Mill will be distributed as dividends in future periods, (2) the dividends received from Mill are eligible for the 80% dividends received deduction, and (3) North's income tax rate is 30%. The change in the amount of deferred income tax to be reported by North for year 3 is a) $0 b) $12,000 c) $24,000 d) $60,000

answer: b Under the equity method, North included revenue of $300,000 ($600,000 × 50%) in its book income. The dividends received of $100,000 ($200,000 × 50%) were reported as a reduction of the investment in Mill Corp. account. Taxable income, however, included the dividends but excluded the undistributed earnings. The total difference between book and taxable income is, therefore, $200,000. This entire amount will eventually be included in taxable income when distributed as dividends. Then, however, there would be an 80% DRD (permanent difference) of $160,000 ($200,000 × 80%). The remaining $40,000 is a temporary difference which should be reflected as a $12,000 increase in the deferred tax liability account ($40,000 × 30%). Then dividends, 0 income for books, 100k income for tax, after the 80% DRD, 20k is taxable NOW, cause they received them this year. so, 6000 in tax for the current year.

On September 29, year 2, Wall Co. paid $860,000 for all the issued and outstanding common stock of Hart Corp. On that date, the carrying amounts of Hart's recorded assets and liabilities were $800,000 and $180,000, respectively. Hart's recorded assets and liabilities had fair values of $840,000 and $140,000, respectively. In Wall's September 30, year 2 balance sheet, what amount should be reported as goodwill? a) $20,000 b) $160,000 c) $180,000 d) $240,000

answer: b Wall Co. purchased 100% of the stock of Hart Corp. for $860,000. The amount of goodwill that should be reported on the September 30, year 2 balance sheet would be the amount paid in excess of the FV of the net identifiable assets. The FV of the net assets would be calculated by taking the FV of the assets and subtracting the FV of the liabilities. The FV of the net assets would be $840,000 - 140,000 = 700,000. Goodwill will equal Wall Co. purchased 100% of the stock of Hart Corp. for $860,000. The amount of goodwill that should be reported on the September 30, year 2 balance sheet would be the amount paid in excess of the FV of the net identifiable assets. The FV of the net assets would be calculated by taking the FV of the assets and subtracting the FV of the liabilities. The FV of the net assets would be $840,000 - 140,000 = 700,000. Goodwill will equal *핵심: make sure to deduct FMV liabilty from the FMV asset. Dont just add them both. **핵심! If acquisition cost is greater than FV of acquired asset during the acquisition period, it is considered as 손실, and we call it as "goodwill" however, if acquisition cost is lower than FV of acquired asset cost during the acquisition period, we call this as revenue 이익, "bargain purchase". Be careful with goodwill vs bargain purchase

A private not‐for‐profit college should prepare a statement of financial position and which of the following financial statements? I. Statement of activities. II. Statement of functional expenses. III. Statement of cash flows. a) I and III. b) II and III. c) III only. d) I, II, and III.

answer: b because only voluntary health and welfare organizations are required to report a statement of functional expenses. only voluntary health and welfare org (such as boys scout & homeless shelter..meaning, volunteer로 운영되는 shelter) requires statement of functional expense A private not‐for‐profit college should prepare a (1) statement of financial position-> this is like balance sheet, (2) cash flow (3) statement of activities - this is like income statement.

A donated plant asset for which the fair value has been determined, and for which incidental costs were incurred in acceptance of the asset, should be recorded at an amount equal to its a) Incidental costs incurred. b)Fair value and incidental costs incurred. c)Book value on books of donor and incidental costs incurred. d)Book value on books of donor.

answer: b donated capital must be recorded at fair value, not book value..

On January 2 of the current year, Peace Co. paid $310,000 to purchase 75% of the voting shares of Surge Co. Peace reported retained earnings of $80,000, and Surge reported contributed capital of $300,000 and retained earnings of $100,000. The purchase differential was attributed to depreciable assets with a remaining useful life of 10 years. Peace used the equity method in accounting for its investment in Surge. Surge reported net income of $20,000 and paid dividends of $8,000 during the current year. Peace reported income, exclusive of its income from Surge, of $30,000 and paid dividends of $15,000 during the current year. What amount will Peace report as dividends declared and paid in its current year's consolidated statement of retained earnings? a) $8,000 b) $15,000 c) $21,000 d) $23,000

answer: b over 50% share를 갖고있다는것은, equity method가 아니고, consolidated method로 취급이 된다. peace company will report only the dividend of the parent company ($15,000) in the consolidated financial statements, not its dividend plus that recorded under the equity method ($15,000 + $6,000). Peace acquired a greater than 50% share of the voting interests in Surge. Accordingly, Peace must consolidate Surge unless it does not have control. Moreover, the equity method is not appropriate except in parent-only statements. The consolidated statements should report only dividends paid to parties outside the consolidated entity.

If a corporation sells some of its treasury stock at a price that exceeds its cost, this excess should be a) Reported as a gain in the income statement. b) Treated as a reduction in the carrying amount of remaining treasury stock. c) Credited to additional paid‐in capital. d) Credited to retained earnings. You Answered Correctly!

answer: c The excess of selling price over cost is credited to an additional paid‐in capital account. If the corporation were to sell some of its treasury stock, the cash received is debited to Cash, the cost of the shares sold is credited to the stockholders' equity account Treasury Stock, and the difference goes to another stockholders' equity account. **핵심 ***Note that the difference DOES NOT GO TO INCOME STATEMENT, as there can be no income statement recognition of gains or losses on treasury stock transactions. If the corporation sells 30 of the 100 shares of its treasury stock for $29 per share, the entry will be: Gain or loss on treasury stock NEVER EVER AFFECT AND GO TO INCOME STATEMENT!!

Nolan owns 100% of the capital stock of both Twill Corp. and Webb Corp. Twill purchases merchandise inventory from Webb at 140% of Webb's cost. During year 2, merchandise that cost Webb $40,000 was sold to Twill. Twill sold all of this merchandise to unrelated customers for $81,200 during year 2. In preparing combined financial statements for year 2, Nolan's bookkeeper disregarded the common ownership of Twill and Webb. By what amount was unadjusted revenue overstated in the combined income statement for year 2? a) $16,000 b) $40,000 c) $56,000 d) $81,200

answer: c since the entire amount of the merchandise sold to Twill is an overstatement, not just the difference between the cost and markup on cost. Merchandise that cost Twill $40,000 was sold to Webb at 140% of Twill's cost ($56,000) and must be eliminated. The elimination entry will be: Dr. Sales (40,000*1.4)-Webb 56,000 Cr. Cost of Sales-Webb 40,000 Cr. cost of sales-twill 16,000 When we eliminate the cost of sales, a corresponding debit has to be given to Sales/Revenue account without which it will be overstated.

On April 1, year 2, Union Company paid $1,600,000 for all the issued and outstanding common stock of Cable Corporation in a transaction properly accounted for as an acquisition. The recorded assets and liabilities of Cable Corporation on April 1, year 2, were as follows: Cash $ 160,000 Inventory $ 480,000 Property, plant and equipment (net) $960,000 Liabilities (360,000) On April 1, year 2, it was determined that Cable's inventory had a fair value of $460,000, and the property, plant and equipment (net) had a fair value of $1,040,000. What is the amount of goodwill resulting from the business combination? a) $0 b) 20,000 c) $300,000 d) $360,000

answer: c ($1,600,000 acquisition cost − (160,000 cash + 460,000 FMV inventory + 1,040,000 FMV PP&E − 360,000 Liability) = $300,000) The definition of goodwill is the excess cost of an acquired firm over the current fair value of the separately identifiable net assets of the acquired firm. You will only see goodwill as an asset on an entity's balance sheet if that entity has acquired another firm. Entities do not record their own goodwill, only goodwill in acquired firms. If XYZ, Inc. purchases (acquires) Widget, Inc., then XYZ will record an asset named called goodwill on its balance sheet based on the acquisition of Widget. Any such goodwill amount recorded by XYZ has nothing to do with the company's own goodwill. **Goodwill equals the price paid for the acquired company minus the fair market value of its net identifiable assets. To figure "net" identifiable assets, subtract the liabilities on the acquired company's balance sheet from the fair value of its identifiable assets. **핵심! If acquisition cost is greater than FV of acquired asset during the acquisition period, it is considered as 손실, and we call it as "goodwill" however, if acquisition cost is lower than FV of acquired asset cost during the acquisition period, we call this as revenue 이익, "bargain purchase". Be careful with goodwill vs bargain purchase

Lang Co. uses the installment method of revenue recognition. The following data pertain to Lang's installment sales for the years ended December 31, year 1 and year 2: Year 1 Year 2 Installment receivables at year‐end on year 1 sales $60,000 $30,000 Installment receivables at year‐end on year 2 sales ‐‐ 69,000 Installment sales 80,000 90,000 Cost of sales 40,000 60,000 What amount should Lang report as deferred gross profit in its December 31, year 2 balance sheet? a) $23,000 b) $33,000 c) $38,000 d) $43,000

answer: c ***If collection of the full account is somewhat uncertain, we need to use the installment method for the contract*** Under the installment method, gross profit is deferred at the time of sale and is recognized by applying the gross profit rate to subsequent cash collections. Therefore, at each year‐end, deferred gross profit can be computed by multiplying the gross profit percentage by the AR balance. The gross profit percentage for each year is computed by dividing that year's gross profit by installment sales. 여기서 deferred gross profit은 year 2에 일어난 A/R를 뜻한다. installment method에선, A/R을 잡으면, 상대계정이 revenue가 아니다. (Debit A/R, Credit Inventory, Credit Deferred gross profit) Year 1 Year 2 Installment sales (A) $80,000 $90,000 Cost of sales (40,000)(60,000) Gross profit (B) $40,000 $30,000 Gross profit % (B ÷ A) 50% 33 1/3% The 12/31/Y2 deferred gross profit is computed by multiplying the gross profit percentages by the year‐end receivables resulting from that year's sales. Year 1 Year 2 12/31/Y2 AR from year 1 sales $30,000 12/31/Y2 AR from year 2 sales $69,000 x 50% x 33 1/3% $15,000 $23,000 Therefore, 12/31/Y2 deferred GP is $38,000 ($15,000 + $23,000).

Key Corp. issued 1,000 shares of its nonvoting preferred stock for all of Lev Corp.'s outstanding common stock. At the date of the transaction, Key's nonvoting preferred stock had a market value of $100 per share, and Lev's tangible net assets had a book value of $60,000. In addition, Key issued 100 shares of its nonvoting preferred stock to an individual as a finder's fee for arranging the transaction. As a result of this capital transaction, Key's total net assets would increase by a) $0 b) $ 60,000 c) $100,000 d) $110,000

answer: c A business acquisition is accounted for using fair values; the net assets acquired are recorded at their fair value or the fair value of the stock issued, whichever is more objectively determinable. In this case, the fair value of the stock issued is a better measure of the value of the purchase (1,000 shares × $100 per share = $100,000). The total cost of acquiring the net assets is the fair value of the preferred stock ($100,000). The finder's fee is treated as an expense of the period. Direct out-of-pocket costs such as finder's fee or a legal fee are expensed from the income. Stock registration and issuance costs such as SEC filling fees are a direct reduction of the value of the stock issued are directly deducted from the APIC if there was any gain from selling the stock.

On January 1, year 1, Vick Company as lessee signed a 10‐year noncancelable lease for a machine stipulating annual payments of $20,000. The first payment was made on January 1, year 1. Vick appropriately treated this transaction as a capital lease. The 10 lease payments have a present value of $135,000 at January 1, year 1, based on implicit interest of 10%. For the year ended December 31, year 1, Vick should record interest expense of a) $0 b) $6,500 c) $11,500 d) $13,500

answer: c At the inception of the lease on 1/1/Y1, the capitalized lease liability was $135,000. The 10 lease payments have a present value of $135,000<- future value를 현재 돈으로 convert한것...결국엔 $135,000만 다 갚으면 빛 청산하는것. $135,000에서 매년 $20,000을 deduct한다. $20,000 already includes both principle and interest payment. The first payment, also on 1/1/Y1, consisted "entirely of principal" and reduced the liability to $115,000 ($135,000 − $20,000). 여기서 핵심은, 1/1 즉, end of the year이 아닌, beginning of the year에 annual payment가 나갈때는, interest incur이 되어있지 않다. 오직 principle로만 treat해야한다. 1년도 채 되지 않고, contract을 맺자마자 하루도 않됬는데 왜 이자가 붙으면 말이 안됨..따라서, 첫번째 payment which was paid on 1/1/1/ $20,000 needs to be treated as fully principle paid 1/1/1에annual payment가 한번에 pay가 됬다. interests는 당여한 include가 안됬다. full 1년이 채우지 않았는데 어떻게 interest가 나오겠어? 따라서, 12/31/1 year end에 interest는 accrual liability로 book에는 적어나야하고, 다만, 1/1/year2에 된다. 지불될때 annual payment for second year $20,000안에 포함이되서 지불된다. ($135,000 total liability에서 작년에 첫번쨰 annual payment $20,000을 제외하면 $115,000이 year 2 beginning balance다. $115,000 x10% = 11,500 interest로 나가고, $20,000 annual paymnet for 2nd year에서 $11,500 interest 뺀 금액 8,500금액이 실직적으로, principle 금액이라 생각하기. year 2 ending balance of liability payment is $135,000 - $20,000 - $8,500 = $106,500. notice, only the principle amount is deducted from total liability. we don't decrease it by interest. 12/31/year2에 book에다 interest liability accrual로 $106,500 x 10% = $10,650 잡아두기.. *notice, it's asking interest for December 31, year 1.. 일년이 지났으니 interest accrue되는건 당여한것.

On April 1, year 1, Pine Construction Company entered into a fixed‐price contract to construct an apartment building for $6,000,000. Pine appropriately accounts for this contract under the percentage‐of‐completion method. Information relating to the contract is as follows: At Dec 31, year 1 At Dec 31, year 2 Percentage of completion 20% 60% Est. costs at completion $4,500,000 $4,800,000 Income recognized (cumulative) $ 300,000 $ 720,000 *여기서 중요한것은, 문제에서 주워진, Estimated costs at completion이란, estimated remaining balance of cost가 아니고, 실제 총 예상되는 공사비용으로 봐야한다.* What is the amount of contract costs incurred during the year ended December 31, year 2? a) $1,200,000 b) $1,920,000 c) $1,980,000 d) $2,880,000

answer: c Based on the information given, it must be assumed that costs incurred are used to measure the extent of progress toward project completion. At 12/31/Y1, the project is 20% complete and total estimated costs are $4,500,000. Therefore, costs incurred as of 12/31/Y1 are 20% of $4,500,000, or $900,000. At 12/31/Y2, the project is 60% complete and total estimated costs are $4,800,000. Therefore, costs incurred as of 12/31/Y2 are 60% of $4,800,000, or $2,880,000. The costs incurred during year 2 are $2,880,000 less $900,000, or $1,980,000. year 1: 20% is completed and the estimated total cost at completion is: 4,500,000. 20%*4,500,000 = 900,000 costs incurred in year 1 year 2: 60% is completed and the estimated total cost at completion is: 4,800,000. year 2 60%*4,800,000 = 2,880,000 of total costs incurred so far. But you already recorded 900,000 in the previous year. 2,880,000 - 900,000 = 1,980,000 costs incurred in 2008 다른 설명: Actual cost incurred up to Dec 31,2007: 4,500,000*20% = 900,000 Actual cost incurred up to Dec 31,2008: 4,800,000*60%= 2,880,000 Actual cost incurred in year of 2008 = 2,880,000 - 900,000 = 1,980,000

On December 31, year 1, Case, Inc. had 300,000 shares of common stock issued and outstanding. Case issued a 10% stock dividend on July 1, year 2. On October 1, year 2, Case purchased 24,000 shares of its common stock for treasury, and recorded the purchase by the cost method. What is the number of shares that should be used in computing basic earnings per share for the year ended December 31, year 2? a) 306,000 b) 309,000 c) 324,000 d) 330,000 EPS Formula: First, subtract the preferred dividends paid from the net income. This will tell you the total earnings available to common shareholders. Next, divide the earnings total you just calculated by the number of outstanding shares listed on the balance sheet. This will give you the EPS.

answer: c For EPS purposes, shares of stock issued as a result of stock dividends or splits should be considered outstanding for the entire period in which they were issued. Therefore, both the original 300,000 shares and the additional 30,000 shares (10% × 300,000) are treated as outstanding for the entire year. The 10/1/Y2 purchase of 24,000 treasury shares results in a weighted‐average deduction of 6,000 shares (3/12 × 24,000) because the shares were not outstanding for only 3 months during year 2. Therefore, the number of shares for EPS computations is 324,000. Outstanding 12/31/Y1 --> 300,000 Stock dividend (10% × 12/12 x300,000)-> 30,000 10/1 purchase (3/12 × 24,000)->(6,000) 300,000 +30,000 -6,000 =324,000 **핵심 포인트는** stock issued for dividend또한 regular cash dividend 처럼, entire period로 recognize하기.. no matter which month it was issued, just use 12/12 month.. 그리고, treasury stock은 무족건 common stock에서 뺀다. 하지만, 이떄 조심해야하는게 acquired한 date 기준으로 weight 해주기.. 10/1에 acquire헀으면, 3/12 로 곱해주기.

Which of the following is a governmental fund that uses the current financial resources measurement focus? a) Enterprise fund. b) Internal service fund. c) Special revenue fund. d) Private‐purpose trust fund.

answer: c Governmental fund types (general, special revenue, debt service, capital projects, and permanent trust) use the current financial resources measurement focus and the modified accrual basis of accounting. 여기서, "current financial resource measurement focus"란, the focus is on cash and/or assets that are expected to be converted to cash within the accounting period, or shortly thereafter. This measurement looks at whether the financial resources obtained during the accounting period are enough to cover all claims made against the fund during the same period. However, it doesn't focus on long-term capital assets and obligations Proprietary and fiduciary fund types use the "flow of economic resources measurement focus" and accrual accounting. The "flow of economic resources" measurement focus measures all assets that are available to the entity, not only cash or soon to be cash assets. Both long-term assets and long-term liabilities are measured when using the "flow of economic resources" measurement focus. In addition *current financial resource measurement focus - on cash and/or assets that are expected to be converted to cash within the accounting period, or shortly thereafter (basically current cash가 될애들만 신경쓰고 있음. long term asset은 안보고) 반면에, *"flow of economic resources" 더 나아가서, long term capital asset또 focus한다. Answer에서 나온 special revenue란, revenue earmarked for specific purpose (Motor fuel tax, Motel tax, cigar tax)

The following information is available for Cooke Company for year 2: Net sales $1,800,000 Freight‐in 45,000 Purchase discounts 25,000 Ending inventory 120,000 The gross margin is 40% of net sales. What is the cost of goods available for sale? a) $ 840,000 b) $ 960,000 c) $1,200,000 d) $1,220,000

answer: c Gross margin is 40% of net sales ($1,800,000), or $720,000. Therefore, cost of goods sold is $1,080,000 ($1,800,000 net sales less $720,000 gross margin). Finally, cost of goods available for sale is $1,200,000 ($1,080,000 cost of goods sold plus $120,000 ending inventory). The amounts for freight‐in ($45,000) and purchase discounts ($25,000) are not necessary for the computation. 여기서 중요한것은, it didn't just ask for COGS. It asked for COGS Available for sale. (COGS available for sale = COGS Inventory + Ending Inventory )

During the current year Knoxx County levied property taxes of $2,000,000, of which 1% is expected to be uncollectible. The following amounts were collected during the current year: Prior year taxes collected within the 60 days of the current year $ 50,000 Prior year taxes collected between 60 and 90 days into the current year 120,000 Current year taxes collected in the current year 1,800,000 Current year taxes collected within the first 60 days of the subsequent year 80,000 What amount of property tax revenue should Knoxx County report in its entity‐wide statement of activities? a) $1,800,000 b) $1,970,000 c) $1,980,000 d) $2,000,000

answer: c I think the entity-wide is equal to government-wide. So the statement is under economic resources measurement focus and the full accrual basis. Under full accrual basis, the $2,000,000 is earned and should be recorded in full amount. The 1% estimated noncollectable AR should be debit at bad debt expense and credit allowance for doubtful accounts. The revenue shouldnt be deduct by doubtful account. I thinks this should be reconcile from funds accounting to government-wide. **나의 생각은 이렇다. Property tax는 얼래 government fund type이고, government fund type은 modified accrual basis다. it means, it recognize income when cash is actually received or cash is collected within 60 days after fiscal year end. 하지만, 여기서 또 얘기하는건, proprety tax revenue need to report in its entity- wide statement라고 한다. 즉 government wide를 애기한다. government wide는 무족건, full accrual method를 주장하고있다. 따라서, modified accrual basis must be reconcile to full accrual method. 그렇기에, full 일반 accrual basis에서 revenue를 인식하는 식으로 이 문제를 풀어야한다. AR $2,0000,000 - Bad debt (2,000,000 x 1%) = $1,980,000

Sayon Co. issues 200,000 shares of $5 par value common stock to acquire Trask Co. in a purchase‐business combination. The market value of Sayon's common stock is $12. Legal and consulting fees incurred in relationship to the purchase are $110,000. Registration and issuance costs for the common stock are $35,000. What should be recorded in Sayon's additional paid‐in capital account for this business combination? a) $1,545,000 b) $1,400,000 c) $1,365,000 d) $1,255,000

answer: c Per ASC Topic 805 the finder's and consultant's fees should be expensed. The SEC registration costs should be netted against the additional paid‐in capital account. Journal Entry # 1 to Record the investment Investment 2,400,000(200,000*12) Common Stock(200,000*5)=1,000,000 APIC(2,000,000*7)= 1,400,000 Journal Entry # 2 to record the costs APIC 35,000 Legal & consult Expense 110,000 Cash 145,000 Registration and issuance costs for stocks are always treated as reduction of Additional paid in Capital. They are not expensed.

Wilburn Corp. signs an agreement to lease land and a building for 20 years. At the end of the lease, the property will not transfer to Wilburn. The life of the building is estimated to be 20 years. Wilburn prepares its financial statements in accordance with IFRS. How should Wilburn account for the lease? a) The lease is recorded as an operating lease. b) The lease is recorded as a finance lease. c) The land is recorded as an operating lease and the building is recorded as a finance lease. d) The land is recorded as a finance lease, and the building is recorded as an operating lease.

answer: c IFRS provides that because land has an indefinite life, if title is not expected to pass by the end of the lease term, then the substantial risks and rewards of ownership do not transfer. Thus, the lease should be separated into two components. The land should be recorded as an operating lease and the building should be recorded as a finance lease. 여기서, financial lease란 capital lease다. 핵심**US GAAP uses the term "capital lease," whereas IFRS uses the term "finance lease."여기 문제에서는 IFRS가 등장하니깐, capital lease라고 불르지않고, financing lease라 불렀다. lease agreement term이 20년이라 한다. 여기서, land는 사실상, useful life가 없다. depreciate하지 않으니깐, land has indefinite life. However, questions tells us building's useful life is 20 years. 이뜻은, 적어도, building은 capital lease로 treat되어야한다는것. LAND- treated as operating lease Building- treated as capital lease ------------------------------------ Ex: Erdman Corp. signs a lease to rent equipment for ten years. The lease payments of $20,000 per year are due on January 2 each year. At the end of the lease term, Erdman may purchase the equipment for $500. The equipment is estimated to have a useful life of 10 years. Erdman prepares its financial statements in accordance with IFRS. Erdman should classify this lease as a(n): a) Operating lease. b) Capital lease. c) Sales‐type lease. d) Finance lease. answer:d US GAAP uses the term "capital lease," whereas IFRS uses the term "finance lease."

LaValley Corp. issues monthly financial statements to its creditors. LaValley should perform assessments of hedge effectiveness on a(n) a) Semiannual basis. b) Annual basis. c) Monthly basis. d) Quarterly basis.

answer: c If a company issues monthly financial statements, the assessment of hedge effectiveness should be performed on a monthly basis. The effectiveness of the hedging relationship must be assessed when financial statements are prepared and at least every 3 months.

According to the IASB Framework, the two criteria required for incorporating items into the income statement or statement of financial position are that a) It meets the definition of relevance and reliability. b) It satisfies the criteria of capital maintenance. c) It meets the definition of an element and can be measured reliably. d) It meets the requirements of comparability and consistency.

answer: c In order for an item to be recognized in the financial statements, IFRS requires that it meet the definition of an element and can be measured reliably.

Selected information for Irvington Company is as follows: December 31 Year 1 Year 2 Preferred stock, 8%, par $100, nonconvertible, noncumulative $125,000 $125,000 Common stock 300,000 400,000 Retained earnings 75,000 185,000 Dividends paid on preferred stock for year ended 10,000 10,000 Net income year ended 60,000 120,000 Irvington's return on common stockholders' equity, rounded to the nearest percentage point, for year 2 is a) 17% b) 19% c) 23% d) 25%

answer: c Irvington's return on common stockholders' equity for year 2 is computed by dividing net income available to common stockholders (net income less preferred dividends) by average common stockholders' equity. $120,000 − $10,000 ($375,000 + $585,000)/2 = 23%

Melville Company leased equipment from Rice Corporation on July 1, year 1, for an 8‐year period expiring June 30, year 9. Equal payments under the lease are $600,000 and are due on July 1 of each year. The first payment was made on July 1, year 1. The rate of interest contemplated by Melville and Rice is 10%. The cash selling price of the equipment is $3,520,000 and the cost of the equipment on Rice's accounting records is $2,800,000. Assuming that the lease is appropriately recorded as a sales‐type lease, what is the amount of profit on the sale and interest income that Rice should record for the year ended December 31, year 1? a) $0 and $0. b) $0 and $146,000. c) $720,000 and $146,000. d) $720,000 and $160,000. .

answer: c Melville's gross profit is the difference between the present value of the lease payments, $3,520,000 (which is also the cash selling price of the equipment), and the cost of goods sold ($2,800,000), or $720,000. Interest income is found by multiplying the book value of the receivable from the lessee (total lease payments receivable minus unearned interest) outstanding during year 1 ($3,520,000 initial balance less $600,000 payment made on 7/1/Y1) times the implicit interest rate (10%) for 1/2 of a year. Therefore, interest income is $146,000 ($2,920,000 × 10% × 1/2).

In accounting for a long‐term construction contract using the percentage‐of‐completion method, the amount of income recognized in any year would be added to a) Deferred revenues. b) Progress billings on contracts. c) Construction in progress. d) Property, plant, and equipment

answer: c Note, that when company is using percentage of completion method, they can recognize the gain as partially, over the life of the contract. However, completion contract method can only recognize the gross profit or gain only after the completion of the contract. 둘이 비슷하지만, 틀리점은 아마 이것일꺼다. Let's say 20% of $1,000 contract was finished so far, so company billed the vendor for 20% which is 200 DR Account Receivable $200 CR Construction in Progress $200 Instead of Recording as A/R (debit) and Revenue (Credit), we would put Construction in progress as (Credit) The percentage-of-completion method ordinarily is to be used for the accounting of long-term construction contracts except in two situations: 1) Where reasonably reliable estimates cannot be made; or (합리적으로 신뢰할만한 견적을 제출할 수없는 경우는 percentage of completion method를 쓸수가 없다) 2) Where the results of using the completed-contract method do not differ materially from those obtained by using the percentage-of-completion method

On March 1, year 1, a suit was filed against Dean Company for patent infringement. Dean's legal counsel believes an unfavorable outcome is probable, and estimates that Dean will have to pay between $500,000 and $900,000 in damages. However, Dean's legal counsel is of the opinion that $600,000 is a better estimate than any other amount in the range. The situation was unchanged when the December 31, year 1 financial statements were released on February 24, year 2. How much of a liability should Dean report on its balance sheet at December 31, year 1 in connection with this suit? a) $0 b) $500,000 c) $600,000 d) $900,000

answer: c Per ASC Topic 450, an estimated loss from a loss contingency should be accrued only if it is probable that a liability exists at the balance sheet date and if the loss is reasonably estimable. The contingency meets the probable requirement. In addition, the best estimate of the loss is $600,000. Therefore, a contingent loss of $600,000 should be reported on December 31, year 1.

The following items relate to the preparation of a statement of cash flows: Year 2 Year 1 Cash $150,000 $100,000 Dividends payable 35,000 0 Common stock 600,000 450,000 Retained earnings 280,000 165,000 Year 2 Net sales $3,200,000 CGS (2,500,000) Expenses (500,000) Net income $ 200,000 Capital stock was sold to provide additional working capital. Under financing activities, cash dividend payments during year 2 amounted to $115,000 $ 85,000 $ 50,000 $ 35,000

answer: c Retained earnings increased $115,000 ($280,000 − $165,000) even though net income was $200,000 for year 2. This indicates that dividends declared during this period amounted to $85,000 ($200,000 − $115,000). However, $35,000 of this represents a liability at the end of year 2. Therefore, cash dividend payments for year 2 is $50,000 ($85,000 − $35,000). 쉽게 설명하면, year 2의 net income은 $200,000이다. And this net income goes into year 2's retained earning. $165,000 +200,000 = $365,000 하지만, year2 의 net income은 $365,000아닌 $280,000으로 기록됬다. ($365,000 -280,000 =85,000 difference) 이뜻은, $85,000 dividend가 pay됬다는것. 근데 잘 보면, dividend payable liabliyt로 $35,000이 잡혀있고, future에 나가기로 했다. 그뜻은 $85,000 dividend중 일부 $35,000은 향후 pay를 하겠다고 잡아둔것이고 $50,000은 cash로 dividend를 pay 한것으로 보여진다.

Bard Co., a calendar‐year corporation, reported income before income tax expense of $10,000 and income tax expense of $1,500 in its interim income statement for the first quarter of the year. Bard had income before income tax expense of $20,000 for the second quarter and an estimated effective annual rate of 25%. What amount should Bard report as income tax expense in its interim income statement for the second quarter? a) $3,500 b) $5,000 c) $6,000 d) $7,500

answer: c Tax expense for interim reporting purposes is calculated as year‐to‐date income times the estimated annual effective tax rate less the tax expense recognized in previous quarters. Therefore, the tax expense for the second quarter is calculated as $6,000 [($30,000 × 25%) - $1,500], and this answer is correct. (sum of income before tax for quarter 1 & 2) x 25% annual effective rate - $1,500 income tax expense reported in quarter1

On January 2, year 3, Morey Corp. granted Dean, its president, 20,000 stock appreciation rights for past services. Those rights are exercisable immediately and expire on January 1, year 4. On exercise, Dean is entitled to receive cash for the excess of the stock's market price on the exercise date over the market price on the grant date. Dean did not exercise any of the rights during year 3. The market price of Morey's stock was $30 on January 2, year 3, and $45 on December 31, year 3. Morey should recognize compensation expense under the stock appreciation rights plan for year 3 of a) $0 b) $100,000 c) $300,000 d) $600,000

answer: c The 20,000 stock appreciation rights (SAR) each entitle the holder to receive cash equal to the excess of the market price of the stock on the exercise date over the market price on the grant date ($30). Since these SAR are payment for " past services and are exercisable immediately" <--this is 핵심 , there is no required service period. Therefore, the expense computed at 12/31/Y1 does not have to be allocated to more than one period. At 12/31/Y1, compensation expense is measured based on the excess of the 12/31/Y1 market price ($45) over the predetermined price ($30), resulting in compensation expense of $300,000 [20,000 ($45 - $30)]. ***여기서나오는 service period란, The service period is the period from the grant date, to the exercise date is on January 2, year 6, which is 3 years from the grant date, then Dean is required to work three years before the SAR could be exercised, the expense would be allocated over the three years of required service ($300,000 × 1/3 = $100,000).

On January 2, year 3, Morey Corp. granted Dean, its president, 20,000 stock appreciation rights for past services. Those rights are exercisable immediately and expire on January 1, year 4. On exercise, Dean is entitled to receive cash for the excess of the stock's market price on the exercise date over the market price on the grant date. Dean did not exercise any of the rights during year 3. The market price of Morey's stock was $30 on January 2, year 3, and $45 on December 31, year 3. Morey should recognize compensation expense under the stock appreciation rights plan for year 3 of a) $0 b) $100,000 c) $300,000 d) $600,000

answer: c The 20,000 stock appreciation rights (SAR) each entitle the holder to receive cash equal to the excess of the market price of the stock on the exercise date over the market price on the grant date ($30). Since these SAR are payment for past services and are exercisable immediately, there is no required service period. Therefore, the expense computed at 12/31/Y1 does not have to be allocated to more than one period. At 12/31/Y1, compensation expense is measured based on the excess of the 12/31/Y1 market price ($45) over the predetermined price ($30), resulting in compensation expense of $300,000 [20,000 ($45 - $30)]. Note that if Dean were required to work three years before the SAR could be exercised, the expense would be allocated over the three years of required service ($300,000 × 1/3 = $100,000). Stock appreciation rights (SARs) is a method for companies to give their management or employees a bonus if the company performs well financially. Such a method is called a 'plan'. SARs resemble employee stock options in that the holder/employee benefits from an increase in stock price.

Wolf Co.'s grant of 30,000 stock appreciation rights enables key employees to receive cash equal to the difference between $20 and the market price of the stock on the date each right is exercised. The service period is year 1 through year 3, and the rights are exercisable in year 4 and year 5. The market price of the stock was $25 and $28 at December 31, year 1 and year 2, respectively. What amount should Wolf report as the liability under the stock appreciation rights plan in its December 31, year 2 balance sheet? a) $0 b) $130,000 c) $160,000 d) $240,000

answer: c The 30,000 stock appreciation rights (SARs) entitle the holders to receive cash equal to the excess of the stock's market price on the exercise date over $20. On 12/31/Y2, the estimate of total SARs compensation expense is $240,000: 30,000 x ($28 -$20) Because the required service period is 3 years, this total expense will be allocated over a 3-year period. By the end of Year 2, the second year, two-thirds of the total estimated compensation expense should be accrued, resulting in a SARs liability of $160,000: (2/3 x $240,000) Stock appreciation rights (SARs) is a method for companies to give their management or employees a bonus if the company performs well financially. Such a method is called a 'plan'. SARs resemble employee stock options in that the holder/employee benefits from an increase in stock price.

Grant, Inc. has current receivables from affiliated companies at December 31, year 2, as follows: • A $50,000 cash advance to Adams Corporation. Grant owns 30% of the voting stock of Adams and accounts for the investment by the equity method. • A receivable of $160,000 from Bullard Corporation for administrative and selling services. Bullard is 100% owned by Grant and is included in Grant's consolidated statements. • A receivable of $100,000 from Carpenter Corporation for merchandise sales on open account. Carpenter is a 90% owned, unconsolidated subsidiary of Grant. In the current assets section of its December 31, year 2 consolidated balance sheet, Grant should report accounts receivable from investees in the total amount of a) $ 90,000 b) $140,000 c) $150,000 d) $310,000

answer: c The accounts receivable from investees to be reported on the balance sheet should only include the receivables from investees considered unconsolidated subsidiaries. The receivables from the unconsolidated subsidiaries ($50,000 + $100,000) would not be eliminated and, therefore, would be reported as receivables in the consolidated balance sheet. However, the $160,000 receivable from the consolidated subsidiary would be eliminated on the consolidated worksheet and thus not reported on the consolidated balance sheet.

On January 1, year 3, Fay Corporation established an employee stock ownership plan (ESOP). Selected transactions relating to the ESOP during year 3 were as follows: On April 1, year 3, Fay contributed $30,000 cash and 3,000 shares of its $10 par common stock to the ESOP. On this date the market price of the stock was $18 a share. On October 1, year 3, the ESOP borrowed $100,000 from Union National Bank and acquired 5,000 shares of Fay's common stock in the open market at $17 a share. The note is for 1 year, bears interest at 10%, and is guaranteed by Fay. In its year 3 income statement, how much should Fay report as compensation expense relating to the ESOP? a) $184,000 b) $120,000 c) $ 84,000 d) $ 60,000

answer: c The amount contributed or committed to be contributed to an employee stock ownership plan (ESOP) in a given year should be the measure of the amount to be charged to expense by the employer. Therefore, Fay should record year 3 compensation expense of $84,000 [contribution of $30,000 cash and common stock with a FV of $54,000 (3,000 × $18)].

If one Canadian dollar can be exchanged for 90 cents of United States money, what fraction should be used to compute the indirect quotation of the exchange rate expressed in Canadian dollars? a) 1.10/1 b) 1/1.10 c) 1/.90 d) .90/1

answer: c The direct quotation is the rate expressed in it's domestic currency..note, in this case, domestic currency is canadian dollar. It means that $.90 can be exchanged for 1 Canadian dollar. The direct quotation is $.90/1. (쉽게 생각해서, domestic currency인 dollar기준으로 생각하기. 1 canadian dollar 기준으로 생각하기. $0.90 = 1 Canadian dollar로 문제가 나왔다. In this case 0.90/1 is direct quotation. The indirect quotation is the inverse of the direct quotation or 1/$.90. 미국 달라 $1불 기준으로 캐나디언 환률을 계산하는것. A "direct quote" is when the currency pair uses the domestic currency as the base currency. An "indirect quote" is when a currency pair uses the domestic currency as the quoted currency. If you're looking at the U.S. dollar as the foreign currency, a direct currency quote would be CAD/USD 0.72, which means $1 Canadian dollar would purchase 72 cents U.S. For an indirect quote, USD/CAD 1.21 means that $1 U.S. dollar will purchase $1.21 Canadian dollars. Direct Quote - It refers to the number of units of domestic currency which is required to buy one unit of foreign currency. 미국인 기준으로 봤을때, Domestic currency는 달라이다. Direct Quote법에서는, 미국돈 1달라로, 얼마나 많은 rupees (인디아돈)을 환전할수있는지 보느것이다. Indirect Quote - It refers to the number of units of foreign currency required to buy 1 unit of domestic currency. 미국인 기준으로 봤을때, Domestic currency는 달라이다. Indirect Quote 법에서는, 1 rupee로 얼마나 많은 미국 달라를 환전할수있는지 보느것이다. (ignorant하게 무족건 질문에서 얘기하는 US dollar가 domestic currency라고 생각하지말고. 천천히 문제 읽기...왜나면 그때그때마다 domestic currency는 바뀐다...위에 질문처럼, canada돈이 domestic currency가 될수도있다) Examples Example 1: An Australian resident converted 500,000 AUD to 45,139,200 JPY. Find the indirect quote for the Australian resident. since we talking about Austrialian resident, we know AUD is the domestic currency in this case. Solution For an Australian resident, Australian Dollar (AUD) is the domestic currency while Japanese Yen (JPY) is the foreign currency. Indirect quote expresses foreign currency in terms of one unit of domestic currency. Indirect quote in this situation would be 90.2784 JPY/AUD (45,139,200 JPY per 500,000 AUD). Relevant direct quote would be 0.011077 JPY/AUD (=1/(90.2784 JPY/AUD)). . -----------

Esker Inc. specializes in real estate transactions other than retail land sales. On January 1, year 1, Esker consummated a sale of property to Kame Ltd. The amount of profit on the sale is determinable and Esker is not obligated to perform any additional activities to earn the profit. Kame's initial and continuing investments were adequate to demonstrate a commitment to pay for the property. However, Esker's receivable may be subject to future subordination. Esker should account for the sale using the a) Deposit method. b) Reduced profit method. c) Cost recovery method. d) Full accrual method.

answer: c The problem states that the sale has been consummated and that Kame's initial and continuing investments are adequate to demonstrate a commitment to pay for the property. However, the fact that Esker's receivable is subject to future subordination precludes recognition of the profit in full. Instead, the cost recovery method must be used to account for the sale.

Roe Company is preparing a statement of cash flows for the year ended December 31, year 2. It has the following account balances: 12/31/Y1 12/31/Y2 Machinery $250,000 $320,000 Accum depreciation—machinery 102,000 120,000 Loss on sale of machinery 0 4,000 During year 2, Roe sold for $26,000 a machine that cost $40,000, and purchased several items of machinery. Depreciation on machinery for year 2 was a) $18,000 b) $24,000 c) $28,000 d) $32,000

answer: c This answer is correct. To determine the amount of depreciation expense, both the machinery and the accumulated depreciation (AD) accounts must be analyzed. Machinery was sold during year 2 for $26,000, resulting in a loss of $4,000. Therefore, the machinery sold must have had a book value of $30,000 (Book value − $26,000 = $4,000). Since the machinery sold originally cost$40,000, the AD at the time of sale must have been $10,000 ($40,000 − AD = $30,000). Now a T‐account is set up for AD

Lee, Inc. acquired 30% of Polk Corp.'s voting stock on January 1, year 1, for $100,000. Lee uses the equity method to account for its investment in Polk. During year 1, Polk earned $40,000 and paid dividends of $25,000. Lee's 30% interest in Polk gives Lee the ability to exercise significant influence over Polk's operating and financial policies. During year 2, Polk earned $50,000 and paid dividends of $15,000 on April 1 and $15,000 on October 1. Polk's income was earned evenly throughout the year. On July 1, year 2, Lee sold half of its stock in Polk for $66,000 cash. What should be the gain on sale of this investment in Lee's year 2 income statement? a) $16,000 b) $13,750 c) $12,250 d) $10,000

answer: c This investment is accounted for using the equity method since Lee owns 30% of Polk's voting stock and has the ability to exercise significant influence. (보통 상대방 회사의 20%가 넘는 Common stock을 갖고있으면, it is equity method. note: this only applies to voting stock which is C/S. not Preferred stock which don't have voting right) Under the equity method, the "carrying amount" of the investment is increased by the investor's share of investee earnings and decreased by dividends received. Therefore at 12/31/Y1, the carrying amount is $104,500 [$100,000 + (30% × $40,000) − (30% × $25,000)]. By 7/1/Y2, Polk earned year 2 income of $25,000 ($50,000 × 6/12) and paid dividends of $15,000. The carrying amount at 7/1/Y2 is $107,500 [$104,500 + (30% × $25,000) − (30% × $15,000)]. year 2현재 기준의 carrying investment 금액은 $107,500. 여기에 only half of stock was sold At 7/1/Y2, half of the stock is sold for $66,000, resulting in a gain of $12,250 [$66,000 − (1/2 × $107,500)].

Conn Company purchased a new machine for $480,000 on January 1, year 1, and leased it to East the same day. The machine has an estimated 12‐year life, and will be depreciated $40,000 per year. The lease is for a 3‐year period expiring January 1, year 4, at an annual rental of $85,000. Additionally, East paid $30,000 to Conn as a lease bonus to obtain the 3‐year lease. For year 1 Conn incurred insurance expense of $8,000 for the leased machine. What is Conn's year 1 operating profit on this leased asset? a) $67,000 b) $55,000 c) $47,000 d) $37,000

answer: c This lease is an operating lease because it does not meet any of the four criteria to be a capital lease as described in ASC Topic 840. The lessor (Conn) should recognize as revenue the year 1 rental payment ($85,000) plus a proportionate fraction of the lease bonus ($30,000/ 3‐year lease term = $10,000 per year). Therefore, total revenue for year 1 is $95,000 ($85,000 + $10,000). Year 1 expenses total $48,000 (depreciation of $40,000 and insurance of $8,000). Thus, operating profit on the leased asset is $47,000 ($95,000 revenues less $48,000 expenses).

Which of the following provides the holder the right to sell the underlying at an exercise or strike price, anytime during a specified period of time a gain accrues to the holder as the market price of the underlying falls below the strike price? a) Call option. b) Forward contract. c) Put option. d )Swaption.

answer: c This meets the definition of an American put option. An American call option provides the holder the right to acquire an underlying at an exercise or strike price, anytime during the option term. The forward contract is an agreement between two parties to buy and sell a specific quantity of a commodity, foreign currency, or financial instrument at an agreed‐upon price, with delivery and/or settlement at a designated future date. A swaption is an option on a swap that provides the holder with the right to enter into a swap at a specified future date at specified terms or to extend or terminate the life of an existing swap.

Steam Co. acquired equipment under a capital lease for six years. Minimum lease payments were $60,000 payable annually at year‐end. The interest rate was 5% with an annuity factor for six years of 5.0757. The present value of the payments was equal to the fair market value of the equipment. What amount should Steam report as interest expense at the end of the first year of the lease? a) $0 b) $3,000 c) $15,227 d) $18,000

answer: c This problem requires you to calculate the present value of the minimum lease payments, which is the present value of the ordinary annuity of $60,000 at 5% for 6 periods, or $304,542 (5.0757 × $60,000). $304,542 x 0.05 = $15,227. Pv of lease = 60,000 * 5.0757 = 304,542. At beginning of lease : Debit Lease Asset 304,542 Credit Lease Liability 304,542 When Payment is made at year end: Debit Interest Expense 15,227 (304,542 *.05) Debit Lease Expense 44,773 (60,000 - 15,227) Credit Cash 60,000 The lease expense will reduce the lease liability to 259,769 (304,542 - 44,773) which will be the new carrying value for next year. The 60,000 is just a routine payment that is made every period.

Lee Corporation's checkbook balance on December 31, year 1, was $4,000. In addition, Lee held the following items in its safe on December 31: Check payable to Lee Corporation, dated 1/2/Y2, not included in 12/31 checkbook balance $1,000 Check payable to Lee Corporation, deposited 12/20, and included in 12/31 checkbook balance, but returned by bank on 12/30, stamped "NSF." The check was redeposited 1/2/Y2, and cleared 1/7. 200 Postage stamps received from mail‐order customers 75 Check drawn on Lee Corporation's account, payable to vendor, dated and recorded 12/31, but not mailed until 1/15/Y2 500 The proper amount to be shown as Cash on Lee's balance sheet at December 31, year 1, is a) $3,800 b) $4,000 c) $4,300 d) $4,875

answer: c To be classified as cash, resources must be readily available for the payment of current obligations and must be free from any restrictions that limit its use in satisfying debts. In this situation the $1,000 postdated check should be classified as a receivable. The $75 of postage stamps should be included in office supplies inventory. Since the company still maintains physical control of the $500 drawn against the checking account it should be added back to the cash balance. (12/31/year 1으로 check를 지불할려고 끊어났어도, 정작 check를 본인이 physically hold하고 있고 vendor한테 보내지 않았더라면, cash에 다시 이 돈을 포함하기. 12/31/year 1 기준으로 빡구당한, check receipt또한 cash에서 빼주기. Redopsit이 1/2/year2에 되었으니깐.) Checkbook balance $4,000 Check drawn but not mail + 500 Less: NSF check − 200 Corrected cash balance $4,300

New England Co. had net cash provided by operating activities of $351,000; net cash used by investing activities of $420,000; and cash provided by financing activities of $250,000. New England's cash balance was $27,000 on January 1. During the year, there was a sale of land that resulted in a gain of $25,000 and proceeds of $40,000 were received from the sale. What was New England's cash balance at the end of the year? a) $ 27,000 b) $ 40,000 c) $208,000 d) $248,000

answer: c To calculate the cash balance at the end of the year, you should combine the effects of the changes in operating, investing, and financing activities, and add the beginning cash balance. $351,000 − $420,000 + $250,000 = change in cash of $181,000 + beginning cash balance of $27,000 = $208,000 ending balance in cash. We are assuming $40,000 gain is already included in investing activities. i think the reason is because, "net cash" provided by operating activities, "net cash" investing activities, "net cash financing"...여기서 NET 이라는단어가 이미 모든걸 다 포함한금액이라 생가이 든다.

The following items relate to the preparation of a statement of cash flows: Year 2 Year 1 Cash $150,000 $100,000 Inventory 330,000 210,000 Prepaid exp. 50,000 25,000 Noncurrent assets 565,000 300,000 Accum. deprec. (55,000) (25,000) Accrued exp. 70,000 65,000 Year 2 Net sales $ 3,200,000 CGS (2,500,000) Expenses (500,000) Net income $ 200,000 The direct approach is used for operating activities. Under operating activities, cash paid for operating expenses for year 2 amounted to (the questions is asking "CASH PAID for OPERATING EXPENSE. THUS, it's asking about cash payment rather than cash received. so don't include cash inflow..just include cash outflow) a) $380,000 b) $435,000 c) $490,000 d) $410,000

answer: c Total cash paid for operating expenses for year 2 would be calculated as follows: Expenses per statement $500,000 Add: Increase in prepaid expenses 25,000 Less: Depreciation expense (30,000) Increase in accrued expenses (5,000) Total cash paid for operating expenses $490,000 The increase in prepaid expenses is added because it represents cash paid out in excess of the expense recognized. Depreciation is subtracted because it does not require an actual outflow of cash. The increase in accrued expense is subtracted because it represents the amount of expense recognized in excess of the amount of cash paid out. The direct method to calculate cash flow from operating activities involves determination of various types of cash receipts and payments such as cash receipts from customers, cash paid to suppliers, cash paid for salaries, etc. and then putting them together under the cash flow from operating section of cash flow statement. These figures are calculated using the beginning and ending balances of various accounts of the business and the net increase or decrease in the account

The balance in Bart Corp.'s foreign exchange loss account was $13,000 at December 31, year 1, before any necessary year‐end adjustment relating to the following: • Bart had a $20,000 loss resulting from the translation of the accounts of its wholly owned foreign subsidiary for the year ended December 31, year 1. • Bart had an account payable due to an unrelated foreign supplier payable in the local currency of the foreign supplier on January 27, year 2. The US dollar equivalent of the payable was $100,000 on the November 28, year 1 invoice date, and it was $106,000 on December 31, year 1. In Bart's year 1 consolidated income statement, what amount should be included as foreign exchange loss? a) $33,000 b)$27,000 c) $19,000 d) $13,000 .

answer: c Translation adjustments result from translating an entity's financial statements into the reporting currency. Such adjustments, which result when the entity's functional currency is the foreign currency, should not be included in net income. Instead, such adjustments should be reported as components of "other comprehensive income" and accumulated other comprehensive income in stockholders' equity. (Note that if the functional currency was the reporting currency, a remeasurement process would have been used instead of translation, with the resulting gain or loss included in income.) The $20,000 translation loss is not reported on the income statement. In contrast, gains and losses which result from foreign exchange transactions (purchases/sales) are reported on the income statement. Therefore, Bart should report the $13,000 foreign exchange loss, plus the $6,000 foreign exchange loss ($106,000 year‐end liability less $100,000 original liability), for a total loss of $19,000.

On June 1 of the current year, a company entered into a real estate lease agreement for a new building. The lease is an operating lease and is fully executed on that day. According to the terms of the lease, payments of $28,900 per month are scheduled to begin on October 1 of the current year and to continue each month thereafter for 56 months. The lease terms spans five years. The company has a calendar year‐end. What amount is the company's lease expense for the current calendar year? a) $ 86,700 b) $161,838 c) $188,813 d) $202,300

answer: c When a lease contains future rent steps or "payments are not required to start immediately" <- 핵심 포인트, rent expense should be recognized on a straight line basis over the lease term starting when the lessee has possession of the asset. This answer is correct because the contract was executed on June 1. Payments of $28,900 span 56 months starting on October 1. Total lease cost equals $28,900 × 56 months = $1,618,400. Converting this to a 5‐year lease term results in a monthly payment of $26,973.33 ($1,618,400/60 = $26,973.33). Seven months (June 1−Dec 31) have passed since the execution of the lease, resulting in a lease expense of $188,813 for the year ($26,973.33 × 7 months = $188,813). Step to memorize: 1) If lease payments are not required to start immediately, like in this case, where, lease contract was signed on June but lease payment is schedule to be paid on October, we need to treat it as "Straight-line basis" 2) Must recalculate the monthly payment which needs to be paid immediately upon the lease contract date.

Certain balance sheet accounts in a foreign subsidiary of the Brogan Company at December 31, year 1, have been remeasured into United States dollars as follows: Current Historical Equity securities carried at cost $100,000 110,000 Marketable equity securities carried at current market price 120,000 125,000 Inventories carried at cost 130,000 132,000 Inventories carried at NRV 80,000 84,000 What amount should be shown in Brogan's balance sheet at December 31, year 1, as a result of the above information? a) $430,000 b) $436,000 c) $442,000 d) $451,000

answer: c When remeasuring into US dollars, the current rate will be used for monetary items and for nonmonetary items carried at market. The historical rate is to be used for nonmonetary items carried at cost. The inventory carried at net realizable value should be remeasured using current rates because net realizable value is a market concept. Per the summary below, the items would be translated at $442,000. (This question applies to foreign affiliates who has financial statements that are in the local currency but the functional currency is dollars, so the Temporal-Rate Method must be used. ) Historical rate: Securities at cost $110,000 Inventories at cost 132,000 Current rate: Securities at market $120,000 Inventory at market 80,000 $442,000 ***Nonmonetary assets are assets a company holds for which it is not possible to precisely determine a dollar value. These are assets whose dollar value may fluctuate and that changes substantially over time; and assets that are not readily or easily convertible into cash or cash equivalents**

When should a lessor recognize in income a nonrefundable lease bonus paid by a lessee on signing an operating lease? a) When received. b) At the inception of the lease. c )At the expiration of the lease. d) Over the life of the lease.

answer: d ASC Topic 840 specifies that, in an operating lease, the lesser should recognize rental revenues on a straight‐line basis. This means that a lease bonus should be recorded as unearned revenue and recognized as rental revenue over the life of the lease.

Young & Jamison's modified cash‐basis financial statements indicate cash paid for operating expenses of $150,000, end‐of‐year prepaid expenses of $15,000, and accrued liabilities of $25,000. At the beginning of the year, Young & Jamison had prepaid expenses of $10,000, while accrued liabilities were $5,000. If cash paid for operating expenses is converted to accrual‐basis operating expenses, what would be the amount of operating expenses? a) $125,000 b) $135,000 c) $165,000 d) $175,000

answer: c because it adds both yearly changes in prepaid expenses and accrued liabilities to cash operating expense to arrive at accrual operating expenses ($150,000 + $5,000 + $20,000 = $175,000). To convert to accrual‐basis operating expenses, the cash paid of $150,000 is adjusted by subtracting the increase in the prepaid expense account, and adding the increase in accrued liabilities. ($150,000 - 5,000 + 20,000 = $165,000). 쉽게 생각하면, cash basis method를 쓰다가, year2 에 서 갑자기 accrual basis로 전환하게 됬다. accrual method로 바뀌면, operating expense가 어떻게 변하냐..뭐 이런 내용. Year 1 ending balance vs Year 2 ending balance 비교하는게 제일 쉬운길이다. Year 1 Ending Balance Prepaid expense $10,000 Accrual liability $5,000 Year 2 Ending Balance Prepaid expense $15,000 Accrual liability $25,000 Operating expense 150,000 그럼 difference를 찾기. Prepaid was increased by $5,000 and accrual liabillity was increased by $20,000 명심하기, prepaid expense는 accrual basis상으로는 asset계정이다. liablity 가 아니다...해당되는 sales가 일어나기전에 미리 낸것은....sales가 실제 일어나는 point때 liablity 계정으로 바뀐다. operating expense 150,000 -$5,000 +20,000 accrual increase = $165,000

Sun Co. was constructing fixed assets that qualified for interest capitalization. Sun had the following outstanding debt issuances during the entire year of construction: $6,000,000 face value, 8% interest $8,000,000 face value, 9% interest None of the borrowings were specified for the construction of the qualified fixed asset. Average expenditures for the year were $1,000,000. What interest rate should Sun use to calculate capitalized interest on the construction? a) 8.00% b) 8.50% c) 8.57% d) 9.00%

answer: c because the interest rate used to capitalize interest on self‐constructed assets is the user's interest on specific borrowings to construct the assets. When there are no specific borrowings for the self‐constructed assets, the weighted‐average interest rate on other borrowings is used to capitalize interest. In Sun's case, the total face value of the two borrowings is $14,000,000. The weighted‐average interest rate is calculated as 8.57% ($6,000,000/14,000,000 × 8%) + ($8,000,000/14,000,000 × 9%). The interest will be capitalized, whether or not it's a specific borrowing or general debt during the period of construction. In this question, interest rate on the individual borrowings cannot be taken, since they were not borrowed for construction purpose. This is a General Debt. So, the weighted average rate is computed and interest capitalized in $ will be 8.57% 1,000,000(Avg Expenditure)

A nongovernmental not‐for‐profit organization borrowed $5,000, which it used to purchase a truck. In which section of the organization's statement of cash flows should the transaction be reported? a) In cash inflow and cash outflow from investing activities. b) In cash inflow and cash outflow from financing activities. c) In cash inflow from financing activities and cash outflow from investing activities. d) In cash inflow from operating activities and cash outflow from investing activities.

answer: c because the item involves an inflow from financing activities and an outflow from investing activities. Financing activity = obtaining cash from creditor. In this case, $5,000 was loan that received. Investment activity= acquiring investment and productive long live asset. in this case, is truck Loan $5,000을 받으면, financing activity로 들어갔다가, 그 돈을 갚을때는 investment activity로 나간다.

When a company discontinues an operation and disposes of the discontinued operation (component), the transaction should be included in the earnings statement as a gain or loss on disposal reported as a) prior period adjustment. b) An extraordinary item. c) An amount after continuing operations and before extraordinary items. d) A bulk sale of fixed assets included in earnings from continuing operations.

answer: c continuing operation earning 다음에, discontinuing operating gain/loss 그다음이 extraordinary item gain/loss로 표시한다.

On January 2, year 1, Troquel Corporation bought 15% of Zafacon Corporation's capital stock for $30,000. Troquel accounts for this investment by the "cost adjusted for fair value method" and carries the securities in an available‐for‐sale portfolio. Zafacon's net income for the years ended December 31, year 1, and December 31, year 2, was $10,000 and $50,000, respectively. During year 2, Zafacon declared a dividend of $70,000. No dividends were declared in year 1. How much should Troquel show on its year 2 income statement as income from this investment? a) $ 1,575 b) $ 7,500 c) $ 9,000 d) $10,500

answer: c if it's commons stock was acquired less than 20% we use cost method. if it's more than 20% we use equity method. if it's more than 50% we use consolidated method. Under the cost adjusted for fair value method, the investor records the investment at cost, adjusts the carrying amount for subsequent changes in fair value, and records dividends received as income. However, dividends received in excess of the investor's share of investee's earnings since acquisition are recorded as a reduction in the investment account. Such distributions are treated as a return of capital (liquidating dividend). In this case, Troquel's share of the dividend is $10,500 ($70,000 × 15%). However, Troquel's share of Zafacon's earnings since acquisition is only $9,000 ($60,000 × 15%). Thus, only $9,000 can appropriately be recorded as investment income.

The general fund of Karsten City received a $30,000 unrestricted grant from the State on June 12, year 1. The cash was received by the general fund on June 12 and was used to pay for capital expenditures that were incurred on June 29. Accounts payable related to the capital expenditures were paid in July year 1. Karsten City's fiscal year ends on June 30. What account should be credited in the general fund on the date the grant was received? a) Deferred inflow of resources. b) Unreserved fund balance. c) Revenue. d) Fund balance restricted for capital expenditures.

answer: c the general fund of Karsten should record the receipt of the unrestricted grant as revenue. The grant was used to pay for expenditures incurred during the year ended June 30, year 1. Under the modified accrual basis of accounting, the grant was both measurable and available during the year ended June 30, year 1. unrestricted grant를 donation을 받으면= revenue로 인식하기. it goes into operating activity. restricted grant를 donation을 받으면= it doesn't include as revenue but goes to financing activity.

For a capital lease, the amount recorded initially by the lessee as a liability should a) Exceed the present value at the beginning of the lease term of minimum lease payments during the lease term. b) Exceed the total of the minimum lease payments during the lease term. c) Not exceed the fair value of the leased property at the inception of the lease. d) Equal the total of the minimum lease payments during the lease term.

answer: c the liability must be recorded at the present value of the minimum lease payments. (이뜻은, future value principle + interest 금액을 현제의 돈 가치로, present value로 계산해서 lease시작시에 장부에 기록하는것이다) If a lease is classified as a capital lease, the lessee must record an asset and a liability, each for an amount equal to the present value of the minimum lease payments at the beginning of the lease. (notice this is only referring to "Capital lease")

Sanni Co. had $150,000 in cash‐basis pretax income for the year. At the current year‐end, accounts receivable decreased by $20,000 and accounts payable increased by $16,000 from their previous year‐end balances. Compared to the accrual‐basis method of accounting, Sanni's cash‐basis pretax income is a) Higher by $4,000 b) Lower by $4,000 c) Higher by $36,000 d) Lower by $36,000

answer: c Because accounts receivable decreased by $20,000, the cash received was $20,000 more than the accrual‐basis sales. Since accounts payable increased by $16,000 during the year, accrual‐basis expenses were $16,000 more than cash payments. Therefore, accrual‐basis net income is equal to $114,000 ($150,000 - 20,000 - $16,000), and therefore, cash‐basis pretax income is $36,000 ($150,000 - $114,000) higher than accrual‐basis income. Cash Basis Income - 15,000 A/R Decrease - (20,000) - The money being collected for AR is being counted as revenue for cash basis accounting. This is not revenue for accrual, thus we subtract it from net income and revenues. A/P Increase - (16,000) - Since these expenses are being paid on credit and no cash is changing hands, this is not an expense for cash basis accounting. In accrual basis accounting, this expense is recognized even if no cash changes hands, thus we subtract it from net income or add it to expenses. Accrual Income= (21,000) Difference - 15,000-(21000)= 36,000 Cash basis income is 36,000 higher than accrual basis. An easy way to remember this (opposite works for the cash flow statement using the indirect method

The present value of pension benefits accrued to date using assumptions as to future compensation levels is the a) Prior service cost. b)Accumulated benefit obligation. c)Projected benefit obligation. d) Accrued pension cost.

answer: c The projected benefit obligation is the present value of pension benefits accrued to date using assumptions as to future compensation levels. Present value (PV) is the current worth of a future sum of money Projected benefit obligation (PBO)- 향후 직원들이 받을 pension 이익을 현재 가치로 생각하는것.

At year‐end, Rim Co. held several investments with the intent of selling them in the near term. The investments consisted of $100,000, 8%, five‐year bonds, purchased for $92,000, and equity securities purchased for $35,000. At year‐end, the bonds were selling on the open market for $105,000 and the equity securities had a market value of $50,000. What amount should Rim report as trading securities in its year‐end balance sheet? a) $ 50,000 b) $127,000 c) $142,000 d) $155,000

answer: d Answer (D) is correct. Trading securities are debt securities not classified as held to maturity and equity securities with readily determinable fair values that are bought and held primarily for sale in the near term. Hence, the bonds and the equity securities are trading securities. They are initially recorded at cost but are subsequently measured at fair value at each balance sheet date. Quoted market prices in active markets are the best evidence of fair value. Based on market quotes at year end, the bonds had a fair value of $105,000, and the equity securities had a fair value of $50,000. The total is $155,000. Both securities are trading securities and both should be valued at market value, or $155,000 ($105,000 + $50,000). Marketable debt (other than those intended to be held until maturity) and marketable equity securities are reported at fair value in the balance sheet. The exceptions are marketable debt securities that a company plans to hold to maturity (reported at amortized cost) and marketable equity securities that provide the company the ability to significantly influence the investee (equity method required) or to control the investee (consolidation required). Marketable securities must be classified as held-to-maturity (debt securities only in this category), available-for-sale securities, and trading securities. Accounting for held-to-maturity securities does not report fair value in the balance sheet or fair value changes in the income statement. Available-for-sale securities are reported at fair value in the balance sheet, but changes in fair value are reported as other comprehensive income, not included in the income statement. Trading securities (specified here) are reported at fair value in the balance sheet, and changes in fair value are reported in other income in the income statement. Holding securities to sell them in the near term implies that the securities are trading securities

On January 1 of the current year, Tell Co. leased equipment from Swill Co. under a nine‐year sales‐type lease. The equipment had a cost of $400,000, and an estimated useful life of fifteen years. Semiannual lease payments of $44,000 are due every January 1 and July 1. "The present value of lease payments at 12% was $505,000, which equals the sales price of the equipment." Using the straight‐line method, what amount should Tell recognize as depreciation expense on the equipment in the current year? a) $26,667 b) $33,667 c) $44,444 d) $56,111

answer: d Tell Co. must treat the lease as a capital lease because the present value of the minimum lease payments exceeds 90% of the fair value (sales price) of the equipment. Tell's cost equals the present value of $505,000. The question does not indicate or imply that Tell guarantees any residual value or that ownership transfers at the end of the 9-year lease. Therefore, the depreciable cost of $505,000 must be charged to depreciation over the period of use, which is the lease term of 9 years. The depreciation expense for the current year (one full year's depreciation) is $505,000 ÷ 9 years, or $56,111. Although the question says this is sales-typle lease, when we look at the question closely, we know this is capital lease. The questions says "The present value of lease payments at 12% was $505,000, which equals the sales price of the equipment." <-this means $505,000 is same as FMV of the equipment. PV of asset must be greater than 90% FMV of lease asset, in order for this lease asset to be capital asset. 따라서, $505,000 PV를 이용해서, depreciate하기..over the lease term of 9 yrs.

The completed‐contract method of accounting for long‐term construction‐type contracts is preferable when a) A contractor is involved in numerous projects. b) The contracts are of a relatively long duration. c) Estimates of costs to complete and extent of progress toward completion are reasonably dependable. d) Lack of dependable estimates or inherent hazards cause forecasts to be doubtful.

answer: d The completed contract method is used to recognize all of the revenue and profit associated with a project only after the project has been completed. This method is used when there is uncertainty about the collection of funds due from a customer under the terms of a contract. Given these issues, the method should only be used under the following circumstances: When it is not possible to derive dependable estimates about the percentage of completion of a project; or When there are inherent hazards that may interfere with completion of a project; or When contracts are of such a short-term nature that the results reported under the completed contract method and the percentage of completion method would not vary materially (신뢰할만한 견적이나 위험 요소가 있어 예측이 의심 스러울때 쓴다)

Main, a pharmaceutical company, leased office space from Ash. Main took possession and began to use the building on July 1, year 1. Rent was due the first day of each month. Monthly lease payments escalated over the 5‐year period of the lease as follows: Period Lease payment July 1, year 1 - Sep 30, year 1 $0 - rent October, 1, year 1- June 30, year 2 17,500 July 1, year 2 - June 30, year 3 19,000 July 1, year 3 - June 30, year 4 20,500 July 1, year 4 - June 30, year 5 23,000 July 1, year 5 - June 30, year 6 24,500 What amount would Main show as deferred rent expense at December 31, year 4? a) $50,658 b) $52,580 c) $68,575 d) $71,550

answer: d The problem involves "uneven rent payments, and accounting standards require rent expense to be recognized evenly over the contract period". The rent per month to be paid over the 60‐month period can be calculated as follows: First 3 months $ 0 Next 9 months 17,500 × 9 157,500 Year 2 19,000 × 12 228,000 Year 3 20,500 × 12 246,000 Year 4 23,000 × 12 276,000 Year 5 24,500 × 12 294,000 Total rent payments $1,201,500 Divided by 60 months ÷ 60 months $ 20,025 per month The amount of rent paid as of December 31, year 4, is July 1, year 1 to June 30, year 2 $157,500 July 1, year 2 to June 30, year 3 228,000 July 1, year 3 to June 30, year 4 246,000 July 1, year 4 to December 30, year 4 138,000 (6 months) Total rent paid $769,500 The amount of rent that should be recognized as total rent expense for the period July 1, year 1, to December 31, year 4, is $841,050 (42 months × $20,025 per month). The correct answer is $71,550 ($841,050 − $769,500), the amount of rent that should be recorded as deferred/accrued rent expense. Note that the question asks for deferred rent but in this case the rent is actually accrued (payable).

Ian Co. is calculating earnings per share amounts for inclusion in Ian's annual report to shareholders. Ian has obtained the following information from the controller's office as well as shareholder services: Net income from January 1 to December 31 $125,000 Number of outstanding shares: January 1 to March 31 15,000 April 1 to May 31 12,500 June 1 to December 31 17,000 In addition, Ian has issued 10,000 incentive stock options with an exercise price of $30 to its employees and a year‐end market price of $25 per share. What amount is Ian's diluted earnings per share for the year ended December 31? a) $4.63 b) $4.85 c) $7.35 d) $7.94

answer: d The weighted‐average shares outstanding is computed by multiplying the number of common shares outstanding times the portion of the year the shares were outstanding. Number of outstanding shares: January 1 to March 31: 15,000 x 3/12= 3,750 April 1 to May 31: 12,500 x 2/12= 2,084 June 1 to December 31: 17,000 x 7/12= 9,916 Total weighted‐average shares outstanding: (3,750 + 2,084 + 9,916= 15,750) **Weighted average 계산하는게 핵심 포인트..제일 중요.. 12 month를 거꿀로 생각하지말고...그냥..쉽게 생각하기 앞에서부터. 4/1-5/31는 정확히 2월달이니깐, 2/12 기준으로 계산하기. 복잡하게 카운트 하지 말기. The diluted earnings per share is then calculated by adding the total weighted‐average common shares outstanding to the weighted‐average shares of all potentially dilutive securities. Ian Co. has 10,000 incentive stock options with an exercise price of $30. If the treasury stock method is used to account for these potentially dilutive securities, it is assumed that the 10,000 options are exercised at $30 (10,000 × $30 = $300,000), and the $300,000 would be used to retire as many shares as possible at the year‐end price of $25 per share ($300,000/$25 = 12,000 shares retired.) Since 10,000 additional shares would be issued and 12.000 shares would be retired, the incentive stock options are considered antidilutive and are not included in the calculation of diluted earnings per share. Therefore, diluted earnings per share is calculated by dividing $125,000 net income by the 15,750 weighted‐average common shares outstanding which equals $7.94 ($125,000 ÷ 15,750). 여기서 anti-dilution이란, Not all security mechanisms result in decreased EPS, and some even increase EPS. If securities are retired, converted or affected through certain corporate activities, and the transaction results in an increased EPS, then the action is considered to be antidilutive. *remember, generally, when Diluate EPS is calculated, Dilute EPS is lower than basic EPS..but in this case, Diluate EPS was increased because exercise value $30 was greater than Market rate which is $25. In this case, this becomes anti-dilution and we can't further calculate for Diluate EPS ..그냥 Diluate 무시하고, basic EPS로 계산하기. **If options were converted they would become common stock. They are anti dilutive because the market price is greater than the option price.**(so, originally, $30 x 10,000 need to be added as common stock but since Option value is greater than market price, this becomes anti-dilution.** There are anti-dilution provisions in lots of convertible securities that make them maybe anti-dilutive but the classic example is convertible preferred stock which may be either dilutive or anti-dilutive. So lets say that there are 100,000 shares of common and 100,000 shares of convertible preferred each convertible into 1 share of common. EPS = $1 without conversion. Dividends on preferred = $1.50 EPS after conversion = (100K + 150K)/200K = $1.25 EPS without conversion = $1 Thus the convertible preferred is antidilutive. antidilutive Of or relating to the conversion of convertible securities into common stock when such conversion would result in an increase in diluted earnings per share or a decrease in diluted loss per share. For example, it is an antidilutive conversion if outstanding warrants are assumed to be exercised in order to acquire shares of common stock at a higher price than the market price of the stock. Such a conversion would result in an increase in diluted earnings per share. Conversions that would increase earnings per share or reduce loss per share are not generally used in calculating diluted earnings per share.

Elaine Corporation was organized on January 1, year 1, with an authorization of 1,000,000 shares of common stock with a par value of $5 per share. During year 1, the corporation had the following capital transactions: January 4—issued 200,000 shares @ $5 per share. April 8—issued 100,000 shares @ $7 per share. June 9—issued 30,000 shares @ $10 per share. July 29—purchased 50,000 shares @ $4 per share. December 31—sold 50,000 shares held in treasury @ $8 per share. Elaine used the cost method to record the purchase and reissuance of the treasury shares. What should be the balance in the account "capital in excess of par value" as of December 31, year 1? a) $400,000 b) $450,000 c) $500,000 d) $550,000

answer: d This answer is correct. The change in the account resulting from each transaction described should be identified Effect Jan. 4‐ Shares issued @ par $0 April 8‐ Shares issued @ $2 above par (100,000 × $2) $200,000 June 9‐ Shares issued at $5 above par (30,000 × $5) $150,000 July 29‐ Purchased treasury shares $0 December 31‐ Sold at $4 above cost (50,000 × $4) $200,000 Change in "capital in excess of par" $550,000 Since Elaine uses the cost method, there is no effect on "capital in excess of par" when the shares are purchased, but the account is increased when the treasury shares are reissued above cost. Remember: "capital in excess of par value" includes APIC from all sources.

Grey Company purchased a machine on January 1, year 1, for $500,000. The machine has an estimated useful life of 5 years and a salvage value of $50,000. Depreciation was computed by the 150% declining balance method. The accumulated depreciation balance at December 31, year 2, should be a) $180,000 b) $229,500 c) 245,000 d) $255,000

answer: d **Salvage value is ignored when using a Decline Balance approach (both for 200% and 150%) The formula for 150% DB depreciation is 150% of the straight‐line rate multiplied by the beginning‐of‐the‐year book value. Since the straight‐line rate is 20% (100%/5 years), the DB rate is 30% (150% × 20%). The book value for the first year is $500,000 (original cost). Therefore, year 1 depreciation is $150,000 ($500,000 cost × 30%). The book value for the second year is $350,000 ($500,000 original cost - $150,000 accumulated depreciation). Therefore, year 2 depreciation is $105,000 ($350,000 × 30%). The total accumulated depreciation at 12/31/Y2 is $255,000 ($150,000 + $105,000).

Kresley Co. has provided the following year 2 current account balances for the preparation of the annual statement of cash flows: Jan 1 Dec 31 Accounts receivable $11,500 $14,500 Allowance for uncollectible 400 500 Prepaid rent expense 6,200 4,100 Accounts payable 9,700 11,200 Kresley's year 2 net income is $75,000. Net cash provided by operating activities in the statement of cash flows should be a) $72,700 b) $74,300 c) $75,500 d) $75,700

answer: d *note that this is using "indirect method" Net income $75,000 Increase in AR (3,000) Increase in Allow. for D.A. 100 Decrease in prepaid rent 2,100 Increase in AP 1,500 Cash provided by operating activities $75,700 The increase in AR is deducted from net income because it indicates that cash collected is less than sales revenue. (A/R이 높은게 오히려 안좋은거다 왜나면 그만큼, actual cash received가 적은거니깐. A/R은 cash를 받아놓은게 아니다) Decrease in A/R means that cash is actually received. The increase in the allowance account is added to net income because it reflects an expense (bad debt expense) which was not a cash payment. allowance account는 그냥 수금이 안될거같은 A/R을 줄여주는거지, cash expense가 오고 가고 했던게 아니니깐, 그냥 net income에다 다시 더해주기. (this is reason why we also add up depreciation and amortization to net income) cash affect를 주지 않는것들은 그냥 다 더하는것 같다. The decrease in prepaid rent is added because it too reflects an expense (rent expense) which was not a cash payment (it was an allocation of previously recorded prepaid rent). Finally, the increase in AP is added because it also represents an expense (cost of goods sold) which was not yet paid. Increased (higher) AP amount is better for cash position because liability 빛만 쌓아놓고, 정작 cash는 나가지 않는상태이니깐.. 반대로, decreased amount in A/P balance is bad because in order to reduce down A/P, it means company paid cash. This is why decrease A/P is subtracted from net income.

Marr Corp. reported rental revenue of $2,210,000 in its cash basis federal income tax return for the year ended November 30, year 2. Additional information is as follows: Rents receivable ‐ November 30, year 2 $ 1,060,000 Rents receivable ‐ November 30, year 1 800,000 Uncollectible rents written off during the fiscal year 30,000 Under the accrual basis, Marr should report rental revenue of A) $1,920,000 B) $1,980,000 C) $2,440,000 D) $2,500,000

answer: d 800,000 +X -2,210,000- 30,000= 1,060,000 then x = 2,500,000 Beginning receivable +Rental Revenue Earned- Collection- written off = Ending receivable.

The vested benefits of an employee in a pension plan represent a) Benefits to be paid to the retired employee in the current year. b) Benefits to be paid to the retired employee in the subsequent year. c) Benefits accumulated in the hands of an independent trustee. d) Benefits that are not contingent (not subject to, not depend on) on the employee's continuing in the service of the employer.

answer: d A vested benefit is a financial incentive of employment that an employee is fully entitled to. Employers sometimes offer their employees benefits that they acquire full ownership of gradually or suddenly, as they accumulate more time with the company. vested benefits are ones you've earned that can't be taken away, even if terminated tomorrow. In a typical pension plan these are defined in terms of annual or monthly income. A vested benefit of $100/mo. means that no matter what happens, you are entitled to receive $100/month at normal retirement age

On June 1, year 1, Ward Corp. established a defined benefit pension plan for its employees. The following information was available at May 31, year 3: Projected benefit obligation $14,500,000 Accumulated benefit obligation 12,000,000 Pension asset/liability (200,000) Plan assets at fair market value 7,000,000 Unrecognized prior service cost 2,550,000 To report the proper pension liability in Ward's May 31, year 3 balance sheet, what is the amount of the adjustment required? a) $2,250,000 b) $4,750,000 c) $4,800,000 d) $7,300,000

answer: d ASC Topic 715 requires the recognition of pension liability if the projected benefit obligation exceeds the fair value of the plan assets. The adjustment needed is the amount to bring the liability equal to $7,500,000 ($14,500,000 less $7,000,000). There is currently $200,000 in the unfunded pension asset (liability) account. Therefore, an adjusting entry is made for $7,300,000. Ward currently has an unfunded accrued liability of $200,000. The proper pension liability should be the difference between Projected Benefit obligation (liability) of 14,500,000 and Plan Assets (asset) 7,000,000 which is $7,500,000. Since they already have a 200,000 liability on the books, the adjustment is for $7,300,000.

Connor Corporation signed a lease on January 1, year 1, to rent equipment for 10 years. The lease was appropriately treated as a capital lease. On January 1, year 4 Connor renegotiated the lease terms. The new lease agreement does not contain a bargain purchase option, nor transfer of title. The new lease terms are for a shorter length of time, which is not greater than 75% of the economic useful life of the asset. The present value of the minimum lease payments under the new agreement is less than 90% of the fair market value of the leased asset. How should Connor account for the change in the lease agreement? a) Reduce the leased asset account by the gross value in the reduction of payments. b) Remove the leased asset from the books and treat the lease as an operating lease. c) Make no change until the end of the lease term at which time a gain or loss will be recognized for the reduction in payments. d) Treat the new lease as a sales‐leaseback.

answer: d ASC Topic 840 requires that capital leases that are modified so that the resulting lease agreement is classified as an operating lease be accounted for under sale‐leaseback provisions.

What is the present value of all future retirement payments attributed by the pension benefit formula to employee services rendered prior to that date only? a) Service cost. b) Interest cost. c) Projected benefit obligation. d) Accumulated benefit obligation.

answer: d Accumulated benefit obligation = present value of all future retirement payments (현재 차곡차곡 쌓인 value) The accumulated benefit obligation (ABO) is estimated based on the assumption that the pension plan is to be terminated "immediately"; it does not consider any future salary increase. This differs from the projected benefit obligation, which assumes that the pension plan is ongoing, and thus accounts for future salary increases. projected benefit obligation is based on future salaries.

An investor uses the cost adjusted for fair value method to account for an investment in common stock that is carried in an available‐for‐sale portfolio. A portion of the dividends received this year were in excess of the investor's share of investee's earnings subsequent to the date of investment. The amount of dividend revenue that should be reported in the investor's income statement for this year would be a) Zero. b) The total amount of dividends received this year. c) The portion of the dividends received this year that were in excess of the investor's share of investee's earnings subsequent to the date of investment. d) The portion of the dividends received this year that were not in excess of the investor's share of investee's earnings subsequent to the date of investment.

answer: d An investor using the cost adjusted for fair value method to account for a stock investment credits dividend revenue for amounts received from an investee unless cumulative dividends received exceed the investor's share of earnings since acquisition. Dividends declared by the investee in excess of its earnings since acquisition by the investor are viewed by the investor as liquidating dividends and reduce the investment amount. Dividends received that were not in excess of the investor's share of earnings since acquisition should be recorded as dividend revenue. Cost Method :- Any dividends received would be credited to Income statement of the investor. However, if such dividends are in excess of the investor's share of investee's undistributed earnings/retained earnings, then that excess would be credited to "investment in investee" account. This excess is called liquidating dividend. Thus, only the portion which is not in excess of the investor's share of investee's undistributed earnings/retained earnings would be credited to IS, rest would be credited to Investment in investee a/c. (Liquidating dividend) is a type of nondividend distribution made by a corporation or a partnership to its shareholders during its partial or complete liquidation. Liquidating distributions are not paid solely out of the profits of the corporation.

Gains from remeasuring a foreign subsidiary's financial statements from the local currency, which is not the functional currency, into the parent company's currency should be reported as a(n) a_ Deferred foreign exchange gain. b) "Other comprehensive income" and as a separate component of stockholders' equity. c) Extraordinary item, net of income taxes. d) Part of continuing operations.

answer: d Be careful!! the questions is talking about "Remeasuring foreign sub's financial statement" and not "translating" Re-measurement is the key word. As stated in the solution if the subsidiaries foreign currency is the local currency (european sub using the Euro) then the financial statements are translated to the US dollar, with the difference going to OCI as "foreign currency translation adjustment". The Other Comprehensive Income can be a separate statement, or merged into the balance sheet as a line item in the retained earnings section. So the question asked about a re-measurement in the INCOME STATEMENT (cap for emphasis to avoid confusion - I know this is difficult). Income statement adjustments arise from when the foreign subsidiary does not use the local currency (typically a low-volume to no-trading, or high-inflationary currency) and the foreign sub typically uses the parent's functional currency (USD). This adjustment goes to the consolidated income statement as a line item. Simply 요약: if subsidiary is using foreign currency which is their local currency, let's say Euro, and if parent is using US Dollar, there will be currency translation and might recognize either gain or loss which will be reported in "OCI" (other comprehensive income) However, if foreign subsidiary is using the same currency as their parent's company, it will need to "REMEASURE" (instead of translate) and this remeasure amount is affecting income statement. it affects net income, just like "currency transaction"

The following items relate to the preparation of a statement of cash flows: Year 2 Year 1 Cash $150,000 $100,000 MES* (Available for sale) 40,000 0 AR net 420,000 290,000 Inventory 330,000 210,000 Noncurrent assets 565,000 300,000 Accum. Deprec. (5,000) (25,000) Year 2 Net sales $3,200,000 CGS (2,500,000) Expenses (500,000) Net income $ 200,000 *Cost and market value All accounts receivable relate to trade merchandise. Cash discounts are not allowed to customers but a service charge is added to an account for late payment. The allowance for doubtful accounts at the end of year 2 was the same as the end of year 1; no receivables were charged against the allowance during year 2. Under investing activities, cash outflows during year 2 totaled a) $ 40,000 b) $265,000 c) $275,000 d) $305,000

answer: d Cash outflows from investing activities is comprised of the $265,000 increase in noncurrent assets ($565,000 −; $300,000) plus the increase in marketable equity securities of $40,000 for a total of $305,000. Under investing activities, in this case, only "AFS" and "Non current assets" Inventory, cash, A/R, Depreciation belongs to "operating activity" and not investing activities. In year 2, it seems like "AFS"

On November 1, year 1, Beni Corp. was awarded a judgment of $1,500,000 in connection with a lawsuit. The decision is being appealed by the defendant, and it is expected that the appeal process will be completed by the end of year 2. Beni's attorney feels that it is highly probable that an award will be upheld on appeal, but that the judgment may be reduced by an estimated 40%. In addition to footnote disclosure, what amount should be reported as a receivable in Beni's balance sheet at December 31, year 1? a) $1,500,000 b) $ 900,000 c) $ 600,000 d) $0 You Answered Incorrectly. This answer is incorrect. Per ASC Topic 450, gain contingencies should not be reflected in the accounts since to do so might be to recognize revenue prior to its realization.

answer: d Gain contingency is an uncertain situation that will be resolved in the future, possibly resulting in a gain. The accounting standards do not allow the recognition of a gain contingency prior to settlement of the underlying event. Doing so might result in the excessively early recognition of revenue However, unlike gain contingency rule, "loss contingencies are reported on the balance sheet and footnotes on the financial statements, if they are probable and their quantity can be reasonably estimated."

The following items were among Wood Township's expenditures from the general fund during the year ended June 30, year 1: Furniture for Township Hall $10,000 Minicomputer for tax collector's office 15,000 The amount that should be classified as fixed assets in Wood's general fund balance sheet at June 30, year 1, is a) $25,000 b)$15,000 c) $10,000 d) $0

answer: d General fixed assets are reported in the governmental activity column in the government‐wide Statement of Net Position rather than in governmental funds.

Green Co. incurred leasehold improvement costs for its leased property. The estimated useful life of the improvements was 15 years. The remaining term of the nonrenewable lease was 20 years. These costs should be a) Expensed as incurred. b) Capitalized and depreciated over 20 years. c) Capitalized and expensed in the year in which the lease expires. d) Capitalized and depreciated over 15 year

answer: d Leasehold improvements should be depreciated over the remaining useful life of the leasehold improvement or the remaining life of the lease, whichever is shorter. Therefore, the leasehold improvement should be capitalized and depreciated over 15 years. For leasehold improvement, if you see that (1) estimated useful life and (2) remaining term of the lease agreement, choose whichever is the lesser one to capitalize and depreciate the cost.

On December 31, year 1, Madrid Township paid a contractor $2,000,000 for the total cost of a new firehouse built in year 1 on Township‐owned land. Financing was by means of a $1,500,000 general obligation bond issue sold at face value on December 31, year 1, with the remaining $500,000 transferred from the general fund. What should be reported on Madrid's year 1 financial statements for the Capital Project Fund? a) Revenues, $1,500,000; Expenditures, $1,500,000. b) Revenues, $1,500,000; Other financing sources, $500,000; Expenditures, $2,000,000. c) Revenues, $2,000,000; Expenditures, $2,000,000. d) Other financing sources, $2,000,000; Expenditures, $2,000,000.

answer: d Neither proceeds from a general obligation bond nor transfers from the general fund are revenues.

During the current year, the local humane society, a nongovernmental not‐for‐profit organization, received a $100,000 permanent endowment from Cobb. Cobb stipulated that the income must be used to care for older horses that can no longer race. The endowment reported income of $8,000 in the current year. What amount of unrestricted contribution revenue should the humane society report for the current year? a) $108,000 b) $100,000 c) $ 8,000 d) $0

answer: d The endowment and the income from the fund are both restricted. (i think interest income from endowment is acceptable and it is reclassify as unrestricted income or temporary income)

Rice Corp. adopted a defined benefit pension plan on January 1, year 1. The plan does not provide any retroactive benefits for existing employees. The pension funding payment is made to the trustee on December 31 each year. The following information is available for year 1 and year 2: Year 1 Year 2 Service cost $150,000 $165,000 Funding payment $170,000 185,000 Interest on projected benefit ‐‐ 15,000 obligation Actual return on plan assets ‐‐ 18,000 Experience gains or losses ‐‐ ‐‐ In its December 31, year 2 balance sheet, Rice should report pension asset/liability of a)$20,000 b)$25,000 c)$40,000 d)$43,000

answer: d Pension asset/liability is the cumulative excess of the amount funded over the amount recorded as pension expense. In year 1, funding was $170,000 and the only element of pension expense which applied was service cost ($150,000), so pension asset/liability was $20,000 at 12/31/Y1. In year 2, pension expense was $162,000 as computed below. Service cost $165,000 Interest on P.B.O. 15,000 Actual return on plan assets (18,000) Pension expense $162,000 Therefore, the year 2 funding ($185,000) exceeded pension expense ($162,000) by $23,000, increasing pension asset/liability to $43,000 ($20,000 + $23,000). The term pension service cost refers to the present value of the projected retirement benefits earned by plan participants in the current period. Generally, a company's pension service cost is the amount it must set aside in the current period to match the retirement benefits accrued by plan participants.

Dahlia, Inc. signed a lease to rent equipment on July 1, year 1. On January 1, year 3, Dahlia decides that the equipment is no longer needed, and the company pays a $20,000 penalty to cancel the lease. How should the cost of termination be disclosed on Dahlia's income statement? a) Recognize the cost of termination as a discontinued operation net of tax. b) Recognize the cost of termination as an extraordinary item net of tax. c) Recognize the cost of termination as a component of other comprehensive income. d) Recognize the cost of termination as a part of income from continuing operations.

answer: d Per ASC Topic 820, termination costs of an operating lease are included in calculating income from continuing operations. If the termination of a lease is associated with the exit or disposal of a discontinued operation, these costs are included in the results of discontinued operations.

Albert University, a private not‐for‐profit university, had the following cash inflows during the year ended June 30, year 1: I. $500,000 from students for tuition. II. $300,000 from a donor who stipulated that the money be invested indefinitely. III. $100,000 from a donor who stipulated that the money be spent in accordance with the wishes of Albert's governing board. (meaning it's unrestricted fund) On Albert University's statement of cash flows for the year ended June 30, year 1, what amount of these cash flows should be reported as operating activities? a) $900,000 b) $400,000 c) $800,000 d) $600,000

answer: d Student tuition $500,000 + donation of unrestriction $100,000 = $600,000 should be revenue. Only unrestricted donation should be considered as revenue and donation that has restriction should not be part of operating activity. restricted donation fund should be reported in financing activity

Winn Co. sells subscriptions to a specialized directory that is published semiannually and shipped to subscribers on April 15 and October 15. Subscriptions received after the March 31 and September 30 cutoff dates are held for the next publication. Cash from subscribers is received evenly during the year and is credited to deferred subscription revenue. Data relating to year 2 are as follows: Deferred subscription revenue, 1/1/Y2 $ 750,000 Cash receipts from subscribers 3,600,000 In its December 31, year 2 balance sheet, Winn should report deferred subscription revenue of a) $2,700,000 b) $1,800,000 c) $1,650,000 d) $ 900,000

answer: d The 12/31/Y2 balance of deferred subscription revenue should reflect the liability for subscriptions still outstanding at that time. The 12/31/Y1 deferred revenue ($750,000) would have been earned when the April 15 directory was mailed, and therefore is no longer a liability. The cash collected through the September 30 cutoff date (9/12 × $3,600,000 = $2,700,000) would also have been earned when the April 15 and October 15 directories were mailed (note that the cash was received evenly throughout the year). However, the cash collected after September 30 (3/12 × $3,600,000 = $900,000) will not be earned until the 4/15/Y3 directory is mailed, and therefore is deferred revenue at 12/31/Y2. ***.Cash received after 9/30 isn't going to be recognized as revenue until the next publication. So any October, November or December (3 months out of 12l) cash receipts are liabilities on the balance sheet until the next publication date.***In this question, "evenly throughout the year" just lets us know that we have received $300,000 in subscription payments from customers each month of the year ($3,600,000/12). All of these receipts except for the receipts in October, November, and December can be recognized as revenue since the obligations of those receipts were met on April 15 and October 15. Receipts for October-December cannot be recognized as revenue until the next publication date of April 15, year 2.

Under IFRS, the approach used in segment reporting is known as a) Segment approach. b)Revenue approach. c)CFO approach. d)Management approach.

answer: d The approach to segment reporting is known as the management approach whereby segments are organized by the way management organizes segments internally to make operating decisions and assess performance.

Timp, Inc. had the following common stock balances and transactions during year 2: 1/1/Y2 Common stock outstanding 39,000 2/1/Y2 Issued a common stock dividend 3,000 7/1/Y2 Issued common stock for cash 8,000 12/31/Y2 Common stock outstanding 50,000 What was Timp's year 2 weighted‐average shares outstanding? a) 40,000 b) 44,250 c) 44,500 d) 46,000

answer: d The computation of weighted‐average shares outstanding is Date # of Fraction WA 1/1 39,000 × 12/12= 39,000 2/1 3,000 × 12/12= 3,000 7/1 8,000 × 6/12 = 4,000 Total: 46,000 The 3,000 shares issued as a result of a stock dividend are weighted at 12/12 instead of 11/12 because for EPS purposes stock dividends are treated as if they occurred at the beginning of the year. *여기서 핵심* stock dividend would be treated as if it had occurred at the beginning of the fiscal year*

Gar, Inc.'s trial balance reflected the following liability account balances at December 31, year 1: Accounts payable $19,000 Bonds payable, due year 2 34,000 Deferred income tax liability 4,000 Discount on bonds payable 2,000 Dividends payable on 2/15/Y2 5,000 Income tax payable 9,000 Notes payable, due 1/19/Y3 6,000 The deferred income tax liability is based on temporary differences stemming from different depreciation methods for financial reporting and income taxes. In Gar's December 31, year 1 balance sheet, the current liabilities total was $71,000 $69,000 $67,000 $65,000

answer: d The current liabilities consist of all payable due within one year. Accounts payable $19,000 +) Bonds payable, due year 2. 34,000 -) Discount on bonds payable 2,000 +) Dividends payable 5,000 +) Income tax payable 9,000 Total current liability: $65,000 The "deferred income tax payable" of $4,000 is a separate "deferred category" on the balance sheet, and is not considered a current item. Deferred income taxes are taxes that a company will eventually pay on its taxable income, but which are not yet due for payment. ... The tax liability is frequently recorded as a long-term liability in the balance sheet, since there is usually no expectation of paying it within the next 12 months. The "notes payable" due 1/19/year3 are due after one year and are considered a long-term liability. Dividends payable are dividends that a company's board of directors has declared to be payable to its shareholders. Until such time as the company actually pays the shareholders, the cash amount of the dividend is recorded within a dividends payable account as a current liability. Dividends payable are nearly always classified as a short-term liability, since the intention of the board of directors is to pay the dividends within one year. Thus, dividends payable should be included in any short-term liquidity calculations, such as the current ratio or the quick ratio.

Kent, Inc.'s reconciliation between financial statement and taxable income for Year 2 follows: Pretax financial income $150,000 Permanent difference (12,000) 138,000 Temporary difference—depreciation (9,000) Taxable income $129,000 Additional information At 12/31/Y1 12/31/Y2 Cumulative temporary difference (future taxable amounts) $11,000 $20,000 The enacted tax rate was 34% for Year 1, and 40% for Year 2 and years thereafter. In its December 31, Year 2, balance sheet, what amount should Kent report as deferred income tax liability? a) $3,600 b) $6,800 c )$7,340 d) $8,000

answer: d The deferred tax liability to be reported at 12/31/Y3 is equal to the future taxable amounts that exist as a result of past transactions multiplied by the appropriate tax rate. The deferred tax liability is computed using the current tax rate unless there is a different enacted future tax rate that will be in effect when the temporary differences become taxable. In this case, the current tax rate at 12/31/Y3 is 40%, and there is no other future tax rate enacted, so the 12/31/Y3 deferred tax liability is $8,000 (40% × future taxable amounts of $20,000).

Disclosure of credit risk of financial instruments with off‐balance‐sheet risk does not have to include a) The amount of accounting loss the entity would incur should any party to the financial instrument fail to perform. b) The entity's policy of requiring collateral or security. c) The class of financial instruments held. d) The specific names of the parties associated with the financial instrument.

answer: d The following disclosures are required about credit risk for financial instruments with off‐balance‐sheet credit risk: • The amount of accounting loss the entity would incur should any party to the financial instrument fail to perform according to the terms of the contract and the collateral, if any, is of no value. • The class of financial instruments held • Categorization between instruments held for trading purposes and purposes other than trading. (이게 무슨말이냐면, risk of loss of principal or loss of a financial reward stemming from a borrower's failure to repay a loan or otherwise meet a contractual obligation. )..

On December 30, year 1, Ames Co. leased equipment under a capital lease for 10 years. It contracted to pay $40,000 annual rent on December 31, year 1, and on December 31 of each of the next 9 years. The capital lease liability was recorded at $270,000 on December 30, year 1, before the first payment. The equipment's useful life is 12 years, and the interest rate implicit in the lease is 10%. Ames uses the straight‐line method to depreciate all equipment. In recording the December 31, year 2, payment, by what amount should Ames reduce the capital lease liability? a) $27,000 b) $23,000 c) $22,500 d) $17,000

answer: d The initial lease obligation at 12/31/Y1 was $270,000. The first payment was made the same day, and therefore consisted entirely of principal reduction. After the payment, the lease obligation was $230,000 ($270,000 − $40,000). The next lease payment, on 12/31/Y2, consists of both principal and interest. The interest portion is $23,000 ($230,000 × 10%), so the reduction in the lease liability is $17,000 ($40,000 − $23,000).

The following information is available from Sand Corp.'s accounting records for the year ended December 31, year 2: Cash received from customers $870,000 Rent received 10,000 Cash paid to suppliers and employees 510,000 Taxes paid 110,000 Cash dividends paid 30,000 Net cash flow provided by operations for year 2 was a) $220,000 b) $230,000 c) $250,000 d) $260,000

answer: d The net cash flow provided by operations can be computed using either the indirect approach or the direct approach. In this question, the direct approach is used. Under the direct approach, each income statement item is directly adjusted for changes in related balance sheet items, and these adjusted amounts are used to compute the net cash flow. The items in this question have already been adjusted, so the net cash flow can be computed as follows: Cash received from customers $870,000 Rent received 10,000 Cash paid to suppliers and employees (510,000) Taxes paid (110,000) Net cash flow provided by operations $260,000 *cash dividend는 financing activity라서 operating activity에 포함 하지 않음* (어떻게 보면, direct method cash flow가 더 쉬운거같다. 그냥 cash inflow 는 더하고, cash outflow는 빼주는것이니깐. indirect method같은경우는 "increased in inventory"할때 increase 된만큼 net income에서 빼주고 "increased in depreciation"할때 increase된만큼, depreciation을 빼줬다..더 생각할게 많고 골치 아푼것같다.

The lessee's net carrying value of an asset arising from the capitalization of a lease would be periodically reduced by the a) Total minimum lease payment. b) Portion of minimum lease payment allocable to interest. c) Portion of minimum lease payment allocable to reduction of principal. d) Depreciation/amortization of the asset.

answer: d The solutions approach is to prepare the journal entry for the lease payment Capital lease obligation (principal) xxx Interest expense xxx Cash xxx and the journal entry for the lease amortization. Amortization of leased asset xxx Accum amortization/depreciation xxx Therefore, only the amortization of the leased asset results in a reduction of the carrying value of the asset. *notice, reduction of carrying value asset is different from paying capital lease obligation and interest..

Abbey Corporation prepares its financial statements in accordance with IFRS. Abbey acquired equipment by signing a $100,000 note payable with the seller of the equipment. How should this transaction be reported on the statement of cash flows? a) As an outflow of cash from investing activities and an inflow of cash from financing activities. b) As an inflow of cash from financing activities and an outflow of cash from operating activities. c) At the bottom of the statement of cash flows as a significant noncash transaction. d) In the notes to the financial statements as a significant noncash transaction.

answer: d This transaction did not involve an exchange of cash; therefore, it is not included on the statement of cash flows. IFRS requires that significant noncash transactions be reported in the notes to the financial statements.

Taft Corp., which began business on January 1, year 1, appropriately uses the installment sales method of accounting. The following data are available for December 31, year 1 and year 2. Balance of deferred gross profit on sales account Year 1 Year 2 Year 1 sales $300,000 $120,000 Year 2 sales ‐ 440,000 Gross profit on sales 30% 40% The installment accounts receivable balance at December 31, year 2, is a) $1,000,000 b) $1,100,000 c) $1,400,000 d) $1,500,000

answer: d When using the installment sales method, the balance of the deferred gross profit account represents the gross profit not yet recognized because the related receivable has not yet been collected. The formula below expresses this relationship. Deferred GP = GP rate × Accounts Receivable This equation can be rearranged as follows: Deferred GP / GP rate = Accounts Receivable Therefore, the installment accounts receivable balance at 12/31/Y2 can be computed as follows: From year 1 sales: $120,000 / 30%= $400,000 From year 2 sales: $440,000 / 40%= 1,100,000 Total $1,500,000

The following information pertains to Kane Co.'s defined benefit pension plan: Pension asset (liability), January 1, year 5 $ 2,000 Service cost 19,000 Interest cost 38,000 Actual return on plan assets 22,000 Amortization of unrecognized prior service cost 52,000 Employer contributions 40,000 The fair value of plan assets exceeds the projected benefit obligation. In its December 31, year 5 income statement, what amount should Kane report as pension cost? a) $45,000 b) $49,000 c) $67,000 d) $87,000

answer: d Year 5 pension expense, computed below, is recorded by debiting pension expense and crediting the pension asset/liability account. Service cost $19,000 Interest cost 38,000 Actual return on plan assets (22,000) Amort. of unrecognized PSC 52,000 Pension expense $ 87,00

An entity sponsors a defined benefit pension plan that is underfunded by $800,000. A $500,000 increase in the fair value of plan assets would have which of the following effects on the financial statements of the entity? a) An increase in the assets of the entity. b) An increase in accumulated other comprehensive income of the entity for the full amount of the increase in the value of the assets. c) A decrease in accumulated other comprehensive income of the entity for the full amount of the increase in the value of the assets. d) A decrease in the liabilities of the entity.

answer: d because changes in the fair value (if unexpected) affect other comprehensive income through amortization in the event the amount of the change exceeds a corrido The plan is currently underfunded and remains underfunded after the asset increases. Reported pension liability is the underfunded amount, the difference between PBO and plan assets. This firm's reported pension liability decreased from $800,000 to $300,000 ($800,000 -$500,000)owing to the asset increase. • Funded Status: Difference between plan assets (PA) and Projected Benefit Obligation (PBO) - Overfunded when PA>PBO • Asset to the firms? - Underfunded when PA<PBO • Liability to the firm?

On January 1, year 2, an intangible asset with a 35‐year estimated useful life was acquired. On January 1, year 6, a review was made of the estimated useful life, and it was determined that the intangible asset had an estimated useful life of 45 more years. As a result of the review a) The original cost at January 1, year 2, should be amortized over a 50‐year life. b) The original cost at January 1, year 2, should be amortized over the remaining 30‐year life. c) The unamortized cost at January 1, year 6, should be amortized over a 40‐year life. d) The unamortized cost at January 1, year 6, should be amortized over a 45‐year life.

answer: d because if the estimated useful life of an intangible asset is revised, the unamortized cost should be allocated over the remaining periods of the new useful life

The governing board of Smithson Hospital, a nonprofit hospital affiliated with a religious organization, acquired 100 BMI Company bonds for $103,000 on June 30, year 1. The bonds pay interest on June 30 and December 30. On December 31, year 1, interest of $3,000 was received from BMI, and the fair value of the BMI bonds was $105,000. The governing board acquired the BMI bonds with cash which was unrestricted, and it classified the bonds as trading securities at December 31, year 1, since it intends to sell all of the bonds in January year 2. As a result of the investment in BMI bonds, what amount should be included in revenue, gains, and other support on the statement of operations for the year ended December 31, year 1? a)$0 b) $3,000 c) $2,000 d) $5,000

answer: d because unrealized gains on trading securities should be included as part of the amount reported for revenue, gains, and other support on the statement of operations. Fair value of trading security was $105,000 and purchased value $103,000. there is $2,000 unrealized gain. notice that, trading security doesn't go into other comprehensive income but it directly affect the net income. so, we need to add $2,000 to $3,000 interest received. Therefore $5,000

Encumbrances would not appear in which fund? a) Capital Projects. b) Special Revenue. c) General. d) Enterprise.

answer: d fixed dollar budgets normally are not adopted for proprietary funds. Thus, the Enterprise Fund would not integrate a budget, and encumbrances would not be appropriate in the Enterprise Fund accounting system Remember, in governmental accounting there are 3 types of fund: (1) governmental fund (2) propreitary fund (3) fidicuary fund. 여기서, governmental fund만, 순수하게 contribution, donation으로 받은 버젯을 뜻한다. Governmental fund includes: general, special revenue, debt service, capital projects, and permanent trust. Propreitary includes internal service fund and enterprise fund. (정부가 service를 제공헤서 revenue를 창출하는것. external user한테 (시민들이나,다른 정부 기간한테 서비스 제공하고 돈 버는것. 어떻게 보면, public company랑 제일 비슷한 구조다. For profit 처럼 운영되는 fund. 따라서, governmental fund type랑 틀리게 appropriation, fund balance, encumbrance이런 계념으로 budger를 잡는게 아니다.

Roe Company is preparing a statement of cash flows for the year ended December 31, year 2. It has the following account balances: 12/31/Y1 12/31/Y2 Machinery $250,000 $320,000 Accum depreciaton 102,000 120,000 Loss sale on machine 4,000 - During year 2, Roe sold for $26,000 a machine that cost $40,000, and purchased several items of machinery. Machinery purchases for year 2 amounted to a)$ 34,000 b)$ 70,000 c)$ 96,000 d)$110,000

answer: d 쉽게 생각하기. During year 2, company sold machine that cost $40,000..이뜻은, 이미 year 1에 purchase했던 machine을 year2때 판것이다. $250,000 안에 $40,000이 포함됬었다.. 그리고 새로운 purchase로 balance sheet machinery accout가 $320,000로 기록이 되면서, ($320,000 -$250,000= $70,000 purchase금액이 늘은것이다) $250,000 -$40,000 sold machinery= $210,000 이 사실상 beginning machinery balance가 되는것이다. year2에.. 그리고, year2의 machinery ending balance는 $320,000이니깐, (320,000 -210,000 = 110,000 purchase amount for year2 가 된다.) Note that when machinery is sold, the credit to the machinery account is the original cost ($40,000), not the book value ($30,000) or the cash proceeds ($26,000).

Beth Co. leased equipment to Wolf, Inc. on April 1, year 1. The lease is appropriately recorded as a direct financing lease by Beth. The lease is for an 8‐year period expiring March 31, year 9. The first equal annual payment of $500,000 was made on April 1, year 1. Beth had purchased the equipment on January 1, year 1, for $2,800,000. The equipment has an estimated useful life of 8 years with no residual value expected. Beth uses straight‐line depreciation and takes a full year's depreciation in the year of purchase. The cash selling price of the equipment is $2,934,000. Assuming an interest rate of 10%, what amount of interest income should Beth record in year 1 as a result of the lease? a) $0 b) $182,550 c) $243,400 d) $280,000

answer:b The present value of the eight $500,000 lease payments is given to be $2,934,000 (cash selling price of the equipment). 핵심* cash selling price of the equipment means PV (present value of lease payment) Since $500,000 is paid at the inception of the lease, the book value of the lease payments receivable (total minimum lease payments minus unearned interest income) outstanding for the last 9 months is $2,434,000. The 10% interest thereon is $243,400, but only 3/4 (9 months/12 months) of this amount, or $182,550, is associated with the period ending December 31, year 1. Direct-financing lease is basically the coupling of a sale and financing transaction. In this case, the lessor removes the leased asset from its books and replaces it with a receivable from the lessee. The only income recognized by the lessor is the interest received

Which of the following classifications is required for reporting of expenses by all not-for-profit entities? A. Natural classification in the statement of activities or notes to the financial statements B. Functional classification in the statement of activities or notes to the financial statements C. Functional classification in the statement of activities and natural classification in a matrix format in a separate document D. Functional classification in the statement of activities and natural classification in the notes to the financial documents

the answer is B. Financial reporting for a not-for-profit should provide information about the service efforts of the entity. Therefore, the FASB Accounting Standards Codification requires expenses to be reported by functional classification (i.e., program services, management, fundraising, etc.). Only those not-for-profits that are voluntary health and welfare entities must augment the functional classification of expenses that appears in the statement of activities with a natural classification of expenses, displayed in a matrix format, that is shown in a separate document, a statement of functional expenses. **not for profit (ngo) entity requires expenses to be supported by functional classification (ex: separating out the expenses by program services, management, fundraising, etc) but, for NGO that are Voluntary health and welfare such as boys scout or homeless, they requires expenses to be supported by functional classification that appears in the statement of activities, which is similar to income statement, with natural classification of expense, displayed in matrix format. (natural classification이란, functional 에서 더 쪼개서, 그안에서, wage 는 wage 끼리만, utility 는 utility 끼리만 또 따로, functional 안에서 split하고 쪼개서 보고하는것. Functional classification is a method of grouping expenses according to the purpose for which costs are incurred. Governments report expenditures in the governmental fund records and expenses in the government-wide records by activity or departmental function aimed towards accomplishing a major service or regulatory responsibility. Functional classification for not-for-profit entities would separate expenses among (1) program, (2) fundraising, and (3) management and general (or supporting). Natural expense classification is a method of grouping expenses according to the kinds of economic benefits received in incurring those expenses. Examples of natural expense classifications include salaries and wages, employee benefits, supplies, rent, and utilities statement of activities -The complete set of financial statements for a not-for-profit entity includes a statement of activities, which is analogous (유사한) to the income statement of a business entity. In governmental accounting, a statement of activities is required for government-wide financial reporting. The formats of the not-for-profit and the governmental statement of activities differ. *statement of activities = income statement*

On January 1, year 1, Parr, Inc. purchased a machine for $600,000 and established an annual depreciation charge of $100,000 over a 6‐year life. During year 3, after issuing its year 2 financial statements, Parr performed the undiscounted future cash flow recovery test and concluded that the machine's operational value was impaired. It also concluded that $120,000 is a reasonable estimate of the fair value. The machine will be used during the period January 1, year 3, through December 31, year 5. In Parr's December 31, year 3 balance sheet, the machine should be reported at a carrying amount of a) $ 80,000 b)$120,000 c)$300,000 d) $360,000

answer: a

Which of the following is an appropriate market approach for determining fair value measurements? a) Using relevant information from recent transactions. b) Using present value techniques to discount cash flows. c) Using the current replacement cost of the asset. d) Using the undiscounted cash flows from the asset.

answer: a Market approach- Relevant information from recent transactions is a market approach to determining fair value measurement. Income approach- Using present value techniques to discount cash flows is an income approach. Fair value approach - Current replacement cost is a cost approach to measuring fair value.

Brass Co. reported income before income tax expense of $60,000 for year 2. Brass had no permanent or temporary timing differences for tax purposes. Brass has an effective tax rate of 30% and a $40,000 net operating loss carryforward from year 1. What is the maximum income tax benefit that Brass can realize from the loss carryforward for year 2? a) $12,000 b) $18,000 c) $20,000 d) $40,000

answer: a The $40,000 net operating loss carryforward is the amount of loss in year 1 which may be carried forward to future years to offset the tax due in those years. The value of the carryforward is the net operating loss multiplied by the tax rate for the year in which the carryforward is expected to be realized. Therefore, the maximum income tax benefit that Brass can realize from the carryforward in year 2 is $12,000 $60,000 x 30% tax rate = $18,000 (Before applying carryforward from PY) if we use $40,000 carryforward from PY, it is $60,000 - $40,000 = $20,000 x 30% tax rate = $6,000 so the benefit is $18,000 - $6,000 = $12,000.

Under IFRS, a provision is a) An event which is not recognized because it is not probable or cannot be measured reliably. b) An event which is probable and measurable. c) An event which is probable, but not measurable. d) An event which is probable, possible, or remote and measurable.

answer:b because a provision is probable and measureable. provision is not possible or remote. financial note disclose할때도 possible 또는 remote경우에 include하지 않는것처럼, provision또한 possible & remote한것에대해서 작성하는게 아니고 오직 "Probable and measure하는것에만" 포함하는것.

Faucet Company has 2,500,000 shares of common stock outstanding on December 31, year 1. An additional 500,000 shares of common stock were issued on April 1, year 2, and 250,000 more on July 1, year 2. On October 1, year 2, Faucet issued 5,000, $1,000 face value, 7% convertible bonds. Each bond is convertible into 40 shares of common stock. No bonds were converted into common stock in year 2. What is the number of shares to be used in computing basic earnings per share and diluted earnings per share, respectively, for the year ended December 31, year 2? a) 2,875,000 and 2,975,000. b) 2,875,000 and 3,075,000. c) 3,000,000 and 3,050,000. d) 3,000,000 and 3,200,000.

Answer: c *remember, when we are calculating Basic EPS or Dilute EPS, we have to use weighted average share outstanding when calculating denominator of the formula To determine the number of shares used for the calculation of basic earnings per share (BEPS) and diluted earnings per share (DEPS), first determine the weighted‐average shares outstanding. 1/1/Year2 Outstanding 2,500,000 4/1/Year 2 Issuance (500,000 Outstanding Stock × 9/12month= 375,000) 7/1/Year2 Issuance (250,000 Outstanding Stock × 6/12 month= 125,000) $2,500,000 + 375,000 + 125,000 = 3,000,000 For DEPS the bonds would be considered converted. Since there were 5,000 bonds, each convertible into 40 shares of stock, this results in an additional 50,000 equivalent shares (5,000 × 40 × 3/12 Month.). Thus, there would be 3,000,000 shares outstanding for BEPS and 3,050,000 shares outstanding for DEPS.

Which of the following should be disclosed in the summary of significant accounting policies? a) Composition of plant assets. b) Pro forma effect of retroactive application of an accounting change. c) Basis of consolidation. d) Maturity dates of long‐term debt.

Answer: c This answer is correct because ASC Topic 235 states that the summary of significant accounting policies should encompass those accounting principles and methods that involve a selection from existing acceptable alternatives (or are peculiar to the industry in which the entity operates). Of the answers listed, only basis of consolidation involves a choice among acceptable methods.

In determining diluted earnings per share, a potentially dilutive security was antidilutive in year 1 and dilutive in year 2. The common stock equivalent would be included in the computation for Answer: Year 1 Year 2 No Yes

In computing diluted EPS, only dilutive common stock equivalents are included. Furthermore, the determination of whether a potentially dilutive security is dilutive must be made at every reporting date. A situation may arise in which a potentially dilutive security may be dilutive at one reporting date and antidilutive the next. In this circumstance, the potentially dilutive security should only be included in the diluted EPS calculation on the date on which it is dilutive. Therefore, the potentially dilutive security would be included in the year 2 computation but not the year 1 computation. *Diluted EPS is a performance metric used to measure the quality of a company's earnings per share (EPS) if all convertible securities were exercised. Convertible securities are all outstanding convertible preferred shares, convertible debentures, stock options (primarily employee-based) and warrants.

In an arm's‐length transaction, Company A and Company B exchanged nonmonetary assets with no monetary consideration involved. The exchange was deemed to have commercial substance for both Company A and Company B, and the fair values of the nonmonetary assets were both clearly evident. The accounting for the exchange should be based on the: Fair value of the asset surrendered. because an exchange of nonmonetary assets that has commercial substance is recorded at the fair value of the asset surrendered. (여기서 asset surrendered란 내가 exchange transaction에서 상대방한텥 exchange한 나의 asset을 가르킨다. asset received는 내가 상대방한테 받은 asset.) 따라서, non-monetary asset exchange할시에는, fair value of asset i surrendered (내가 상대방한테 주는 나의 original asset)으로 기록해야한다.

Monetary assets and liabilities (cash, liquid securities, accounts payable and receivable, debt) Nonmonetary assets and liabilities (fixed assets and inventory)

A machine with a 4‐year estimated useful life and an estimated 15% salvage value was acquired on January 1. Would depreciation expense using the sum‐of‐the‐years' digits method of depreciation be higher or lower than depreciation expense using the double‐declining balance method of depreciation in the first and second years? First year Second year Lower Higher

Sum-of-the year's digit will have lower depreciation amount in first year but higher depreciation amount than double declining balance in second year, because the equation for calculating sum‐of‐the‐years' digits (SYD) depreciation is SYD depr. = Years remaining SYD × (Cost - Salvage value) Year 1: 4/10(100% - 15%) = 34.0% Year 2: 3/10(100% - 15%) = 25.5% (여기서 10이란, useful life 4year를 다 더한거. (1+2+3+4= 10 year) DDB = 200%/Useful life × Book value Year 1: 200%/4(1.00) = 50.0% Year 2: 200%/4(1.00 − 0.50) =25.0% Therefore:Year 1: SYD < DDB (34.0% < 50.0%) Year 2: SYD > DDB (25.5% > 25.0%) Recall that salvage value is included in the SYD calculation and not in the DDB calculation

Which of the following is a research and development cost? a) Development or improvement of techniques and processes. b) Offshore "oil" exploration that is the primary activity of a company. c) Research and development performed under contract for others. d) Market research related to a major product for the company.

Answer:A This answer is correct because development or improvement of techniques and processes is included in research and development cost. R&D specifically excludes activities in extractive industries. (Ex: oil, mining) The market research is for an existing major product and is not the discovery of new knowledge that may be useful in developing a new product or improving existing products or services. Extractive Defintion: Any processes that involve the extraction of raw materials from the earth to be used by consumers. The extractive industry consists of any operations that remove metals, mineral and aggregates from the earth. Examples of extractive processes include oil and gas extraction, mining, dredging and quarrying

An impairment loss for a long‐lived asset, which is being used in the operations of a business, is measured by the excess of the asset's carrying amount over its a) Expected undiscounted selling price, less expected costs of disposal. b) Fair value. c) Expected undiscounted future cash flows from use and disposal. d) Fair value less cost to sell. ASC Topic 360 explains that an impairment loss shall be measured as the amount by which the carrying amount of the asset exceeds the fair value of the asset. The fair value of an asset is determined by the guidelines set in ASC Topic 820. Fair value is determined by using the principal or most advantageous market and assumes the asset is used in its highest and best use.

Answer: B ASC Topic 360 explains that an impairment loss shall be measured as the amount by which the carrying amount of the asset exceeds the fair value of the asset. The fair value of an asset is determined by the guidelines set in ASC Topic 820. Fair value is determined by using the principal or most advantageous market and assumes the asset is used in its highest and best use. Definition of Impairment loss: An impairment loss is a recognized reduction in the carrying amount of an asset that is triggered by a decline in its fair value. When the fair value of an asset declines below its carrying amount, the difference is written off. Carrying amount is the acquisition cost of an asset, less any subsequent depreciation and impairment charges. Example: During June year 1, Maxwell Corporation determined that actual costs incurred associated with the equipment used in its assembly line significantly exceeded original expected costs. At June 30, year 1, Maxwell had compiled the following information: Original cost of the equipment $800,000 Accumulated depreciation $300,000 Expected net future cash inflows (undiscounted) related to the continued use and eventual disposal of the equipment $450,000 Fair value of the equipment $375,000 What is the amount of impairment loss that should be reported on Maxwell's income statement prepared for the period ended June 30, year 1? a) $125,000 b) $350,000 c) $375,000 d) $ 50,000 answer: a The undiscounted expected net future cash inflows ($450,000) are less than the carrying amount of the equipment ($500,000). Therefore, the equipment is deemed impaired. The impairment loss is calculated by subtracting the fair value of the equipment ($375,000) from its carrying value ($500,000). (Note: Both values are at the impairment date, June 30, year 1.) The impairment loss that should be reported on Maxwell's June 30, year 1 income statement is $125,000.

Alfisol, Inc. offers sales discounts of 2% on all credit sales paid within 15 days. For year 1, gross credit sales totaled $150,000 and 75% of Alfisol's customers took advantage of the discount. Under the "net method" (여기서 핵심 단어는 "USE NET METHOD!"): A) Each sale should not be recorded until payment is received and it is known whether the discount is taken. B) Allowance for sales discounts must be credited for $2,250. C) For cash receipts within the discount period, sales discounts must be debited for $2,250. D) For cash receipts after the discount period, discounts not taken must be credited for $750.

Answer: D Under the "NET METHOD", sales are initially recorded net of discounts. 상대방이 discount period를 이용할걸 미리 예측하고, D/C된금액으로 record를 미리 하는것. Payments received after the discount period total $37,500 ($150,000 × 25%), and the amount of discounts forfeited is $750 ($37,500 × 2%). Under the net method, the entry to record these receipts is: Upon sale: AR 147,000 Sales 147,000 ($150,000 x 98% = $147,000. <-assume 2% discount will be applied.) Receipts within discount period (75%) Cash 110,250 AR 110,250 Receipts within non-discount period (25%) Cash 37,500 AR 36,750 Discounts not taken 750 Remember, there's also "Gross method" Which is different from "Net method". Please know the difference!! Under the gross method, sales are initially recorded at the gross amount. Payments received within the discount period total $112,500 ($150,000 × 75%). Under the gross method, the entry to record these receipts is Cash 110,250 Sales discounts 2,250 AR 112,500 Alternatively, an allowance account may be established, in which case the following entries would be recorded. Upon sale: AR 150,000 Sales 150,000 Allow for sales D/C 2,250 Sales discounts 2,250 Receipts within discount period (75%) Cash 110,250 Allow for sales D/C 2, 250 AR 112,500 Receipts within non-discount period (25%) Cash 37,500 AR 37,500 Note that under the allowance method, sales discounts is debited at the time of sale instead of at the time of cash receipt.

Substantial doubt about an entity's ability to continue as a going concern: A) Must be evaluated by management on an interim and annual basis. B) Is an evaluation for auditors to make, not management. C) Is an optional evaluation for management. D) Must be evaluated by management on an annual basis only.

Answer: a Substantial doubt about an entity's ability to continue as a going concern must be evaluated by management on an interim and annual basis. Key word: "Must be" evaluated by "Management (not auditor), "Interim and Annual basis" (not just end of year)

Smith Co. has a checking account at Small Bank and an interest-bearing savings account at Big Bank. On December 31, year 1, the bank reconciliations for Smith are as follows: Big Bank- Bank balance $150,000 Deposit in transit 5,000 Book balance 155,000 Small Bank- Bank balance $ 1,500 Outstanding checks (8,500) Book balance (7,000) What amount should be classified as cash on Smith's balance sheet at December 31, year 1? a) $148,000 b) $151,000 c) $155,000 d) $156,000

Answer: c This answer is correct because the cash on Smith's balance sheet is equal to the cash in the savings account in Big Bank of $155,000. The negative balance in the checking account at Small Bank would be reclassified as a payable and is reported as a liability on the balance sheet. deposits in transit and outstanding checks should be considered when determining "cash" and negative cash balances should be recorded as a liability, not netted with positive cash balances 아직 check outstanding (company's payment remit to other party)가 bank에서 빠져나가지 않았으면, cash로 인식되나보다)

Megan Corporation values its inventory at the lower of cost or net realizable value as required by IFRS. Megan has the following information regarding its inventory. Historical cost $10,000 Estimated selling price 9,000 Estimated costs to complete and sell 500 Replacement cost 8,000 What is the amount for inventory that Megan should report on the balance sheet under the lower of cost or net realizable value method? A) $10,000 B) $9,000 C) $8,500 D) $7,500 IFRS requires inventory to be reported at the lower of cost or net realizable value (LCNRV). This method requires net realizable value to be calculated as the estimated selling price less estimated costs of completion and estimated costs to sell. Therefore, the NRV is $8,500 ($9,000 − $500). The lower of cost or net realizable value is determined by comparing the cost of $10,000 to the NRV of $8,500, and using the lower amount. Inventory should be reported at $8,500.

Answer: c IFRS requires inventory to be reported at the lower of cost or net realizable value (LCNRV). This method requires net realizable value to be calculated as the estimated selling price less estimated costs of completion and estimated costs to sell. Therefore, the NRV is $8,500 ($9,000 − $500). The lower of cost or net realizable value is determined by comparing the cost of $10,000 to the NRV of $8,500, and using the lower amount. Inventory should be reported at $8,500.

The purchase of treasury stock a) Decreases common stock authorized. b) Decreases common stock issued. c) Decreases common stock outstanding. d) Has no effect on common stock outstanding.

Answer: c because only the common stock outstanding will be decreased by the amount of treasury stock purchased. When a company reacquires its own stock, the purchase does not reduce the number of shares issued or authorized, but does reduce the number of shares outstanding and the total stockholders' equity. Outstanding shares refer to a company's stock currently held by all its shareholders, including share blocks held by institutional investors and restricted shares owned by the company's officers and insiders. Outstanding shares are shown on a company's balance sheet under the heading "Capital Stock." (회사 본인이 자신회사의 stock을 다시 shareholder나 investor로부터 사들이면, 그들의 소유가 줄어진다, meaning outstanding shares (refers to sock held by all its shareholders) will be decrease. 핵심: once stock has been issued, the total common stock issued is decreased only when it is repurchased and retired.

A firm has basic earnings per share of $1.29. If the tax rate is 30%, which of the following securities would be dilutive? a. Six percent, $100 par cumulative convertible preferred stock, issued at par, with each preferred share convertible into four shares of common stock. b. Seven percent convertible bonds, issued at par, with each $1,000 bond convertible into 40 shares of common stock. c. Ten percent convertible bonds, issued at par, with each $1,000 bond convertible into 20 shares of common stock. d. Cumulative 8%, $50 par preferred stock.

Choice "b" is correct. <CAS해설> a. (6% x $100)/4 = $1.5/share > $1.29/share(BEPS) : anti-dilutive이므로 제외 b. {7% x 1,000 x (1-30%)}/40 = $1.225/share < $1,29/share(BEPS) : dulitive c. {10% x 1,000 x (1-30%)}/20 = $3.5/share > $1.29/share(BEPS) : anti-dilutive이므로 제외 d. "convertible"이 아니므로 dilutive와 무관 *여기서 핵심은 : Generally, if a company has convertible securities, the diluted EPS is less than its basic EPS.* and note net income is after tax. 이미 tax가 반영된것이 EPS이기에, dilute eps구할때도 tax 반영해서 구하는것. <BECKER 해설> A dilutive security will produce an earnings per share number below basic earnings per share. (dilutive security will give earnings per share number lower than $1.29 which is basic earnings per share) The formula for basic earnings per share is income available to common shareholders divided by the weighted average number of common shares outstanding. Basic earnings per share is $1.29, and a dilutive security will result in a lower earnings per share number. If the seven percent convertible bonds are converted, the company will save $49 on each bond ($1,000 x .07 x (1 - .30)), but 40 new shares of stock will be issued. This equates to $1.225 per 1 new share, which is a lower ratio than $1.29 per share. So these securities will be dilutive. Choice "d" is incorrect. There is no indication given that the shares are convertible, so they will not be dilutive. Choice "c" is incorrect. If the ten percent convertible bonds are converted, the company will save $70 on each bond ($1,000 x .10 x (1 - .30)) and 20 new shares of stock will be issued. This equates to $3.50 per 1 new share, which is a higher ratio than $1.29 per share. So these securities will be anti-dilutive. Choice "a" is incorrect. If the convertible preferred stock is converted, the company's earnings per share will increase in the numerator by the $6 dividend that will no longer be paid, while the denominator will increase by 4 for the new shares of common stock issued. That equates to $1.50 per share, which is higher than $1.29.

Compensatory stock options were granted to executives on January 1, year 1, for services to be rendered during year 1, year 2, and year 3. The options are accounted for under ASC Topic 718. The fair value of the option was measured at the grant‐date fair value using the observable market price of an option with similar terms. The fair value of the options was in excess of the amount the executives must pay for the stock. The stock options were exercised on December 30, year 3. Compensation expense should be recognized in the income statement in which of the following years? Year 1 Year 2 Year 3 Yes Yes Yes

This answer is correct because per ASC Topic 718, compensation expense resulting from compensatory stock option plans is typically determined at the grant date, and then allocated to the periods in which the employees perform the services. 여기서 Key word는 "compensation stock"이다.. service를 제공한 사람들한테 주는 보상.

Last year, Katt Co. reduced the carrying amount of its long‐lived assets used in operations from $120,000 to $100,000, in connection with its annual impairment review. During the current year, Katt determined that the fair value of the same assets had increased to $130,000. What amount should Katt record as restoration of previously recognized impairment loss in the current year's financial statements? a) $0 b) $10,000 c)$20,000 d)$30,000

answer: A This answer is correct because under U.S. GAAP, previously recognized impairments of fixed assets may not be recovered or reversed. FASB follows GAAP, and GAAP does not allow recovery of impairments. If something is written down, it is left as it is, and it is not written back up, even if it was an error. You cannot really mess up with impairments. But as for IFRS, they allow impairment recovery Generally, there is "Revaluation method" where Carrying value is compared with FMV, and we recognize the gain or loss from this remeasurement. Ex: At the end of the year 1, company revalued its building to FMV of $2,700,000 and there was $200,000 revaluation gain. This means that our carrying value of building was $2,500,000. We would recognize $200,000 in OCI, as revaluation surplus account. At end of the year 2, the company revalued its building to FMV $2,600,000, causing $1,000,000 loss. First ,we want to offset $200,000 revaluation gain from year 1 in the OCI to decrease $1,000,000 loss. (1,000,000 -200,000 surplus = 800,000) Here, we will deduct 800,000 directly from the Income statement. 위에 Katt Company example은 아마, impair loss를 바로 income statement에서 deduct한것으로 보여진다. (The loss was not reported in the OCI in this case) An impairment loss makes it into the "total operating expenses" section of an income statement and, thus, decreases corporate net income. Also known as an impairment charge, an impairment loss happens when a company writes off products or assets that it considers damaged, unusable or less worthy -- operationally and financially speaking.

On April 1, Aloe, Inc. factored $80,000 of its accounts receivable without recourse. The factor retained 10% of the accounts receivable as an allowance for sales returns and charged a 5% commission on the gross amount of the factored receivables. What amount of cash did Aloe receive from the factored receivables? a) $68,000 b) $68,400 c) $72,000 d) $76,000

answer: a answer is correct. The proceeds from the factored accounts receivable is $80,000 less the 10% factor holdback of $8,000 ($80,000 × 10%), and less the 5% commission on the gross amount of receivables ($80,000 × 5%). Therefore, this answer is correct because the amount received from factoring the accounts receivable is $68,000 ($80,000 − $8,000 − $4,000) Factoring is a financial transaction and a type of debtor finance in which a business sells its accounts receivable (i.e., invoices) to a third party (called a factor) at a discount. A business will sometimes factor its receivable assets to meet its present and immediate cash needs. (In this question, company named Aloe, sold its A/R balance to other party without recourse, meaning Aloe don't take any responsibility on the uncollected A/R amount as factored company (other party buying A/R from Aloe) assumes the risk of noncollectable amount. Factoring A/R is when business sells its invoices, or receivables, to a third-party financial company known as a "factor." The factor then collects payment on those invoices from the business's customers. In Non-Recourse factoring, a company sells their accounts receivable to a factor, whom then supplies the cash needed to cover the invoices. The difference with non-recourse as opposed to recourse factoring is that the company has no liability with any uncollected invoices. The factor absorbs all the risk

The following information pertains to Lark Corp.'s available‐for‐sale securities portfolio: December 31 Year 2 Year 1 Cost $200,000 $200,000 Market value 240,000 180,000 Lark does not elect to use the fair value option for reporting financial assets. Differences between cost and market values are considered to be temporary. The decline in market value was properly accounted for at December 31, year 1. What is the amount of "Unrealized gain (loss) on marketable equity securities," a component of other comprehensive income, for the year ended to December 31, year 2? a) $60,000 b) $40,000 c) $20,000 d) $0

answer: a (여기서 제일 중요한것은, sales가 일어나지않으면, unrealized gain(loss)이고 이금액은, OCI (other comprehensive income)으로 balance sheet안에 들어간다. 명심해야할께 OCI do "accumulates"! 작년에 accumulate된거 이번년에도 accumulate하기. The amount by which an available‐for‐sale portfolio's aggregate carrying value exceeds its market value should be recognized as an unrealized loss and shown as "Other comprehensive income" which with net income make up comprehensive income. Note that at the end of each year the unrealized gain(loss) is closed to "Accumulated other comprehensive income," not "Retained earnings." In future years the portfolio's aggregate market value increases, the unrealized loss is removed from the accumulated other comprehensive income account and eventually an unrealized gain may be established. Lark Corp's journal entries for year 1 and year 2 were Year 1: Unrealized loss on MES 20,000 Investment in MES 20,000 Year 2: Investment in MES 60,000 Unrealized gain in MES 60,000 Therefore, the unrealized gain (loss) account would increase by $60,000 in year 2 for the recovery in market value of the previously recognized unrealized loss of $20,000 in year 1 plus the additional $40,000 increase in market value above cost.

Anchor Co. owns 40% of Main Co.'s common stock outstanding and 75% of Main's noncumulative preferred stock outstanding. Anchor exercises significant influence over Main's operations. During the current period, Main declared dividends of $200,000 on its common stock and $100,000 on its noncumulative preferred stock. *** "Anchor does not elect the fair value option for reporting its investment in Main. "*** What amount of dividend income should Anchor report on its income statement for the current period related to its investment in Main? a) $ 75,000 b) $ 80,000 c) $120,000 d) $225,000

answer: a Assuming Anchor accounts for its investment in Main common stock using the equity method, Anchor would recognize investment income in the amount of 40% of Main's income. Anchor's 40% share of the dividends on common stock declared by Main would reduce Anchor's investment account. In addition, Anchor would also recognize its proportionate share of preferred dividends from Main, $75,000 ($100,000 × 75%), as dividend income on the income statement. Therefore, this answer is correct. ***Note that if Anchor elected the fair value option for reporting its financial assets*****, both the preferred and common stock dividends would be included as dividend income for the period. Anchor would recognize 40% of the common stock dividends declared by Main (40% × $200,000 = $80,000) plus 75% of the preferred stock dividends declared by Main (75% × $100,000 = $75,000) for a total of $155,000 ($80,000 + $75,000) dividend income. Since this number is not one of the possible answers listed in the problem, Anchor must be using the equity method to account for its investment in the common stock of Main. (FMV option을 elect하게되면, Common stock도 아무리, equity method도라도 같이 계산해서 포함되나보다)

White Airlines sold a used jet aircraft to Brown Company for $800,000, accepting a 5‐year 6% note for the entire amount. Brown's incremental borrowing rate was 14%. The annual payment of principal and interest on the note was to be $189,930. The aircraft could have been sold at an established cash price of $651,460. The present value of an ordinary annuity of $1 at 8% for five periods is 3.99. The aircraft should be capitalized on Brown's books at a) $651,460 b) $757,820 c) $800,000 d) $949,650

answer: a Brown's 14% incremental borrowing rate is significantly higher than the stated rate of 6%. Therefore, the stated rate is unreasonable and the acquisition should not be recorded at the face value ($800,000) of the note. *핵심: when incremental borrowing rate is "significantly higher" than stated rate, the stated rate is incorrect and the company should not be using the acquired cost as it's face value. it must use The cost of the aircraft is the present value of the note and stated interest payments discounted at 14% or the fair market value of the aircraft, whichever is more clearly evident. Since the aircraft has an established cash price of $651,460, this amount is an appropriate basis for recording the transaction. incremental borrowing rate = Interest rate a lessee would have to pay if person decides to lease similiar asset

An issuer of bonds uses a sinking fund for the retirement of the bonds. Cash was transferred to the sinking fund and subsequently used to purchase investments. The sinking fund I. Increases by revenue earned on the investments. II. Is not affected by revenue earned on the investments. III. Decreases when the investments are purchased. I only. I and III. II and III. III only.

answer: a Businesses occasionally accumulate a fund of cash and/or investments for a specific purpose, such as the retirement of bonds in this problem. These funds are referred to as "sinking funds." The sinking fund is increased when periodic additions are made to the fund and when revenue is earned on the investments held in the fund. When cash is used to purchase investments, the components of the fund change (i.e., cash is invested and replaced by bonds or other securities), but the total fund balance is not affected.

Clarion had the following investments in its portfolio that were purchased during year 2. Investment Classification Cost Fair Value CS Company X $100,000 $121,000 Bond Available‐for‐sale $ 96,000 $101,000 Bond Held‐to‐maturity $ 64,000 $ 63,000 On December 31, year 2, the amortized cost of Bond Y was $97,000, and the amortized cost of Bond Z was $63,500. Clarion "does not elect the fair value option" for reporting financial assets. What amount should Clarion record as an unrealized gain in its year 2 income statement? $21,000 $25,000 $26,000 $0

answer: a If Clarion does not elect the fair value option for valuing its financial assets, the rules of ASC Topic 320 apply. Both the Company's stock trading security investment as well as the Company Y bond available‐for‐sale security investment would be reported at fair value. However, only the $21,000 unrealized gain associated with the Company's stock trading security investment would be reported in earnings of the period. If Clarion does not elect the fair value option for valuing its financial assets, the bond investments (bond available for sale & bond held to maturity) would be reported at amortized cost Trading securities gain should be on I/S but the bond gain in OCI

On January 2, year 1, Smith purchased the net assets of Jones' Cleaning, a sole proprietorship, for $350,000, and commenced operations of Spiffy Cleaning, a sole proprietorship. The assets had a carrying amount of $375,000 and a market value of $360,000. In Spiffy's cash‐basis financial statements for the year ended December 31, year 1, Spiffy reported revenues in excess of expenses of $60,000. Smith's drawings during year 1 were $20,000. In Spiffy's financial statements, what amount should be reported as Capital‐Smith? a) $390,000 b) $400,000 c) $410,000 d) $415,000

answer: a The ending balance in Smith's capital account on either the accrual or cash basis is computed as follows: Beginning capital + Investments + Income - Drawings = Ending capital Smith's beginning capital balance is measured as the cost of the assets purchased to establish the business ($350,000). The previously ecorded value ($375,000) and estimated market value ($360,000) are irrelevant and do not affect beginning capital. No additional investments were made; cash basis income was $60,000 and drawings were $20,000. Therefore, the ending capital balance is $390,000 ($350,000 + $60,000 − $20,000 "Statement of Owner's Equity", or "Statement of Changes in Owner's Equity", summarizes the items affecting the capital account of a sole proprietorship business. A sole proprietorship's capital is affected by four items: owner's contributions, owner's withdrawals, income, and expenses. Capital is increased by owner contributions and income, and decreased by withdrawals and expenses. The Statement of Owner's Equity, which is prepared for the sole proprietorship type of business, shows the movement in capital as a result of those four elements.

Parker Corporation prepares its financial statements in accordance with IFRS. Parker uses the revaluation model for reporting plant, property, and equipment. Parker paid $400,000 for equipment on January 5, year 1. The equipment is valued at $410,000 on December 31, year 1. The $10,000 gain should be included in a) A revaluation surplus account in other comprehensive income. b) Gain from revaluation on the income statement. c) Income for the period. d) An extraordinary gain on the income statement.

answer: a When the revaluation method is used for reporting plant, property, and equipment under IFRS, any gain or loss is recorded in a revaluation surplus account which is classified as other comprehensive income. (key word here is :Revaluation Method) Other comprehensive income is those revenues, expenses, gains, and losses under both GAAP and IFRS that are excluded from net income on the income statement. This means that they are instead listed after net income on the income statement. Revenues, expenses, gains and losses appear in other comprehensive income when they have not yet been realized. Something has been realized when the underlying transaction has been completed, such as when an investment is sold. Thus, if your company has invested in bonds, and the value of those bonds changes, you recognize the difference as a gain or loss in other comprehensive income. Once you sell the bonds, you have then realized the gain or loss associated with the bonds, and can then shift the gain or loss out of other comprehensive income and into a line item higher in the income statement, so that it is a part of net income. Examples of items that may be classified in other comprehensive income are: Unrealized holding gains or losses on investments that are classified as available for sale Foreign currency translation gains or losses Pension plan gains or losses Pension prior service costs or credits

Bake Co.'s trial balance included the following at December 31, year 1: Accounts payable $80,000 Bonds payable, due year 2 300,000 Discount on bonds payable 15,000 Deferred income tax liability 25,000 The deferred income tax liability is not related to an asset for financial accounting purposes and is expected to reverse in year 2. What amount should be included in the current liability section of Bake's December 31, year 2 balance sheet? (Question is only asking about current-liability) a) $365,000 b) $390,000 c) $395,000 d) $420,000

answer: a because all deferred taxes are classified as noncurrent regardless. <-이게 핵심이네. "All deferred taxes are non-current". The accounts payable and the bond payable are classified as current liabilities because they are due within the next 12 months. The bond payable is valued at its carrying value of $285,000 ($300,000 − $15,000 discount). This answer is correct because the amount of current liabilities is equal to $365,000 ($80,000 + $285,000).

In determining whether to accrue employees' compensation for future absences, one of the conditions that must be met is that the employer has an obligation to make payment even if an employee terminates. This an example of a(n) a) Vested right. b) Accumulated right. c) Contingent right. d) Estimable right.

answer: a because vested rights are those rights which are not contingent (contingent= chance to change depend on different situation) on an employee's future service; the employer has an obligation to make payment even if an employee terminates employment (ASC Topic 710). vested 뜻은 - secured in the possession of or assigned to a person. *Accumulated rights are rights which have been earned in the past and which carry over into future periods, but which are lost if an employee terminates employment. *Contingent rights are totally dependent on future actions and are not the result of past service. (Contingent 뜻이란, subject to change depend on the situation이라는것) *Estimable rights do not exist with regard to employee compensation.

The following items relate to the preparation of a statement of cash flows: Year2 Year 1 AP $265,000 $220,000 Divid payable 35,000 0 Note payable 250,000 0 Common stock 600,000 450,000 Retained earnings 280,000 165,000 Year 2 Net sales $3,200,000 CGS (2,500,000) Expenses (500,000) Net income $ 200,000 All accounts payable relate to trade merchandise. Accounts payable are recorded net and always are paid to take all of the discount allowed. The proceeds from the note payable were used to finance a new store building. Capital stock was sold to provide additional working capital. Under "financing activities", cash inflows during year 2 totaled: (notice, the question is specifically asking about cash flow's "Financing activity") a) $400,000 b)$250,000 c)$150,000 d) $ 70,000

answer: answer A Cash inflows from financing would include the long‐term note of $250,000 and the issuance of common stock of $150,000 ($600,000 − $450,000) for a total of $400,000. Must know that there are 3 types of activities in cash flow and financing is just one of the type. Financing activity includes non-current liability + anything that affects owner's equity.

In April year 1, Delta Hospital purchased medicines from Field Pharmaceutical Co. at a cost of $5,000. However, Field notified Delta that the invoice was being canceled and that the medicines were being donated to Delta. Delta should record this donation of medicines as a) A memorandum entry only. b) A $5,000 credit to nonoperating expenses. c) A $5,000 credit to operating expenses. d) Other operating revenue of $5,000.

answer: d Contributions received shall be recognized as revenues in the period received and shall be measured at "fair value". Thus, this answer is correct since the contribution of medicine is recognized as revenues.

Hines Company leased a new machine from Ashwood Company on December 31, year 1, under a lease with the following pertinent information: Lease term 8 years Annual rental payable at the beginning of each lease year $ 50,000 Useful life of the machine 10 years Present value of the 8 lease payments at 12/31/Y1 $258,000 The machine reverts to Ashwood at lease expiration date and has a fair value of $280,000 at the inception of the lease. Hines uses the straight‐line method of depreciation. For the year ended December 31, year 2, how much depreciation (amortization) should Hines record for the capitalized leased machine? a) $35,000 b) $32,250 c)$28,000 d) $25,800

answer: b Per ASC Topic 840, the lessee records the asset at the lower of (1) the present value of the minimum lease payments or (2) the fair market value of the leased asset. In this case, the present value ($258,000) is less than the fair market value ($280,000); therefore, $258,000 is capitalized. Since the machine reverts to the lessor at the end of the lease, the lessee should depreciate it over the lease term (8 years) even though it is less than the useful life (10 years). Depreciation expense is $32,250 ($258,000/8 years).

On January 1, year 2, Pall Corp. granted stock options to key employees for the purchase of 40,000 shares of the company's common stock at $25 per share. The options are intended to compensate employees for the next 2 years. The options are exercisable within a 4‐year period beginning January 1, year 3, by grantees still in the employment of the company. The market price of Pall's common stock was $25 per share at the date of grant. The value of each option under the Black‐Scholes model was $8.00. No stock options were terminated during the year. What amount should Pall charge to compensation expense for the year ended December 31, year 2? a) $320,000 b) $160,000 c) $ 80,000 d) $0

answer: b Employee compensation expense as the result of a stock option plan is calculated as the **value of each option at the date of grant times the number of option shares.** Option shares x Value of each option =Total compensation expense 40,000 × $8.00 =$320,000 핵심단어 *(stock option plan is calculated as the "Value of each option at the date of grant times the number of option shares.) This total compensation expense must be recognized over the period for which the option plan represents compensation. These options were intended to compensate employees for the years year 2 and year 3. Therefore, year 2 compensation expense is $160,000 ($320,000 ÷ 2).

A derivative financial instrument is best described as a) Evidence of an ownership interest in an entity such as shares of common stock. b) A contract that has its settlement value tied to an underlying notional amount. c) A contract that conveys to a second entity a right to receive cash from a first entity. d) A contract that conveys to a second entity a right to future collections on accounts receivable

answer: b A derivative is a security with a price that is dependent upon or derived from one or more underlying assets. The derivative itself is a contract between two or more parties based upon the asset or assets. Its value is determined by fluctuations in the underlying asset. The most common underlying assets include stocks, bonds, commodities, currencies, interest rates and market indexes.

The following balances are included in the subsidiary records of Burwood Village's Parks and Recreation Department at March 31, year 2: Appropriations—supplies $7,500 Expenditures—supplies 4,500 Encumbrances—supply orders 750 How much does the Department have available for additional purchases of supplies? a) $0 b) $2,250 c) $3,000 d) $6,750

answer: b GASB 1 states that appropriations constitute maximum expenditure authorizations during the fiscal year and cannot legally be exceeded unless subsequently amended by the legislative body. An encumbrance reduces appropriation authority and is formally recorded in the accounting records. Appropriations $7,500 Less: Encumbrances $ 750 Expenditures $4,500 5,250 Unencumbered balance $2,250 The unencumbered balance is the amount of resources that can still be obligated or expended without exceeding the legal or authorized limit. You book the appropriation with the budget, as mentioned, as to book the amount of monies the entity is allowed to expend for the fiscal year. An encumbrance is a control account used to book a purchase, usually in the form of a purchase order. I.e. the governmental entity has issued a purchase order to purchase the good/service but hasn't actually expended the money for the purchase yet. When the entity pays for the good/service the encumbrance is negated and the expenditure is booked. Appropriation" means an authorization by the General Assembly to make expenditures and incur liabilities for specific purposes. "Encumbrances" means obligations in the form of purchase orders or contracts which are to be met from an appropriation and for which a part of the appropriation is reserved. Expenditures" means amounts paid for all purposes, including expenses, provisions for retirement of debt and capital outlay.

Bond Company purchased a machine on January 1, year 1, for $3,000,000. At the date of acquisition, the machine had an estimated useful life of 6 years with no salvage. The machine is being depreciated on a straight‐line basis. On January 1, year 4, Bond determined, as a result of additional information, that the machine had an estimated useful life of 8 years from the date of acquisition with no salvage. An accounting change was made in year 4 to reflect this additional information. What is the amount of depreciation expense on this machine that should be charged in Bond's income statement for the year ended December 31, year 4? a) $100,000 b) $300,000 c) $375,000 d) $500,000

answer: b Per ASC 250‐10‐45‐17, changes in accounting estimates are reflected prospectively (i.e., no retroactive adjustment). As computed below, the depreciation base beginning in year 4 is $1,500,000. The depreciation taken in year 1‐year 3 was $1,500,000 ($3,000,000/6 years × 3 years). The remaining depreciation base of $1,500,000 ($3,000,000 - $1,500,000) is divided by 5 years (8 years - 3 years since acquisition), resulting in $300,000 per year depreciation. $3,000,000 Cost (no salvage) $1,500,000 Year 1, year 2, year 3 depreciation (3,000,000/6yr x 3 yr) $1,500,000 Remained to be depreciated for year 4‐year 8 $1,500,000/5 years = $300,000. (총 8년에서 3년어치는 이미 depreciation 했으니, 5 년이 남은것으로 판단하기)

Tang City received land from a donor who stipulated that the land must remain intact (intact = untouched), but any income generated from the property may be used for general government services. In which fund should Tang City record the donated land? a) Special revenue. b) Permanent. c) Private‐purpose trust. d) Agency.

answer: b Permanent funds are used to report resources that are legally restricted to the extent that only earnings, and not principal, may be used to support governmental programs.

The following information pertains to Gali Co.'s defined benefit pension plan for year 2: Fair value of plan assets, beginning of year $350,000 Fair value of plan assets, end of year 525,000 Employer contributions 110,000 Benefits paid 85,000 In computing pension expense, what amount should Gali use as "actual return on plan" assets? a) $ 65,000 b) $150,000 c) $175,000 d) $260,000

answer: b The "actual return on plan assets" represents the return earned on "accumulated" pension fund assets. It is calculated as the difference in the fair value of plan assets at the beginning and the end of the period adjusted for contributions made to the plan and benefit payments made by the plan during the period. This calculation separates out the change in the FV of plan assets that is due to interest and dividends earned. The formula for determining the actual return is as follows: Beginning plan assets + Actual return on plan (aka: accumulated pension) + employer's contributions = Ending plan asset. $350,000 + x + $110,000 -$85,000= $525,000 X를 구하면 $150,000 and this is "actual return on plan"

On January 1, year 1, Nobb Corp. signed a 12‐year lease for warehouse space. Nobb has an option to renew the lease for an additional 8‐year period on or before January 1, year 5. During January year 3, Nobb made substantial improvements to the warehouse. The cost of these improvements was $540,000, with an estimated useful life of 15 years. At December 31, year 3, Nobb intended to exercise the renewal option. Nobb has taken a full year's amortization on this leasehold. In Nobb's December 31, year 3 balance sheet, the carrying amount of this leasehold improvement should be a) $486,000 b) $504,000 c) $510,000 d) $513,000

answer: b The cost of the leasehold improvements ($540,000) should be amortized over the remaining life of the lease, or over the useful life of the improvements, whichever is shorter. <-핵심 단어. Leasehold improvement should be amortize over remaining life of Lease or Useful life whichever is "SHORTER". 그리고 제일 중요한건, it is allowed to amortize (capitalized) because it means capital lease requirement. 4개중에 하나의 조건이 맞는다. To be considered a capital lease, a lease must satisfy any one of the four criteria. (1) The lease has bargain purchase option (2) have transfer transfer title. (3) The lease term is 75% or more of the useful life (4) the PV of the lease payments is 90% or more of the FV of the asset *In this case, Nobb company has right to bargain purchase option. (remember, 만약에 4가지중에 단 하나라도 해당이 안되면, 그냥 operating lease다) **The lease shall be accounted for as an operating lease when none of the four requirements applicable to both lessees and lessors is met. Even if one or more was met, the lease would still be classified as an operating lease if the payments are not reasonably predictable ****(ex:The Morn Company leased equipment to the Lizard Company on May 1, year 1. At that time the collectibility of the minimum lease payments was not reasonably predictable. )<-- this is when it becomes operating lease The remaining life of the lease should include periods covered by a renewal option if it is probable that the option will be exercised. In this case, the remaining life of the lease is 18 years (12 years of original lease + 8 years in option period − 2 years past), and the useful life of the improvements is 15 years. Therefore, amortization is based on a 15‐year life ($540,000 ÷ 15 = $36,000). The 12/31/Y3 carrying amount is $504,000 ($540,000 − $36,000).

On August 1, year 1, Bamco Corporation purchased a new machine on a deferred payment basis. A down payment of $1,000 was made and 4 monthly installments of $2,500 each are to be made beginning on September 1, year 1. The cash equivalent price of the machine was $9,500. Bamco incurred and paid installation costs amounting to $300. The amount to be capitalized as the cost of the machine is a) $ 9,500 b) $ 9,800 c) $11,000 d) $11,300

answer: b The total of the payments made by Bamco for the machine was $11,000 (4 payments of $2,500 + $1,000 down payment). However, the cash equivalent price of the machine was $9,500. The difference of $1,500 ($11,000 - $9,500) is considered interest and is not capitalized because it is ready for use when acquired. In addition, the $300 installation cost is capitalized, as all costs required to get goods in their operating condition and location should be capitalized. Thus the total cost of the machine is $9,800 ($9,500 + $300). (핵심: you don't capitalize interest payment)

Abbot Co. is being sued for illness caused to local residents as a result of negligence on the company's part in permitting the local residents to be exposed to highly toxic chemicals from its plant. Abbot's lawyer states that it is probable that Abbot will lose the suit and be found liable for a judgment costing Abbot anywhere from $500,000 to $2,500,000. However, the lawyer states that the most probable cost is $1,000,000. As a result of the above facts, Abbot should accrue a) A loss contingency of $500,000 and disclose an additional contingency of up to $2,000,000. b) A loss contingency of $1,000,000 and disclose an additional contingency of up to $1,500,000. c) A loss contingency of $1,000,000 but not disclose any additional contingency. d) No loss contingency but disclose a contingency of $500,000 to $2,500,000.

answer: b This answer is correct because ASC Topic 450 requires that estimated losses from loss contingencies be accrued if the contingency is probable (as opposed to reasonably possible or remote) and the amount of loss can be reasonably estimated. ASC Topic 450 states that a range of loss rather than an estimate of a single loss amount is a basis for recording a probable loss. Furthermore, the loss should be recorded at the best estimate within the range. If there is no best estimate, one should use the minimum. The excess of the recorded amount within the range should be disclosed. Accordingly, a $1,000,000 loss contingency should be recorded in the accounts, because the lawyer stated that the most probable loss was $1,000,000. Additionally, $1,500,000 should be disclosed as a possible contingency because the range of the possible loss was up to $2,500,000.

The controller of Peabody, Inc. has been asked to present an analysis of accounts receivable collections at the upcoming staff meeting. The following information is used: 12/31, year 2 12/31, year 1 A/R $100,000 $130,000 Allowance, doubtful (20,000) (40,000) Sales 400,000 200,000 Cost of goods sold 350,000 170,000 What is the receivables turnover ratio as of December 31, year 2? a) 5 b)4.7 c)3.5 d) 0.6

answer: b This answer is correct. Accounts receivables turnover is calculated as net credit sales divided by average accounts receivable. Average accounts receivable is calculated as (beginning of year A/R plus end of year A/R) divided by 2. If an allowance for doubtful accounts is used, the net realizable value of accounts receivable should be used to calculate average accounts receivable. Therefore, the accounts receivable turnover is 4.7. $400,000/(($80,000 + $90,000)/2) = 4.7 times

In a statement of cash flows, proceeds from issuing equity instruments should be classified as cash inflows from a) Lending activities. b) Operating activities. c) Investing activities. d) Financing activities.

answer: d Per ASC Topic 230, financing activities include obtaining resources from owners and providing them with a return on, and a return of, their investment. Proceeds from issuing equity instruments are specifically identified as cash inflows from financing activities (ex: Issuing 500 shares of common stock is inflow, income to financing activity)

On January 1, year 1, Glen Co. leased a building to Dix Corp. for a 10‐year term at an annual rental of $50,000. At inception of the lease, Glen received the first 2 years' rent of $100,000 and a security deposit of $100,000. This deposit will not be returned to Dix upon expiration of the lease but will be applied to payment of rent for the last 2 years of the lease. What portion of the $200,000 should be shown as a current and long‐term liability, respectively, in Glen's December 31, year 1 balance sheet? Current Liability Long‐term liability a) $ 0 $ 200,000 b) $ 50,000 $ 100,000 c) $ 100,000 $ 100,000 d) $ 100,000 $ 50,000

answer: b This answer is correct. At 1/1/Y1, Glen would record as a current liability unearned rent of $50,000, and as a long‐term liability unearned rent of $150,000. During year 1, the current portion of unearned rent was earned and would be recognized as revenue. At 12/31/Y1, the portion of the long‐term liability representing the second year's rent ($50,000) would be reclassified as current, leaving as a long‐term liability, the $100,000 representing the last 2 years' rent.

A company issued 10‐year term bonds at a discount in year 1. Bond issue costs were incurred at that time. The company uses the effective interest method to amortize bond issue costs. Reporting the bond issue costs as a deferred charge would result in a) More of a reduction in net income in year 2 than reporting the bond issue costs as a reduction of the related debt liability. b) The same reduction in net income in year 2 as reporting the bond issue costs as a reduction of the related debt liability. c) Less of a reduction in net income in year 2 than reporting the bond issue costs as a reduction of the related debt liability. d) No reduction in net income in year 2. .

answer: b When bonds are issued, the issuing company often incurs printing costs, legal fees, commissions, and other similar expenses. Per ASC 835‐30‐45‐3, bond issue costs are debited to a deferred charge account and amortized similarly to the bond discount. The amortization expense related to the bond issue costs is the same regardless of how the bond issue costs are reported on the balance sheet (i.e., as a deferred charge or as a reduction of the liability). what it is mean by "issuing bond at discount"? A bond that is issued for less than its par (or face) value, or a bond currently trading for less than its par value in the secondary market. A bond is considered a discount bond when it has a lower interest rate than the current market rate, and consequently is sold at a lower price.

Which statement is true with respect to push‐down accounting? a) IFRS permits the use of push‐down accounting. b) IFRS does not permit the use of push‐down accounting. c) SEC accounting does not permit the use of push‐down accounting. d) Both SEC accounting and IFRS permit the use of push‐down accounting.

answer: b because IFRS disallows the use of push‐down accounting. The push-down method of accounting is a way for a company to account for the controlling purchase of a subsidiary. When a company purchases another, the question arises as to how to value the subsidiary company's assets and liabilities. The push-down method addresses this question. Push down method is allowed by GAAP but disallowed by IFRS. Definition: Pushdown accounting is the process of using the acquiring entity's basis of accounting to prepare the financial statements of the acquired entity. This means that the assets and liabilities of the acquiree are updated to their fair values as of the acquisition date. These changes appear in the financial statements of the newly-acquired entity. Pushdown accounting is not required for entities that are not registrants with the Securities and Exchange Commission (i.e., public entities).

A short‐term marketable debt security was purchased on September 1, year 1, between interest dates. The next interest payment date was February 1, year 2. On the balance sheet at December 31, year 1, the debt security should be carried at a) Market value plus the accrued interest paid. b) Market value. c) Cost plus the accrued interest paid. d) Cost.

answer: b because a short‐term marketable debt security would be carried in a trading portfolio. Securities in trading portfolios are carried at market value. When an investor holds a group of various financial instruments, it is called a portfolio. The portfolio may contain assets such as stocks and shares, funds, currencies, derivatives and bonds. The investor may be an individual trader or a financial institution such as a bank, fund or venture capital company.

For IFRS reporting purposes, currencies are defined as a) International and presentation. b) Foreign, functional, and presentation. c) Domestic and international. d) Functional, international, and presentation.

answer: b because for IFRS reporting purposes, currencies are defined as foreign, functional, and presentation. • Functional Currency • Definition: Under international financial reporting standards, a functional currency is the currency used in the primary economic environment where an entity operates. This is the environment in which an entity primarily generates and expends cash. You should consider the following primary factors in determining an entity's functional currency: The same Standard defines presentation currency as "the currency in which the financial statements are presented". The functional currency is determined by looking at a number of relevant factors. This currency should be the currency in which an entity usually generates and spends cash. Functional currency should be the one in which the business transactions of an entity are normally denominated. All of the transactions which are not in the functional currency are treated as foreign transactions. Following five factors need to be considered when determining a functional currency. Functional currency is the currency: • That mainly affects the prices at which the goods or services are sold • Of the country whose regulations, market conditions and competitive forces mainly affect the pricing policy of the entity • That influences the costs and expenses of the entity • In which the funds are usually generated • In which receipts from operating activities are retained

Pahn, a nongovernmental not‐for‐profit organization, received an unconditional pledge of $50,000. The donor stipulated that the pledge must be used in the next fiscal year. Pahn received and spent the $50,000 in the next year. For the current fiscal year, what element of Pahn's statement of financial position will increase as a result of the unconditional pledge? a) Cash and cash equivalents. b) Pledge receivables. c)Unrestricted support. d) Deferred contributions.

answer: b cash and cash equivalents does not increase until the cash is received. Pledge receivable is when donor "promise" to non profit or gov that they will make contribution in the future, so non profit is treating this amount as "Pledge receivables" (앞으로 받을것으로. 무슨 돈을 받는다고? donor가 약속하고 맹세한 그돈!) I think the journal entry will be Dr. Pledge receivable Cr. Temporarily restricted net asset. We are yet to receive the money but we can recognize it as temporary restricted contribution. Once the cash is received (Dr. Cash, Cr. Pledge receivable)

Gordon Ltd., a 100% owned British subsidiary of a US parent company, reports its financial statements in local currency, the British pound. A local newspaper published the following US exchange rates to the British pound at year‐end: (U.S. parent company owes subsidiary in British and subsidiary need to follow U.S Exchange rate) Current Rate $1.50 Historical rate (acquisition) 1.70 Average rate 1.55 Inventory (FIFO) 1.60 Which currency ratio should Gordon use to convert its income statement to US dollars at yearend? a) 1.50 b) 1.55 c)1.60 d) 1.70

answer: b the current rate method requires income statement items (revenues and expenses) to be translated using the weighted‐average rate, $1.55. Per ASC Topic 830, if the functional currency equals the local currency, (meaning, functional currency refers to the main currency used by a business or unit of a business, generally the currency that parent company uses), company'the current rate method is used. the current rate method requires income statement items. If British subsidiary also followed US dollar (in this case, functional currency that its parents use), then British subsidiary can use current rate. However, since, British sub used Pound instead of dollar, we need to use "average rate" to convert If the foreign affiliate's financial statements are in the local currency and this is also the functional currency then the Current-Rate Method must be used. (이뜻은, 일본에 있는 subsidiary가 본인나라의 functional currency를 쓰고있다는것. 즉, 일본에서 일어나는 세일이 미국보다 더 영향력있고, 더 많고, expense또한 일본에서 더 많이 쓰이고, 마켓이 일본이 더 클때, 일본이 yen becomes functional currency. 보통은, functional currency로 Financial statement를 만들지만, parent가 미국회사다 보니 나중에 어쩔수없이, yen에서 dollar로 바꿔야한다. 이런경우엔, we use Current Rate Method) If the foreign affiliates' financial statements are in the local currency but the functional currency is dollars then the Temporal-Rate Method must be used. 이뜻은, local currency가 functional currency로 감히 불리지 않을만큼, 마켓의 영향력이 없다는뜻. (Temporal Rate Method는 조심해야하는게 이안에서도 여러가지 조건이 있다. Monetary asset is translated using current rate, income statement is translated using weighted average, and nonmetary asset is translated using historical cost. )

Under the lower of cost or market method, the replacement cost of an inventory item would be used as the designated market value a) When it is below the net realizable value less the normal profit margin. b) When it is below the net realizable value and above the net realizable value less the normal profit margin. c) When it is above the net realizable value. d) Regardless of net realizable value.

answer: b This answer is correct. ASC Topic 330 "market" is equal to current replacement cost, subject to the following constraints: (1) market cannot exceed the net realizable value (NRV) of an item, and (2) market cannot be below NRV less the normal profit margin

Northstar Co. acquired a registered trademark for $600,000. The trademark has a remaining legal life of five years, but can be renewed every 10 years for a nominal fee. Northstar expects to renew the trademark indefinitely. What amount of amortization expense should Northstar record for the trademark in the current year? a) $0 b) $15,000 c) $40,000 d) $120,000

answer: b. For US GAAP purposes, assets with indefinite lives are not amortized but tested for impairment instead. Because the trademark is expected to be renewed indefinitely, it is treated as an asset with an indefinite life and is not amortized.)

Stam Co. incurred the following research and development project costs during the current year: Equipment purchased for "current and future" projects $100,000 Equipment purchased for "current" projects only $200,000 Research and development "salaries" for current projects 400,000 Legal fees to obtain patent 50,000 Material and labor costs for prototype product 600,000 The equipment has a five‐year useful life and is depreciated using the straight‐line method. What amount should Stam recognize as research and development expense at year‐end? *question is only asking about current portion expense* a) $ 450,000 b) $1,000,000 c) $1,220,000 d) $1,350,000

answer: c *핵심: Include only "current" portion. Equipment purchased for "current and future" projects $100,000 (Here, $100,00 / 5yr = $20,000 applies to current year) For this question, you must stick to the Matching Principle. Since the equipment is deemed to have "future use," the equipment will be capitalized and depreciation will be matched to revenue in future periods. 밑에 자세한 설명 보기. Equipment purchased for "current" projects only $200,000 (Here, all $200,00 applies to current year) Research and development "salaries" for current projects 400,000 (Since this doesn't have indication if this is for future or current, let's just assume that $400,000 refers to all current) Legal fees to obtain patent 50,000 (Legal fee is not R&D expense. it capitalize ) Material and labor costs for prototype product 600,000 (Since this doesn't have indication if this is for future or current, let's just assume that $600,000 refers to all current) Therefore, the total R&D at year‐end is equal to $1,220,000 ($20,000 + $200,000 + $400,000 + $600,000). *핵심: Legal fees to obtain patent is not R&D capital . R&D is expense in the current year. To determine CY's R&D, will only include costs is defined as "Current" and exclude future project portion. Legal feel to obtain patent is always capitalize cost rather than expense. Also, for "Material and labor cost for prototype product, since it doesn't have any indication whether the cost is for current or future, we will just assume that it is current project. ---------------------------------------------- Be careful with the term "R&D equipment which is used only for the current project". Vs "R&D equipment which is used both for current & future project". It is treated different. If Equipment is purchased for R&D purpose, and is planned to used only for the current project, company have to expense the cost of equipment immediately ,rather than capitalizing over the life of the assets. (So, don't depreciate the equipment by the estimated life of the asset. Just recognize the expense in the same current year) However, if equipment is intended to use for both current project and some future project, it means that the equipment will provide more benefit throughout the year so we need to only deduct the depreciation amount throughout the life of the assets. Different Question: On Jan 1 of the current yr Jambon purchased an equipment for use in developing a new product. Jambon uses straight-line depreciation. The equipment could provide benefits for over 10 yr period. However the new product development is expected to take 5 yrs & the equipment can be used for only this project. Jambons current year expense equals:- a)The total cost of the equipment b)1/5 th of cost of the equipment c)1/10 th of the cost of the equipment d)Zero Answer: a The reason Answer is A is that the equipment can only be used for this project and there is no future benefit. Therefore, you don't take any depreciation expense and expense the full cost of the equipment. Since the machine will be only used for this current project, the cost of the machine is treated as research and development costs. Remember, we will always expense research and development costs instead of capitalizing them.

Unrealized gains on investments which are permanently restricted as to use by donors are reported by a private, nonprofit hospital on the a) Statement of operations. b) Statement of cash flows. c) Statement of changes in net assets. d) Statement of operations and statement of cash flows.

answer: c According to the AICPA Audit and Accounting Guide, Health Care Organizations, investment returns not restricted by donors are reported on the statement of operations. The statement of operations explains the change in the hospital's unrestricted net assets for a period. Consequently, investment returns which are permanently restricted would not be reported on the statement of operations. Investment returns which are realized in cash are reported on the statement of cash flows. However, investment returns which are not realized in cash are not reported on the statement of cash flows. Investment returns, whether realized or unrealized, which are restricted by donors are reported on the statement of changes in net assets. Unrealized gains on investments which are permanently restricted represent investment returns which would be reported as an increase in permanently restricted net assets on the statement of changes in net assets. 핵심: investment returns not restricted by donors are reported on the statement of operations. The statement of operations explains the change in the hospital's unrestricted net assets for a period. However, if investment returns which are permanently restricted, would be reported on the statement of changes in net assets. Investment returns which are realized in cash are reported on the statement of cash flows. However, investment returns which are not realized in cash are not reported on the statement of cash flows.

Tone Company is the defendant in a lawsuit filed by Witt in year 1 disputing the validity of a copyright held by Tone. At December 31, year 1, Tone determined that Witt would probably be successful against Tone for an estimated amount of $400,000. Appropriately, a $400,000 loss was accrued by a charge to income for the year ended December 31, year 1. On December 15, year 2, Tone and Witt agreed to a settlement providing for cash payment of $250,000 by Tone to Witt, and transfer of Tone's copyright to Witt. The carrying amount of the copyright on Tone's accounting records was $60,000 at December 15, year 2. What would be the effect of the settlement on Tone's income before income tax in year 2? a) No effect. b) $ 60,000 decrease. c) $ 90,000 increase. d) $150,000 increase.

answer: c At 12/31/Y1, the contingent liability from the lawsuit met ASC Topic 450's criteria for accrual (probable and reasonably estimable), so a loss and liability of $400,000 was recognized. In year 2, the lawsuit was settled and the actual loss was $310,000 ($60,000 copyright transfer and $250,000 cash payment). This is a change in estimate which should be accounted for in the period of change per ASC Topic 250. Therefore the $90,000 difference will be reflected in year 2 income. The journal entry on 12/15/Y2 to record the settlement would be Lawsuit liability 400,000 Gain from settlement 90,000 Cash 250,000 Copyright 60,000

The lessee's (물건을 빌리는자) balance sheet liability for a capital lease would be periodically reduced by the total a) Minimum lease payment plus the amortization of the related asset. b) Minimum lease payment less the amortization of the related asset. c) Minimum lease payment less the portion of the minimum lease payment allocable to interest. d) Minimum lease payment.

answer: c During the lease term, each minimum lease payment consists of both interest and reduction of the lease obligation. Lease amortization produces a constant periodic rate of interest on the remaining balance of the obligation, known as the "effective interest" method.

Park City uses encumbrance accounting and formally integrates its budget into the general fund's accounting records. For the year ending July 31, year 1, the following budget was adopted: Estimated revenues $30,000,000 Appropriations 27,000,000 Estimated transfer to debt service fund 900,000 When Park's budget is adopted and recorded, Park's budgetary fund balance would be a) Credited for $3,000,000. b) Debited for $3,000,000. c) Credited for $2,100,000. d) Debited for $2,100,000.

answer: c In certain governmental funds, such as the general fund, an annual budget is recorded in the accounts. The entry to record Park City's fiscal year 1 budget is as follows: Est.Revenue 30,000,000 Est. Other Financing Uses 900,000 Appropriations 27,000,000 Budgetary Fund Balance 2,100,000 When Park's budget is adopted and recorded, Budgetary Fund Balance is credited for $2,100,000. In law and government, appropriation (from Latin appropriare, "to make one's own", later "to set aside") is the act of setting apart something for its application to a particular usage, to the exclusion of all other uses.

According to ASC Topic 820, which level has the lowest priority for valuation purposes? a) Level 1 b) Level 2 c) Level 3 d) Level 4

answer: c Level 3 is the lowest priority for valuation purposes and includes unobservable inputs, such as an entity's own assumptions, expected cash flows, or a firm's forecasts. A Financial Accounting Standards Board (FASB) Statement that requires all publicly-traded companies in the U.S. to classify their assets based on the certainty with which fair values can be calculated. This statement created three asset categories: Level 1, Level 2 and Level 3. Level 1 assets are the easiest to value accurately based on standard market-based prices and Level 3 are the most difficult. FASB 157 was passed to help investors and regulators understand how accurate a given company's asset estimates truly were. Level 2 assets are the middle classification based on how reliably their fair market values can be calculated. Level 1 assets are the easiest (such as listed stocks, bonds), while Level 3 assets can only be valued based on internal models or "guesstimates" and have no observable market prices.

On November 15, year 2, Celt, Inc., a US company, ordered merchandise FOB shipping point from a foreign company for 200,000 LCUs. The merchandise was shipped and invoiced to Celt on December 10, year 2. Celt paid the invoice on January 10, year 3. The spot rates for LCUs on the respective dates are as follows: November 15, year 2 $ .4955 December 10, year 2 .4875 December 31, year 2 .4675 January 10, year 3 .4475 In Celt's December 31, year 2 income statement, the foreign exchange transaction gain is a) $9,600 b) $8,000 c) $4,000 d) $1,600

answer: c No journal entry is prepared on 11/15/Y2 when the goods are ordered, so the spot rate for LCUs on that date ($.4955) is not relevant to the solution of this problem. On 12/10/Y2, Celt would record the purchase and related accounts payable at $97,500 (200,000 LCUs × $.4875). A foreign exchange transaction gain (loss) is to be recognized if the spot rate on the settlement date (or any intervening balance sheet date) is different from the rate on the transaction date. At 12/31/Y2, the spot rate is $.4675, which means the account payable must be adjusted down to $93,500 (200,000 LCUs × $.4675), resulting in a gain of $4,000. A shortcut approach is to multiply the change in the exchange rate ($.4875 − $.4675 = $.02) by the payable balance in LCUs ($.02 × 200,000 LCUs = $4,000). *FOB Shipping 일 경우, once product is shipped, buyer need to book J/E for A/P using the current foreign exchange. 그리고, balance sheet date기준, 즉, 12/31 end of year의 foreign transaction이랑 비교한후, loss또는 gain이 있는지 보는것**

The following information relates to the year 3 activity of the defined benefit pension plan of Lindy Corp., a company whose stock is publicly traded: Service cost $150,000 Return on plan assets 40,000 Interest cost on pension benefit obligation 82,000 Amortization of actuarial loss 15,000 Amortization of unrecognized net obligation 35,000 Lindy's year 3 pension cost is a) $322,000 b)$287,000 c)$242,000 d)$158,000

answer: c Pension cost is a net amount calculated by adding or subtracting the factors as follows: Service cost $150,000 Return on plan assets (40,000) Interest cost on pension benefit obligation 82,000 Amortization of actuarial loss 15,000 Amortization of unrecognized net obligation 35,000 Pension cost $242,000 Service cost and interest on the projected benefit obligation always increase pension expense; return on plan assets almost always decreases pension expense. Amortization of actuarial loss and amortization of a net obligation both increase pension expense The term return on plan assets refers to the dividends, interest, and capital gains generated by assets held in a company's pension fund. Accounting rules require companies to differentiate between the expected and actual return on their plan's assets. Funded Status = Plan Assets - Projected Benefit Obligation (PBO) if plan asset < projected benefit obligation, it is "underfunded" therefore, must pay if plan asset > projected benefit obligation, it is "overfunded"

For the year ended December 31, year 3, Colt Corp. has a loss carryforward of $180,000 available to offset future taxable income. At December 31, year 3, all available evidence concerning future profitability is positive. Assume an income tax rate of 30%. What amount of the tax benefit should be reported in Colt's year 3 income statement? a) $180,000 b) $126,000 c) $ 54,000 d) $0

answer: c Per ASC Topic 740 <- Fin 48, a deferred tax liability or asset is recognized for all temporary differences and operating loss and tax credit carryforwards. When a deferred tax asset is recorded, the entity must consider the need for a valuation allowance. In assessing the need for a valuation allowance, provisions in the tax law that may limit utilization of an operating loss or tax credit carryforward are applied in determining whether it is more likely than not that some portion of the deferred tax asset will not be realized by reduction of taxable income or taxes payable during the carryforward period. For Colt Corp., it is not more likely than not that a part or all of the benefit will not be realized in future years. Therefore, the tax benefit of the loss carryforward to be recorded in Colt's income statement is $54,000 ($180,000 × 30%). Loss carryforward refers to an accounting technique that applies the current year's net operating losses to future years' profits to reduce tax liability and track profits accurately. FIN 48 (mostly codified at ASC 740-10) is an official interpretation of United States accounting rules that requires businesses to analyze and disclose income tax risks. It was effective in 2007 for publicly traded entities, and is now effective for all entities adhering to US GAAP. A business may recognize an income tax benefit only if it is more likely than not that the benefit will be sustained. The amount of benefit recognized is based on relative probable outcomes.

Jerry Corp., a company whose stock is publicly traded, provides a noncontributory defined benefit pension plan for its employees. The company's actuary has provided the following information for the year ended December 31, year 5: Projected benefit obligation- $400,000 Accumulated benefit obligation- 350,000 Plan assets (fair value)- 410,000 Service cost- 120,000 Interest on projected benefit obligation- 12,000 Amortization of unrecognized prior service cost- 30,000 Expected and actual return on plan assets- 41,000 The market‐related asset value equals the fair value of plan assets. Prior contributions to the defined benefit pension plan equaled the amount of net periodic pension cost accrued for the previous year‐end. No contributions have been made for year 5 pension cost. In its December 31, year 5 balance sheet, Jerry should report a pension asset of a) $203,000 b) $121,000 c) $ 10,000 d) $0

answer: c Since prior contributions equaled the amount of net pension cost previously accrued, there is no pension asset/liability at 1/1/Y5. Year 5 pension expense is $121,000, as computed below. Service cost $120,000 Interest on Proj benefit oblig. 12,000 Amort. of unrec. PSC 30,000 Return on plan assets (41,000) Pension cost $121,000 Although no year 5 contributions have been made, there is no liability at 12/31/Y5 because the plan assets exceed the Projected benefit obligation. Therefore, a pension plan asset of $10,000 is recognized. Plan asset $410,000 -projected benefit obligation $400,000 = $10,000

Brockton City serves as collecting agency for the local independent school district and for a local water district. For this purpose, Brockton has created a single agency fund and charges the other entities a fee of 1% of the gross amounts collected. (The service fee is treated as general‐fund revenue.) During the latest fiscal year a gross amount of $268,000 was collected for the independent school district and $80,000 for the water district. As a consequence of the foregoing, Brockton's General Fund should a) Recognize receipts of $348,000. b) Recognize receipts of $344,520. c) Record revenue of $3,480. d) Record encumbrances of $344,520.

answer: c The Agency Fund collected $348,000 during the last fiscal year. Therefore, the General Fund would recognize $3,480 ($348,000 × .01) as revenue. Agency fund란 다른 gov agency를 위해 돈만 관리하고 대신 받아주고, 돈이 수금되면, 바로 전달해주는 애들.. Agency fund only have balance sheet. income을 관리하는 그런 계념이 아니다. 언제나 asset & liability 계정 금액이 똑같다. 돈이 들어오면, asset에다 기록하고, 바로 liabilty에 똑같은 금액 기록하기. 본인의 수수료 (commission)만 떄고, 바로 돌려줘야 하는 돈이니깐. Ex: Revenue of general fund. if agency fund helps other gov sector with collecting AR and receive 1% commission. Cash $100 Due to other gov fund $99 Due to general fund $1 In general fund: Due to general fund $1 Rev control $1 (commission을 revenue로 전환하는 book entry)

When would a company use the installment sales method of revenue recognition? a) When collectability of installment accounts receivable is reasonably predictable. b) When repossessions of merchandise sold on the installment plan may result in a future gain or loss. c) When installment sales are material, and there is no reasonable basis for estimating collectability. d) When collection expenses and bad debts on installment accounts receivable are deemed to be immaterial.

answer: c The installment sales method of revenue recognition is used when sales are material, and the collection of the sales price is not reasonably assured. note that installment method is a revenue recognition method used under US GAAP, not IFRS. (따라서, 질문에 IFRS가 나오면, 무족건 installment sales method는 IFRS에 인정된 방법이 아니다라는것 기억하기)

Green Corp. owns 30% of the outstanding common stock and 100% of the outstanding noncumulative nonvoting preferred stock of Axel Corp. In year 2, Axel declared dividends of $100,000 on its common stock and $60,000 on its preferred stock. Green exercises significant influence over Axel's operations. Green uses the equity method to account for its investment in Axel. What amount of dividend revenue should Green report in its income statement for the year ended December 31, year 2? a) $0 b) $30,000 c) $60,000 d) $90,000

answer: c The preferred dividends received by Green (100% × $60,000 = $60,000) are reported as dividend revenue in year 2. The common dividends received (30% × $100,000 = $30,000) are not reported as dividend revenue. Because the equity method is used (due to 30% ownership and significant influence), the common dividends received are recorded as a reduction of the investment account rather than as dividend revenue. 핵심 summary: Preferred dividends are not affected by equity rules. Equity rules only applies to common stock. We use the equity method when we hold more than 20% of investee's voting stock. Preferred stock is usually non voting, so we don't apply equity method to it.

In its December 31, year 1 balance sheet, Fleet Co. reported accounts receivable of $100,000 before allowance for uncollectible accounts of $10,000. Credit sales during year 2 were $611,000, and collections from customers, excluding recoveries, totaled $591,000. During year 2, accounts receivable of $45,000 were written off and $17,000 were recovered. Fleet estimated that $15,000 of the accounts receivable at December 31, year 2, were uncollectible. In its December 31, year 2 balance sheet, what amount should Fleet report as accounts receivable before allowance for uncollectible accounts? a) $58,000 b) $67,000 c) $75,000 d) $82,000

answer: c The question asked for AR ending balance AR beginning balance : 100 credit sale: Dr. AR 611<- Cr. Sale 611 payment from customer: Dr. Cash 591 Cr. AR 591<- AR written off: Dr. Allowance for doubtful debt 45 Cr. AR 45<- So you have +100+611-591-45=75 recover bad debt is irrelevant because it's not affecting A/R Account. (Basically allowance for doubtful debt으로 write off로 처리되었던게 나중에 다시 돈이 수금되서 reverse해주는것) Dr. cash 17 Cr. Allowance 17 Bad Debt estimate is irrelevant Dr. bad debt expense 15 cr. allowance 15

Reserves for contingencies for general or unspecified business risks should a) Be accrued in the financial statements and disclosed in the notes thereto. b) Not be accrued in the financial statements but should be disclosed in the notes thereto. c) Not be accrued in the financial statements and need not be disclosed in the notes thereto. d) Be accrued in the financial statements but need not be disclosed in the notes thereto.

answer: c This answer is correct because ASC Topic 450 states that no accrual, loss, or disclosure should be made for general or unspecified business risks and that they need not be disclosed. An estimated loss for a loss contingency shall be accrued only if the loss is probable and the amount of the loss is reasonably estimated. 핵심단어-> "General or unspecified business risks do not meet these conditions."

The accounting for special revenue funds is most similar to which type of fund? a) Capital Projects. b) Enterprise. c) General. d) Internal Service.

answer: c because a Special Revenue Fund is like the General Fund, a governmental fund that accounts for "current activities". The General Fund accounts for all transactions not accounted for in any other fund, which generally includes general revenues and expenditures for ordinary operations. (note: general fund are NOT restricted to specific purpose) A Special Revenue Fund accounts for revenues (ear marked) that are legally restricted to collected for a specific purpose, and include library taxes, motor fuel taxes, continuing federal housing grants, or gas taxes. Internal Service Fund accounts for the financing of goods or services provided by one department or agency to other departments or agencies of the governmental unit, or to other governmental units, on a cost‐reimbursement basis. In an Internal Service Fund, the accounting procedures are similar to those for private enterprises. Capital Projects Fund accounts for the proceeds of a bond issue, federal grant, etc., to be used for a capital project such as a new school or library.

When a company purchases land with a building on it and immediately tears down the building so that the land can be used for the construction of a plant, the costs incurred to tear down the building should be a) Expensed as incurred. b) Added to the cost of the plant. c) Added to the cost of the land. d) Amortized over the estimated time period between the tearing down of the building and the completion of the plant.

answer: c because in general, an asset should be recorded at the cost of getting it ready for its intended use. If land is purchased with the intention of constructing a building on it, the cost of tearing down an old building on that land is part of the cost of getting the land ready for its intended use and should be added to the cost of that land. This cost attaches to the land and, therefore, should not be depreciated. Added to the cost of the plant is incorrect answer because costs to tear down the building were incurred to get the land ready for its intended use. Therefore, these costs will be added to the cost of the land, not the plant.

Milton Co. pledged some of its accounts receivable to Good Neighbor Financing Corporation in return for a loan. Which of the following statements is correct? a) Good Neighbor Financing cannot take title to the receivables if Milton does not repay the loan. Title can only be taken if the receivables are factored. b) Good Neighbor Financing will assume the responsibility of collecting the receivables. c) Milton will retain control of the receivables. d) Good Neighbor Financing will take title to the receivables and will return title to Milton after the loan is paid.

answer: c because pledging accounts receivable is treated as a borrowing with the accounts receivable used as collateral for the loan. Therefore, Milton retains control of the receivables.

The correction of an error in the financial statements of a prior period should be reflected, net of applicable income taxes, in the current a) Income statement after income from continuing operations and before discontinued operations. b) Income statement after income from continuing operations and after discontinued operations. c) Retained earnings statement as an adjustment of the opening balance. d) Retained earnings statement after net income but before dividends.

answer: c because the correction of an error in the financial statements of a prior period is a prior period adjustment which is to be shown net of tax as an adjustment to the beginning balance of retained earnings.

On September 30, World Co. borrowed $1,000,000 on a 9% note payable. World paid the first of four quarterly payments of $264,200 when due on December 30. In its December 31 balance sheet, what amount should World report as note payable? a) $735,800 b) $750,000 c) $758,300 d) $825,800

answer: c 우선, 내가 알고 넘어가야 할 부분은, it's asking "note payable" balance as of Dec. 31, end of year. note payable must only include principle amount and not include interest. 따라서, 제일 먼제 해야하는게, 12월달에 pay한 Q1 payment를 분석해서, 이 금액의서 오직 순수히 principle에 해당되는 금액을 빼오기. Interest expense is calculated as $1,000,000 × 9% × 3/12 months = $22,500. (핵심: interest를 계산할때 제일 중요한건 interest pay했을 시점에서 12개월기준으로 나눠야함) The payment of $264,200 less $22,500 in interest is equal to $241,700, which is the amount of the payment which is applied to the principal balance of the note. Therefore, this answer is correct because the carrying value of the note on December 31 is $758,300 ($1,000,000 − $241,700)

On December 31, year 1, the New Bite Company had capitalized costs for a new computer software product with an economic life of 4 years. Sales for year 2 were 10% of expected total sales of the software. At December 31, year 2, the software had a net realizable value equal to 80% of the capitalized cost. The unamortized cost reported on the December 31, year 2 balance sheet should be (The question is asking about "Unamortized cost as of Dec. 31, year 2 for "software") a) Net realizable value. b) 90% of net realizable value. c) 75% of capitalized cost. d) 90% of capitalized cost.

answer: c (note, this is question about amoritization on "SOFTWARE" this rule "ASC 985" only applies to software and not to anything else. one time question같지만, 그냥 외우기) Per ASC 985, the annual amortization of capitalized software costs shall be the "greater" of (1) The ratio of the software's current sales to its expected total sales, or (2) The straight‐line method over the economic life of the product. In this case, the ratio of current to expected total sales is 10% (given). The annual straight‐line rate is 25% per year (1/economic life of 4 years). The straight‐line amortization should be used in year 2, since it is the higher of the two. The unamortized cost on the 12/31/Y2 balance sheet should, therefore, be 75% (100% − 25% amortization) 75% is "unamortized cost as of 12/31/year 2.

On January 1, year 2, Karva Company granted James Dean, the president, an option to purchase 1,000 shares of Karva's $30 par value common stock at $40 per share. The option becomes exercisable on January 1, year 4, after Dean has completed 2 years of service. Assume that the fair value at the grant date of Karva's options with similar terms and conditions is $15. As a result of the option granted to Dean, Karva should recognize compensation expense in year 2 of a) $0 b) $ 5,000 c) $ 7,500 d) $15,000

answer: c The options are measured at OPTION VALUE at grant date of ($15 × 1,000 options)/2 years service period = $7,500 compensation expense in year 2.

On 1/31/Y1, Clay Company leased a new machine from Saxe Corp. The following data relate to the lease transaction at its inception: Lease term 10 years Annual rental payable at beginning of each lease year $50,000 Useful life of machine 15 years Implicit interest rate 10% Present value of an annuity of 1 in advance for 10 periods at 10% 6.76 Present value of annuity of 1 in arrears for 10 periods at 10% 6.15 Fair value of the machine $400,000 The lease has no renewal option, and the possession of the machine reverts to Saxe when the lease terminates. At the inception of the lease, Clay should record a lease liability of a) $400,000 b)$338,000 c) $307,500 d) $0

answer: d "At the inception of a lease, the lessee records a lease liability if the lease is considered to be a capital lease." <-이게 핵심 To be considered a capital lease, a lease must satisfy any one of the four criteria specified This lease does not satisfy any of the four criteria. The lease has no bargain purchase option and does not transfer title. The lease term is not 75% or more of the useful life (10 years out of 15 years is 67%) and the PV of the lease payments is not 90% or more of the FV of the asset [(6.76 × $50,000) / $400,000 = 84.5%]. Therefore, this is an operating lease, not a capital lease, and no liability is recorded at the lease's inception.

The France Company owns a foreign subsidiary with 2,400,000 local currency units (LCU) of property, plant, and equipment before accumulated depreciation at December 31, year 3. Of this amount, 1,500,000 LCU were acquired in year 1 when the rate of exchange was 1.5 LCU to $1, and 900,000 LCU were acquired in year 2 when the rate of exchange was 1.6 LCU to $1. The rate of exchange in effect at December 31, year 3, was 1.9 LCU to $1. The weighted average of exchange rates which were in effect during year 3 was 1.8 LCU to $1. Assuming that the property, plant, and equipment are depreciated using the straight‐line method over a 10‐year period with no salvage value, how much depreciation expense relating to the foreign subsidiary's property, plant, and equipment should be charged in France's income statement for year 3? Assume the US dollar is the functional currency. a) $126,316 b) $133,333 c) $150,000 d) $156,250

answer: d ASC Topic 830 requires remeasurement when the US dollar is the functional currency. Remeasurement means that all assets and liabilities on the balance sheet and revenues and expenses on the income statement are translated at the rates in effect when the transactions originally occurred (e.g., depreciation is translated at the exchange rate in effect at the original transaction date) (i.e., the historical rate). Since the useful life of the fixed assets is 10 years with no salvage value, depreciation will be 150,000 LCU for the equipment acquired in year 1 and 90,000 LCU for the equipment acquired in year 2. These are converted to dollars at their respective historical rates of 1.5 and 1.6 LCU. $1,500,000 × 10% ÷ 1.5 = $100,000 $ 900,000 × 10% ÷ 1.6 = 56,250 $156,250

In accordance with GASB 33, Accounting and Financial Reporting for Nonexchange Transactions, which of the following items is classified as a derived tax revenue? a) Property taxes. b) Fines. c) Grants. d) Motor fuel taxes.

answer: d Accounting for non-exchange transaction definition: In which government gives (or receives) value without directly receiving (or giving) equal value in exchange. Gov (정부기간) 에서는 꼭 어떤한 service를 회사나 개인들한테 공급하지 않아도 저절로 정부는 그들한테서 돈을 받을수가 있다는 그런 내용. this is why it is called accounting for non-exchange transaction. There are 4 types: (1) derived tax revenue-citizen's sales tax, income tax. 사람들이 일을해서 수입이 들어면, 정부가 tax를 때린다. (무엇으로 인해 사람들이 돈이 들어면, 정부는 택스때림. 사람들이 물건을 쇼핑하고 사면, 그걸로 인해서 ,정부는 sales use tax를 챙긴다) (2) imposed non-exchanged transaction- 가만히 있어도 tax가 들어오는것.. house purchase가 없더라고 기존에 property owner들이 property tax내야된다. (딱히 새로운 transaction을 사람들한테서 발생하지 않아도, 기존에 오래전에 집을 산사람은 앞으로도 쭈욱 property tax를 내야한다.) (3) grant- grant is recognized as "revenue" when the money has been used. (Rather than when money is received) (4) grant - unrestricted DONATIONS Derived tax revenues, which result from assessments imposed on exchange transactions (for example, income taxes, sales taxes, and other assessments on earnings or consumption) Derived tax revenues result from taxes assessed on exchange transactions. For derived tax revenues in the govt-wide financial statements, the revenues are recognized when the uderlying transactions occur. For example when a sale is made, the govt should then recognize the resulting sale tax Imposed nonexchange revenues result from assessments by governments on nongovernmental entities, including individuals, other than assessments on exchange transactions. Examples include property (ad valorem) taxes; fines and penalties; and property forfeitures, such as seizures and escheats.

Rue Co.'s allowance for uncollectible accounts had a credit balance of $12,000 at December 31, Year 1. During Year 2, Rue wrote off uncollectible accounts of $48,000. The aging of accounts receivable indicated that a $50,000 allowance for uncollectible accounts was required at December 31, Year 2. What amount of uncollectible accounts expense should Rue report for Year 2? a) $48,000 b) $50,000 c) $60,000 d) $86,000

answer: d An examination of the T‐account indicates the beginning balance in Allowance for uncollectible accounts has a credit balance of $12,000. (Debit "bad expense" credit "allowance for uncollectible" 계정으로 작년에 잡아둔게 이번년에 beginning balance가 되어버린다. When an account is written off, a debit entry is made to the Allowance for uncollectible accounts. (when account is written off, meaning, reversing uncollectible account to debit and crediting A/R) If the aging schedule indicates that a $50,000 allowance is required, then the ending balance in the Allowance for uncollectible accounts must be $50,000. Therefore, this answer is correct because $86,000 ($12,000 − $48,000 − $50,000) must be debited to Bad debt expense and credited to the Allowance for uncollectible accounts.

Seco Corp. was incorporated on January 2, year 1. The following information pertains to Seco's common stock transactions: Year 1 Jan 2 Number of shares authorized 80,000 Feb 1 Number of shares issue 60,000 July 1 Number of shares reacquired but not canceled 5,000 December 1 2‐for‐1 stock split At December 31, year 1, the number of shares of Seco's common stock outstanding is a) 150,000 b) 120,000 c)115,000 d) 110,000

answer: d Before the stock split, 60,000 shares of common stock were issued, of which 5,000 shares were reacquired. Any time stock is reacquired, it is no longer considered to be outstanding. Therefore, there were 55,000 (60,000 − 5,000) shares outstanding before the 2‐for‐1 stock split and 110,000 (55,000 × 2) shares outstanding after the stock split. How to calculate outstanding shares: Add together the numbers of preferred and common shares outstanding, and subtract the number of treasury shares. The result is the total number of shares outstanding **number of stock issue - repurchased stock (treasury stock) number of shares authorized란 별 의미가 없다. Authorized stock represents the maximum number of common shares that can be issued legally by the company as stated in the company's chart

On May 1, Year 1, Reynolds purchased 5,000 shares of common stock of Haywood Corp. for $250,000 and classified the investment as available‐for‐sale securities. On December 31, Year 1, the Haywood stock had a fair value of $257,000. Reynolds Corp. prepares its financial statements in accordance with IFRS. Reynolds elects to use fair value through profit or loss to record its investments in available‐for‐sale securities. How is the gain on the investment in Haywood stock reported in Reynolds's Year 1 financial statements? a) As a $7,000 gain in other comprehensive income. b) No gain or loss is reported in Year 1. c) As a $7,000 prior‐period adjustment to retained earnings. d) As a $7,000 gain in current earnings of the period.

answer: d IFRS requires that if an asset is classified as fair value through profit or loss, it is remeasured to fair value and any profit or loss is recorded in the period. Therefore, Reynolds should recognize a $7,000 gain in current earnings of the period. (여기서 핵심단어는 : company elected to use FMV option through profit or loss) Available for sale의 사실상 revaluation method for loss and gain은 other comprehensive income에 report되는데, 아마, FMV option을 elect하면, 바로 그해에 income statement에 보고되나보다. Financial assets at fair value through profit or loss include financial assets held-for-trading and financial assets designated upon initial recognition at fair value through profit or loss. Financial assets are classified as held-for-trading if they are acquired for the purpose of selling in the near term. Financial assets at fair value through profit and loss are carried in the consolidated balance sheet at fair value with gains or losses recognized in the consolidated statement of income under "Gains or Losses on Derivative Financial Instrument Transactions" for derivative instruments and "Other Income or Expense" for non-derivative financial assets

Thorpe Co.'s income statement for the year ended December 31, year 3, reported net income of $74,100. The auditor raised questions about the following amounts that had been included in net income: Unrealized loss on decline in market value of available‐for‐sale marketable equity securities $(5,400) Gain on early retirement of bonds payable 33,000 Adjustment to profits of prior years for errors in depreciation (net of $3,750 tax effect) (7,500) Loss from fire (net of $7,000 tax effect) (14,000) Thorpe did not elect the fair value option for reporting any of its financial assets. The loss from the fire was an infrequent but not unusual occurrence in Thorpe's line of business. Thorpe's December 31, year 3 income statement should report net income of a) $65,000 b) $66,100 c) $81,600 d) $87,000

answer: d Net income as reported ($74,100) properly included the gain on early retirement of bonds payable ($33,000) and the loss from fire ($14,000). The fact that the loss was reported net of taxes in the income statement was incorrect, but does not cause the net income amount to be in error. However, the other two items should not be reported in the income statement. If Thorpe does not elect the fair value option, the rules of ASC Topic 320 apply. Therefore, an unrealized loss on available‐for‐sale investments in stock ($5,400) is reported in "other comprehensive income," net of tax under one of three acceptable alternatives and as part of "accumulated other comprehensive income" in the stockholders' equity section. A correction of an error ($7,500) is treated as a prior period adjustment. It is reported in the financial statements as an adjustment to the beginning balance of retained earnings, rather than in the income statement. Since both of these items were subtracted in the computation of reported net income, they must be added back to compute the correct net income of $87,000 ($74,100 + $5,400 + $7,500).

On July 1, year 1, Link Development Company purchased a tract of land for $900,000. Additional costs of $150,000 were incurred in subdividing the land during July through December year 1. Of the tract acreage, 70% was subdivided into residential lots as shown below and 30% was conveyed to the city for roads and a park. (real estate company has donated the land to government and here, seems like 70% of land is intended to use as building residential house and 30% of land will be used for park and road. *여기서 tricky 한것은, we know gov can generate sales from building housing rather than building park..so, we will treat 100% cost toward residential lots Lot class Number of lots Sales price per lot A 100 $12,000 B 100 8,000 C 200 5,000 Under the relative sales value method, the cost allocated to each Class A lot should be $2,625 $2,940 $3,600 $4,200

answer: d Per ASC 970, real estate donated to municipalities or other governmental agencies for uses that will benefit the project shall be allocated as a common cost of the project. None of the cost should be allocated to land donated to the city, since that land will not directly generate revenue (and therefore has no sales value). Therefore, the total cost of acquiring the land ($900,000 + $150,000 = $1,050,000) should be allocated to the lots which will generate revenue. The $1,050,000 cost is allocated based on the relative sales value of the lots, as computed below. Lot class # of Sales price Total sales value A 100 × $12,000 = $1,200,000 B 100 × 8,000 = 800,000 C 200 × 5,000 = 1,000,000 $3,000,000 Total cost Fraction allocated to Class A Allocated cost # of Cost per lot in Class A $1,050,000 × ($1,200/$3,000) = $420,000 ÷ 100 = $4,200

Information pertaining to dividends from Wray Corp.'s common stock investments for the year ended December 31, year 2, follows: On September 8, year 2, Wray received a $50,000 cash dividend from Seco, Inc., in which Wray owns a 30% interest. A majority of Wray's directors are also directors of Seco. "The equity method of accounting is used." <-이게 힌트라고 볼수있다. On October 15, year 2, Wray received a $6,000 liquidating dividend from King Co. Wray owns a 5% interest in King Co. "liquid dividend can be hint" Wray owns a 2% interest in Bow Corp., which declared a $200,000 cash dividend on November 27, year 2, to stockholders of record on December 15, year 2, payable on January 5, year 3. What amount should Wray report as dividend income in its income statement for the year ended December 31, year 2? a) $60,000 b) $56,000 c) $10,000 d) $ 4,000

answer: d The cash dividend from Seco ($50,000) is recorded as a reduction of the investment account, rather than as dividend revenue, because the equity method is used. The equity method is used because Wray's ownership interest is greater than 20% and it can exercise significant influence over Seco. Equity Method Example: ABC International acquires a 30% interest in Blue Widgets Corporation. In the most recent reporting period, Blue Widgets recognizes $1,000,000 of net income. Under the requirements of the equity method, ABC records $300,000 of this net income amount as earnings on its investment (as reported on the ABC income statement), which also increases the amount of its investment (as reported on the ABC balance sheet). Investors do not treat dividends as revenue under the equity method. Instead, the investor subtracts the cash dividend amount from the investment carrying value. This treatment recognizes that the value of the investment has decreased by the cash distribution. Since the investor immediately records this effect on its balance sheet, it would constitute double counting to also book the dividend as revenue. The liquidating dividend from King ($6,000) is also recorded as a reduction of the investment account because it is a return of, rather than a return on investment. What is a 'Liquidating Dividend' A type of payment made by a corporation to its shareholders during its partial or full liquidation. For the most part, such a distribution is made from the company's capital base, and as a return of capital, is typically not taxable for shareholders. This distinguishes a liquidating dividend from regular dividends, which are issued from the company's operating profits or retained earnings Wray's share of the Bow dividend (2% × $200,000 = $4,000) is recorded as dividend revenue in year 2 even though it is not received until year 3. Since the date of record was in year 2, Wray should accrue the revenue by debiting dividends receivable and crediting dividend revenue.

Main, a pharmaceutical company, leased office space from Ash. Main took possession and began to use the building on July 1, year 1. Rent was due the first day of each month. Monthly lease payments escalated over the 5‐year period of the lease as follows: Period Lease payment July 1, year 1 - September 30, year 1 $0 - October, 1, year 1- June 30, year 2 17,500 July 1, year 2 - June 30, year 3 19,000 July 1, year 3 - June 30, year 4 20,500 July 1, year 4 - June 30, year 5 23,000 July 1, year 5 - June 30, year 6 24,500 What amount would Main show as deferred rent expense at December 31, year 4? a) $50,658 b) $52,580 c) $68,575 d) $71,550

answer: d The problem involves uneven rent payments, and accounting standards require rent expense to be recognized evenly over the contract period. The rent per month to be paid over the 60‐month period can be calculated as follows: First 3 months $ 0 Next 9 months 17,500 × 9 157,500 Year 2 19,000 × 12 228,000 Year 3 20,500 × 12 246,000 Year 4 23,000 × 12 276,000 Year 5 24,500 × 12 294,000 Total rent payments $1,201,500 Divided by 60 months ÷ 60 months $ 20,025 per month The amount of rent paid as of December 31, year 4, is July 1, year 1 to June 30, year 2 $157,500 July 1, year 2 to June 30, year 3 228,000 July 1, year 3 to June 30, year 4 246,000 July 1, year 4 to December 30, year 4 138,000 (6 months) Total rent paid $769,500 The amount of rent that should be recognized as total rent expense for the period July 1, year 1, to December 31, year 4, is $841,050 (42 months × $20,025 per month). The correct answer is $71,550 ($841,050 − $769,500), the amount of rent that should be recorded as deferred/accrued rent expense. Note that the question asks for deferred rent but in this case the rent is actually accrued (payable).

On December 31, year 1, Clark Company, an investment banker, purchased marketable equity securities with the intent to sell them for a quick profit. Pertinent data are as follows: Security Cost Market value at 12/31/Y2 W $24,000 $26,000 X 36,000 33,000 Y 72,000 65,000 On December 31, year 2, Clark reclassified its investment in security Y from trading to available‐for‐sale because Clark intends to retain security Y as a long‐term investment. Assume Clark does not elect the fair value option for reporting financial assets. What total amount of loss on these securities should be included in Clark's income statement for the year ended December 31, year 2? a) $0 b) $1,000 c) $7,000 d) $8,000

answer: d Unrealized losses on trading securities are reported in the income statement. The amount of the unrealized loss at 12/31/Y2 is determined by comparing the total carrying value of the portfolio at 1/01/Y2 to its total market value at 12/31/Y2. The carrying value at 1/01/Y2 is its cost of $132,000 ($24,000 + $36,000 + $72,000) and the market value at 12/31/Y2 is $124,000 ($26,000 + $33,000 + $65,000). Therefore, $8,000 total unrealized loss on trading securities is recognized in income. The unrealized gain or loss on trading securities on the date that they are transferred from that category will have already been recognized in earnings and shall not be reversed. Transfer to or from Trading is one of the easier ones to remember. Transfer to Trading from any category realize gains and losses on IS right away. Transfer from Trading to any other category do not reverse what you have recognized to that point (12/31/YEAR2)에 Security Y 를 trading security에서 AFS로 transfer한다고 해도, 12/31/year2 에 이미 security Y가 trading security에 당시의 loss ($8,000 <- 72,000 -65,000)를 income statement에 directly report가 된상황이다. You can't never reverse what you have recognized from the income statement from this point. <- 핵심 tricky하네. Even if the company don't elect FMV election, gain/loss incur from trading security directly reports in the income statement.

Deferred income tax expense resulting from temporary differences related to depreciation of plant assets should be presented in a statement of cash flows (using indirect approach for operating activities) as a(n) a) Noncash financing and investing activity reported in a separate schedule. b) Financing activity. c) Deduction from net income. d) Addition to net income.

answer: d When using the indirect approach to determine cash flows from operating activities, noncash deductions from net income should be added back to determine cash flows from operating activities. Since the deferred income tax expense did not use cash but was subtracted in determining net income, it is proper to add it back to net income in cash flows from operating activities.

A necessary condition for the recording of a pension liability is present when a) Projected benefit obligation exceeds accumulated benefit obligation. b) The market‐related asset value exceeds accumulated benefit obligation. c) Accumulated benefit obligation exceeds the fair value of plan assets. d) Projected benefit obligation exceeds pension plan assets.

answer: d ASC Topic 715 requires that if the projected benefit obligation exceeds the fair value of plan assets, a liability must be recognized in the balance sheet The projected benefit obligation (PBO) is a pension concept in accounting. The PBO is the present value of an employee's pension. In other words, a pension liability is the difference between the total amount due to retired employee and the actual amount of money the company has on hand to make those payments. A pension plan is a retirement plan that requires an employer to make contributions into a pool of funds set aside for a worker's future benefit. The pool of funds is invested on the employee's behalf, and the earnings on the investments generate income to the worker upon retirement. Funded Status = Plan Assets - Projected Benefit Obligation (PBO) if plan asset < projected benefit obligation, it is "underfunded" therefore, must pay if plan asset > projected benefit obligation, it is "overfunded"

Which of the following is generally associated with payables classified as accounts payable? Periodic payment of interest: No Secured by collateral: No .

because in general, accounts payable are liabilities to suppliers that are incurred in the regular course of a company's business. Short‐term liabilities such as accounts payable do not usually provide for the periodic payment of interest or a security interest in collateral.


Related study sets

Prep U Chapter 34: Assessment and Management of Patients with Inflammatory Rheumatic Disorders

View Set

Chapter 13: Supply Chain Process Integration

View Set

Chapter 1 UAS Foundations: Progress Check

View Set

Genomics - All the experiment techniques

View Set

MATERNAL NEWBORN - PRACTICE QUESTIONS (DRAFT)

View Set